Π‘ΠΈΠ»Π° Ρ‚ΠΎΠΊΠ° напряТСниС сопротивлСниС: 1. ЭлСктричСскоС сопротивлСниС. Π—Π°ΠΊΠΎΠ½ Ома для участка элСктричСской Ρ†Π΅ΠΏΠΈ

Π‘ΠΎΠ΄Π΅Ρ€ΠΆΠ°Π½ΠΈΠ΅

Π—Π°Π΄Π°Ρ‡ΠΈ

Π—Π°Π΄Π°Ρ‡ΠΈ ΠΊ ΡƒΡ€ΠΎΠΊΡƒ 50/14

1.Β Β Β Β Β  ΠšΠΎΡΠΌΠΈΡ‡Π΅ΡΠΊΠ°Ρ Ρ€Π°ΠΊΠ΅Ρ‚Π° ΠΏΡ€ΠΈ стартС с Π—Π΅ΠΌΠ»ΠΈ двиТСтся Π²Π΅Ρ€Ρ‚ΠΈΠΊΠ°Π»ΡŒΠ½ΠΎ Π²Π²Π΅Ρ€Ρ… с ускорСниСм a = 25 ΠΌ/с2. ΠžΠΏΡ€Π΅Π΄Π΅Π»ΠΈΡ‚Π΅ вСс космонавта массой m = 100 ΠΊΠ³. УскорСниС свободного падСния ΡΡ‡ΠΈΡ‚Π°Ρ‚ΡŒ Ρ€Π°Π²Π½Ρ‹ΠΌ 10 ΠΌ/с2.

2.Β Β Β Β Β  ΠŸΠ°Ρ€Π°ΡˆΡŽΡ‚ΠΈΡΡ‚, достигнув Π² затяТном ΠΏΡ€Ρ‹ΠΆΠΊΠ΅ скорости Ο…1 = 60 ΠΌ/с, раскрыл ΠΏΠ°Ρ€Π°ΡˆΡŽΡ‚, послС Ρ‡Π΅Π³ΠΎ Π΅Π³ΠΎ ΡΠΊΠΎΡ€ΠΎΡΡ‚ΡŒ Π·Π° t = 2 с ΡƒΠΌΠ΅Π½ΡŒΡˆΠΈΠ»Π°ΡΡŒ Π΄ΠΎ Ο…2 = 10 ΠΌ/с. Π§Π΅ΠΌΡƒ Ρ€Π°Π²Π΅Π½ вСс ΠΏΠ°Ρ€Π°ΡˆΡŽΡ‚ΠΈΡΡ‚Π° массой m = 70 ΠΊΠ³ Π²ΠΎ врСмя тормоТСния? УскорСниС свободного падСния ΡΡ‡ΠΈΡ‚Π°Ρ‚ΡŒ Ρ€Π°Π²Π½Ρ‹ΠΌ 10 ΠΌ/с2.

3.Β Β Β Β Β  Π‘Π°ΠΌΠΎΠ»Π΅Ρ‚, двигаясь с постоянной ΡΠΊΠΎΡ€ΠΎΡΡ‚ΡŒΡŽ 720 ΠΊΠΌ/Ρ‡, ΡΠΎΠ²Π΅Ρ€ΡˆΠ°Π΅Ρ‚ Ρ„ΠΈΠ³ΡƒΡ€Ρƒ Π²Ρ‹ΡΡˆΠ΅Π³ΠΎ ΠΏΠΈΠ»ΠΎΡ‚Π°ΠΆΠ° – Β«ΠΌΠ΅Ρ€Ρ‚Π²ΡƒΡŽ ΠΏΠ΅Ρ‚Π»ΡŽΒ» – радиусом 1000 ΠΌ. Π§Π΅ΠΌΡƒ Ρ€Π°Π²Π½Π° ΠΏΠ΅Ρ€Π΅Π³Ρ€ΡƒΠ·ΠΊΠ° Π»Π΅Ρ‚Ρ‡ΠΈΠΊΠ° Π² Π²Π΅Ρ€Ρ…Π½Π΅ΠΉ Ρ‚ΠΎΡ‡ΠΊΠ΅ ΠΏΠ΅Ρ‚Π»ΠΈ? (g = 10 ΠΌ/с2).

Β 

Π—Π°Π΄Π°Ρ‡ΠΈ Π΄/Π· ΠΊ ΡƒΡ€ΠΎΠΊΡƒ 48/12

1.Β Β Β Β Β Β Β Β  Π’ΠΎ сколько Ρ€Π°Π· измСнится сила ВсСмирного тяготСния, Ссли массу ΠΎΠ΄Π½ΠΎΠ³ΠΎ Ρ‚Π΅Π»Π° ΡƒΠ²Π΅Π»ΠΈΡ‡ΠΈΡ‚ΡŒ Π² 3 Ρ€Π°Π·Π°, Π° Π΄Ρ€ΡƒΠ³ΠΎΠ³ΠΎ ΡƒΠΌΠ΅Π½ΡŒΡˆΠΈΡ‚ΡŒ Π² 9 Ρ€Π°Π·?

2.Β Β Β Β Β Β Β Β  Π’ΠΎ сколько Ρ€Π°Π· измСнится сила ВсСмирного тяготСния, Ссли расстояниС ΠΌΠ΅ΠΆΠ΄Ρƒ Ρ‚Π΅Π»Π°ΠΌΠΈ ΡƒΠΌΠ΅Π½ΡŒΡˆΠΈΡ‚ΡŒ Π² 5 Ρ€Π°Π·?

3.Β Β Β Β Β Β Β Β  Π‘ ΠΊΠ°ΠΊΠΈΠΌ ускорСниСм всплываСт Ρ‚Π΅Π»ΠΎ массой 25 ΠΊΠ³, Ссли Π½Π° Π½Π΅Π³ΠΎ дСйствуСт сила АрхимСда 300 Н?

Π—Π°Π΄Π°Ρ‡ΠΈ Π΄/Π· ΠΊ ΡƒΡ€ΠΎΠΊΡƒ 60 Β 

1. ΠŸΠΎΡ‡Π΅ΠΌΡƒ Π½Π΅Π²ΠΎΠ·ΠΌΠΎΠΆΠ½ΠΎ, ΠΈΠ· полоТСния сидя прямо Π½Π° стулС, Π²ΡΡ‚Π°Ρ‚ΡŒ Π½Π° Π½ΠΎΠ³ΠΈ, Π½Π΅ наклонившись ΠΏΡ€Π΅Π΄Π²Π°Ρ€ΠΈΡ‚Π΅Π»ΡŒΠ½ΠΎ Π²ΠΏΠ΅Ρ€Π΅Π΄?

2. ΠŸΠΎΡ‡Π΅ΠΌΡƒ ΠΎΠ΄Π½ΠΎΡ€ΠΎΠ΄Π½Ρ‹ΠΉ ΠΏΡ€ΡΠΌΠΎΡƒΠ³ΠΎΠ»ΡŒΠ½Ρ‹ΠΉ ΠΊΠΈΡ€ΠΏΠΈΡ‡ ΠΌΠΎΠΆΠ½ΠΎ ΠΏΠΎΠ»ΠΎΠΆΠΈΡ‚ΡŒ Π½Π° ΠΊΡ€Π°ΠΉ стола, Ρ‚ΠΎΠ»ΡŒΠΊΠΎ Ссли с края стола свисаСт Π½Π΅ Π±ΠΎΠ»Π΅Π΅ ΠΏΠΎΠ»ΠΎΠ²ΠΈΠ½Ρ‹ Π΄Π»ΠΈΠ½Ρ‹ ΠΊΠΈΡ€ΠΏΠΈΡ‡Π°?

3. ΠŸΠΎΡ‡Π΅ΠΌΡƒ Π²Ρ‹ Π²Ρ‹Π½ΡƒΠΆΠ΄Π΅Π½Ρ‹ ΠΎΡ‚ΠΊΠ»ΠΎΠ½ΡΡ‚ΡŒΡΡ Π½Π°Π·Π°Π΄, ΠΊΠΎΠ³Π΄Π° нСсСтС Π² Ρ€ΡƒΠΊΠ°Ρ… тяТСлый Π³Ρ€ΡƒΠ·?

Π—Π°Π΄Π°Ρ‡ΠΈ Π΄/Π· ΠΊ ΡƒΡ€ΠΎΠΊΡƒ 58/7Β 

1. Какова срСдняя сила давлСния F Π½Π° ΠΏΠ»Π΅Ρ‡ΠΎ ΠΏΡ€ΠΈ ΡΡ‚Ρ€Π΅Π»ΡŒΠ±Π΅ ΠΈΠ· Π°Π²Ρ‚ΠΎΠΌΠ°Ρ‚Π°, Ссли масса ΠΏΡƒΠ»ΠΈ m = 10 Π³, Π° ΡΠΊΠΎΡ€ΠΎΡΡ‚ΡŒ ΠΏΡƒΠ»ΠΈ ΠΏΡ€ΠΈ Π²Ρ‹Π»Π΅Ρ‚Π΅ ΠΈΠ· ΠΊΠ°Π½Π°Π»Π° ствола v = 300 ΠΌ/с? Автомат Π΄Π΅Π»Π°Π΅Ρ‚ 300 выстрСлов Π² ΠΌΠΈΠ½ΡƒΡ‚Ρƒ.

2. Для провСдСния ΠΎΠ³Π½Π΅Π²Ρ‹Ρ… испытаний Тидкостный Ρ€Π°ΠΊΠ΅Ρ‚Π½Ρ‹ΠΉ Π΄Π²ΠΈΠ³Π°Ρ‚Π΅Π»ΡŒ Π·Π°ΠΊΡ€Π΅ΠΏΠΈΠ»ΠΈ Π½Π° стСндС. Π‘ ΠΊΠ°ΠΊΠΎΠΉ силой ΠΎΠ½ дСйствуСт Π½Π° стСнд, Ссли ΡΠΊΠΎΡ€ΠΎΡΡ‚ΡŒ истСчСния ΠΏΡ€ΠΎΠ΄ΡƒΠΊΡ‚ΠΎΠ² сгорания ΠΈΠ· сопла 150 ΠΌ/с, Π° расход Ρ‚ΠΎΠΏΠ»ΠΈΠ²Π° Π·Π° 5 сСкунд составил 30 ΠΊΠ³?

3. Π Π°ΠΊΠ΅Ρ‚Π° массой 1000 ΠΊΠ³ Π½Π΅ΠΏΠΎΠ΄Π²ΠΈΠΆΠ½ΠΎ зависла Π½Π°Π΄ ΠΏΠΎΠ²Π΅Ρ€Ρ…Π½ΠΎΡΡ‚ΡŒΡŽ Π·Π΅ΠΌΠ»ΠΈ. Бколько Ρ‚ΠΎΠΏΠ»ΠΈΠ²Π° Π² Π΅Π΄ΠΈΠ½ΠΈΡ†Ρƒ Π²Ρ€Π΅ΠΌΠ΅Π½ΠΈ сТигаСт Ρ€Π°ΠΊΠ΅Ρ‚Π°, Ссли ΡΠΊΠΎΡ€ΠΎΡΡ‚ΡŒ истСчСния ΠΏΡ€ΠΎΠ΄ΡƒΠΊΡ‚ΠΎΠ² сгорания ΠΈΠ· Ρ€Π°ΠΊΠ΅Ρ‚Ρ‹ Ρ€Π°Π²Π½Π° 2 ΠΊΠΌ/с?

Π—Π°ΠΊΠΎΠ½ Ома — Ρ„ΠΈΠ·ΠΈΠΊΠ° процСсса Π½Π° ΠΏΡ€ΠΈΠΌΠ΅Ρ€Π΅ двиТСния Π²ΠΎΠ΄Ρ‹. Π€ΠΎΡ€ΠΌΡƒΠ»Ρ‹ зависимости сопротивлСния, напряТСния, силы Ρ‚ΠΎΠΊΠ° ΠΈ мощности

БущСствуСт всСго 2 Π±Π°Π·ΠΎΠ²Ρ‹Ρ… Ρ„ΠΎΡ€ΠΌΡƒΠ»Ρ‹ ΠΊΠΎΡ‚ΠΎΡ€Ρ‹Π΅ ΠΏΠΎΠΌΠΎΠ³ΡƒΡ‚ Π²Π°ΠΌ ΠΏΠΎΠ½ΡΡ‚ΡŒ взаимосвязь ΠΌΠ΅ΠΆΠ΄Ρƒ силой Ρ‚ΠΎΠΊΠ°(АмСр), напряТСниСм(Π’ΠΎΠ»ΡŒΡ‚), сопротивлСниСм (Ом) ΠΈ ΠΌΠΎΡ‰Π½ΠΎΡΡ‚ΡŒΡŽ (Π’Π°Ρ‚Ρ‚).
Зная хотя Π±Ρ‹ Π΄Π²Π° ΠΈΠ· пСрСчислСнных ΠΏΠ°Ρ€Π°ΠΌΠ΅Ρ‚Ρ€Π° Π²Ρ‹ всСгда ΠΌΠΎΠΆΠ΅Ρ‚Π΅ Ρ€Π°ΡΡΡ‡ΠΈΡ‚Π°Ρ‚ΡŒ Π΄Π²Π° Π΄Ρ€ΡƒΠ³ΠΈΡ….
Β 

Π—ΠΠšΠžΠ ОМА

Базовая Ρ„ΠΎΡ€ΠΌΡƒΠ»Π° P=I*E E=I*R Β 
РасчСт напряТСния E=P/I E=I*R E=SQR(P*R)
РасчСт силы Ρ‚ΠΎΠΊΠ° I=P/E I=E/R I=SQR(P/R)
РасчСт мощности P=I*E P=E 2 /R P=I 2 *R
РасчСт сопротивлСния R=E 2 /P R=E/I R=P/I 2
P — ΠœΠΎΡ‰Π½ΠΎΡΡ‚ΡŒ (Π’Π°Ρ‚Ρ‚)
E — НапряТСниС (Π’ΠΎΠ»ΡŒΡ‚)
I — Π‘ΠΈΠ»Π° Ρ‚ΠΎΠΊΠ° (АмпСр)
R — ЭлСктричСскоС сопротивлСниС (Ом)
SQR — ΠΊΠ²Π°Π΄Ρ€Π°Ρ‚Π½Ρ‹ΠΉ ΠΊΠΎΡ€Π΅Π½ΡŒ

Β 


Для справки:

ΠœΡ‹ ΠΈΡΠΏΠΎΠ»ΡŒΠ·ΡƒΠ΅ΠΌ ΠΏΠ΅Ρ€Π΅ΠΌΠ΅Π½Π½ΡƒΡŽ E для обозначСния напряТСния, ΠΈΠ½ΠΎΠ³Π΄Π° Π²Ρ‹ ΠΌΠΎΠΆΠ΅Ρ‚Π΅ Π²ΡΡ‚Ρ€Π΅Ρ‚ΠΈΡ‚ΡŒΒ  ΠΎΠ±ΠΎΠ·Π½Π°Ρ‡Π΅Π½ΠΈΠ΅ V для напряТСния. НС Π΄Π°ΠΉΡ‚Π΅ сСбя Π·Π°ΠΏΡƒΡ‚Π°Ρ‚ΡŒ названиям ΠΏΠ΅Ρ€Π΅ΠΌΠ΅Π½Π½Ρ‹Ρ….

ИзмСнСниС сопротивлСния:

На ΡΠ»Π΅Π΄ΡƒΡŽΡ‰Π΅ΠΉ схСмС Π²Ρ‹ Π²ΠΈΠ΄ΠΈΡ‚Π΅ Ρ€Π°Π·Π½ΠΎΡΡ‚ΡŒ сопротивлСний ΠΌΠ΅ΠΆΠ΄Ρƒ систСмами ΠΈΠ·ΠΎΠ±Ρ€Π°ΠΆΠ΅Π½Π½Ρ‹ΠΌΠΈ Π½Π° ΠΏΡ€Π°Π²ΠΎΠΉ ΠΈ Π»Π΅Π²ΠΎΠΉ сторонС рисунка. Π‘ΠΎΠΏΡ€ΠΎΡ‚ΠΈΠ²Π»Π΅Π½ΠΈΠ΅ давлСнию Π²ΠΎΠ΄Ρ‹ Π² ΠΊΡ€Π°Π½Π΅ противодСйствуСт Π·Π°Π΄Π²ΠΈΠΆΠΊΠ°, Π² зависимости ΠΎΡ‚ стСпСни открытия Π·Π°Π΄Π²ΠΈΠΆΠΊΠΈ измСняСтся сопротивлСниС.

Π‘ΠΎΠΏΡ€ΠΎΡ‚ΠΈΠ²Π»Π΅Π½ΠΈΠ΅ Π² ΠΏΡ€ΠΎΠ²ΠΎΠ΄Π½ΠΈΠΊΠ΅ ΠΈΠ·ΠΎΠ±Ρ€Π°ΠΆΠ΅Π½ΠΎ Π² Π²ΠΈΠ΄Π΅ суТСния ΠΏΡ€ΠΎΠ²ΠΎΠ΄Π½ΠΈΠΊΠ°, Ρ‡Π΅ΠΌ Π±ΠΎΠ»Π΅Π΅ ΡƒΠ·ΠΊΠΈΠΉ ΠΏΡ€ΠΎΠ²ΠΎΠ΄Π½ΠΈΠΊ Ρ‚Π΅ΠΌ большС ΠΎΠ½ противодСйствуСт ΠΏΡ€ΠΎΡ…ΠΎΠΆΠ΄Π΅Π½ΠΈΡŽ Ρ‚ΠΎΠΊΠ°.

Π’Ρ‹ ΠΌΠΎΠΆΠ΅Ρ‚Π΅ Π·Π°ΠΌΠ΅Ρ‚ΠΈΡ‚ΡŒ Ρ‡Ρ‚ΠΎ Π½Π° ΠΏΡ€Π°Π²ΠΎΠΉ ΠΈ Π½Π° Π»Π΅Π²ΠΎΠΉ сторонС схСмы напряТСниС ΠΈ Π΄Π°Π²Π»Π΅Π½ΠΈΠ΅ Π²ΠΎΠ΄Ρ‹ ΠΎΠ΄ΠΈΠ½Π°ΠΊΠΎΠ²ΠΎ.

Π’Π°ΠΌ Π½Π΅ΠΎΠ±Ρ…ΠΎΠ΄ΠΈΠΌΠΎ ΠΎΠ±Ρ€Π°Ρ‚ΠΈΡ‚ΡŒ Π²Π½ΠΈΠΌΠ°Π½ΠΈΠ΅ Π½Π° самый Π²Π°ΠΆΠ½Ρ‹ΠΉ Ρ„Π°ΠΊΡ‚.

Π’ зависимости ΠΎΡ‚ сопротивлСния  увСличиваСтся ΠΈ ΡƒΠΌΠ΅Π½ΡŒΡˆΠ°Π΅Ρ‚ΡΡ сила Ρ‚ΠΎΠΊΠ°.

Π‘Π»Π΅Π²Π° ΠΏΡ€ΠΈ ΠΏΠΎΠ»Π½ΠΎΡΡ‚ΡŒΡŽ ΠΎΡ‚ΠΊΡ€Ρ‹Ρ‚ΠΎΠΉ Π·Π°Π΄Π²ΠΈΠΆΠΊΠ΅ ΠΌΡ‹ Π²ΠΈΠ΄ΠΈΠΌ самый большой ΠΏΠΎΡ‚ΠΎΠΊ Π²ΠΎΠ΄Ρ‹. И ΠΏΡ€ΠΈ самом Π½ΠΈΠ·ΠΊΠΎΠΌ сопротивлСнии, Π²ΠΈΠ΄ΠΈΠΌ самый большой ΠΏΠΎΡ‚ΠΎΠΊ элСктронов (АмпСраТ) Π² ΠΏΡ€ΠΎΠ²ΠΎΠ΄Π½ΠΈΠΊΠ΅.

Π‘ΠΏΡ€Π°Π²Π° Π·Π°Π΄Π²ΠΈΠΆΠΊΠ° Π·Π°ΠΊΡ€Ρ‹Ρ‚Π° Π½Π°ΠΌΠ½ΠΎΠ³ΠΎ большС ΠΈ ΠΏΠΎΡ‚ΠΎΠΊ Π²ΠΎΠ΄Ρ‹ Ρ‚ΠΎΠΆΠ΅ стал Π½Π°ΠΌΠ½ΠΎΠ³ΠΎ большС.

ΡƒΠΆΠ΅Π½ΠΈΠ΅ ΠΏΡ€ΠΎΠ²ΠΎΠ΄Π½ΠΈΠΊΠ° Ρ‚ΠΎΠΆΠ΅ ΡƒΠΌΠ΅Π½ΡŒΡˆΠΈΠ»ΠΎΡΡŒ Π²Π΄Π²ΠΎΠ΅, я Π·Π½Π°Ρ‡ΠΈΡ‚ Π²Π΄Π²ΠΎΠ΅ ΡƒΠ²Π΅Π»ΠΈΡ‡ΠΈΠ»ΠΎΡΡŒ сопротивлСниС ΠΏΡ€ΠΎΡ‚Π΅ΠΊΠ°Π½ΠΈΡŽ Ρ‚ΠΎΠΊΠ°. Как ΠΌΡ‹ Π²ΠΈΠ΄ΠΈΠΌ Ρ‡Π΅Ρ€Π΅Π· ΠΏΡ€ΠΎΠ²ΠΎΠ΄Π½ΠΈΠΊ ΠΈΠ· Π·Π° выского сопротивлСния ΠΏΡ€ΠΎΡ‚Π΅ΠΊΠ°Π΅Ρ‚ Π² Π΄Π²Π° Ρ€Π°Π·Π° мСньшС элСктронов.


Для справки

ΠžΠ±Ρ€Π°Ρ‚ΠΈΡ‚Π΅ Π²Π½ΠΈΠΌΠ°Π½ΠΈΠ΅ Ρ‡Ρ‚ΠΎ суТСниС ΠΏΡ€ΠΎΠ²ΠΎΠ΄Π½ΠΈΠΊΠ° ΠΈΠ·ΠΎΠ±Ρ€Π°ΠΆΠ΅Π½Π½ΠΎΠ΅ Π½Π° схСмС ΠΈΡΠΏΠΎΠ»ΡŒΠ·ΡƒΠ΅Ρ‚ΡΡ Ρ‚ΠΎΠ»ΡŒΠΊΠΎ для ΠΏΡ€ΠΈΠΌΠ΅Ρ€Π° сопротивлСния ΠΏΡ€ΠΎΡ‚Π΅ΠΊΠ°Π½ΠΈΡŽ Ρ‚ΠΎΠΊΠ°. Π’ Ρ€Π΅Π°Π»ΡŒΠ½Ρ‹Ρ… условиях суТСния ΠΏΡ€ΠΎΠ²ΠΎΠ΄Π½ΠΈΠΊΠ° Π½Π΅ сильно влияСт Π½Π° ΠΏΡ€ΠΎΡ‚Π΅ΠΊΠ°ΡŽΡ‰ΠΈΠΉ Ρ‚ΠΎΠΊ. Π—Π½Π°Ρ‡ΠΈΡ‚Π΅Π»ΡŒΠ½ΠΎ большСС сопротивлСниС ΠΌΠΎΠ³ΡƒΡ‚ ΠΎΠΊΠ°Π·Ρ‹Π²Π°Ρ‚ΡŒ ΠΏΠΎΠ»ΡƒΠΏΡ€ΠΎΠ²ΠΎΠ΄Π½ΠΈΠΊΠΈ ΠΈ диэлСктрики.

Π‘ΡƒΠΆΠ°ΡŽΡ‰ΠΈΠΉΡΡ ΠΏΡ€ΠΎΠ²ΠΎΠ΄Π½ΠΈΠΊ Π½Π° схСмС ΠΈΠ·ΠΎΠ±Ρ€Π°ΠΆΠ΅Π½ лишь для ΠΏΡ€ΠΈΠΌΠ΅Ρ€Π°, для понимания сути происходящСго процСсса.

Π€ΠΎΡ€ΠΌΡƒΠ»Π° Π·Π°ΠΊΠΎΠ½Π° Ома — Π·Π°Π²ΠΈΡΠΈΠΌΠΎΡΡ‚ΡŒ сопротивлСния ΠΈ силы Ρ‚ΠΎΠΊΠ°

I = E/R

Как Π²Ρ‹ Π²ΠΈΠ΄ΠΈΡ‚Π΅ ΠΈΠ· Ρ„ΠΎΡ€ΠΌΡƒΠ»Ρ‹, сила Ρ‚ΠΎΠΊΠ° ΠΎΠ±Ρ€Π°Ρ‚Π½Π°ΠΏΡ€ΠΎΠΏΠΎΡ€Ρ†ΠΈΠΎΠ½Π°Π»ΡŒΠ½Π° ΡΠΎΠΏΡ€ΠΎΡ‚ΠΈΠ²Π»Π΅Π½ΠΈΡŽ Ρ†Π΅ΠΏΠΈ.

Π‘ΠΎΠ»ΡŒΡˆΠ΅ сопротивлСниС = МСньшС Ρ‚ΠΎΠΊ

Β 

* ΠΏΡ€ΠΈ условии Ρ‡Ρ‚ΠΎ напряТСниС постоянно.
Β 

ИзмСнСниС напряТСния.

На ΠΈΠ·ΠΎΠ±Ρ€Π°ΠΆΠ΅Π½Π½ΠΎΠΉ схСмС Π²ΠΎ всСх систСмах сопротивлСниС ΠΈΠΌΠ΅Π΅Ρ‚ ΠΎΠ΄ΠΈΠ½Π°ΠΊΠΎΠ²ΡƒΡŽ Π²Π΅Π»ΠΈΡ‡ΠΈΠ½Ρƒ.
Π’ этот Ρ€Π°Π· Π½Π° ΠΊΠ°Ρ€Ρ‚ΠΈΠ½ΠΊΠ΅ измСняСтся сопротивлСниС/Π΄Π°Π²Π»Π΅Π½ΠΈΠ΅.

Π’Ρ‹ ΠΌΠΎΠΆΠ΅Ρ‚Π΅ ΡƒΠ²ΠΈΠ΄Π΅Ρ‚ΡŒ Ρ‡Ρ‚ΠΎ ΠΏΡ€ΠΈ ΡƒΠ²Π΅Π»ΠΈΡ‡Π΅Π½ΠΈΠΈ напряТСния ΠΏΡ€ΠΈΠ²ΠΎΠ΄ΠΈΡ‚ ΠΊ ΡƒΠ²Π΅Π»ΠΈΡ‡Π΅Π½ΠΈΡŽ ΠΏΡ€ΠΎΡ‚Π΅ΠΊΠ°ΡŽΡ‰Π΅Π³ΠΎ Ρ‚ΠΎΠΊΠ° Π΄Π°ΠΆΠ΅ ΠΏΡ€ΠΈ постоянном сопротивлСнии.

Π€ΠΎΡ€ΠΌΡƒΠ»Π° Π·Π°ΠΊΠΎΠ½Π° Ома — Π·Π°Π²ΠΈΡΠΈΠΌΠΎΡΡ‚ΡŒ напряТСния ΠΈ силы Ρ‚ΠΎΠΊΠ°

I = E/R

ΠžΠ±Ρ€Π°Ρ‚ΠΈΡ‚Π΅ Π²Π½ΠΈΠΌΠ°Π½ΠΈΠ΅ Ρ‡Ρ‚ΠΎ сила Ρ‚ΠΎΠΊΠ° ΠΏΡ€ΠΎΡ‚Π΅ΠΊΠ°ΡŽΡ‰Π΅Π³ΠΎ Π² ΠΏΡ€ΠΎΠ²ΠΎΠ΄Π½ΠΈΠΊΠ΅ ΠΏΡ€ΡΠΌΠΎΠΏΡ€ΠΎΠΏΠΎΡ€Ρ†ΠΈΠΎΠ½Π°Π»ΡŒΠ½Π° Π½Π°ΠΏΡ€ΡΠΆΠ΅Π½ΠΈΡŽ.

Π‘ΠΎΠ»ΡŒΡˆΠ΅ напряТСниС = Π‘ΠΎΠ»ΡŒΡˆΠ΅ сила Ρ‚ΠΎΠΊΠ°

Β 

* ΠΏΡ€ΠΈ условии Ρ‡Ρ‚ΠΎ сопротивлСниС постоянно.
Β 

ΠœΠ°Ρ‚Π΅ΠΌΠ°Ρ‚ΠΈΡ‡Π΅ΡΠΊΠΈΠΉ рассчСт


Рассмотрим ΠΏΡ€ΠΈΠΌΠ΅Ρ€.
Π£ нас Π΅ΡΡ‚ΡŒ аккумуляторная батарСя с напряТСниСм питания 12 Π’ΠΎΠ»ΡŒΡ‚. К Π½Π΅ΠΉ Π½Π°ΠΏΡ€ΡΠΌΡƒΡŽ ΠΏΠΎΠ΄ΠΊΠ»ΡŽΡ‡Π΅Π½ рСзистор (сопротивлСниС) 10 Ом. Для Ρ‚ΠΎΠ³ΠΎ Ρ‡Ρ‚ΠΎ Π±Ρ‹ Ρ€Π°ΡΡΡ‡ΠΈΡ‚Π°Ρ‚ΡŒ какая ΠΌΠΎΡ‰Π½ΠΎΡΡ‚ΡŒ ΠΏΡ€ΠΈΠ»ΠΎΠΆΠ΅Π½Π° ΠΊ Π½Π°ΡˆΠ΅ΠΌΡƒ рСзистору, ΠΌΠΎΠΆΠ½ΠΎ Π²ΠΎΡΠΏΠΎΠ»ΡŒΠ·ΠΎΠ²Π°Ρ‚ΡŒΡΡ Ρ„ΠΎΡ€ΠΌΡƒΠ»ΠΎΠΉ.

P = E2/R
P = 122/10
P = 144/10.
P = 14.4 watts

ΠœΠΎΡ‰Π½ΠΎΡΡ‚ΡŒ рассСиваСмая Π½Π° рСзисторС состовляСт 14,4 Π’Π°Ρ‚Ρ‚Π°.

Если Π²Ρ‹ Ρ…ΠΎΡ‚ΠΈΡ‚Π΅ ΠΎΠΏΡ€Π΅Π΄Π΅Π»ΠΈΡ‚ΡŒ Π²Π΅Π»ΠΈΡ‡ΠΈΠ½Ρƒ Ρ‚ΠΎΠΊΠ° ΠΏΡ€ΠΎΡ‚Π΅ΠΊΠ°ΡŽΡ‰Π΅Π³ΠΎ Ρ‡Π΅Ρ€Π΅Π· ΠΏΡ€ΠΎΠ²ΠΎΠ΄Π½ΠΈΠΊ, ΠΌΡ‹ ΠΈΡΠΏΠΎΠ»ΡŒΠ·ΡƒΠ΅ΠΌ Π΄Ρ€ΡƒΠ³ΡƒΡŽ Ρ„ΠΎΡ€ΠΌΡƒΠ»Ρƒ

I = E/R
I = 12/10
I = 1.2 amps

Π‘ΠΈΠ»Π° Ρ‚ΠΎΠΊΠ° ΠΏΡ€ΠΎΡ‚Π΅ΠΊΠ°ΡŽΡ‰Π΅Π³ΠΎ Ρ‡Π΅Ρ€Π΅Π· Ρ†Π΅ΠΏΡŒ составляСт 1,2 АмпСра
—————-
ΠšΠ°Π»ΡŒΠΊΡƒΠ»ΡΡ‚ΠΎΡ€Ρ‹ зависимости напряТСния, силы Ρ‚ΠΎΠΊΠ° ΠΈ сопротивлСния.
Β 

1. ΠšΠ°Π»ΡŒΠΊΡƒΠ»ΡΡ‚ΠΎΡ€ рассСиваСмой мощности  ΠΈ ΠΏΡ€ΠΎΡ‚Π΅ΠΊΠ°ΡŽΡ‰Π΅ΠΉ силы Ρ‚ΠΎΠΊΠ° Π² зависимости ΠΎΡ‚ сопротивлСния ΠΈ ΠΏΡ€ΠΈΠ»ΠΎΠΆΠ΅Π½Π½ΠΎΠ³ΠΎ напряТСния.

Β 


Π”Π΅ΠΌΠΎ Π·Π°ΠΊΠΎΠ½Π° Ома Π² Ρ€Π΅Π°Π»ΡŒΠ½ΠΎΠΌ Π²Ρ€Π΅ΠΌΠ΅Π½ΠΈ.

Для справки
Π’ Π΄Π°Π½Π½ΠΎΠΌ ΠΏΡ€ΠΈΠΌΠ΅Ρ€Π΅ Π²Ρ‹ ΠΌΠΎΠΆΠ΅Ρ‚Π΅ ΡƒΠ²Π΅Π»ΠΈΡ‡ΠΈΠ²Π°Ρ‚ΡŒ напряТСниС ΠΈ сопротивлСниС Ρ†Π΅ΠΏΠΈ. Π”Π°Π½Π½Ρ‹Π΅ измСнСния Π² Ρ€Π΅Π°Π»ΡŒΠ½ΠΎΠΌ Π²Ρ€Π΅ΠΌΠ΅Π½ΠΈ Π±ΡƒΠ΄ΡƒΡ‚ ΠΈΠ·ΠΌΠ΅Π½ΡΡ‚ΡŒ силу Ρ‚ΠΎΠΊΠ° ΠΏΡ€ΠΎΡ‚Π΅ΠΊΠ°ΡŽΡ‰Π΅Π³ΠΎ Π² Ρ†Π΅ΠΏΠΈ ΠΈ ΠΌΠΎΡ‰Π½ΠΎΡΡ‚ΡŒ Ρ€Π°ΡΡΠ΅ΠΈΠ²Π°Π΅ΠΌΡƒΡŽ Π½Π° сопротивлСнии.

Если Ρ€Π°ΡΡΠΌΠ°Ρ‚Ρ€ΠΈΠ²Π°Ρ‚ΡŒ Π°ΡƒΠ΄ΠΈΠΎ систСмы — Π²Ρ‹ Π΄ΠΎΠ»ΠΆΠ½Ρ‹ ΠΏΠΎΠΌΠ½ΠΈΡ‚ΡŒ Ρ‡Ρ‚ΠΎ ΡƒΡΠΈΠ»ΠΈΡ‚Π΅Π»ΡŒ Π²Ρ‹Π΄Π°Π΅Ρ‚ ΠΎΠΏΡ€Π΅Π΄Π΅Π»Π΅Π½Π½ΠΎΠ΅ напряТСниС Π½Π° ΠΎΠΏΡ€Π΅Π΄Π΅Π»Π΅Π½Π½ΡƒΡŽ Π½Π°Π³Ρ€ΡƒΠ·ΠΊΡƒ (сопротивлСниС). Π‘ΠΎΠΎΡ‚Π½ΠΎΡˆΠ΅Π½ΠΈΠ΅ Π΄Π²ΡƒΡ… этих Π²Π΅Π»ΠΈΡ‡ΠΈΠ½ опрСдСляСт ΠΌΠΎΡ‰Π½ΠΎΡΡ‚ΡŒ.

Π£ΡΠΈΠ»ΠΈΡ‚Π΅Π»ΡŒ ΠΌΠΎΠΆΠ΅Ρ‚ Π²Ρ‹Π΄Π°Ρ‚ΡŒ ΠΎΠ³Ρ€Π°Π½ΠΈΡ‡Π΅Π½Π½ΡƒΡŽ Π²Π΅Π»ΠΈΡ‡ΠΈΠ½Ρƒ напряТСния Π² зависимости ΠΎΡ‚ Π²Π½ΡƒΡ‚Ρ€Π΅Π½Π½Π΅Π³ΠΎ Π±Π»ΠΎΠΊΠ° питания ΠΈ источника Ρ‚ΠΎΠΊΠ°. Π’Π°ΠΊ ΠΆΠ΅ Ρ‚ΠΎΡ‡Π½ΠΎ ΠΎΠ³Ρ€Π°Π½ΠΈΡ‡Π΅Π½Π° ΠΈ ΠΌΠΎΡ‰Π½ΠΎΡΡ‚ΡŒ ΠΊΠΎΡ‚ΠΎΡ€ΡƒΡŽ ΠΌΠΎΠΆΠ΅Ρ‚ ΠΏΠΎΠ΄Π°Ρ‚ΡŒ ΡƒΡΠΈΠ»ΠΈΡ‚Π΅Π»ΡŒ Π½Π° ΠΎΠΏΡ€Π΅Π΄Π΅Π»Π΅Π½Π½ΡƒΡŽ Π½Π°Π³Ρ€ΡƒΠ·ΠΊΡƒ (ΠΊ ΠΏΡ€ΠΈΠΌΠ΅Ρ€Ρƒ 4 Ома).
Для Ρ‚ΠΎΠ³ΠΎ Ρ‡Ρ‚ΠΎ Π±Ρ‹ ΠΏΠΎΠ»ΡƒΡ‡ΠΈΡ‚ΡŒ большС мощности, Π²Ρ‹ ΠΌΠΎΠΆΠ΅Ρ‚Π΅ ΠΏΠΎΠ΄ΠΊΠ»ΡŽΡ‡ΠΈΡ‚ΡŒ ΠΊ ΡƒΡΠΈΠ»ΠΈΡ‚Π΅Π»ΡŽ Π½Π°Π³Ρ€ΡƒΠ·ΠΊΡƒ с мСньшим сопротивлСниСм (ΠΊ ΠΏΡ€ΠΈΠΌΠ΅Ρ€Ρƒ 2 Ома). Π£Ρ‡Ρ‚ΠΈΡ‚Π΅ Ρ‡Ρ‚ΠΎ ΠΏΡ€ΠΈ использовании Π½Π°Π³Ρ€ΡƒΠ·ΠΊΠΈ с мСньшим сопротивлСниСм — скаТСм Π² Π΄Π²Π° Ρ€Π°Π·Π° (Π±Ρ‹Π»ΠΎ 4 Ома, стало 2 Ома) — ΠΌΠΎΡ‰Π½ΠΎΡΡ‚ΡŒ Ρ‚ΠΎΠΆΠ΅ возрастСт Π² Π΄Π²Π° Ρ€Π°Π·Π°.(ΠΏΡ€ΠΈ условии Ρ‡Ρ‚ΠΎ Π΄Π°Π½Π½ΡƒΡŽ ΠΌΠΎΡ‰Π½ΠΎΡΡ‚ΡŒ ΠΌΠΎΠΆΠ΅Ρ‚ ΠΎΠ±Π΅ΡΠΏΠ΅Ρ‡ΠΈΡ‚ΡŒ Π²Π½ΡƒΡ‚Ρ€Π΅Π½Π½ΠΈΠΉ Π±Π»ΠΎΠΊ питания ΠΈ источник Ρ‚ΠΎΠΊΠ°).
Если ΠΌΡ‹ возьмСм для ΠΏΡ€ΠΈΠΌΠ΅Ρ€Π° ΠΌΠΎΠ½ΠΎ ΡƒΡΠΈΠ»ΠΈΡ‚Π΅Π»ΡŒ ΠΌΠΎΡ‰Π½ΠΎΡΡ‚ΡŒΡŽ 100 Π’Π°Ρ‚Ρ‚ Π½Π° Π½Π°Π³Ρ€ΡƒΠ·ΠΊΡƒ 4 Ома, зная Ρ‡Ρ‚ΠΎ ΠΎΠ½ ΠΌΠΎΠΆΠ΅Ρ‚ Π²Ρ‹Π΄Π°Ρ‚ΡŒ напряТСниС Π½Π΅ Π±ΠΎΠ»Π΅Π΅ 20 Π’ΠΎΠ»ΡŒΡ‚ Π½Π° Π½Π°Π³Ρ€ΡƒΠ·ΠΊΡƒ.
Если Π²Ρ‹ поставитС Π½Π° нашСм ΠΊΠ°Π»ΡŒΠΊΡƒΠ»ΡΡ‚ΠΎΡ€Π΅ Π±Π΅Π³ΡƒΠ½ΠΊΠΈ
НапряТСниС 20 Π’ΠΎΠ»ΡŒΡ‚
Π‘ΠΎΠΏΡ€ΠΎΡ‚ΠΈΠ²Π»Π΅Π½ΠΈΠ΅ 4 Ома
Π’Ρ‹ ΠΏΠΎΠ»ΡƒΡ‡ΠΈΡ‚Π΅
ΠœΠΎΡ‰Π½ΠΎΡΡ‚ΡŒ 100 Π’Π°Ρ‚Ρ‚ Β 
Β 
Если Π²Ρ‹ сдвинСтС Π±Π΅Π³ΡƒΠ½ΠΎΠΊ сопротивлСния Π½Π° Π²Π΅Π»ΠΈΡ‡ΠΈΠ½Ρƒ 2 Ома, Π²Ρ‹ ΡƒΠ²ΠΈΠ΄ΠΈΡ‚Π΅ ΠΊΠ°ΠΊ ΠΌΠΎΡ‰Π½ΠΎΡΡ‚ΡŒ удвоится ΠΈ составит 200 Π’Π°Ρ‚Ρ‚.

Π’ ΠΎΠ±Ρ‰Π΅ΠΌ ΠΏΡ€ΠΈΠΌΠ΅Ρ€Π΅ источником Ρ‚ΠΎΠΊΠ° являСтся аккумуляторная батарСя (Π° Π½Π΅ ΡƒΡΠΈΠ»ΠΈΡ‚Π΅Π»ΡŒ Π·Π²ΡƒΠΊΠ°) Π½ΠΎ зависимости силы Ρ‚ΠΎΠΊΠ°, напряТСния, сопротивлСния ΠΈ сопротивлСния ΠΎΠ΄ΠΈΠ½Π°ΠΊΠΎΠ²Ρ‹ Π²ΠΎ всСх цСпях.
Β 

Β 

ΠžΡΠ½ΠΎΠ²Π½Ρ‹Π΅ элСктротСхничСскиС Ρ„ΠΎΡ€ΠΌΡƒΠ»Ρ‹. ΠœΠΎΡ‰Π½ΠΎΡΡ‚ΡŒ. Π‘ΠΎΠΏΡ€ΠΎΡ‚ΠΈΠ²Π»Π΅Π½ΠΈΠ΅. Π’ΠΎΠΊ. НапряТСниС. Π—Π°ΠΊΠΎΠ½ Ома.

ЭлСктричСскоС напряТСниС:

  • U = R* I — Π—Π°ΠΊΠΎΠ½ Ома для участка Ρ†Π΅ΠΏΠΈ
  • U = P / I
  • U = (P*R)1/2

ЭлСктричСская ΠΌΠΎΡ‰Π½ΠΎΡΡ‚ΡŒ:

  • P= U* I
  • P= R* I
    2
  • P = U 2/ R

ЭлСктричСский Ρ‚ΠΎΠΊ:

  • I = U / R
  • I = P/ E
  • I = (P / R)1/2

ЭлСктричСскоС сопротивлСниС:

  • R = U / I
  • R = U 2/ P
  • R = P / I2

НЕ Π—ΠΠ‘Π«Π’ΠΠ•Πœ: Π—Π°ΠΊΠΎΠ½Ρ‹ ΠšΠΈΡ€Ρ…Π³ΠΎΡ„Π° ΠΎΠ½ΠΈ ΠΆΠ΅ ΠŸΡ€Π°Π²ΠΈΠ»Π° ΠšΠΈΡ€Ρ…Π³ΠΎΡ„Π° для Ρ‚ΠΎΠΊΠ° ΠΈ напряТСния.

ЦСпь ΠΏΠ΅Ρ€Π΅ΠΌΠ΅Π½Π½ΠΎΠ³ΠΎ ΡΠΈΠ½ΡƒΡΠΎΠΈΠ΄Π°Π»ΡŒΠ½ΠΎΠ³ΠΎ Ρ‚ΠΎΠΊΠ° c частотой Ο‰.

ΠŸΡ€ΠΈΠΌΠ΅Π½ΠΈΠΌΠΎΡΡ‚ΡŒ Ρ„ΠΎΡ€ΠΌΡƒΠ»: ΠΏΡ€Π΅Π½Π΅Π±Ρ€Π΅Π³Π°Π΅ΠΌ Π·Π°Π²ΠΈΡΠΈΠΌΠΎΡΡ‚ΡŒΡŽ сопротивлСний ΠΎΡ‚ силы Ρ‚ΠΎΠΊΠ° ΠΈ частоты.

Напомним, Ρ‡Ρ‚ΠΎ любой сигнал, ΠΌΠΎΠΆΠ΅Ρ‚ Π±Ρ‹Ρ‚ΡŒ с любой Ρ‚ΠΎΡ‡Π½ΠΎΡΡ‚ΡŒΡŽ Ρ€Π°Π·Π»ΠΎΠΆΠ΅Π½ Π² ряд Π€ΡƒΡ€ΡŒΠ΅, Ρ‚.Π΅. Π² ΠΏΡ€Π΅Π΄ΠΏΠΎΠ»ΠΎΠΆΠ΅Π½ΠΈΠΈ, Ρ‡Ρ‚ΠΎ ΠΏΠ°Ρ€Π°ΠΌΠ΅Ρ‚Ρ€Ρ‹ сСти
частотнонСзависимы — данная Ρ„ΠΎΡ€ΠΌΡƒΠ»ΠΈΡ€ΠΎΠ²ΠΊΠ° ΠΏΡ€ΠΈΠΌΠ΅Π½ΠΈΠΌΠ° ΠΊΠΎ всСм Π³Π°Ρ€ΠΌΠΎΠ½ΠΈΠΊΠ°ΠΌ любого сигнала.

Π—Π°ΠΊΠΎΠ½ Ома для Ρ†Π΅ΠΏΠ΅ΠΉ ΠΏΠ΅Ρ€Π΅ΠΌΠ΅Π½Π½ΠΎΠ³ΠΎ Ρ‚ΠΎΠΊΠ°

:

  • U = U0eiΟ‰tΒ  напряТСниС ΠΈΠ»ΠΈ Ρ€Π°Π·Π½ΠΎΡΡ‚ΡŒ ΠΏΠΎΡ‚Π΅Π½Ρ†ΠΈΠ°Π»ΠΎΠ²,
  • IΒ  сила Ρ‚ΠΎΠΊΠ°,
  • Z = Reβ€”iφ  комплСксноС сопротивлСниС (импСданс)
  • R = (Ra2+Rr2)1/2Β  ΠΏΠΎΠ»Π½ΠΎΠ΅ сопротивлСниС,
  • Rr = Ο‰LΒ β€” 1/Ο‰CΒ  Ρ€Π΅Π°ΠΊΡ‚ΠΈΠ²Π½ΠΎΠ΅ сопротивлСниС (Ρ€Π°Π·Π½ΠΎΡΡ‚ΡŒ ΠΈΠ½Π΄ΡƒΠΊΡ‚ΠΈΠ²Π½ΠΎΠ³ΠΎ ΠΈ Смкостного),
  • RΠ°Β  Π°ΠΊΡ‚ΠΈΠ²Π½ΠΎΠ΅ (омичСскоС) сопротивлСниС, Π½Π΅ зависящСС ΠΎΡ‚ частоты,
  • Ο† = arctg Rr/RaΒ β€” сдвиг Ρ„Π°Π· ΠΌΠ΅ΠΆΠ΄Ρƒ напряТСниСм ΠΈ Ρ‚ΠΎΠΊΠΎΠΌ.

Π‘ΠΈΠ»Π° токаНапряТСниСБопротивлСниСЗакон Ома

Π‘ΠΈΠ»Π° Ρ‚ΠΎΠΊΠ° НапряТСниС Π‘ΠΎΠΏΡ€ΠΎΡ‚ΠΈΠ²Π»Π΅Π½ΠΈΠ΅ Π—Π°ΠΊΠΎΠ½ Ома

Ѐизика, 8 класс

ΠšΡ€ΠΈΡΡ‚Π°Π»Π»ΠΈΡ‡Π΅ΡΠΊΠ°Ρ Ρ€Π΅ΡˆΠ΅Ρ‚ΠΊΠ° ΠΌΠ΅Ρ‚Π°Π»Π»Π°

Π’ ΡƒΠ·Π»Π°Ρ… кристалличСской Ρ€Π΅ΡˆΠ΅Ρ‚ΠΊΠΈ располоТСны Β«+Β» ΠΈΠΎΠ½Ρ‹, ΠΌΠ΅ΠΆΠ΄Ρƒ ΠΊΠΎΡ‚ΠΎΡ€Ρ‹ΠΌΠΈ Ρ…Π°ΠΎΡ‚ΠΈΡ‡Π½ΠΎ двиТутся  свободныС Β  элСктроны

ΠœΠ΅Ρ‚Π°Π»Π»Ρ‹ ΡΠ²Π»ΡΡŽΡ‚ΡΡ Ρ…ΠΎΡ€ΠΎΡˆΠΈΠΌΠΈ ΠΏΡ€ΠΎΠ²ΠΎΠ΄Π½ΠΈΠΊΠ°ΠΌΠΈ благодаря свободным заряТСнным частицам – элСктронам

ЭлСктричСский Ρ‚ΠΎΠΊ

ЭлСктричСский Ρ‚ΠΎΠΊ — упорядочСнноС (Π½Π°ΠΏΡ€Π°Π²Π»Π΅Π½Π½ΠΎΠ΅) Π΄Π²ΠΈΠΆΠ΅Π½ΠΈΠ΅ заряТСнных частиц

Условия возникновСния элСктричСского Ρ‚ΠΎΠΊΠ° Π² ΠΏΡ€ΠΎΠ²ΠΎΠ΄Π½ΠΈΠΊΠ΅ :

  • Π½Π°Π»ΠΈΡ‡ΠΈΠ΅ свободных заряТСнных частиц (элСктронов, ΠΈΠΎΠ½ΠΎΠ²)
  • элСктричСскоС ΠΏΠΎΠ»Π΅

НаправлСниС элСктричСского Ρ‚ΠΎΠΊΠ°: ΠΎΡ‚ + ΠΊ –

Π² ΠΌΠ΅Ρ‚Π°Π»Π»Π΅:

  • элСктроны двиТутся ΠΎΡ‚ – ΠΊ +
  • Ρ‚ΠΎΠΊ Π½Π°ΠΏΡ€Π°Π²Π»Π΅Π½ Π² сторону, ΠΏΡ€ΠΎΡ‚ΠΈΠ²ΠΎΠΏΠΎΠ»ΠΎΠΆΠ½ΡƒΡŽ Π½Π°ΠΏΡ€Π°Π²Π»Π΅Π½ΠΈΡŽ двиТСния элСктронов

Π‘ΠΈΠ»Π° Ρ‚ΠΎΠΊΠ°

Π‘ΠΈΠ»Π° Ρ‚ΠΎΠΊΠ° Β — физичСская Π²Π΅Π»ΠΈΡ‡ΠΈΠ½Π°, равная заряду, ΠΏΡ€ΠΎΡˆΠ΅Π΄ΡˆΠ΅ΠΌΡƒ Ρ‡Π΅Ρ€Π΅Π· ΠΏΠΎΠΏΠ΅Ρ€Π΅Ρ‡Π½ΠΎΠ΅ сСчСниС ΠΏΡ€ΠΎΠ²ΠΎΠ΄Π½ΠΈΠΊΠ° Π·Π° Π΅Π΄ΠΈΠ½ΠΈΡ†Ρƒ Π²Ρ€Π΅ΠΌΠ΅Π½ΠΈ

ΠžΠ±ΠΎΠ·Π½Π°Ρ‡Π΅Π½ΠΈΠ΅: I

Π•Π΄ΠΈΠ½ΠΈΡ†Π° измСрСния: 1А (АмпСр)

Π€ΠΎΡ€ΠΌΡƒΠ»Π°:

Π˜Π·ΠΌΠ΅Ρ€ΠΈΡ‚Π΅Π»ΡŒΠ½Ρ‹ΠΉ ΠΏΡ€ΠΈΠ±ΠΎΡ€: Π°ΠΌΠΏΠ΅Ρ€ΠΌΠ΅Ρ‚Ρ€

АмпСрмСтр

А

АмпСрмСтр Π²ΠΊΠ»ΡŽΡ‡Π°Π΅Ρ‚ΡΡ ΠΏ ΠΎ с Π» Π΅ Π΄ ΠΎ Π² Π° Ρ‚ Π΅ Π» ь Π½ ΠΎ

А

ΠŸΡ€ΠΈ Π²ΠΊΠ»ΡŽΡ‡Π΅Π½ΠΈΠΈ Π°ΠΌΠΏΠ΅Ρ€ΠΌΠ΅Ρ‚Ρ€Π° Π² Ρ†Π΅ΠΏΡŒ Π½Π΅ ΠΈΠΌΠ΅Π΅Ρ‚ значСния, с ΠΊΠ°ΠΊΠΎΠΉ стороны (слСва ΠΈΠ»ΠΈ справа) ΠΎΡ‚ исслСдуСмого элСмСнта Π΅Π³ΠΎ ΠΏΠΎΠ΄ΠΊΠ»ΡŽΡ‡Π°Ρ‚ΡŒ.Β 

АмпСрмСтр Π»Π°Π±ΠΎΡ€Π°Ρ‚ΠΎΡ€Π½Ρ‹ΠΉ

Π¨ΠΊΠ°Π»Π° Π°ΠΌΠΏΠ΅Ρ€ΠΌΠ΅Ρ‚Ρ€Π°

Π¦Π΅Π½Π° дСлСния ΠΈ ΠΏΡ€Π΅Π΄Π΅Π»Ρ‹ измСрСния ΠΏΡ€ΠΈΠ±ΠΎΡ€Π°

ЦСна дСлСния:

ΠŸΡ€Π΅Π΄Π΅Π»Ρ‹ измСрСния:

НапряТСниС

НапряТСниС  – скалярная физичСская Π²Π΅Π»ΠΈΡ‡ΠΈΠ½Π°, равная Ρ€Π°Π±ΠΎΡ‚Π΅ элСктричСского поля ΠΏΠΎ ΠΏΠ΅Ρ€Π΅ΠΌΠ΅Ρ‰Π΅Π½ΠΈΡŽ Π΅Π΄ΠΈΠ½ΠΈΡ‡Π½ΠΎΠ³ΠΎ ΠΏΠΎΠ»ΠΎΠΆΠΈΡ‚Π΅Π»ΡŒΠ½ΠΎΠ³ΠΎ заряда

ΠžΠ±ΠΎΠ·Π½Π°Ρ‡Π΅Π½ΠΈΠ΅: U

Π•Π΄ΠΈΠ½ΠΈΡ†Π° измСрСния Π² БИ: 1Π’ (Π²ΠΎΠ»ΡŒΡ‚)

Π€ΠΎΡ€ΠΌΡƒΠ»Π°:

Π˜Π·ΠΌΠ΅Ρ€ΠΈΡ‚Π΅Π»ΡŒΠ½Ρ‹ΠΉ ΠΏΡ€ΠΈΠ±ΠΎΡ€: Π²ΠΎΠ»ΡŒΡ‚ΠΌΠ΅Ρ‚Ρ€

Π’ΠΎΠ»ΡŒΡ‚ΠΌΠ΅Ρ‚Ρ€

V

Π’ΠΎΠ»ΡŒΡ‚ΠΌΠ΅Ρ‚Ρ€ Π²ΠΊΠ»ΡŽΡ‡Π°Π΅Ρ‚ΡΡ ΠΏ Π° Ρ€ Π° Π» Π» Π΅ Π» ь Π½ ΠΎ

V

Π’ΠΎΠ»ΡŒΡ‚ΠΌΠ΅Ρ‚Ρ€ Π»Π°Π±ΠΎΡ€Π°Ρ‚ΠΎΡ€Π½Ρ‹ΠΉ

Π’ΠΎΠ»ΡŒΡ‚ΠΌΠ΅Ρ‚Ρ€ Π‘Π‘Π‘Π , 1940 Π³ΠΎΠ΄

Π¨ΠΊΠ°Π»Π° Π²ΠΎΠ»ΡŒΡ‚ΠΌΠ΅Ρ‚Ρ€Π°

Π¦Π΅Π½Π° дСлСния ΠΈ ΠΏΡ€Π΅Π΄Π΅Π»Ρ‹ измСрСния ΠΏΡ€ΠΈΠ±ΠΎΡ€Π°

ЦСна дСлСния:

ΠŸΡ€Π΅Π΄Π΅Π»Ρ‹ измСрСния:

Π’ΠΎΠΊ Ρ‡Π΅Ρ€Π΅Π· Π»Π°ΠΌΠΏΠΎΡ‡ΠΊΡƒ ΠΈ напряТСниС Π½Π° Π½Π΅ΠΉ

А

V

Π‘ΠΎΠΏΡ€ΠΎΡ‚ΠΈΠ²Π»Π΅Π½ΠΈΠ΅

Π‘ΠΎΠΏΡ€ΠΎΡ‚ΠΈΠ²Π»Π΅Π½ΠΈΠ΅ – скалярная физичСская Π²Π΅Π»ΠΈΡ‡ΠΈΠ½Π°, Ρ…Π°Ρ€Π°ΠΊΡ‚Π΅Ρ€ΠΈΠ·ΡƒΡŽΡ‰Π°Ρ свойство ΠΏΡ€ΠΎΠ²ΠΎΠ΄Π½ΠΈΠΊΠ° ΠΏΡ€ΠΎΡ‚ΠΈΠ²ΠΎΠ΄Π΅ΠΉΡΡ‚Π²ΠΎΠ²Π°Ρ‚ΡŒ элСктричСскому Ρ‚ΠΎΠΊΡƒ

ΠžΠ±ΠΎΠ·Π½Π°Ρ‡Π΅Π½ΠΈΠ΅: R

Π•Π΄ΠΈΠ½ΠΈΡ†Π° измСрСния: 1 Ом (Ом)

Π˜Π·ΠΌΠ΅Ρ€ΠΈΡ‚Π΅Π»ΡŒΠ½Ρ‹ΠΉ ΠΏΡ€ΠΈΠ±ΠΎΡ€: ΠžΠΌΠΌΠ΅Ρ‚Ρ€

ΠŸΡ€ΠΈΡ‡ΠΈΠ½Π° элСктричСского сопротивлСния:

взаимодСйствиС элСктронов ΠΏΡ€ΠΈ ΠΈΡ… Π΄Π²ΠΈΠΆΠ΅Π½ΠΈΠΈ ΠΏΠΎ ΠΏΡ€ΠΎΠ²ΠΎΠ΄Π½ΠΈΠΊΡƒ с ΠΈΠΎΠ½Π°ΠΌΠΈ кристалличСской Ρ€Π΅ΡˆΠ΅Ρ‚ΠΊΠΈ.

+

+

+

+

+

+

+

+

НаправлСнному двиТСнию элСктронов ΠΌΠ΅ΡˆΠ°ΡŽΡ‚ ΠΈΡ… столкновСния с ΠΊΠΎΠ»Π΅Π±Π»ΡŽΡ‰ΠΈΠΌΠΈΡΡ тяТСлыми ΠΈ большими ΠΈΠΎΠ½Π°ΠΌΠΈ кристалличСской Ρ€Π΅ΡˆΠ΅Ρ‚ΠΊΠΈ. Π­Ρ‚ΠΎ ΠΈ создаСт сопротивлСниС двиТСнию элСктронов β€” Π²Ρ‹Π·Ρ‹Π²Π°Π΅Ρ‚ элСктричСскоС сопротивлСниС ΠΌΠ΅Ρ‚Π°Π»Π»Π°.

ЭлСктричСскоС сопротивлСниС ΠΌΠ΅Ρ‚Π°Π»Π»ΠΎΠ² прямо ΠΏΡ€ΠΎΠΏΠΎΡ€Ρ†ΠΈΠΎΠ½Π°Π»ΡŒΠ½ΠΎ Π΄Π»ΠΈΠ½Π΅ ΠΏΡ€ΠΎΠ²ΠΎΠ΄Π½ΠΈΠΊΠ° ΠΈ ΠΎΠ±Ρ€Π°Ρ‚Π½ΠΎ ΠΏΡ€ΠΎΠΏΠΎΡ€Ρ†ΠΈΠΎΠ½Π°Π»ΡŒΠ½ΠΎ ΠΏΠ»ΠΎΡ‰Π°Π΄ΠΈ Π΅Π³ΠΎ ΠΏΠΎΠΏΠ΅Ρ€Π΅Ρ‡Π½ΠΎΠ³ΠΎ сСчСния:

 – ΡƒΠ΄Π΅Π»ΡŒΠ½ΠΎΠ΅ сопротивлСниС

l – Π΄Π»ΠΈΠ½Π° ΠΏΡ€ΠΎΠ²ΠΎΠ΄Π½ΠΈΠΊΠ°

S – ΠΏΠ»ΠΎΡ‰Π°Π΄ΡŒ ΠΏΠΎΠΏΠ΅Ρ€Π΅Ρ‡Π½ΠΎΠ³ΠΎ сСчСния ΠΏΡ€ΠΎΠ²ΠΎΠ΄Π½ΠΈΠΊΠ°

УдСльноС сопротивлСниС – скалярная физичСская Π²Π΅Π»ΠΈΡ‡ΠΈΠ½Π°, числСнно равная ΡΠΎΠΏΡ€ΠΎΡ‚ΠΈΠ²Π»Π΅Π½ΠΈΡŽ цилиндричСского ΠΏΡ€ΠΎΠ²ΠΎΠ΄Π½ΠΈΠΊΠ° Π΅Π΄ΠΈΠ½ΠΈΡ‡Π½ΠΎΠΉ Π΄Π»ΠΈΠ½Ρ‹ ΠΈ Π΅Π΄ΠΈΠ½ΠΈΡ‡Π½ΠΎΠΉ ΠΏΠ»ΠΎΡ‰Π°Π΄ΠΈ ΠΏΠΎΠΏΠ΅Ρ€Π΅Ρ‡Π½ΠΎΠ³ΠΎ сСчСния

Зависит ΠΎΡ‚ вСщСства ΠΈ Π΅Π³ΠΎ состояния (Ρ‚Π΅ΠΌΠΏΠ΅Ρ€Π°Ρ‚ΡƒΡ€Ρ‹)

Π•Π΄ΠΈΠ½ΠΈΡ†Π° измСрСния: 1 Ом οƒ— ΠΌ

РСзистор – устройство с постоянным сопротивлСниСм.

РСостат – устройство с ΠΏΠ΅Ρ€Π΅ΠΌΠ΅Π½Π½Ρ‹ΠΌ сопротивлСниСм, ΠΏΡ€Π΅Π΄Π½Π°Π·Π½Π°Ρ‡Π΅Π½Π½ΠΎΠ΅ для рСгулирования силы Ρ‚ΠΎΠΊΠ° ΠΈ напряТСния Π² элСктричСской Ρ†Π΅ΠΏΠΈ.

Π€ΠΈΠ·ΠΈΠΊΠ°, 7-11 ΠΊΠ». Π‘ΠΈΠ±Π»ΠΈΠΎΡ‚Π΅ΠΊΠ° наглядных пособий (1Π‘)

Π—Π°Π²ΠΈΡΠΈΠΌΠΎΡΡ‚ΡŒ силы Ρ‚ΠΎΠΊΠ°

ΠΎΡ‚ напряТСния ΠΈ сопротивлСния

V

А

R

Π Π΅Π·ΡƒΠ»ΡŒΡ‚Π°Ρ‚Ρ‹

I , А

R 1

Π’Π°Π±Π»ΠΈΡ†Π° 1

R =

R 2

β„–

1

U , Π’

2

I , А

3

R 3

U , Π’

R 1

Π—Π°ΠΊΠΎΠ½ Ома

Π‘ΠΈΠ»Π° Ρ‚ΠΎΠΊΠ° Π½Π° участкС Ρ†Π΅ΠΏΠΈ прямо ΠΏΡ€ΠΎΠΏΠΎΡ€Ρ†ΠΈΠΎΠ½Π°Π»ΡŒΠ½Π° Π½Π°ΠΏΡ€ΡΠΆΠ΅Π½ΠΈΡŽ Π½Π° Π΅Π³ΠΎ ΠΊΠΎΠ½Ρ†Π°Ρ… ΠΈ ΠΎΠ±Ρ€Π°Ρ‚Π½ΠΎ ΠΏΡ€ΠΎΠΏΠΎΡ€Ρ†ΠΈΠΎΠ½Π°Π»ΡŒΠ½Π° Π΅Π³ΠΎ ΡΠΎΠΏΡ€ΠΎΡ‚ΠΈΠ²Π»Π΅Π½ΠΈΡŽ

Π—Π°ΠΊΠΎΠ½Β ΠžΠΌΠ°Β Π½Π°Π³Π»ΡΠ΄Π½ΠΎ

АмпСр АндрС ΠœΠ°Ρ€ΠΈ

1775-1836

Π“Π΅ΠΎΡ€Π³ Ом

1787 — 1854

АлСсандро Π’ΠΎΠ»ΡŒΡ‚Π°

1745 — 1827

Вопросы для самоконтроля

  • Π”Π°ΠΉΡ‚Π΅ ΠΎΠΏΡ€Π΅Π΄Π΅Π»Π΅Π½ΠΈΠ΅ элСктричСского Ρ‚ΠΎΠΊΠ°.
  • ΠŸΡ€ΠΈ ΠΊΠ°ΠΊΠΈΡ… условиях Π²ΠΎΠ·Π½ΠΈΠΊΠ°Π΅Ρ‚ элСктричСский Ρ‚ΠΎΠΊ?
  • Π§Π΅ΠΌ отличаСтся Π΄Π²ΠΈΠΆΠ΅Π½ΠΈΠ΅ заряТСнных частиц Π² ΠΏΡ€ΠΎΠ²ΠΎΠ΄Π½ΠΈΠΊΠ΅ Π² отсутствиС ΠΈ ΠΏΡ€ΠΈ Π½Π°Π»ΠΈΡ‡ΠΈΠΈ внСшнСго элСктричСского поля?
  • Как Π½Π°ΠΏΡ€Π°Π²Π»Π΅Π½ элСктричСский Ρ‚ΠΎΠΊ?
  • Π’ ΠΊΠ°ΠΊΠΎΠΌ Π½Π°ΠΏΡ€Π°Π²Π»Π΅Π½ΠΈΠΈ двиТутся элСктроны Π² мСталличСском ΠΏΡ€ΠΎΠ²ΠΎΠ΄Π½ΠΈΠΊΠ΅, ΠΏΠΎ ΠΊΠΎΡ‚ΠΎΡ€ΠΎΠΌΡƒ ΠΏΡ€ΠΎΡ‚Π΅ΠΊΠ°Π΅Ρ‚ элСктричСский Ρ‚ΠΎΠΊ?
  • Π§Ρ‚ΠΎ Π½Π°Π·Ρ‹Π²Π°ΡŽΡ‚ силой Ρ‚ΠΎΠΊΠ°?
  • Какова Π΅Π΄ΠΈΠ½ΠΈΡ†Π° измСрСния силы Ρ‚ΠΎΠΊΠ°?
  • Каким ΠΏΡ€ΠΈΠ±ΠΎΡ€ΠΎΠΌ ΠΈΠ·ΠΌΠ΅Ρ€ΡΡŽΡ‚ силу Ρ‚ΠΎΠΊΠ°? Как ΠΎΠ½ ΠΏΠΎΠ΄ΠΊΠ»ΡŽΡ‡Π°Π΅Ρ‚ΡΡ?
  • Π§Ρ‚ΠΎ Ρ‚Π°ΠΊΠΎΠ΅ напряТСниС?
  • Какова Π΅Π΄ΠΈΠ½ΠΈΡ†Π° напряТСния?
  • Каким ΠΏΡ€ΠΈΠ±ΠΎΡ€ΠΎΠΌ ΠΈΠ·ΠΌΠ΅Ρ€ΡΡŽΡ‚ напряТСниС? Как ΠΎΠ½ ΠΏΠΎΠ΄ΠΊΠ»ΡŽΡ‡Π°Π΅Ρ‚ΡΡ?
  • Π§Ρ‚ΠΎ Ρ‚Π°ΠΊΠΎΠ΅ сопротивлСниС? Какова ΠΏΡ€ΠΈΡ‡ΠΈΠ½Π° сопротивлСния?
  • Какова Π΅Π΄ΠΈΠ½ΠΈΡ†Π° сопротивлСния?
  • Каким ΠΏΡ€ΠΈΠ±ΠΎΡ€ΠΎΠΌ ΠΈΠ·ΠΌΠ΅Ρ€ΡΡŽΡ‚ сопротивлСниС? Как ΠΎΠ½ ΠΏΠΎΠ΄ΠΊΠ»ΡŽΡ‡Π°Π΅Ρ‚ΡΡ?
  • Π‘Ρ„ΠΎΡ€ΠΌΡƒΠ»ΠΈΡ€ΡƒΠΉΡ‚Π΅ Π·Π°ΠΊΠΎΠ½ Ома для участка Ρ†Π΅ΠΏΠΈ.

Онлайн ΠΊΠ°Π»ΡŒΠΊΡƒΠ»ΡΡ‚ΠΎΡ€ — Π·Π°ΠΊΠΎΠ½ Ома (Ρ‚ΠΎΠΊ, напряТСниС, сопротивлСниС) + ΠœΠΎΡ‰Π½ΠΎΡΡ‚ΡŒ :: ΠΠ²Ρ‚ΠΎΠœΠΎΡ‚ΠΎΠ“Π°Ρ€Π°ΠΆ

ΠŸΡ€ΠΈΡ‡ΠΈΠ½ΠΎΠΉ написания Π΄Π°Π½Π½ΠΎΠΉ ΡΡ‚Π°Ρ‚ΡŒΠΈ явилась Π½Π΅ ΡΠ»ΠΎΠΆΠ½ΠΎΡΡ‚ΡŒ этих Ρ„ΠΎΡ€ΠΌΡƒΠ», Π° Ρ‚ΠΎ, Ρ‡Ρ‚ΠΎ Π² Ρ…ΠΎΠ΄Π΅ проСктирования ΠΈ Ρ€Π°Π·Ρ€Π°Π±ΠΎΡ‚ΠΊΠΈ ΠΊΠ°ΠΊΠΈΡ…-Π»ΠΈΠ±ΠΎ схСм часто приходится ΠΏΠ΅Ρ€Π΅Π±ΠΈΡ€Π°Ρ‚ΡŒ ряд Π·Π½Π°Ρ‡Π΅Π½ΠΈΠΉ Ρ‡Ρ‚ΠΎΠ±Ρ‹ Π²Ρ‹ΠΉΡ‚ΠΈ Π½Π° Ρ‚Ρ€Π΅Π±ΡƒΠ΅ΠΌΡ‹Π΅ ΠΏΠ°Ρ€Π°ΠΌΠ΅Ρ‚Ρ€Ρ‹ ΠΈΠ»ΠΈ ΡΠ±Π°Π»Π°Π½ΡΠΈΡ€ΠΎΠ²Π°Ρ‚ΡŒ схСму. Данная ΡΡ‚Π°Ρ‚ΡŒΡ ΠΈ ΠΊΠ°Π»ΡŒΠΊΡƒΠ»ΡΡ‚ΠΎΡ€ Π² Π½Π΅ΠΉ ΠΏΠΎΠ·Π²ΠΎΠ»ΠΈΡ‚ ΡƒΠΏΡ€ΠΎΡΡ‚ΠΈΡ‚ΡŒ этот ΠΏΠΎΠ΄Π±ΠΎΡ€ ΠΈ ΡƒΡΠΊΠΎΡ€ΠΈΡ‚ΡŒ процСсс Ρ€Π΅Π°Π»ΠΈΠ·Π°Ρ†ΠΈΠΈ Π·Π°Π΄ΡƒΠΌΠ°Π½Π½ΠΎΠ³ΠΎ. Π’Π°ΠΊΠΆΠ΅ Π² ΠΊΠΎΠ½Ρ†Π΅ ΡΡ‚Π°Ρ‚ΡŒΠΈ ΠΏΡ€ΠΈΠ²Π΅Π΄Ρƒ нСсколько ΠΌΠ΅Ρ‚ΠΎΠ΄ΠΈΠΊ для запоминания основной Ρ„ΠΎΡ€ΠΌΡƒΠ»Ρ‹ Π·Π°ΠΊΠΎΠ½Π° Ома. Π­Ρ‚Π° информация Π±ΡƒΠ΄Π΅Ρ‚ ΠΏΠΎΠ»Π΅Π·Π½Π° Π½Π°Ρ‡ΠΈΠ½Π°ΡŽΡ‰ΠΈΠΌ. Π€ΠΎΡ€ΠΌΡƒΠ»Π° Ρ…ΠΎΡ‚ΡŒ ΠΈ простая, Π½ΠΎ ΠΈΠ½ΠΎΠ³Π΄Π° Π΅ΡΡ‚ΡŒ Π·Π°ΠΌΠ΅ΡˆΠ°Ρ‚Π΅Π»ΡŒΡΡ‚Π²ΠΎ, Π³Π΄Π΅ ΠΈ ΠΊΠ°ΠΊΠΎΠΉ ΠΏΠ°Ρ€Π°ΠΌΠ΅Ρ‚Ρ€ Π΄ΠΎΠ»ΠΆΠ΅Π½ ΡΡ‚ΠΎΡΡ‚ΡŒ, особСнно это Π±Ρ‹Π²Π°Π΅Ρ‚ ΠΏΠΎΠ½Π°Ρ‡Π°Π»Ρƒ.

Π’ радиоэлСктроникС ΠΈ элСктротСхникС Π·Π°ΠΊΠΎΠ½ Ома ΠΈ Ρ„ΠΎΡ€ΠΌΡƒΠ»Π° расчёта мощности ΠΈΡΠΏΠΎΠ»ΡŒΠ·ΡƒΡŽΡ‚ΡΡ Ρ‡Π°ΡˆΠ΅ Ρ‡Π΅ΠΌ ΠΊΠ°ΠΊΠΈΠ΅-Π»ΠΈΠ±ΠΎ ΠΈΠ· всСх ΠΎΡΡ‚Π°Π»ΡŒΠ½Ρ‹Ρ… Ρ„ΠΎΡ€ΠΌΡƒΠ». Они ΠΎΠΏΡ€Π΅Π΄Π΅Π»ΡΡŽΡ‚ ΠΆΠ΅ΡΡ‚ΠΊΡƒΡŽ взаимосвязь ΠΌΠ΅ΠΆΠ΄Ρƒ Ρ‡Π΅Ρ‚Ρ‹Ρ€ΡŒΠΌΡ самыми Ρ…ΠΎΠ΄ΠΎΠ²Ρ‹ΠΌΠΈ элСктричСскими Π²Π΅Π»ΠΈΡ‡ΠΈΠ½Π°ΠΌΠΈ: Ρ‚ΠΎΠΊΠΎΠΌ, напряТСниСм, сопротивлСниСм ΠΈ ΠΌΠΎΡ‰Π½ΠΎΡΡ‚ΡŒΡŽ.

Π—Π°ΠΊΠΎΠ½ Ома. Π­Ρ‚Ρƒ взаимосвязь выявил ΠΈ Π΄ΠΎΠΊΠ°Π·Π°Π» Π“Π΅ΠΎΡ€Π³ Π‘ΠΈΠΌΠΎΠ½ Ом Π² 1826 Π³ΠΎΠ΄Ρƒ. Для участка Ρ†Π΅ΠΏΠΈ ΠΎΠ½Π° Π·Π²ΡƒΡ‡ΠΈΡ‚ Ρ‚Π°ΠΊ: сила Ρ‚ΠΎΠΊΠ° прямо ΠΏΡ€ΠΎΠΏΠΎΡ€Ρ†ΠΈΠΎΠ½Π°Π»ΡŒΠ½Π° Π½Π°ΠΏΡ€ΡΠΆΠ΅Π½ΠΈΡŽ, ΠΈ ΠΎΠ±Ρ€Π°Ρ‚Π½ΠΎ ΠΏΡ€ΠΎΠΏΠΎΡ€Ρ†ΠΈΠΎΠ½Π°Π»ΡŒΠ½Π° ΡΠΎΠΏΡ€ΠΎΡ‚ΠΈΠ²Π»Π΅Π½ΠΈΡŽ

Π’Π°ΠΊ записываСтся основная Ρ„ΠΎΡ€ΠΌΡƒΠ»Π°:

ΠŸΡƒΡ‚Π΅ΠΌ прСобразования основной Ρ„ΠΎΡ€ΠΌΡƒΠ»Ρ‹ ΠΌΠΎΠΆΠ½ΠΎ Π½Π°ΠΉΡ‚ΠΈ ΠΈ Π΄Ρ€ΡƒΠ³ΠΈΠ΅ Π΄Π²Π΅ Π²Π΅Π»ΠΈΡ‡ΠΈΠ½Ρ‹:

Β  Β  Β Β 

ΠœΠΎΡ‰Π½ΠΎΡΡ‚ΡŒ. Π•Ρ‘ ΠΎΠΏΡ€Π΅Π΄Π΅Π»Π΅Π½ΠΈΠ΅ Π·Π²ΡƒΡ‡ΠΈΡ‚ Ρ‚Π°ΠΊ: ΠΌΠΎΡ‰Π½ΠΎΡΡ‚ΡŒΡŽ называСтся ΠΏΡ€ΠΎΠΈΠ·Π²Π΅Π΄Π΅Π½ΠΈΠ΅ ΠΌΠ³Π½ΠΎΠ²Π΅Π½Π½Ρ‹Ρ… Π·Π½Π°Ρ‡Π΅Π½ΠΈΠΉ напряТСния ΠΈ силы Ρ‚ΠΎΠΊΠ° Π½Π° ΠΊΠ°ΠΊΠΎΠΌ-Π»ΠΈΠ±ΠΎ участкС элСктричСской Ρ†Π΅ΠΏΠΈ.

Π€ΠΎΡ€ΠΌΡƒΠ»Π° ΠΌΠ³Π½ΠΎΠ²Π΅Π½Π½ΠΎΠΉ элСктричСской мощности:

НиТС ΠΏΡ€ΠΈΠ²Π΅Π΄Ρ‘Π½ ΠΎΠ½Π»Π°ΠΉΠ½ ΠΊΠ°Π»ΡŒΠΊΡƒΠ»ΡΡ‚ΠΎΡ€ для расчёта Π·Π°ΠΊΠΎΠ½Π° Ома ΠΈ ΠœΠΎΡ‰Π½ΠΎΡΡ‚ΠΈ. Π”Π°Π½Π½Ρ‹ΠΉ ΠΊΠ°Π»ΡŒΠΊΡƒΠ»ΡΡ‚ΠΎΡ€ позволяСт ΠΎΠΏΡ€Π΅Π΄Π΅Π»ΠΈΡ‚ΡŒ взаимосвязь ΠΌΠ΅ΠΆΠ΄Ρƒ Ρ‡Π΅Ρ‚Ρ‹Ρ€ΡŒΠΌΡ элСктричСскими Π²Π΅Π»ΠΈΡ‡ΠΈΠ½Π°ΠΌΠΈ: Ρ‚ΠΎΠΊΠΎΠΌ, напряТСниСм, сопротивлСниСм ΠΈ ΠΌΠΎΡ‰Π½ΠΎΡΡ‚ΡŒΡŽ. Для этого достаточно ввСсти Π»ΡŽΠ±Ρ‹Π΅ Π΄Π²Π΅ Π²Π΅Π»ΠΈΡ‡ΠΈΠ½Ρ‹. Π‘Ρ‚Ρ€Π΅Π»ΠΊΠ°ΠΌΠΈ Β«Π²Π²Π΅Ρ€Ρ…-Π²Π½ΠΈΠ·Β» ΠΌΠΎΠΆΠ½ΠΎ с шагом Π² Π΅Π΄ΠΈΠ½ΠΈΡ†Ρƒ ΠΌΠ΅Π½ΡΡ‚ΡŒ Π²Π²Π΅Π΄Ρ‘Π½Π½ΠΎΠ΅ Π·Π½Π°Ρ‡Π΅Π½ΠΈΠ΅. Π Π°Π·ΠΌΠ΅Ρ€Π½ΠΎΡΡ‚ΡŒ Π²Π΅Π»ΠΈΡ‡ΠΈΠ½ Ρ‚ΠΎΠΆΠ΅ ΠΌΠΎΠΆΠ½ΠΎ Π²Ρ‹Π±Ρ€Π°Ρ‚ΡŒ. Π’Π°ΠΊΠΆΠ΅ для удобства ΠΏΠΎΠ΄Π±ΠΎΡ€Π° ΠΏΠ°Ρ€Π°ΠΌΠ΅Ρ‚Ρ€ΠΎΠ², ΠΊΠ°Π»ΡŒΠΊΡƒΠ»ΡΡ‚ΠΎΡ€ позволяСт Ρ„ΠΈΠΊΡΠΈΡ€ΠΎΠ²Π°Ρ‚ΡŒ Π΄ΠΎ дСсяти Ρ€Π°Π½Π΅Π΅ Π²Ρ‹ΠΏΠΎΠ»Π½Π΅Π½Π½Ρ‹Ρ… расчётов с Ρ‚Π΅ΠΌΠΈ размСрностями с ΠΊΠΎΡ‚ΠΎΡ€Ρ‹ΠΌΠΈ Π²Ρ‹ΠΏΠΎΠ»Π½ΡΠ»ΠΈΡΡŒ сами расчёты.

Β 

Β 

Когда ΠΌΡ‹ ΡƒΡ‡ΠΈΠ»ΠΈΡΡŒ Π² радиотСхничСском Ρ‚Π΅Ρ…Π½ΠΈΠΊΡƒΠΌΠ΅, Ρ‚ΠΎ ΠΏΡ€ΠΈΡ…ΠΎΠ΄ΠΈΠ»ΠΎΡΡŒ Π·Π°ΠΏΠΎΠΌΠΈΠ½Π°Ρ‚ΡŒ ΠΎΡ‡Π΅Π½ΡŒ ΠΌΠ½ΠΎΠ³ΠΎ всякой всячины. И Ρ‡Ρ‚ΠΎΠ±Ρ‹ ΠΏΡ€ΠΎΡ‰Π΅ Π±Ρ‹Π»ΠΎ Π·Π°ΠΏΠΎΠΌΠ½ΠΈΡ‚ΡŒ, для Π·Π°ΠΊΠΎΠ½Π° Ома Π΅ΡΡ‚ΡŒ Ρ‚Ρ€ΠΈ ΡˆΠΏΠ°Ρ€Π³Π°Π»ΠΊΠΈ. Π’ΠΎΡ‚ ΠΊΠ°ΠΊΠΈΠΌΠΈ ΠΌΠ΅Ρ‚ΠΎΠ΄ΠΈΠΊΠ°ΠΌΠΈ ΠΌΡ‹ пользовались.

Β 

ΠŸΠ΅Ρ€Π²Π°Ρ — мнСмоничСскоС ΠΏΡ€Π°Π²ΠΈΠ»ΠΎ. Если ΠΈΠ· Ρ„ΠΎΡ€ΠΌΡƒΠ»Ρ‹ Π·Π°ΠΊΠΎΠ½Π° Ома Π²Ρ‹Ρ€Π°Π·ΠΈΡ‚ΡŒ сопротивлСниС, Ρ‚ΠΎ R = Ρ€ΡŽΠΌΠΊΠ°.

Вторая — ΠΌΠ΅Ρ‚ΠΎΠ΄ Ρ‚Ρ€Π΅ΡƒΠ³ΠΎΠ»ΡŒΠ½ΠΈΠΊΠ°. Π•Π³ΠΎ Π΅Ρ‰Ρ‘ Π½Π°Π·Ρ‹Π²Π°ΡŽΡ‚ магичСский Ρ‚Ρ€Π΅ΡƒΠ³ΠΎΠ»ΡŒΠ½ΠΈΠΊ Π·Π°ΠΊΠΎΠ½Π° Ома.

Если ΠΎΡ‚ΠΎΡ€Π²Π°Ρ‚ΡŒ Π²Π΅Π»ΠΈΡ‡ΠΈΠ½Ρƒ, ΠΊΠΎΡ‚ΠΎΡ€ΡƒΡŽ трСбуСтся Π½Π°ΠΉΡ‚ΠΈ, Ρ‚ΠΎ Π² ΠΎΡΡ‚Π°Π²ΡˆΠ΅ΠΉΡΡ части ΠΌΡ‹ ΠΏΠΎΠ»ΡƒΡ‡ΠΈΠΌ Ρ„ΠΎΡ€ΠΌΡƒΠ»Ρƒ для Π΅Ρ‘ нахоТдСния.

Π’Ρ€Π΅Ρ‚ΡŒΡ. Она большС являСтся ΡˆΠΏΠ°Ρ€Π³Π°Π»ΠΊΠΎΠΉ, Π² ΠΊΠΎΡ‚ΠΎΡ€ΠΎΠΉ ΠΎΠ±ΡŠΠ΅Π΄ΠΈΠ½Π΅Π½Ρ‹ всС основныС Ρ„ΠΎΡ€ΠΌΡƒΠ»Ρ‹ для Ρ‡Π΅Ρ‚Ρ‹Ρ€Ρ‘Ρ… элСктричСских Π²Π΅Π»ΠΈΡ‡ΠΈΠ½.

ΠŸΠΎΠ»ΡŒΠ·ΠΎΠ²Π°Ρ‚ΡŒΡΡ Сю Ρ‚Π°ΠΊΠΆΠ΅ просто, ΠΊΠ°ΠΊ ΠΈ Ρ‚Ρ€Π΅ΡƒΠ³ΠΎΠ»ΡŒΠ½ΠΈΠΊΠΎΠΌ. Π’Ρ‹Π±ΠΈΡ€Π°Π΅ΠΌ Ρ‚ΠΎΡ‚ ΠΏΠ°Ρ€Π°ΠΌΠ΅Ρ‚Ρ€, ΠΊΠΎΡ‚ΠΎΡ€Ρ‹ΠΉ Ρ…ΠΎΡ‚ΠΈΠΌ Ρ€Π°ΡΡΡ‡ΠΈΡ‚Π°Ρ‚ΡŒ, ΠΎΠ½ Π½Π°Ρ…ΠΎΠ΄ΠΈΡ‚ΡŒΡΡ Π² ΠΌΠ°Π»ΠΎΠΌ ΠΊΡ€ΡƒΠ³Ρƒ Π² Ρ†Π΅Π½Ρ‚Ρ€Π΅ ΠΈ ΠΏΠΎΠ»ΡƒΡ‡Π°Π΅ΠΌ ΠΏΠΎ Ρ‚Ρ€ΠΈ Ρ„ΠΎΡ€ΠΌΡƒΠ»Ρ‹ для Π΅Π³ΠΎ расчёта. Π”Π°Π»Π΅Π΅ Π²Ρ‹Π±ΠΈΡ€Π°Π΅ΠΌ Π½ΡƒΠΆΠ½ΡƒΡŽ.

Π­Ρ‚ΠΎΡ‚ ΠΊΡ€ΡƒΠ³ Ρ‚Π°ΠΊΠΆΠ΅, ΠΊΠ°ΠΊ ΠΈ Ρ‚Ρ€Π΅ΡƒΠ³ΠΎΠ»ΡŒΠ½ΠΈΠΊ ΠΌΠΎΠΆΠ½ΠΎ Π½Π°Π·Π²Π°Ρ‚ΡŒ магичСским.

Β 

ΠΊΠ°ΠΊ связаны ΠΌΠ΅ΠΆΠ΄Ρƒ собой напряТСниС, Ρ‚ΠΎΠΊ ΠΈ сопротивлСниС

Π”ΠΎΠ±Π°Π²Π»Π΅Π½ΠΎ 30 сСнтября 2020 Π² 00:30

Π‘ΠΎΡ…Ρ€Π°Π½ΠΈΡ‚ΡŒ ΠΈΠ»ΠΈ ΠΏΠΎΠ΄Π΅Π»ΠΈΡ‚ΡŒΡΡ

ΠŸΠ΅Ρ€Π²Π°Ρ ΠΈ, Π²ΠΎΠ·ΠΌΠΎΠΆΠ½ΠΎ, самая ваТная взаимосвязь ΠΌΠ΅ΠΆΠ΄Ρƒ Ρ‚ΠΎΠΊΠΎΠΌ, напряТСниСм ΠΈ сопротивлСниСм называСтся Π·Π°ΠΊΠΎΠ½ΠΎΠΌ Ома, ΠΊΠΎΡ‚ΠΎΡ€Ρ‹ΠΉ Π±Ρ‹Π» ΠΎΡ‚ΠΊΡ€Ρ‹Ρ‚ Π“Π΅ΠΎΡ€Π³ΠΎΠΌ Π‘ΠΈΠΌΠΎΠ½ΠΎΠΌ Омом ΠΈ ΠΎΠΏΡƒΠ±Π»ΠΈΠΊΠΎΠ²Π°Π½ Π² Π΅Π³ΠΎ ΡΡ‚Π°Ρ‚ΡŒΠ΅ 1827 Π³ΠΎΠ΄Π° Β«Π“Π°Π»ΡŒΠ²Π°Π½ΠΈΡ‡Π΅ΡΠΊΠ°Ρ Ρ†Π΅ΠΏΡŒ, исслСдованная матСматичСски».

НапряТСниС, Ρ‚ΠΎΠΊ ΠΈ сопротивлСниС

ЭлСктричСская Ρ†Π΅ΠΏΡŒ образуСтся, ΠΊΠΎΠ³Π΄Π° создаСтся проводящий ΠΏΡƒΡ‚ΡŒ, ΠΏΠΎΠ·Π²ΠΎΠ»ΡΡŽΡ‰ΠΈΠΉ элСктричСскому заряду Π½Π΅ΠΏΡ€Π΅Ρ€Ρ‹Π²Π½ΠΎ ΠΏΠ΅Ρ€Π΅ΠΌΠ΅Ρ‰Π°Ρ‚ΡŒΡΡ. Π­Ρ‚ΠΎ Π½Π΅ΠΏΡ€Π΅Ρ€Ρ‹Π²Π½ΠΎΠ΅ Π΄Π²ΠΈΠΆΠ΅Π½ΠΈΠ΅ элСктричСского заряда ΠΏΠΎ ΠΏΡ€ΠΎΠ²ΠΎΠ΄Π½ΠΈΠΊΠ°ΠΌ Ρ†Π΅ΠΏΠΈ называСтся Ρ‚ΠΎΠΊΠΎΠΌ, ΠΈ ΠΎ Π½Π΅ΠΌ часто говорят ΠΊΠ°ΠΊ ΠΎ Β«ΠΏΠΎΡ‚ΠΎΠΊΠ΅Β», ΠΊΠ°ΠΊ ΠΎ ΠΏΠΎΡ‚ΠΎΠΊΠ΅ Тидкости Ρ‡Π΅Ρ€Π΅Π· ΠΏΠΎΠ»ΡƒΡŽ Ρ‚Ρ€ΡƒΠ±Ρƒ.

Π‘ΠΈΠ»Π°, ΠΏΠΎΠ±ΡƒΠΆΠ΄Π°ΡŽΡ‰Π°Ρ носитСли заряда Β«Ρ‚Π΅Ρ‡ΡŒΒ» ΠΏΠΎ Ρ†Π΅ΠΏΠΈ, называСтся напряТСниСм. НапряТСниС – это особая ΠΌΠ΅Ρ€Π° ΠΏΠΎΡ‚Π΅Π½Ρ†ΠΈΠ°Π»ΡŒΠ½ΠΎΠΉ энСргии, которая всСгда ΠΎΡ‚Π½ΠΎΡΠΈΡ‚Π΅Π»ΡŒΠ½Π° ΠΌΠ΅ΠΆΠ΄Ρƒ двумя Ρ‚ΠΎΡ‡ΠΊΠ°ΠΌΠΈ. Когда ΠΌΡ‹ Π³ΠΎΠ²ΠΎΡ€ΠΈΠΌ ΠΎΠ± ΠΎΠΏΡ€Π΅Π΄Π΅Π»Π΅Π½Π½ΠΎΠΉ Π²Π΅Π»ΠΈΡ‡ΠΈΠ½Π΅ напряТСния, ΠΏΡ€ΠΈΡΡƒΡ‚ΡΡ‚Π²ΡƒΡŽΡ‰Π΅Π³ΠΎ Π² Ρ†Π΅ΠΏΠΈ, ΠΌΡ‹ ΠΈΠΌΠ΅Π΅ΠΌ Π² Π²ΠΈΠ΄Ρƒ ΠΈΠ·ΠΌΠ΅Ρ€Π΅Π½ΠΈΠ΅ ΠΏΠΎΡ‚Π΅Π½Ρ†ΠΈΠ°Π»ΡŒΠ½ΠΎΠΉ энСргии для пСрСмСщСния носитСлСй заряда ΠΈΠ· ΠΎΠ΄Π½ΠΎΠΉ ΠΊΠΎΠ½ΠΊΡ€Π΅Ρ‚Π½ΠΎΠΉ Ρ‚ΠΎΡ‡ΠΊΠΈ этой Ρ†Π΅ΠΏΠΈ Π² Π΄Ρ€ΡƒΠ³ΡƒΡŽ ΠΊΠΎΠ½ΠΊΡ€Π΅Ρ‚Π½ΡƒΡŽ Ρ‚ΠΎΡ‡ΠΊΡƒ. Π‘Π΅Π· упоминания Π΄Π²ΡƒΡ… ΠΊΠΎΠ½ΠΊΡ€Π΅Ρ‚Π½Ρ‹Ρ… Ρ‚ΠΎΡ‡Π΅ΠΊ Ρ‚Π΅Ρ€ΠΌΠΈΠ½ «напряТСниС» Π½Π΅ ΠΈΠΌΠ΅Π΅Ρ‚ значСния.

Π’ΠΎΠΊ, ΠΊΠ°ΠΊ ΠΏΡ€Π°Π²ΠΈΠ»ΠΎ, ΠΏΡ€ΠΎΡ…ΠΎΠ΄ΠΈΡ‚ Ρ‡Π΅Ρ€Π΅Π· ΠΏΡ€ΠΎΠ²ΠΎΠ΄Π½ΠΈΠΊΠΈ с Π½Π΅ΠΊΠΎΡ‚ΠΎΡ€ΠΎΠΉ ΡΡ‚Π΅ΠΏΠ΅Π½ΡŒΡŽ трСния ΠΈΠ»ΠΈ противодСйствия двиТСнию. Π­Ρ‚ΠΎ противодСйствиС двиТСнию ΠΏΡ€Π°Π²ΠΈΠ»ΡŒΠ½Π΅Π΅ Π½Π°Π·Ρ‹Π²Π°Ρ‚ΡŒ сопротивлСниСм. Π’Π΅Π»ΠΈΡ‡ΠΈΠ½Π° Ρ‚ΠΎΠΊΠ° Π² Ρ†Π΅ΠΏΠΈ зависит ΠΎΡ‚ Π²Π΅Π»ΠΈΡ‡ΠΈΠ½Ρ‹ напряТСния ΠΈ Π²Π΅Π»ΠΈΡ‡ΠΈΠ½Ρ‹ сопротивлСния Π² Ρ†Π΅ΠΏΠΈ, ΠΏΡ€Π΅ΠΏΡΡ‚ΡΡ‚Π²ΡƒΡŽΡ‰Π΅Π³ΠΎ ΠΏΡ€ΠΎΡ…ΠΎΠΆΠ΄Π΅Π½ΠΈΡŽ Ρ‚ΠΎΠΊΠ°. Как ΠΈ напряТСниС, сопротивлСниС – это Π²Π΅Π»ΠΈΡ‡ΠΈΠ½Π°, измСряСмая ΠΌΠ΅ΠΆΠ΄Ρƒ двумя Ρ‚ΠΎΡ‡ΠΊΠ°ΠΌΠΈ. По этой ΠΏΡ€ΠΈΡ‡ΠΈΠ½Π΅ Π²Π΅Π»ΠΈΡ‡ΠΈΠ½Ρ‹ напряТСния ΠΈ сопротивлСния часто ΡƒΠΊΠ°Π·Ρ‹Π²Π°ΡŽΡ‚ΡΡ ΠΊΠ°ΠΊ Β«ΠΌΠ΅ΠΆΠ΄ΡƒΒ» двумя Ρ‚ΠΎΡ‡ΠΊΠ°ΠΌΠΈ Π² Ρ†Π΅ΠΏΠΈ.

Π•Π΄ΠΈΠ½ΠΈΡ†Ρ‹ измСрСния: Π²ΠΎΠ»ΡŒΡ‚, Π°ΠΌΠΏΠ΅Ρ€ ΠΈ ΠΎΠΌ

Π§Ρ‚ΠΎΠ±Ρ‹ ΠΈΠΌΠ΅Ρ‚ΡŒ Π²ΠΎΠ·ΠΌΠΎΠΆΠ½ΠΎΡΡ‚ΡŒ Π΄Π΅Π»Π°Ρ‚ΡŒ осмыслСнныС утвСрТдСния ΠΎΠ± этих Π²Π΅Π»ΠΈΡ‡ΠΈΠ½Π°Ρ… Π² цСпях, Π½Π°ΠΌ Π½ΡƒΠΆΠ½ΠΎ ΡƒΠΌΠ΅Ρ‚ΡŒ ΠΎΠΏΠΈΡΡ‹Π²Π°Ρ‚ΡŒ ΠΈΡ… количСства Ρ‚Π°ΠΊ ΠΆΠ΅, ΠΊΠ°ΠΊ ΠΌΡ‹ ΠΌΠΎΠ³Π»ΠΈ Π±Ρ‹ количСствСнно ΠΎΠΏΡ€Π΅Π΄Π΅Π»ΠΈΡ‚ΡŒ массу, Ρ‚Π΅ΠΌΠΏΠ΅Ρ€Π°Ρ‚ΡƒΡ€Ρƒ, объСм, Π΄Π»ΠΈΠ½Ρƒ ΠΈΠ»ΠΈ Π»ΡŽΠ±Ρ‹Π΅ Π΄Ρ€ΡƒΠ³ΠΈΠ΅ физичСскиС Π²Π΅Π»ΠΈΡ‡ΠΈΠ½Ρ‹. Для массы ΠΌΡ‹ ΠΌΠΎΠΆΠ΅ΠΌ ΠΈΡΠΏΠΎΠ»ΡŒΠ·ΠΎΠ²Π°Ρ‚ΡŒ Π΅Π΄ΠΈΠ½ΠΈΡ†Ρ‹ Β«ΠΊΠΈΠ»ΠΎΠ³Ρ€Π°ΠΌΠΌΒ» ΠΈΠ»ΠΈ Β«Π³Ρ€Π°ΠΌΠΌΒ». Для Ρ‚Π΅ΠΌΠΏΠ΅Ρ€Π°Ρ‚ΡƒΡ€Ρ‹ ΠΌΡ‹ ΠΌΠΎΠΆΠ΅ΠΌ ΠΈΡΠΏΠΎΠ»ΡŒΠ·ΠΎΠ²Π°Ρ‚ΡŒ градусы Π€Π°Ρ€Π΅Π½Π³Π΅ΠΉΡ‚Π° ΠΈΠ»ΠΈ градусы ЦСльсия. Π’ Ρ‚Π°Π±Π»ΠΈΡ†Π΅ Π½ΠΈΠΆΠ΅ ΠΏΡ€ΠΈΠ²Π΅Π΄Π΅Π½Ρ‹ стандартныС Π΅Π΄ΠΈΠ½ΠΈΡ†Ρ‹ измСрСния элСктричСского Ρ‚ΠΎΠΊΠ°, напряТСния ΠΈ сопротивлСния:

Π•Π΄ΠΈΠ½ΠΈΡ†Ρ‹ измСрСния Ρ‚ΠΎΠΊΠ°, напряТСния, сопротивлСния
Π’Π΅Π»ΠΈΡ‡ΠΈΠ½Π°Π‘ΠΈΠΌΠ²ΠΎΠ»Π•Π΄ΠΈΠ½ΠΈΡ†Π° измСрСнияБокращСниС Π΅Π΄ΠΈΠ½ΠΈΡ†Ρ‹ измСрСния
Π’ΠΎΠΊIАмпСрА
НапряТСниСVΠ’ΠΎΠ»ΡŒΡ‚Π’
Π‘ΠΎΠΏΡ€ΠΎΡ‚ΠΈΠ²Π»Π΅Π½ΠΈΠ΅RОмОм

Β«Π‘ΠΈΠΌΠ²ΠΎΠ»Β», присвоСнный ΠΊΠ°ΠΆΠ΄ΠΎΠΉ Π²Π΅Π»ΠΈΡ‡ΠΈΠ½Π΅, прСдставляСт собой ΡΡ‚Π°Π½Π΄Π°Ρ€Ρ‚Π½ΡƒΡŽ Π±ΡƒΠΊΠ²Ρƒ латинского Π°Π»Ρ„Π°Π²ΠΈΡ‚Π°, ΠΈΡΠΏΠΎΠ»ΡŒΠ·ΡƒΠ΅ΠΌΡƒΡŽ для прСдставлСния этой Π²Π΅Π»ΠΈΡ‡ΠΈΠ½Ρ‹ Π² Ρ„ΠΎΡ€ΠΌΡƒΠ»Π°Ρ…. ΠŸΠΎΠ΄ΠΎΠ±Π½Ρ‹Π΅ стандартизированныС Π±ΡƒΠΊΠ²Ρ‹ распространСны Π²ΠΎ всСх физичСских ΠΈ тСхничСских дисциплинах ΠΈ ΠΏΡ€ΠΈΠ·Π½Π°Π½Ρ‹ Π²ΠΎ всСм ΠΌΠΈΡ€Π΅. Β«Π‘ΠΎΠΊΡ€Π°Ρ‰Π΅Π½ΠΈΠ΅ Π΅Π΄ΠΈΠ½ΠΈΡ†Ρ‹ измСрСния» для ΠΊΠ°ΠΆΠ΄ΠΎΠΉ Π²Π΅Π»ΠΈΡ‡ΠΈΠ½Ρ‹ прСдставляСт собой Π°Π»Ρ„Π°Π²ΠΈΡ‚Π½Ρ‹ΠΉ символ(Ρ‹), ΠΈΡΠΏΠΎΠ»ΡŒΠ·ΡƒΠ΅ΠΌΡ‹ΠΉ Π² качСствС сокращСнного обозначСния ΠΊΠΎΠ½ΠΊΡ€Π΅Ρ‚Π½ΠΎΠΉ Π΅Π΄ΠΈΠ½ΠΈΡ†Ρ‹ измСрСния.

КаТдая Π΅Π΄ΠΈΠ½ΠΈΡ†Π° измСрСния Π½Π°Π·Π²Π°Π½Π° Π² Ρ‡Π΅ΡΡ‚ΡŒ извСстного экспСримСнтатора Π² области элСктричСства: Π°ΠΌΠΏΠ΅Ρ€ Π² Ρ‡Π΅ΡΡ‚ΡŒ Ρ„Ρ€Π°Π½Ρ†ΡƒΠ·Π° АндрС М. АмпСра, Π²ΠΎΠ»ΡŒΡ‚ Π² Ρ‡Π΅ΡΡ‚ΡŒ ΠΈΡ‚Π°Π»ΡŒΡΠ½Ρ†Π° АлСссандро Π’ΠΎΠ»ΡŒΡ‚Π°, Π° ΠΎΠΌ Π² Ρ‡Π΅ΡΡ‚ΡŒ Π½Π΅ΠΌΡ†Π° Π“Π΅ΠΎΡ€Π³Π° Π‘ΠΈΠΌΠΎΠ½Π° Ома.

ΠœΠ°Ρ‚Π΅ΠΌΠ°Ρ‚ΠΈΡ‡Π΅ΡΠΊΠΈΠΉ символ для ΠΊΠ°ΠΆΠ΄ΠΎΠΉ Π²Π΅Π»ΠΈΡ‡ΠΈΠ½Ρ‹ Ρ‚Π°ΠΊΠΆΠ΅ ΠΈΠΌΠ΅Π΅Ρ‚ Π·Π½Π°Ρ‡Π΅Π½ΠΈΠ΅. Β«RΒ» для сопротивлСния ΠΈ Β«VΒ» для напряТСния говорят сами Π·Π° сСбя (Β«ResistanceΒ» ΠΈ Β«VoltageΒ», соотвСтствСнно), Ρ‚ΠΎΠ³Π΄Π° ΠΊΠ°ΠΊ Β«IΒ» для Ρ‚ΠΎΠΊΠ° каТСтся Π½Π΅ΠΌΠ½ΠΎΠ³ΠΎ странным. ΠŸΡ€Π΅Π΄ΠΏΠΎΠ»Π°Π³Π°Π΅Ρ‚ΡΡ, Ρ‡Ρ‚ΠΎ Π±ΡƒΠΊΠ²Π° Β«IΒ» Π΄ΠΎΠ»ΠΆΠ½Π° ΠΏΡ€Π΅Π΄ΡΡ‚Π°Π²Π»ΡΡ‚ΡŒ Β«ΠΈΠ½Ρ‚Π΅Π½ΡΠΈΠ²Π½ΠΎΡΡ‚ΡŒΒ» (Β«IntensityΒ»)(ΠΏΠΎΡ‚ΠΎΠΊΠ° заряда). Будя ΠΏΠΎ исслСдованиям, ΠΊΠΎΡ‚ΠΎΡ€Ρ‹Π΅ ΠΌΠ½Π΅ ΡƒΠ΄Π°Π»ΠΎΡΡŒ провСсти, каТСтся, Ρ‡Ρ‚ΠΎ Π΅ΡΡ‚ΡŒ Π½Π΅ΠΊΠΎΡ‚ΠΎΡ€Ρ‹Π΅ разногласия ΠΏΠΎ ΠΏΠΎΠ²ΠΎΠ΄Ρƒ значСния слова Β«IΒ». Π”Ρ€ΡƒΠ³ΠΎΠΉ символ напряТСния, Β«EΒ», ΠΎΠ·Π½Π°Ρ‡Π°Π΅Ρ‚ Β«ΡΠ»Π΅ΠΊΡ‚Ρ€ΠΎΠ΄Π²ΠΈΠΆΡƒΡ‰ΡƒΡŽ силу» (Β«Electromotive forceΒ»). Π‘ΠΈΠΌΠ²ΠΎΠ»Ρ‹ Β«EΒ» ΠΈ Β«VΒ» ΠΏΠΎ большСй части взаимозамСняСмы, хотя Π² Π½Π΅ΠΊΠΎΡ‚ΠΎΡ€Ρ‹Ρ… тСкстах Β«EΒ» Π·Π°Ρ€Π΅Π·Π΅Ρ€Π²ΠΈΡ€ΠΎΠ²Π°Π½ΠΎ для обозначСния напряТСния Π½Π° источникС (Ρ‚Π°ΠΊΠΎΠΌ ΠΊΠ°ΠΊ батарСя ΠΈΠ»ΠΈ Π³Π΅Π½Π΅Ρ€Π°Ρ‚ΠΎΡ€), Π° Β«V»– для обозначСния напряТСния Π½Π° любом Π΄Ρ€ΡƒΠ³ΠΎΠΌ элСмСнтС.

ВсС эти символы Π²Ρ‹Ρ€Π°ΠΆΠ°ΡŽΡ‚ΡΡ Π·Π°Π³Π»Π°Π²Π½Ρ‹ΠΌΠΈ Π±ΡƒΠΊΠ²Π°ΠΌΠΈ, Π·Π° ΠΈΡΠΊΠ»ΡŽΡ‡Π΅Π½ΠΈΠ΅ΠΌ случаСв, ΠΊΠΎΠ³Π΄Π° Π²Π΅Π»ΠΈΡ‡ΠΈΠ½Π° (особСнно напряТСниС ΠΈΠ»ΠΈ Ρ‚ΠΎΠΊ) описываСтся Π² Ρ‚Π΅Ρ€ΠΌΠΈΠ½Π°Ρ… ΠΊΠΎΡ€ΠΎΡ‚ΠΊΠΎΠ³ΠΎ ΠΏΠ΅Ρ€ΠΈΠΎΠ΄Π° Π²Ρ€Π΅ΠΌΠ΅Π½ΠΈ (Ρ‚Π°ΠΊ Π½Π°Π·Ρ‹Π²Π°Π΅ΠΌΡ‹Π΅ Β«ΠΌΠ³Π½ΠΎΠ²Π΅Π½Π½Ρ‹Π΅Β» значСния). НапримСр, напряТСниС Π±Π°Ρ‚Π°Ρ€Π΅ΠΈ, ΠΊΠΎΡ‚ΠΎΡ€ΠΎΠ΅ ΡΡ‚Π°Π±ΠΈΠ»ΡŒΠ½ΠΎ Π² Ρ‚Π΅Ρ‡Π΅Π½ΠΈΠ΅ Π΄Π»ΠΈΡ‚Π΅Π»ΡŒΠ½ΠΎΠ³ΠΎ ΠΏΠ΅Ρ€ΠΈΠΎΠ΄Π° Π²Ρ€Π΅ΠΌΠ΅Π½ΠΈ, Π±ΡƒΠ΄Π΅Ρ‚ ΠΎΠ±ΠΎΠ·Π½Π°Ρ‡Π°Ρ‚ΡŒΡΡ Π·Π°Π³Π»Π°Π²Π½ΠΎΠΉ Π±ΡƒΠΊΠ²ΠΎΠΉ Β«EΒ», Ρ‚ΠΎΠ³Π΄Π° ΠΊΠ°ΠΊ ΠΏΠΈΠΊΠΎΠ²ΠΎΠ΅ напряТСния ΠΏΡ€ΠΈ ΡƒΠ΄Π°Ρ€Π΅ ΠΌΠΎΠ»Π½ΠΈΠΈ Π² Ρ‚ΠΎΡ‚ самый ΠΌΠΎΠΌΠ΅Π½Ρ‚, ΠΊΠΎΠ³Π΄Π° ΠΎΠ½Π° ΠΏΠΎΠΏΠ°Π΄Π°Π΅Ρ‚ Π² линию элСктропСрСдачи, скорСС всСго, Π±ΡƒΠ΄Π΅Ρ‚ ΠΎΠ±ΠΎΠ·Π½Π°Ρ‡Π°Ρ‚ΡŒΡΡ строчной Π±ΡƒΠΊΠ²ΠΎΠΉ Β«Π΅Β» (ΠΈΠ»ΠΈ строчной Π±ΡƒΠΊΠ²ΠΎΠΉ Β«vΒ»), Ρ‡Ρ‚ΠΎΠ±Ρ‹ ΠΎΡ‚ΠΌΠ΅Ρ‚ΠΈΡ‚ΡŒ это Π·Π½Π°Ρ‡Π΅Π½ΠΈΠ΅ ΠΊΠ°ΠΊ ΠΈΠΌΠ΅ΡŽΡ‰Π΅Π΅ΡΡ Π² ΠΎΠ΄ΠΈΠ½ ΠΌΠΎΠΌΠ΅Π½Ρ‚ Π²Ρ€Π΅ΠΌΠ΅Π½ΠΈ. Π­Ρ‚ΠΎ ΠΆΠ΅ соглашСниС ΠΎ Π½ΠΈΠΆΠ½Π΅ΠΌ рСгистрС справСдливо ΠΈ для Ρ‚ΠΎΠΊΠ°: строчная Π±ΡƒΠΊΠ²Π° Β«iΒ» прСдставляСт Ρ‚ΠΎΠΊ Π² Π½Π΅ΠΊΠΎΡ‚ΠΎΡ€Ρ‹ΠΉ ΠΌΠΎΠΌΠ΅Π½Ρ‚ Π²Ρ€Π΅ΠΌΠ΅Π½ΠΈ. Однако Π±ΠΎΠ»ΡŒΡˆΠΈΠ½ΡΡ‚Π²ΠΎ ΠΈΠ·ΠΌΠ΅Ρ€Π΅Π½ΠΈΠΉ Π² цСпях постоянного Ρ‚ΠΎΠΊΠ°, ΠΊΠΎΡ‚ΠΎΡ€Ρ‹Π΅ ΡΡ‚Π°Π±ΠΈΠ»ΡŒΠ½Ρ‹ Π²ΠΎ Π²Ρ€Π΅ΠΌΠ΅Π½ΠΈ, Π±ΡƒΠ΄ΡƒΡ‚ ΠΎΠ±ΠΎΠ·Π½Π°Ρ‡Π°Ρ‚ΡŒΡΡ Π·Π°Π³Π»Π°Π²Π½Ρ‹ΠΌΠΈ Π±ΡƒΠΊΠ²Π°ΠΌΠΈ.

ΠšΡƒΠ»ΠΎΠ½ ΠΈ элСктричСский заряд

Одна ΠΈΠ· основных Π΅Π΄ΠΈΠ½ΠΈΡ† элСктричСских ΠΈΠ·ΠΌΠ΅Ρ€Π΅Π½ΠΈΠΉ, ΠΊΠΎΡ‚ΠΎΡ€ΡƒΡŽ часто ΠΏΡ€Π΅ΠΏΠΎΠ΄Π°ΡŽΡ‚ Π² Π½Π°Ρ‡Π°Π»Π΅ курсов элСктроники, Π½ΠΎ нСчасто ΠΈΡΠΏΠΎΠ»ΡŒΠ·ΡƒΡŽΡ‚ впослСдствии, – это ΠΊΡƒΠ»ΠΎΠ½ – Π΅Π΄ΠΈΠ½ΠΈΡ†Π° измСрСния элСктричСского заряда, ΠΏΡ€ΠΎΠΏΠΎΡ€Ρ†ΠΈΠΎΠ½Π°Π»ΡŒΠ½Π°Ρ количСству элСктронов Π² нСсбалансированном состоянии. Один ΠΊΡƒΠ»ΠΎΠ½ заряда соотвСтствуСт 6 250 000 000 000 000 000 элСктронов. Π‘ΠΈΠΌΠ²ΠΎΠ»ΠΎΠΌ количСства элСктричСского заряда являСтся заглавная Π±ΡƒΠΊΠ²Π° Β«QΒ», Π° Π΅Π΄ΠΈΠ½ΠΈΡ†Π° измСрСния ΠΊΡƒΠ»ΠΎΠ½ΠΎΠ² обозначаСтся «Кл». Π•Π΄ΠΈΠ½ΠΈΡ†Π° измСрСния Ρ‚ΠΎΠΊΠ°, Π°ΠΌΠΏΠ΅Ρ€, Ρ€Π°Π²Π½Π° 1 ΠΊΡƒΠ»ΠΎΠ½Ρƒ заряда, проходящСму Ρ‡Π΅Ρ€Π΅Π· Π·Π°Π΄Π°Π½Π½ΡƒΡŽ Ρ‚ΠΎΡ‡ΠΊΡƒ Π² Ρ†Π΅ΠΏΠΈ Π·Π° 1 сСкунду. Π’ этом смыслС, Ρ‚ΠΎΠΊ – это ΡΠΊΠΎΡ€ΠΎΡΡ‚ΡŒ двиТСния элСктричСского заряда Ρ‡Π΅Ρ€Π΅Π· ΠΏΡ€ΠΎΠ²ΠΎΠ΄Π½ΠΈΠΊ.

Как ΡƒΠΊΠ°Π·Ρ‹Π²Π°Π»ΠΎΡΡŒ Ρ€Π°Π½Π΅Π΅, напряТСниС – это ΠΌΠ΅Ρ€Π° ΠΏΠΎΡ‚Π΅Π½Ρ†ΠΈΠ°Π»ΡŒΠ½ΠΎΠΉ энСргии Π½Π° Π΅Π΄ΠΈΠ½ΠΈΡ†Ρƒ заряда, доступная для стимулирования протСкания Ρ‚ΠΎΠΊΠ° ΠΈΠ· ΠΎΠ΄Π½ΠΎΠΉ Ρ‚ΠΎΡ‡ΠΊΠΈ Π² Π΄Ρ€ΡƒΠ³ΡƒΡŽ. ΠŸΡ€Π΅ΠΆΠ΄Π΅ Ρ‡Π΅ΠΌ ΠΌΡ‹ смоТСм Ρ‚ΠΎΡ‡Π½ΠΎ ΠΎΠΏΡ€Π΅Π΄Π΅Π»ΠΈΡ‚ΡŒ, Ρ‡Ρ‚ΠΎ Ρ‚Π°ΠΊΠΎΠ΅ Β«Π²ΠΎΠ»ΡŒΡ‚Β», ΠΌΡ‹ Π΄ΠΎΠ»ΠΆΠ½Ρ‹ ΠΏΠΎΠ½ΡΡ‚ΡŒ, ΠΊΠ°ΠΊ ΠΈΠ·ΠΌΠ΅Ρ€ΠΈΡ‚ΡŒ эту Π²Π΅Π»ΠΈΡ‡ΠΈΠ½Ρƒ, ΠΊΠΎΡ‚ΠΎΡ€ΡƒΡŽ ΠΌΡ‹ Π½Π°Π·Ρ‹Π²Π°Π΅ΠΌ Β«ΠΏΠΎΡ‚Π΅Π½Ρ†ΠΈΠ°Π»ΡŒΠ½ΠΎΠΉ энСргиСй». ΠžΠ±Ρ‰Π΅ΠΉ мСтричСской Π΅Π΄ΠΈΠ½ΠΈΡ†Π΅ΠΉ измСрСния энСргии любого Π²ΠΈΠ΄Π° являСтся Π΄ΠΆΠΎΡƒΠ»ΡŒ, Ρ€Π°Π²Π½Ρ‹ΠΉ количСству Ρ€Π°Π±ΠΎΡ‚Ρ‹, ΡΠΎΠ²Π΅Ρ€ΡˆΠ°Π΅ΠΌΠΎΠΉ силой Π² 1 Π½ΡŒΡŽΡ‚ΠΎΠ½ ΠΏΡ€ΠΈ Π΄Π²ΠΈΠΆΠ΅Π½ΠΈΠΈ Π½Π° 1 ΠΌΠ΅Ρ‚Ρ€ (Π² Ρ‚ΠΎΠΌ ΠΆΠ΅ Π½Π°ΠΏΡ€Π°Π²Π»Π΅Π½ΠΈΠΈ). Π’ этих Π½Π°ΡƒΡ‡Π½Ρ‹Ρ… Ρ‚Π΅Ρ€ΠΌΠΈΠ½Π°Ρ… 1 Π²ΠΎΠ»ΡŒΡ‚ Ρ€Π°Π²Π΅Π½ 1 Π΄ΠΆΠΎΡƒΠ»ΡŽ элСктричСской ΠΏΠΎΡ‚Π΅Π½Ρ†ΠΈΠ°Π»ΡŒΠ½ΠΎΠΉ энСргии Π½Π° (Π΄Π΅Π»Π΅Π½Π½ΠΎΠΌΡƒ Π½Π°) 1 ΠΊΡƒΠ»ΠΎΠ½ заряда. Π’Π°ΠΊΠΈΠΌ ΠΎΠ±Ρ€Π°Π·ΠΎΠΌ, 9-Π²ΠΎΠ»ΡŒΡ‚ΠΎΠ²Π°Ρ батарСя выдСляСт 9 Π΄ΠΆΠΎΡƒΠ»Π΅ΠΉ энСргии Π½Π° ΠΊΠ°ΠΆΠ΄Ρ‹ΠΉ ΠΊΡƒΠ»ΠΎΠ½ заряда, проходящСго Ρ‡Π΅Ρ€Π΅Π· Ρ†Π΅ΠΏΡŒ.

Π­Ρ‚ΠΈ Π΅Π΄ΠΈΠ½ΠΈΡ†Ρ‹ ΠΈ символы элСктричСских Π²Π΅Π»ΠΈΡ‡ΠΈΠ½ станут ΠΎΡ‡Π΅Π½ΡŒ Π²Π°ΠΆΠ½Ρ‹, ΠΊΠΎΠ³Π΄Π° ΠΌΡ‹ Π½Π°Ρ‡Π½Π΅ΠΌ ΠΈΡΡΠ»Π΅Π΄ΠΎΠ²Π°Ρ‚ΡŒ ΠΎΡ‚Π½ΠΎΡˆΠ΅Π½ΠΈΡ ΠΌΠ΅ΠΆΠ΄Ρƒ Π½ΠΈΠΌΠΈ Π² цСпях.

Π€ΠΎΡ€ΠΌΡƒΠ»Π° Π·Π°ΠΊΠΎΠ½Π° Ома

ОсновноС ΠΎΡ‚ΠΊΡ€Ρ‹Ρ‚ΠΈΠ΅ Ома Π·Π°ΠΊΠ»ΡŽΡ‡Π°Π»ΠΎΡΡŒ Π² Ρ‚ΠΎΠΌ, Ρ‡Ρ‚ΠΎ Π²Π΅Π»ΠΈΡ‡ΠΈΠ½Π° элСктричСского Ρ‚ΠΎΠΊΠ°, ΠΏΡ€ΠΎΡ‚Π΅ΠΊΠ°ΡŽΡ‰Π΅Π³ΠΎ Ρ‡Π΅Ρ€Π΅Π· мСталличСский ΠΏΡ€ΠΎΠ²ΠΎΠ΄Π½ΠΈΠΊ Π² Ρ†Π΅ΠΏΠΈ, ΠΏΡ€ΠΈ любой Π·Π°Π΄Π°Π½Π½ΠΎΠΉ Ρ‚Π΅ΠΌΠΏΠ΅Ρ€Π°Ρ‚ΡƒΡ€Π΅ прямо ΠΏΡ€ΠΎΠΏΠΎΡ€Ρ†ΠΈΠΎΠ½Π°Π»ΡŒΠ½Π° Π½Π°ΠΏΡ€ΡΠΆΠ΅Π½ΠΈΡŽ, ΠΏΡ€ΠΈΠ»ΠΎΠΆΠ΅Π½Π½ΠΎΠΌΡƒ ΠΊ Π½Π΅ΠΌΡƒ. Ом Π²Ρ‹Ρ€Π°Π·ΠΈΠ» своС ΠΎΡ‚ΠΊΡ€Ρ‹Ρ‚ΠΈΠ΅ Π² Π²ΠΈΠ΄Π΅ простого уравнСния, ΠΎΠΏΠΈΡΡ‹Π²Π°ΡŽΡ‰Π΅Π³ΠΎ взаимосвязь напряТСния, Ρ‚ΠΎΠΊΠ° ΠΈ сопротивлСния:

\[E=IR\]

Π’ этом алгСбраичСском Π²Ρ‹Ρ€Π°ΠΆΠ΅Π½ΠΈΠΈ напряТСниС (E) Ρ€Π°Π²Π½ΠΎ Ρ‚ΠΎΠΊΡƒ (I), ΡƒΠΌΠ½ΠΎΠΆΠ΅Π½Π½ΠΎΠΌΡƒ Π½Π° сопротивлСниС (R). Π˜ΡΠΏΠΎΠ»ΡŒΠ·ΡƒΡ Π°Π»Π³Π΅Π±Ρ€Ρƒ, ΠΌΡ‹ ΠΌΠΎΠΆΠ΅ΠΌ ΠΏΡ€Π΅ΠΎΠ±Ρ€Π°Π·ΠΎΠ²Π°Ρ‚ΡŒ это ΡƒΡ€Π°Π²Π½Π΅Π½ΠΈΠ΅ Π² Π΄Ρ€ΡƒΠ³ΠΈΡ… Π΄Π²Π° Π²Π°Ρ€ΠΈΠ°Π½Ρ‚Π°, Ρ€Π΅ΡˆΠ°Ρ Π΅Π³ΠΎ для I ΠΈ R соотвСтствСнно:

\[I = \frac{E}{R}\]

\[R = \frac{E}{I}\]

Анализ простых схСм с ΠΏΠΎΠΌΠΎΡ‰ΡŒΡŽ Π·Π°ΠΊΠΎΠ½Π° Ома

Π”Π°Π²Π°ΠΉΡ‚Π΅ посмотрим, ΠΊΠ°ΠΊ эти Ρ„ΠΎΡ€ΠΌΡƒΠ»Ρ‹ Ρ€Π°Π±ΠΎΡ‚Π°ΡŽΡ‚, Ρ‡Ρ‚ΠΎΠ±Ρ‹ ΠΏΠΎΠΌΠΎΡ‡ΡŒ Π½Π°ΠΌ Π°Π½Π°Π»ΠΈΠ·ΠΈΡ€ΠΎΠ²Π°Ρ‚ΡŒ простыС схСмы:

Рисунок 1 – ΠŸΡ€ΠΈΠΌΠ΅Ρ€ простой схСмы

Π’ ΠΏΡ€ΠΈΠ²Π΅Π΄Π΅Π½Π½ΠΎΠΉ Π²Ρ‹ΡˆΠ΅ схСмС Π΅ΡΡ‚ΡŒ Ρ‚ΠΎΠ»ΡŒΠΊΠΎ ΠΎΠ΄ΠΈΠ½ источник напряТСния (батарСя слСва) ΠΈ Ρ‚ΠΎΠ»ΡŒΠΊΠΎ ΠΎΠ΄ΠΈΠ½ источник сопротивлСния Ρ‚ΠΎΠΊΡƒ (Π»Π°ΠΌΠΏΠ° справа). Π­Ρ‚ΠΎ позволяСт ΠΎΡ‡Π΅Π½ΡŒ Π»Π΅Π³ΠΊΠΎ ΠΏΡ€ΠΈΠΌΠ΅Π½ΠΈΡ‚ΡŒ Π·Π°ΠΊΠΎΠ½ Ома. Если ΠΌΡ‹ Π·Π½Π°Π΅ΠΌ значСния Π»ΡŽΠ±Ρ‹Ρ… Π΄Π²ΡƒΡ… ΠΈΠ· Ρ‚Ρ€Π΅Ρ… Π²Π΅Π»ΠΈΡ‡ΠΈΠ½ (напряТСния, Ρ‚ΠΎΠΊΠ° ΠΈ сопротивлСния) Π² этой Ρ†Π΅ΠΏΠΈ, ΠΌΡ‹ ΠΌΠΎΠΆΠ΅ΠΌ ΠΈΡΠΏΠΎΠ»ΡŒΠ·ΠΎΠ²Π°Ρ‚ΡŒ Π·Π°ΠΊΠΎΠ½ Ома для опрСдСлСния Ρ‚Ρ€Π΅Ρ‚ΡŒΠ΅ΠΉ.

Π’ этом ΠΏΠ΅Ρ€Π²ΠΎΠΌ ΠΏΡ€ΠΈΠΌΠ΅Ρ€Π΅ ΠΌΡ‹ вычислим Π²Π΅Π»ΠΈΡ‡ΠΈΠ½Ρƒ Ρ‚ΠΎΠΊΠ° (I) Π² Ρ†Π΅ΠΏΠΈ, учитывая значСния напряТСния (E) ΠΈ сопротивлСния (R):

Рисунок 2 – ΠŸΡ€ΠΈΠΌΠ΅Ρ€ 1. Π˜Π·Π²Π΅ΡΡ‚Π½Ρ‹ напряТСниС источника ΠΈ сопротивлСниС Π»Π°ΠΌΠΏΡ‹

Какая Π²Π΅Π»ΠΈΡ‡ΠΈΠ½Π° Ρ‚ΠΎΠΊΠ° (I) Π² этой Ρ†Π΅ΠΏΠΈ?

\[I = \frac{E}{R} = \frac{12 \ Π’}{3 \ Ом} = 4 \ А\]

Π’ΠΎ Π²Ρ‚ΠΎΡ€ΠΎΠΌ ΠΏΡ€ΠΈΠΌΠ΅Ρ€Π΅ ΠΌΡ‹ вычислим Π²Π΅Π»ΠΈΡ‡ΠΈΠ½Ρƒ сопротивлСния (R) Π² Ρ†Π΅ΠΏΠΈ, учитывая значСния напряТСния (E) ΠΈ Ρ‚ΠΎΠΊΠ° (I):

Рисунок 3 – ΠŸΡ€ΠΈΠΌΠ΅Ρ€ 2. Π˜Π·Π²Π΅ΡΡ‚Π½Ρ‹ напряТСниС источника ΠΈ Ρ‚ΠΎΠΊ Π² Ρ†Π΅ΠΏΠΈ

КакоС сопротивлСниС (R) ΠΎΠΊΠ°Π·Ρ‹Π²Π°Π΅Ρ‚ Π»Π°ΠΌΠΏΠ°?

\[R = \frac{E}{I} = \frac{36 \ Π’}{4 \ А} = 9 \ Ом\]

Π’ послСднСм ΠΏΡ€ΠΈΠΌΠ΅Ρ€Π΅ ΠΌΡ‹ рассчитаСм Π²Π΅Π»ΠΈΡ‡ΠΈΠ½Ρƒ напряТСния, ΠΏΠΎΠ΄Π°Π²Π°Π΅ΠΌΠΎΠ³ΠΎ Π±Π°Ρ‚Π°Ρ€Π΅Π΅ΠΉ, с ΡƒΡ‡Π΅Ρ‚ΠΎΠΌ Π·Π½Π°Ρ‡Π΅Π½ΠΈΠΉ Ρ‚ΠΎΠΊΠ° (I) ΠΈ сопротивлСния (R):

Рисунок 4 – ΠŸΡ€ΠΈΠΌΠ΅Ρ€ 3. Π˜Π·Π²Π΅ΡΡ‚Π½Ρ‹ Ρ‚ΠΎΠΊ Π² Ρ†Π΅ΠΏΠΈ ΠΈ сопротивлСниС Π»Π°ΠΌΠΏΡ‹

КакоС напряТСниС обСспСчиваСт батарСя?

\[E = IR = (2 \ А)(7 \ Ом) = 14 \ Π’\]

ΠœΠ΅Ρ‚ΠΎΠ΄Π° Ρ‚Ρ€Π΅ΡƒΠ³ΠΎΠ»ΡŒΠ½ΠΈΠΊΠ° Π·Π°ΠΊΠΎΠ½Π° Ома

Π—Π°ΠΊΠΎΠ½ Ома – ΠΎΡ‡Π΅Π½ΡŒ простой ΠΈ ΠΏΠΎΠ»Π΅Π·Π½Ρ‹ΠΉ инструмСнт для Π°Π½Π°Π»ΠΈΠ·Π° элСктричСских Ρ†Π΅ΠΏΠ΅ΠΉ. Он Ρ‚Π°ΠΊ часто ΠΈΡΠΏΠΎΠ»ΡŒΠ·ΡƒΠ΅Ρ‚ΡΡ ΠΏΡ€ΠΈ ΠΈΠ·ΡƒΡ‡Π΅Π½ΠΈΠΈ элСктричСства ΠΈ элСктроники, Ρ‡Ρ‚ΠΎ студСнт Π΄ΠΎΠ»ΠΆΠ΅Π½ Π·Π°ΠΏΠΎΠΌΠ½ΠΈΡ‚ΡŒ Π΅Π³ΠΎ. Если Π²Ρ‹ Π½Π΅ ΠΎΡ‡Π΅Π½ΡŒ Ρ…ΠΎΡ€ΠΎΡˆΠΎ ΡƒΠΌΠ΅Π΅Ρ‚Π΅ Ρ€Π°Π±ΠΎΡ‚Π°Ρ‚ΡŒ с Ρ„ΠΎΡ€ΠΌΡƒΠ»Π°ΠΌΠΈ, Ρ‚ΠΎ для Π΅Π³ΠΎ запоминания сущСствуСт простой ΠΏΡ€ΠΈΠ΅ΠΌ, ΠΏΠΎΠΌΠΎΠ³Π°ΡŽΡ‰ΠΈΠΉ ΠΈΡΠΏΠΎΠ»ΡŒΠ·ΠΎΠ²Π°Ρ‚ΡŒ Π΅Π³ΠΎ для любой Π²Π΅Π»ΠΈΡ‡ΠΈΠ½Ρ‹, зная Π΄Π²Π΅ Π΄Ρ€ΡƒΠ³ΠΈΡ…. Π‘Π½Π°Ρ‡Π°Π»Π° располоТитС Π±ΡƒΠΊΠ²Ρ‹ E, I ΠΈ R Π² Π²ΠΈΠ΄Π΅ Ρ‚Ρ€Π΅ΡƒΠ³ΠΎΠ»ΡŒΠ½ΠΈΠΊΠ° ΡΠ»Π΅Π΄ΡƒΡŽΡ‰ΠΈΠΌ ΠΎΠ±Ρ€Π°Π·ΠΎΠΌ:

Рисунок 5 – Π’Ρ€Π΅ΡƒΠ³ΠΎΠ»ΡŒΠ½ΠΈΠΊ Π·Π°ΠΊΠΎΠ½Π° Ома

Если Π²Ρ‹ Π·Π½Π°Π΅Ρ‚Π΅ E ΠΈ I ΠΈ Ρ…ΠΎΡ‚ΠΈΡ‚Π΅ ΠΎΠΏΡ€Π΅Π΄Π΅Π»ΠΈΡ‚ΡŒ R, просто ΡƒΠ΄Π°Π»ΠΈΡ‚Π΅ R с ΠΊΠ°Ρ€Ρ‚ΠΈΠ½ΠΊΠΈ ΠΈ посмотритС, Ρ‡Ρ‚ΠΎ ΠΎΡΡ‚Π°Π»ΠΎΡΡŒ:

Рисунок 6 – Π—Π°ΠΊΠΎΠ½ Ома для опрСдСлСния R

Если Π²Ρ‹ Π·Π½Π°Π΅Ρ‚Π΅ E ΠΈ R ΠΈ Ρ…ΠΎΡ‚ΠΈΡ‚Π΅ ΠΎΠΏΡ€Π΅Π΄Π΅Π»ΠΈΡ‚ΡŒ I, ΡƒΠ΄Π°Π»ΠΈΡ‚Π΅ I ΠΈ посмотритС, Ρ‡Ρ‚ΠΎ ΠΎΡΡ‚Π°Π»ΠΎΡΡŒ:

Рисунок 7 – Π—Π°ΠΊΠΎΠ½ Ома для опрСдСлСния I

НаконСц, Ссли Π²Ρ‹ Π·Π½Π°Π΅Ρ‚Π΅ I ΠΈ R ΠΈ Ρ…ΠΎΡ‚ΠΈΡ‚Π΅ ΠΎΠΏΡ€Π΅Π΄Π΅Π»ΠΈΡ‚ΡŒ E, ΡƒΠ΄Π°Π»ΠΈΡ‚Π΅ E ΠΈ посмотритС, Ρ‡Ρ‚ΠΎ ΠΎΡΡ‚Π°Π»ΠΎΡΡŒ:

Рисунок 8 – Π—Π°ΠΊΠΎΠ½ Ома для опрСдСлСния E

Π’ ΠΊΠΎΠ½Ρ†Π΅ ΠΊΠΎΠ½Ρ†ΠΎΠ², Π²Π°ΠΌ придСтся Π½Π°ΡƒΡ‡ΠΈΡ‚ΡŒΡΡ Ρ€Π°Π±ΠΎΡ‚Π°Ρ‚ΡŒ с Ρ„ΠΎΡ€ΠΌΡƒΠ»Π°ΠΌΠΈ, Ρ‡Ρ‚ΠΎΠ±Ρ‹ ΡΠ΅Ρ€ΡŒΠ΅Π·Π½ΠΎ ΠΈΠ·ΡƒΡ‡Π°Ρ‚ΡŒ элСктричСство ΠΈ элСктронику, Π½ΠΎ этот совСт ΠΌΠΎΠΆΠ΅Ρ‚ ΠΎΠ±Π»Π΅Π³Ρ‡ΠΈΡ‚ΡŒ Π·Π°ΠΏΠΎΠΌΠΈΠ½Π°Π½ΠΈΠ΅ Π²Π°ΡˆΠΈΡ… ΠΏΠ΅Ρ€Π²Ρ‹Ρ… вычислСний. Если Π²Π°ΠΌ ΡƒΠ΄ΠΎΠ±Π½ΠΎ Ρ€Π°Π±ΠΎΡ‚Π°Ρ‚ΡŒ с Ρ„ΠΎΡ€ΠΌΡƒΠ»Π°ΠΌΠΈ, всё, Ρ‡Ρ‚ΠΎ Π²Π°ΠΌ Π½ΡƒΠΆΠ½ΠΎ ΡΠ΄Π΅Π»Π°Ρ‚ΡŒ, это Π·Π°Ρ„ΠΈΠΊΡΠΈΡ€ΠΎΠ²Π°Ρ‚ΡŒ Π² памяти E = IR ΠΈ вывСсти ΠΈΠ· Π½Π΅Π΅ Π΄Π²Π΅ Π΄Ρ€ΡƒΠ³ΠΈΠ΅ Ρ„ΠΎΡ€ΠΌΡƒΠ»Ρ‹, ΠΊΠΎΠ³Π΄Π° ΠΎΠ½ΠΈ Π²Π°ΠΌ понадобятся!

РСзюмС

  • НапряТСниС измСряСтся Π² Π²ΠΎΠ»ΡŒΡ‚Π°Ρ…, обозначаСтся Π±ΡƒΠΊΠ²Π°ΠΌΠΈ Β«EΒ» ΠΈΠ»ΠΈ Β«VΒ».
  • Π‘ΠΈΠ»Π° Ρ‚ΠΎΠΊΠ° измСряСтся Π² Π°ΠΌΠΏΠ΅Ρ€Π°Ρ…, обозначаСтся Π±ΡƒΠΊΠ²ΠΎΠΉ Β«IΒ».
  • Π‘ΠΎΠΏΡ€ΠΎΡ‚ΠΈΠ²Π»Π΅Π½ΠΈΠ΅ измСряСтся Π² ΠΎΠΌΠ°Ρ…, обозначаСтся Π±ΡƒΠΊΠ²ΠΎΠΉ Β«RΒ».
  • Π—Π°ΠΊΠΎΠ½ Ома: E = IR; I = E/R; R = E/I

ΠžΡ€ΠΈΠ³ΠΈΠ½Π°Π» ΡΡ‚Π°Ρ‚ΡŒΠΈ:

Π’Π΅Π³ΠΈ

Π—Π°ΠΊΠΎΠ½ ΠžΠΌΠ°Π—Π°Ρ€ΡΠ΄ΠšΡƒΠ»ΠΎΠ½ΠžΠ±ΡƒΡ‡Π΅Π½ΠΈΠ΅Π‘ΠΈΠ»Π° токаБопротивлСниСБхСмотСхникаЭлСктричСский токЭлСктричСскоС напряТСниС

Π‘ΠΎΡ…Ρ€Π°Π½ΠΈΡ‚ΡŒ ΠΈΠ»ΠΈ ΠΏΠΎΠ΄Π΅Π»ΠΈΡ‚ΡŒΡΡ

ΠŸΠΎΡΡ‚ΠΎΡΠ½Π½Ρ‹ΠΉ элСктричСский Ρ‚ΠΎΠΊ. Π‘ΠΈΠ»Π° Ρ‚ΠΎΠΊΠ°. НапряТСниС. ЭлСктричСскоС сопротивлСниС. Π—Π°ΠΊΠΎΠ½ Ома для участка элСктричСской Ρ†Π΅ΠΏΠΈ

1. ЭлСктричСским Ρ‚ΠΎΠΊΠΎΠΌ Π½Π°Π·Ρ‹Π²Π°ΡŽΡ‚ упорядочСнноС Π΄Π²ΠΈΠΆΠ΅Π½ΠΈΠ΅ заряТСнных частиц.

Для Ρ‚ΠΎΠ³ΠΎ Ρ‡Ρ‚ΠΎΠ±Ρ‹ Π² ΠΏΡ€ΠΎΠ²ΠΎΠ΄Π½ΠΈΠΊΠ΅ сущСствовал элСктричСский Ρ‚ΠΎΠΊ, Π½Π΅ΠΎΠ±Ρ…ΠΎΠ΄ΠΈΠΌΡ‹ Π΄Π²Π° условия: Π½Π°Π»ΠΈΡ‡ΠΈΠ΅ свободных заряТСнных частиц ΠΈ элСктричСского поля, ΠΊΠΎΡ‚ΠΎΡ€ΠΎΠ΅ создаёт ΠΈΡ… Π½Π°ΠΏΡ€Π°Π²Π»Π΅Π½Π½ΠΎΠ΅ Π΄Π²ΠΈΠΆΠ΅Π½ΠΈΠ΅.

ΠŸΡ€ΠΈ сущСствовании Ρ‚ΠΎΠΊΠ° Π² Ρ€Π°Π·Π½Ρ‹Ρ… срСдах: Π² ΠΌΠ΅Ρ‚Π°Π»Π»Π°Ρ…, Тидкостях, Π³Π°Π·Π°Ρ… β€” элСктричСский заряд пСрСносится Ρ€Π°Π·Π½Ρ‹ΠΌΠΈ частицами. Π’ ΠΌΠ΅Ρ‚Π°Π»Π»Π°Ρ… этими частицами ΡΠ²Π»ΡΡŽΡ‚ΡΡ элСктроны, Π² Тидкостях заряд пСрСносится ΠΈΠΎΠ½Π°ΠΌΠΈ, Π² Π³Π°Π·Π°Ρ… β€” элСктронами, ΠΏΠΎΠ»ΠΎΠΆΠΈΡ‚Π΅Π»ΡŒΠ½Ρ‹ΠΌΠΈ ΠΈ ΠΎΡ‚Ρ€ΠΈΡ†Π°Ρ‚Π΅Π»ΡŒΠ½Ρ‹ΠΌΠΈ ΠΈΠΎΠ½Π°ΠΌΠΈ.

Дистиллированная Π²ΠΎΠ΄Π° Π½Π΅ ΠΏΡ€ΠΎΠ²ΠΎΠ΄ΠΈΡ‚ элСктричСский Ρ‚ΠΎΠΊ, ΠΏΠΎΡΠΊΠΎΠ»ΡŒΠΊΡƒ ΠΎΠ½Π° Π½Π΅ содСрТит свободных зарядов. Если Π² Π²ΠΎΠ΄Ρƒ Π΄ΠΎΠ±Π°Π²ΠΈΡ‚ΡŒ ΠΏΠΎΠ²Π°Ρ€Π΅Π½Π½ΡƒΡŽ соль ΠΈΠ»ΠΈ ΠΌΠ΅Π΄Π½Ρ‹ΠΉ купорос, Ρ‚ΠΎ Π² Π½Π΅ΠΉ появятся свободныС заряды, ΠΈ ΠΎΠ½Π° станСт ΠΏΡ€ΠΎΠ²ΠΎΠ΄Π½ΠΈΠΊΠΎΠΌ элСктричСского Ρ‚ΠΎΠΊΠ°. Π’ растворС ΠΏΠΎΠ²Π°Ρ€Π΅Π½Π½ΠΎΠΉ соли Π² Π²ΠΎΠ΄Π΅ происходит элСктролитичСская диссоциация — процСсс разлоТСния ΠΌΠΎΠ»Π΅ΠΊΡƒΠ»Ρ‹ ΠΏΠΎΠ²Π°Ρ€Π΅Π½Π½ΠΎΠΉ соли Π½Π° ΠΏΠΎΠ»ΠΎΠΆΠΈΡ‚Π΅Π»ΡŒΠ½Ρ‹ΠΉ ΠΈΠΎΠ½ натрия ΠΈ ΠΎΡ‚Ρ€ΠΈΡ†Π°Ρ‚Π΅Π»ΡŒΠ½Ρ‹ΠΉ ΠΈΠΎΠ½ Ρ…Π»ΠΎΡ€Π°. Если Π² сосуд с раствором ΠΏΠΎΠ²Π°Ρ€Π΅Π½Π½ΠΎΠΉ соли ΠΏΠΎΠΌΠ΅ΡΡ‚ΠΈΡ‚ΡŒ Π΄Π²Π΅ мСталличСскиС пластины, соСдинённыС с источником Ρ‚ΠΎΠΊΠ° (рис. 79), Ρ‚ΠΎ ΠΏΠΎΠ»ΠΎΠΆΠΈΡ‚Π΅Π»ΡŒΠ½Ρ‹ΠΉ ΠΈΠΎΠ½ натрия Π² элСктричСском ΠΏΠΎΠ»Π΅ Π±ΡƒΠ΄Π΅Ρ‚ Π΄Π²ΠΈΠ³Π°Ρ‚ΡŒΡΡ ΠΊ пластинС, соСдинСнной с ΠΎΡ‚Ρ€ΠΈΡ†Π°Ρ‚Π΅Π»ΡŒΠ½Ρ‹ΠΌ полюсом источника Ρ‚ΠΎΠΊΠ°, Π½Π°Π·Ρ‹Π²Π°Π΅ΠΌΡ‹ΠΌ ΠΊΠ°Ρ‚ΠΎΠ΄ΠΎΠΌ, Π° ΠΎΡ‚Ρ€ΠΈΡ†Π°Ρ‚Π΅Π»ΡŒΠ½Ρ‹ΠΉ ΠΈΠΎΠ½ Ρ…Π»ΠΎΡ€Π° β€” с ΠΏΠΎΠ»ΠΎΠΆΠΈΡ‚Π΅Π»ΡŒΠ½Ρ‹ΠΌ полюсом источника Ρ‚ΠΎΠΊΠ°, Π½Π°Π·Ρ‹Π²Π°Π΅ΠΌΡ‹ΠΌ Π°Π½ΠΎΠ΄ΠΎΠΌ.

Π“Π°Π·Ρ‹ Π² ΠΎΠ±Ρ‹Ρ‡Π½Ρ‹Ρ… условиях Ρ‚ΠΎΠΆΠ΅ Π½Π΅ проводят элСктричСский Ρ‚ΠΎΠΊ, Ρ‚Π°ΠΊ ΠΊΠ°ΠΊ Π² Π½ΠΈΡ… Π½Π΅Ρ‚ свободных зарядов. Однако Ссли Π² Π²ΠΎΠ·Π΄ΡƒΡˆΠ½Ρ‹ΠΉ ΠΏΡ€ΠΎΠΌΠ΅ΠΆΡƒΡ‚ΠΎΠΊ ΠΌΠ΅ΠΆΠ΄Ρƒ двумя мСталличСскими пластинами, соСдинёнными с источником Ρ‚ΠΎΠΊΠ°, внСсти Π·Π°ΠΆΠΆΡ‘Π½Π½ΡƒΡŽ спичку ΠΈΠ»ΠΈ спиртовку, Ρ‚ΠΎ Π³Π°Π· станСт ΠΏΡ€ΠΎΠ²ΠΎΠ΄Π½ΠΈΠΊΠΎΠΌ ΠΈ Π³Π°Π»ΡŒΠ²Π°Π½ΠΎΠΌΠ΅Ρ‚Ρ€ зафиксируСт ΠΏΡ€ΠΎΡ‚Π΅ΠΊΠ°Π½ΠΈΠ΅ Ρ‚ΠΎΠΊΠ° Π½ΠΎ Ρ†Π΅ΠΏΠΈ. ΠŸΡ€ΠΈ внСсСнии ΠΏΠ»Π°ΠΌΠ΅Π½ΠΈ Π² Π²ΠΎΠ·Π΄ΡƒΡˆΠ½Ρ‹ΠΉ ΠΏΡ€ΠΎΠΌΠ΅ΠΆΡƒΡ‚ΠΎΠΊ ΠΌΠ΅ΠΆΠ΄Ρƒ пластинами происходит ионизация Π³Π°Π·Π° (рис. 80). ΠŸΡ€ΠΈ этом ΠΎΡ‚ Π°Ρ‚ΠΎΠΌΠ° Β«ΠΎΡ‚Ρ€Ρ‹Π²Π°ΡŽΡ‚ΡΡΒ» элСктроны ΠΈ образуСтся ΠΏΠΎΠ»ΠΎΠΆΠΈΡ‚Π΅Π»ΡŒΠ½Ρ‹ΠΉ ΠΈΠΎΠ½. Π’ΠΎ врСмя двиТСния элСктрон ΠΌΠΎΠΆΠ΅Ρ‚ ΠΏΡ€ΠΈΡΠΎΠ΅Π΄ΠΈΠ½ΠΈΡ‚ΡŒΡΡ ΠΊ Π½Π΅ΠΉΡ‚Ρ€Π°Π»ΡŒΠ½ΠΎΠΌΡƒ Π°Ρ‚ΠΎΠΌΡƒ ΠΈ ΠΎΠ±Ρ€Π°Π·ΠΎΠ²Π°Ρ‚ΡŒ ΠΎΡ‚Ρ€ΠΈΡ†Π°Ρ‚Π΅Π»ΡŒΠ½Ρ‹ΠΉ ΠΈΠΎΠ½. ΠŸΠΎΠ»ΠΎΠΆΠΈΡ‚Π΅Π»ΡŒΠ½Ρ‹Π΅ ΠΈΠΎΠ½Ρ‹ двиТутся ΠΊ ΠΎΡ‚Ρ€ΠΈΡ†Π°Ρ‚Π΅Π»ΡŒΠ½ΠΎΠΌΡƒ элСктроду, Π° ΠΎΡ‚Ρ€ΠΈΡ†Π°Ρ‚Π΅Π»ΡŒΠ½Ρ‹Π΅ ΠΈΠΎΠ½Ρ‹ ΠΈ элСктроны β€” ΠΊ ΠΏΠΎΠ»ΠΎΠΆΠΈΡ‚Π΅Π»ΡŒΠ½ΠΎΠΌΡƒ элСктроду.

2. НаправлСнноС Π΄Π²ΠΈΠΆΠ΅Π½ΠΈΠ΅ зарядов обСспСчиваСтся элСктричСским ΠΏΠΎΠ»Π΅ΠΌ. ЭлСктричСскоС ΠΏΠΎΠ»Π΅ Π² ΠΏΡ€ΠΎΠ²ΠΎΠ΄Π½ΠΈΠΊΠ°Ρ… создаётся ΠΈ поддСрТиваСтся источником Ρ‚ΠΎΠΊΠ°. Π’ источникС Ρ‚ΠΎΠΊΠ° ΡΠΎΠ²Π΅Ρ€ΡˆΠ°Π΅Ρ‚ΡΡ Ρ€Π°Π±ΠΎΡ‚Π° ΠΏΠΎ Ρ€Π°Π·Π΄Π΅Π»Π΅Π½ΠΈΡŽ ΠΏΠΎΠ»ΠΎΠΆΠΈΡ‚Π΅Π»ΡŒΠ½ΠΎ ΠΈ ΠΎΡ‚Ρ€ΠΈΡ†Π°Ρ‚Π΅Π»ΡŒΠ½ΠΎ заряТСнных частиц. Π­Ρ‚ΠΈ частицы Π½Π°ΠΊΠ°ΠΏΠ»ΠΈΠ²Π°ΡŽΡ‚ΡΡ Π½Π° ΠΏΠΎΠ»ΡŽΡΠ°Ρ… источника Ρ‚ΠΎΠΊΠ°. Один полюс источника заряТаСтся ΠΏΠΎΠ»ΠΎΠΆΠΈΡ‚Π΅Π»ΡŒΠ½ΠΎ, Π΄Ρ€ΡƒΠ³ΠΎΠΉ β€” ΠΎΡ‚Ρ€ΠΈΡ†Π°Ρ‚Π΅Π»ΡŒΠ½ΠΎ. ΠœΠ΅ΠΆΠ΄Ρƒ полюсами источника образуСтся элСктричСскоС ΠΏΠΎΠ»Π΅, ΠΏΠΎΠ΄ дСйствиСм ΠΊΠΎΡ‚ΠΎΡ€ΠΎΠ³ΠΎ заряТСнныС частицы Π½Π°Ρ‡ΠΈΠ½Π°ΡŽΡ‚ Π΄Π²ΠΈΠ³Π°Ρ‚ΡŒΡΡ упорядочСнно.

Π’ источникС Ρ‚ΠΎΠΊΠ° ΡΠΎΠ²Π΅Ρ€ΡˆΠ°Π΅Ρ‚ΡΡ Ρ€Π°Π±ΠΎΡ‚Π° ΠΏΡ€ΠΈ Ρ€Π°Π·Π΄Π΅Π»Π΅Π½ΠΈΠΈ заряТСнных частиц. ΠŸΡ€ΠΈ этом Ρ€Π°Π·Π»ΠΈΡ‡Π½Ρ‹Π΅ Π²ΠΈΠ΄Ρ‹ энСргии ΠΏΡ€Π΅Π²Ρ€Π°Ρ‰Π°ΡŽΡ‚ΡΡ Π² ΡΠ»Π΅ΠΊΡ‚Ρ€ΠΈΡ‡Π΅ΡΠΊΡƒΡŽ ΡΠ½Π΅Ρ€Π³ΠΈΡŽ. Π’ элСктрофорной машинС Π² ΡΠ»Π΅ΠΊΡ‚Ρ€ΠΈΡ‡Π΅ΡΠΊΡƒΡŽ ΡΠ½Π΅Ρ€Π³ΠΈΡŽ прСвращаСтся мСханичСская энСргия, Π² Π³Π°Π»ΡŒΠ²Π°Π½ΠΈΡ‡Π΅ΡΠΊΠΎΠΌ элСмСнтС β€” химичСская.

3. ЭлСктричСский Ρ‚ΠΎΠΊ, проходя ΠΏΠΎ Ρ†Π΅ΠΏΠΈ, ΠΏΡ€ΠΎΠΈΠ·Π²ΠΎΠ΄ΠΈΡ‚ Ρ€Π°Π·Π»ΠΈΡ‡Π½Ρ‹Π΅ дСйствия. Π’Π΅ΠΏΠ»ΠΎΠ²ΠΎΠ΅ дСйствиС элСктричСского Ρ‚ΠΎΠΊΠ° Π·Π°ΠΊΠ»ΡŽΡ‡Π°Π΅Ρ‚ΡΡ Π² Ρ‚ΠΎΠΌ, Ρ‡Ρ‚ΠΎ ΠΏΡ€ΠΈ Π΅Π³ΠΎ ΠΏΡ€ΠΎΡ…ΠΎΠΆΠ΄Π΅Π½ΠΈΠΈ ΠΏΠΎ ΠΏΡ€ΠΎΠ²ΠΎΠ΄Π½ΠΈΠΊΡƒ Π² Π½Ρ‘ΠΌ выдСляСтся Π½Π΅ΠΊΠΎΡ‚ΠΎΡ€ΠΎΠ΅ количСство Ρ‚Π΅ΠΏΠ»ΠΎΡ‚Ρ‹. ΠŸΡ€ΠΈΠΌΠ΅Ρ€ примСнСния Ρ‚Π΅ΠΏΠ»ΠΎΠ²ΠΎΠ³ΠΎ дСйствия Ρ‚ΠΎΠΊΠ° β€” ΡΠ»Π΅ΠΊΡ‚Ρ€ΠΎΠ½Π°Π³Ρ€Π΅Π²Π°Ρ‚Π΅Π»ΡŒΠ½Ρ‹Π΅ элСмСнты Ρ‡Π°ΠΉΠ½ΠΈΠΊΠΎΠ², элСктроплит, ΡƒΡ‚ΡŽΠ³ΠΎΠ² ΠΈ ΠΏΡ€. Π’ рядС случаСв Ρ‚Π΅ΠΌΠΏΠ΅Ρ€Π°Ρ‚ΡƒΡ€Π° ΠΏΡ€ΠΎΠ²ΠΎΠ΄Π½ΠΈΠΊΠ° нагрСваСтся Π½Π°ΡΡ‚ΠΎΠ»ΡŒΠΊΠΎ сильно, Ρ‡Ρ‚ΠΎ ΠΌΠΎΠΆΠ½ΠΎ Π½Π°Π±Π»ΡŽΠ΄Π°Ρ‚ΡŒ Π΅Π³ΠΎ свСчСниС. Π­Ρ‚ΠΎ происходит Π² элСктричСских Π»Π°ΠΌΠΏΠΎΡ‡ΠΊΠ°Ρ… накаливания.

ΠœΠ°Π³Π½ΠΈΡ‚Π½ΠΎΠ΅ дСйствиС элСктричСского Ρ‚ΠΎΠΊΠ° проявляСтся Π² Ρ‚ΠΎΠΌ, Ρ‡Ρ‚ΠΎ Π²ΠΎΠΊΡ€ΡƒΠ³ ΠΏΡ€ΠΎΠ²ΠΎΠ΄Π½ΠΈΠΊΠ° с Ρ‚ΠΎΠΊΠΎΠΌ Π²ΠΎΠ·Π½ΠΈΠΊΠ°Π΅Ρ‚ ΠΌΠ°Π³Π½ΠΈΡ‚Π½ΠΎΠ΅ ΠΏΠΎΠ»Π΅, ΠΊΠΎΡ‚ΠΎΡ€ΠΎΠ΅, дСйствуя Π½Π° ΠΌΠ°Π³Π½ΠΈΡ‚Π½ΡƒΡŽ стрСлку, Ρ€Π°ΡΠΏΠΎΠ»ΠΎΠΆΠ΅Π½Π½ΡƒΡŽ рядом с ΠΏΡ€ΠΎΠ²ΠΎΠ΄Π½ΠΈΠΊΠΎΠΌ, заставляСт Π΅Ρ‘ ΠΏΠΎΠ²ΠΎΡ€Π°Ρ‡ΠΈΠ²Π°Ρ‚ΡŒΡΡ (рис. 81).

Благодаря ΠΌΠ°Π³Π½ΠΈΡ‚Π½ΠΎΠΌΡƒ Π΄Π΅ΠΉΡΡ‚Π²ΠΈΡŽ Ρ‚ΠΎΠΊΠ° ΠΌΠΎΠΆΠ½ΠΎ ΠΏΡ€Π΅Π²Ρ€Π°Ρ‚ΠΈΡ‚ΡŒ ΠΆΠ΅Π»Π΅Π·Π½Ρ‹ΠΉ гвоздь Π² элСктромагнит, Π½Π°ΠΌΠΎΡ‚Π°Π² Π½Π° Π½Π΅Π³ΠΎ ΠΏΡ€ΠΎΠ²ΠΎΠ΄, соСдинённый с источником Ρ‚ΠΎΠΊΠ°. ΠŸΡ€ΠΈ пропускании ΠΏΠΎ ΠΏΡ€ΠΎΠ²ΠΎΠ΄Ρƒ элСктричСского Ρ‚ΠΎΠΊΠ° гвоздь Π±ΡƒΠ΄Π΅Ρ‚ ΠΏΡ€ΠΈΡ‚ΡΠ³ΠΈΠ²Π°Ρ‚ΡŒ ΠΆΠ΅Π»Π΅Π·Π½Ρ‹Π΅ ΠΏΡ€Π΅Π΄ΠΌΠ΅Ρ‚Ρ‹.

Π₯имичСскоС дСйствиС элСктричСского Ρ‚ΠΎΠΊΠ° проявляСтся Π² Ρ‚ΠΎΠΌ, Ρ‡Ρ‚ΠΎ ΠΏΡ€ΠΈ Π΅Π³ΠΎ ΠΏΡ€ΠΎΡ…ΠΎΠΆΠ΄Π΅Π½ΠΈΠΈ Π² Тидкости Π½Π° элСктродС выдСляСтся вСщСство. Если Π² стакан с раствором ΠΌΠ΅Π΄Π½ΠΎΠ³ΠΎ купороса ΠΏΠΎΠΌΠ΅ΡΡ‚ΠΈΡ‚ΡŒ ΡƒΠ³ΠΎΠ»ΡŒΠ½Ρ‹Π΅ элСктроды ΠΈ ΠΏΡ€ΠΈΡΠΎΠ΅Π΄ΠΈΠ½ΠΈΡ‚ΡŒ ΠΈΡ… ΠΊ источнику Ρ‚ΠΎΠΊΠ°, Ρ‚ΠΎ, Π²Ρ‹Π½ΡƒΠ² Ρ‡Π΅Ρ€Π΅Π· Π½Π΅ΠΊΠΎΡ‚ΠΎΡ€ΠΎΠ΅ врСмя эти элСктроды ΠΈΠ· раствора, ΠΌΠΎΠΆΠ½ΠΎ ΠΎΠ±Π½Π°Ρ€ΡƒΠΆΠΈΡ‚ΡŒ Π½Π° элСктродС, присоСдинённом ΠΊ ΠΎΡ‚Ρ€ΠΈΡ†Π°Ρ‚Π΅Π»ΡŒΠ½ΠΎΠΌΡƒ ΠΏΠΎΠ»ΡŽΡΡƒ источника (Π½Π° ΠΊΠ°Ρ‚ΠΎΠ΄Π΅), слой чистой ΠΌΠ΅Π΄ΠΈ.

Π­Ρ‚ΠΎ происходит ΠΏΠΎΡ‚ΠΎΠΌΡƒ, Ρ‡Ρ‚ΠΎ ΠΌΠ΅ΠΆΠ΄Ρƒ элСктродами сущСствуСт элСктричСскоС ΠΏΠΎΠ»Π΅, Π² ΠΊΠΎΡ‚ΠΎΡ€ΠΎΠΌ ΠΈΠΎΠ½Ρ‹ (ΠΏΠΎΠ»ΠΎΠΆΠΈΡ‚Π΅Π»ΡŒΠ½ΠΎ заряТСнныС ΠΈΠΎΠ½Ρ‹ ΠΌΠ΅Π΄ΠΈ ΠΈ ΠΎΡ‚Ρ€ΠΈΡ†Π°Ρ‚Π΅Π»ΡŒΠ½ΠΎ заряТСнныС ΠΈΠΎΠ½Ρ‹ кислотного остатка) двиТутся ΠΊ ΡΠΎΠΎΡ‚Π²Π΅Ρ‚ΡΡ‚Π²ΡƒΡŽΡ‰ΠΈΠΌ элСктродам. Достигнув ΠΎΡ‚Ρ€ΠΈΡ†Π°Ρ‚Π΅Π»ΡŒΠ½ΠΎΠ³ΠΎ элСктрода, ΠΈΠΎΠ½Ρ‹ ΠΌΠ΅Π΄ΠΈ ΠΏΠΎΠ»ΡƒΡ‡Π°ΡŽΡ‚ Π½Π΅Π΄ΠΎΡΡ‚Π°ΡŽΡ‰ΠΈΠ΅ элСктроны, ΠΏΡ€ΠΈ этом восстанавливаСтся чистая мСдь.

4. Π₯арактСристикой Ρ‚ΠΎΠΊΠ° Π² Ρ†Π΅ΠΏΠΈ слуТит Π²Π΅Π»ΠΈΡ‡ΠΈΠ½Π°, называСмая силой Ρ‚ΠΎΠΊΠ° ​\( (I) \)​. Π‘ΠΈΠ»ΠΎΠΉ Ρ‚ΠΎΠΊΠ° Π½Π°Π·Ρ‹Π²Π°ΡŽΡ‚ Ρ„ΠΈΠ·ΠΈΡ‡Π΅ΡΠΊΡƒΡŽ Π²Π΅Π»ΠΈΡ‡ΠΈΠ½Ρƒ, Ρ€Π°Π²Π½ΡƒΡŽ ΠΎΡ‚Π½ΠΎΡˆΠ΅Π½ΠΈΡŽ заряда ​\( q \)​, проходящСго Ρ‡Π΅Ρ€Π΅Π· ΠΏΠΎΠΏΠ΅Ρ€Π΅Ρ‡Π½ΠΎΠ΅ сСчСниС ΠΏΡ€ΠΎΠ²ΠΎΠ΄Π½ΠΈΠΊΠ° Π·Π° ΠΏΡ€ΠΎΠΌΠ΅ΠΆΡƒΡ‚ΠΎΠΊ Π²Ρ€Π΅ΠΌΠ΅Π½ΠΈ ​\( t \)​, ΠΊ этому ΠΏΡ€ΠΎΠΌΠ΅ΠΆΡƒΡ‚ΠΊΡƒ Π²Ρ€Π΅ΠΌΠ΅Π½ΠΈ: ​\( I=q/t \)​.

ΠžΠΏΡ€Π΅Π΄Π΅Π»Π΅Π½ΠΈΠ΅ Π΅Π΄ΠΈΠ½ΠΈΡ†Ρ‹ силы Ρ‚ΠΎΠΊΠ° основано Π½Π° ΠΌΠ°Π³Π½ΠΈΡ‚Π½ΠΎΠΌ дСйствии Ρ‚ΠΎΠΊΠ°, Π² частности Π½Π° взаимодСйствии ΠΏΠ°Ρ€Π°Π»Π»Π΅Π»ΡŒΠ½Ρ‹Ρ… ΠΏΡ€ΠΎΠ²ΠΎΠ΄Π½ΠΈΠΊΠΎΠ², ΠΏΠΎ ΠΊΠΎΡ‚ΠΎΡ€Ρ‹ΠΌ ΠΈΠ΄Ρ‘Ρ‚ элСктричСский Ρ‚ΠΎΠΊ. Π’Π°ΠΊΠΈΠ΅ ΠΏΡ€ΠΎΠ²ΠΎΠ΄Π½ΠΈΠΊΠΈ ΠΏΡ€ΠΈΡ‚ΡΠ³ΠΈΠ²Π°ΡŽΡ‚ΡΡ, Ссли Ρ‚ΠΎΠΊ ΠΏΠΎ Π½ΠΈΠΌ ΠΈΠ΄Ρ‘Ρ‚ Π² ΠΎΠ΄Π½ΠΎΠΌ Π½Π°ΠΏΡ€Π°Π²Π»Π΅Π½ΠΈΠΈ, ΠΈ ΠΎΡ‚Ρ‚Π°Π»ΠΊΠΈΠ²Π°ΡŽΡ‚ΡΡ, Ссли Π½Π°ΠΏΡ€Π°Π²Π»Π΅Π½ΠΈΠ΅ Ρ‚ΠΎΠΊΠ° Π² Π½ΠΈΡ… ΠΏΡ€ΠΎΡ‚ΠΈΠ²ΠΎΠΏΠΎΠ»ΠΎΠΆΠ½ΠΎΠ΅.

Π—Π° Π΅Π΄ΠΈΠ½ΠΈΡ†Ρƒ силы Ρ‚ΠΎΠΊΠ° ΠΏΡ€ΠΈΠ½ΠΈΠΌΠ°ΡŽΡ‚ Ρ‚Π°ΠΊΡƒΡŽ силу Ρ‚ΠΎΠΊΠ°, ΠΏΡ€ΠΈ ΠΊΠΎΡ‚ΠΎΡ€ΠΎΠΉ ΠΎΡ‚Ρ€Π΅Π·ΠΊΠΈ ΠΏΠ°Ρ€Π°Π»Π»Π΅Π»ΡŒΠ½Ρ‹Ρ… ΠΏΡ€ΠΎΠ²ΠΎΠ΄Π½ΠΈΠΊΠΎΠ² Π΄Π»ΠΈΠ½ΠΎΠΉ 1 ΠΌ, находящиСся Π½Π° расстоянии 1 ΠΌ Π΄Ρ€ΡƒΠ³ ΠΎΡ‚ Π΄Ρ€ΡƒΠ³Π°, Π²Π·Π°ΠΈΠΌΠΎΠ΄Π΅ΠΉΡΡ‚Π²ΡƒΡŽΡ‚ с силой 2Β·10-7 Н.

Π­Ρ‚Π° Π΅Π΄ΠΈΠ½ΠΈΡ†Π° называСтся Π°ΠΌΠΏΠ΅Ρ€ΠΎΠΌ (1 А).

Зная Ρ„ΠΎΡ€ΠΌΡƒΠ»Ρƒ силы Ρ‚ΠΎΠΊΠ°, ΠΌΠΎΠΆΠ½ΠΎ ΠΏΠΎΠ»ΡƒΡ‡ΠΈΡ‚ΡŒ Π΅Π΄ΠΈΠ½ΠΈΡ†Ρƒ элСктричСского заряда: 1 Кл = 1 А Β· 1 с.

5. ΠŸΡ€ΠΈΠ±ΠΎΡ€, с ΠΏΠΎΠΌΠΎΡ‰ΡŒΡŽ ΠΊΠΎΡ‚ΠΎΡ€ΠΎΠ³ΠΎ ΠΈΠ·ΠΌΠ΅Ρ€ΡΡŽΡ‚ силу Ρ‚ΠΎΠΊΠ° Π² Ρ†Π΅ΠΏΠΈ, называСтся Π°ΠΌΠΏΠ΅Ρ€ΠΌΠ΅Ρ‚Ρ€ΠΎΠΌ. Π•Π³ΠΎ Ρ€Π°Π±ΠΎΡ‚Π° основана Π½Π° ΠΌΠ°Π³Π½ΠΈΡ‚Π½ΠΎΠΌ дСйствии Ρ‚ΠΎΠΊΠ°. ΠžΡΠ½ΠΎΠ²Π½Ρ‹Π΅ части Π°ΠΌΠΏΠ΅Ρ€ΠΌΠ΅Ρ‚Ρ€Π° ΠΌΠ°Π³Π½ΠΈΡ‚ ΠΈ ΠΊΠ°Ρ‚ΡƒΡˆΠΊΠ°. ΠŸΡ€ΠΈ ΠΏΡ€ΠΎΡ…ΠΎΠΆΠ΄Π΅Π½ΠΈΠΈ ΠΏΠΎ ΠΊΠ°Ρ‚ΡƒΡˆΠΊΠ΅ элСктричСского Ρ‚ΠΎΠΊΠ° ΠΎΠ½Π° Π² Ρ€Π΅Π·ΡƒΠ»ΡŒΡ‚Π°Ρ‚Π΅ взаимодСйствия с ΠΌΠ°Π³Π½ΠΈΡ‚ΠΎΠΌ, поворачиваСтся ΠΈ ΠΏΠΎΠ²ΠΎΡ€Π°Ρ‡ΠΈΠ²Π°Π΅Ρ‚ ΡΠΎΠ΅Π΄ΠΈΠ½Ρ‘Π½Π½ΡƒΡŽ с Π½Π΅ΠΉ стрСлку. Π§Π΅ΠΌ большС сила Ρ‚ΠΎΠΊΠ°, проходящСго Ρ‡Π΅Ρ€Π΅Π· ΠΊΠ°Ρ‚ΡƒΡˆΠΊΡƒ, Ρ‚Π΅ΠΌ сильнСС ΠΎΠ½Π° взаимодСйствуСт с ΠΌΠ°Π³Π½ΠΈΡ‚ΠΎΠΌ, Ρ‚Π΅ΠΌ большС ΡƒΠ³ΠΎΠ» ΠΏΠΎΠ²ΠΎΡ€ΠΎΡ‚Π° стрСлки. АмпСрмСтр Π²ΠΊΠ»ΡŽΡ‡Π°Π΅Ρ‚ΡΡ Π² Ρ†Π΅ΠΏΡŒ ΠΏΠΎΡΠ»Π΅Π΄ΠΎΠ²Π°Ρ‚Π΅Π»ΡŒΠ½ΠΎ с Ρ‚Π΅ΠΌ ΠΏΡ€ΠΈΠ±ΠΎΡ€ΠΎΠΌ, силу Ρ‚ΠΎΠΊΠ° Π² ΠΊΠΎΡ‚ΠΎΡ€ΠΎΠΌ Π½ΡƒΠΆΠ½ΠΎ ΠΈΠ·ΠΌΠ΅Ρ€ΠΈΡ‚ΡŒ (рис. 82), ΠΈ ΠΏΠΎΡ‚ΠΎΠΌΡƒ ΠΎΠ½ ΠΈΠΌΠ΅Π΅Ρ‚ ΠΌΠ°Π»ΠΎΠ΅ Π²Π½ΡƒΡ‚Ρ€Π΅Π½Π½Π΅Π΅ сопротивлСниС, ΠΊΠΎΡ‚ΠΎΡ€ΠΎΠ΅ практичСски Π½Π΅ влияСт Π½Π° сопротивлСниС Ρ†Π΅ΠΏΠΈ ΠΈ Π½Π° силу Ρ‚ΠΎΠΊΠ° Π² Ρ†Π΅ΠΏΠΈ.

Π£ ΠΊΠ»Π΅ΠΌΠΌ Π°ΠΌΠΏΠ΅Ρ€ΠΌΠ΅Ρ‚Ρ€Π° стоят Π·Π½Π°ΠΊΠΈ Β«+Β» ΠΈ Β«-Β», ΠΏΡ€ΠΈ Π²ΠΊΠ»ΡŽΡ‡Π΅Π½ΠΈΠΈ Π°ΠΌΠΏΠ΅Ρ€ΠΌΠ΅Ρ‚Ρ€Π° Π² Ρ†Π΅ΠΏΡŒ ΠΊΠ»Π΅ΠΌΠΌΠ° со Π·Π½Π°ΠΊΠΎΠΌ Β«+Β» присоСдиняСтся ΠΊ ΠΏΠΎΠ»ΠΎΠΆΠΈΡ‚Π΅Π»ΡŒΠ½ΠΎΠΌΡƒ ΠΏΠΎΠ»ΡŽΡΡƒ источника Ρ‚ΠΎΠΊΠ°, Π° ΠΊΠ»Π΅ΠΌΠΌΠ° со Π·Π½Π°ΠΊΠΎΠΌ Β«-Β» ΠΊ ΠΎΡ‚Ρ€ΠΈΡ†Π°Ρ‚Π΅Π»ΡŒΠ½ΠΎΠΌΡƒ ΠΏΠΎΠ»ΡŽΡΡƒ источника Ρ‚ΠΎΠΊΠ°.

6. Π˜ΡΡ‚ΠΎΡ‡Π½ΠΈΠΊ Ρ‚ΠΎΠΊΠ° создаёт элСктричСскоС ΠΏΠΎΠ»Π΅, ΠΊΠΎΡ‚ΠΎΡ€ΠΎΠ΅ ΠΏΡ€ΠΈΠ²ΠΎΠ΄ΠΈΡ‚ Π² Π΄Π²ΠΈΠΆΠ΅Π½ΠΈΠ΅ элСктричСскиС заряды. Π₯арактСристикой источника Ρ‚ΠΎΠΊΠ° слуТит Π²Π΅Π»ΠΈΡ‡ΠΈΠ½Π°, называСмая напряТСниСм. Π§Π΅ΠΌ ΠΎΠ½ΠΎ большС, Ρ‚Π΅ΠΌ сильнСС созданноС ΠΈΠΌ ΠΏΠΎΠ»Π΅. НапряТСниС Ρ…Π°Ρ€Π°ΠΊΡ‚Π΅Ρ€ΠΈΠ·ΡƒΠ΅Ρ‚ Ρ€Π°Π±ΠΎΡ‚Ρƒ, ΠΊΠΎΡ‚ΠΎΡ€ΡƒΡŽ ΡΠΎΠ²Π΅Ρ€ΡˆΠ°Π΅Ρ‚ элСктричСскоС ΠΏΠΎΠ»Π΅ ΠΏΠΎ ΠΏΠ΅Ρ€Π΅ΠΌΠ΅Ρ‰Π΅Π½ΠΈΡŽ элСктричСского заряда, Ρ€Π°Π²Π½ΠΎΠ³ΠΎ 1 Кл.

НапряТСниСм ​\( U \)​ Π½Π°Π·Ρ‹Π²Π°ΡŽΡ‚ Ρ„ΠΈΠ·ΠΈΡ‡Π΅ΡΠΊΡƒΡŽ Π²Π΅Π»ΠΈΡ‡ΠΈΠ½Ρƒ, Ρ€Π°Π²Π½ΡƒΡŽ ΠΎΡ‚Π½ΠΎΡˆΠ΅Π½ΠΈΡŽ Ρ€Π°Π±ΠΎΡ‚Ρ‹ ​\( (A) \)​ элСктричСского поля ΠΏΠΎ ΠΏΠ΅Ρ€Π΅ΠΌΠ΅Ρ‰Π΅Π½ΠΈΡŽ элСктричСского заряда ΠΊ заряду ​\( (q) \)​: ​\( U=A/q \)​.

Π’ΠΎΠ·ΠΌΠΎΠΆΠ½ΠΎ Π΄Ρ€ΡƒΠ³ΠΎΠ΅ ΠΎΠΏΡ€Π΅Π΄Π΅Π»Π΅Π½ΠΈΠ΅ понятия напряТСния. Если Ρ‡ΠΈΡΠ»ΠΈΡ‚Π΅Π»ΡŒ ΠΈ Π·Π½Π°ΠΌΠ΅Π½Π°Ρ‚Π΅Π»ΡŒ Π² Ρ„ΠΎΡ€ΠΌΡƒΠ»Π΅ напряТСния ΡƒΠΌΠ½ΠΎΠΆΠΈΡ‚ΡŒ Π½Π° врСмя двиТСния заряда ​\( (t) \)​, Ρ‚ΠΎ ΠΏΠΎΠ»ΡƒΡ‡ΠΈΠΌ: ​\( U=At/qt \)​. Π’ числитСлС этой Π΄Ρ€ΠΎΠ±ΠΈ стоит ΠΌΠΎΡ‰Π½ΠΎΡΡ‚ΡŒ Ρ‚ΠΎΠΊΠ° ​\( (P) \)​, Π° Π² Π·Π½Π°ΠΌΠ΅Π½Π°Ρ‚Π΅Π»Π΅ β€” сила Ρ‚ΠΎΠΊΠ° ​\( (I) \)​: ​\( U=P/I \)​, Ρ‚.Π΅. напряТСниС β€” физичСская Π²Π΅Π»ΠΈΡ‡ΠΈΠ½Π°, равная ΠΎΡ‚Π½ΠΎΡˆΠ΅Π½ΠΈΡŽ мощности элСктричСского Ρ‚ΠΎΠΊΠ° ΠΊ силС Ρ‚ΠΎΠΊΠ° Π² Ρ†Π΅ΠΏΠΈ.

Π•Π΄ΠΈΠ½ΠΈΡ†Π° напряТСния: ​\( [U]=[A]/[q] \)​; ​\( [U] \)​ = 1 Π”ΠΆ/1 Кл = 1 Π’ (ΠΎΠ΄ΠΈΠ½ Π²ΠΎΠ»ΡŒΡ‚).

НапряТСниС ΠΈΠ·ΠΌΠ΅Ρ€ΡΡŽΡ‚ Π²ΠΎΠ»ΡŒΡ‚ΠΌΠ΅Ρ‚Ρ€ΠΎΠΌ. Он ΠΈΠΌΠ΅Π΅Ρ‚ Ρ‚Π°ΠΊΠΎΠ΅ ΠΆΠ΅ устройство, Ρ‡Ρ‚ΠΎ ΠΈ Π°ΠΌΠΏΠ΅Ρ€ΠΌΠ΅Ρ‚Ρ€ ΠΈ Ρ‚Π°ΠΊΠΎΠΉ ΠΆΠ΅ ΠΏΡ€ΠΈΠ½Ρ†ΠΈΠΏ дСйствия, Π½ΠΎ ΠΎΠ½ ΠΏΠΎΠ΄ΠΊΠ»ΡŽΡ‡Π°Π΅Ρ‚ΡΡ ΠΏΠ°Ρ€Π°Π»Π»Π΅Π»ΡŒΠ½ΠΎ Ρ‚ΠΎΠΌΡƒ участку Ρ†Π΅ΠΏΠΈ, напряТСниС Π½Π° ΠΊΠΎΡ‚ΠΎΡ€ΠΎΠΌ хотят ΠΈΠ·ΠΌΠ΅Ρ€ΠΈΡ‚ΡŒ (рис. 83). Π’Π½ΡƒΡ‚Ρ€Π΅Π½Π½Π΅Π΅ сопротивлСниС Π²ΠΎΠ»ΡŒΡ‚ΠΌΠ΅Ρ‚Ρ€Π° достаточно большоС, соотвСтствСнно проходящий Ρ‡Π΅Ρ€Π΅Π· Π½Π΅Π³ΠΎ Ρ‚ΠΎΠΊ ΠΌΠ°Π» ΠΏΠΎ ΡΡ€Π°Π²Π½Π΅Π½ΠΈΡŽ с Ρ‚ΠΎΠΊΠΎΠΌ Π² Ρ†Π΅ΠΏΠΈ.

Π£ ΠΊΠ»Π΅ΠΌΠΌ Π²ΠΎΠ»ΡŒΡ‚ΠΌΠ΅Ρ‚Ρ€Π° стоят Π·Π½Π°ΠΊΠΈ Β«+Β» ΠΈ Β«-Β», ΠΏΡ€ΠΈ Π²ΠΊΠ»ΡŽΡ‡Π΅Π½ΠΈΠΈ Π²ΠΎΠ»ΡŒΡ‚ΠΌΠ΅Ρ‚Ρ€Π° Π² Ρ†Π΅ΠΏΡŒ ΠΊΠ»Π΅ΠΌΠΌΠ° со Π·Π½Π°ΠΊΠΎΠΌ Β«+Β» присоСдиняСтся ΠΊ ΠΏΠΎΠ»ΠΎΠΆΠΈΡ‚Π΅Π»ΡŒΠ½ΠΎΠΌΡƒ ΠΏΠΎΠ»ΡŽΡΡƒ источника Ρ‚ΠΎΠΊΠ°, Π° ΠΊΠ»Π΅ΠΌΠΌΠ° со Π·Π½Π°ΠΊΠΎΠΌ Β«-Β» ΠΊ ΠΎΡ‚Ρ€ΠΈΡ†Π°Ρ‚Π΅Π»ΡŒΠ½ΠΎΠΌΡƒ ΠΏΠΎΠ»ΡŽΡΡƒ источника Ρ‚ΠΎΠΊΠ°.

7. Π‘ΠΎΠ±Ρ€Π°Π² ΡΠ»Π΅ΠΊΡ‚Ρ€ΠΈΡ‡Π΅ΡΠΊΡƒΡŽ Ρ†Π΅ΠΏΡŒ, ΡΠΎΡΡ‚ΠΎΡΡ‰ΡƒΡŽ ΠΈΠ· источника Ρ‚ΠΎΠΊΠ°, рСзистора, Π°ΠΌΠΏΠ΅Ρ€ΠΌΠ΅Ρ‚Ρ€Π°, Π²ΠΎΠ»ΡŒΡ‚ΠΌΠ΅Ρ‚Ρ€Π°, ΠΊΠ»ΡŽΡ‡Π° (рис. 83), ΠΌΠΎΠΆΠ½ΠΎ ΠΏΠΎΠΊΠ°Π·Π°Ρ‚ΡŒ, Ρ‡Ρ‚ΠΎ сила Ρ‚ΠΎΠΊΠ° ​\( (I) \)​, ΠΏΡ€ΠΎΡ‚Π΅ΠΊΠ°ΡŽΡ‰Π΅Π³ΠΎ Ρ‡Π΅Ρ€Π΅Π· рСзистор, прямо ΠΏΡ€ΠΎΠΏΠΎΡ€Ρ†ΠΈΠΎΠ½Π°Π»ΡŒΠ½Π° Π½Π°ΠΏΡ€ΡΠΆΠ΅Π½ΠΈΡŽ ​\( (U) \)​ Π½Π° Π΅Π³ΠΎ ΠΊΠΎΠ½Ρ†Π°Ρ…: ​\( I\sim U \)​. ΠžΡ‚Π½ΠΎΡˆΠ΅Π½ΠΈΠ΅ напряТСния ΠΊ силС Ρ‚ΠΎΠΊΠ° ​\( U/I \)​ β€” Π΅ΡΡ‚ΡŒ Π²Π΅Π»ΠΈΡ‡ΠΈΠ½Π° постоянная. Если Π·Π°ΠΌΠ΅Π½ΠΈΡ‚ΡŒ рСзистор, Π²ΠΊΠ»ΡŽΡ‡Ρ‘Π½Π½Ρ‹ΠΉ Π² Ρ†Π΅ΠΏΡŒ, Π΄Ρ€ΡƒΠ³ΠΈΠΌ рСзистором ΠΈ ΠΏΠΎΠ²Ρ‚ΠΎΡ€ΠΈΡ‚ΡŒ ΠΎΠΏΡ‹Ρ‚, ΠΏΠΎΠ»ΡƒΡ‡ΠΈΠΌ Ρ‚ΠΎΡ‚ ΠΆΠ΅ Ρ€Π΅Π·ΡƒΠ»ΡŒΡ‚Π°Ρ‚: сила Ρ‚ΠΎΠΊΠ° Π² рСзисторС прямо ΠΏΡ€ΠΎΠΏΠΎΡ€Ρ†ΠΈΠΎΠ½Π°Π»ΡŒΠ½Π° Π½Π°ΠΏΡ€ΡΠΆΠ΅Π½ΠΈΡŽ Π½Π° Π΅Π³ΠΎ ΠΊΠΎΠ½Ρ†Π°Ρ…, Π° ΠΎΡ‚Π½ΠΎΡˆΠ΅Π½ΠΈΠ΅ напряТСния ΠΊ силС Ρ‚ΠΎΠΊΠ° Π΅ΡΡ‚ΡŒ Π²Π΅Π»ΠΈΡ‡ΠΈΠ½Π° постоянная. Волько Π² этом случаС Π·Π½Π°Ρ‡Π΅Π½ΠΈΠ΅ ΠΎΡ‚Π½ΠΎΡˆΠ΅Π½ΠΈΡ напряТСния ΠΊ силС Ρ‚ΠΎΠΊΠ° Π±ΡƒΠ΄Π΅Ρ‚ ΠΎΡ‚Π»ΠΈΡ‡Π°Ρ‚ΡŒΡΡ ΠΎΡ‚ ΠΎΡ‚Π½ΠΎΡˆΠ΅Π½ΠΈΡ этих Π²Π΅Π»ΠΈΡ‡ΠΈΠ½ Π² ΠΏΠ΅Ρ€Π²ΠΎΠΌ ΠΎΠΏΡ‹Ρ‚Π΅. ΠŸΡ€ΠΈΡ‡ΠΈΠ½ΠΎΠΉ этого являСтся Ρ‚ΠΎ, Ρ‡Ρ‚ΠΎ Π² Ρ†Π΅ΠΏΡŒ Π²ΠΊΠ»ΡŽΡ‡Π°Π»ΠΈΡΡŒ Ρ€Π°Π·Π½Ρ‹Π΅ рСзисторы. Π‘Π»Π΅Π΄ΠΎΠ²Π°Ρ‚Π΅Π»ΡŒΠ½ΠΎ, сущСствуСт физичСская Π²Π΅Π»ΠΈΡ‡ΠΈΠ½Π°, Ρ…Π°Ρ€Π°ΠΊΡ‚Π΅Ρ€ΠΈΠ·ΡƒΡŽΡ‰Π°Ρ свойства ΠΏΡ€ΠΎΠ²ΠΎΠ΄Π½ΠΈΠΊΠ° (рСзистора), ΠΏΠΎ ΠΊΠΎΡ‚ΠΎΡ€ΠΎΠΌΡƒ Ρ‚Π΅Ρ‡Ρ‘Ρ‚ элСктричСский Ρ‚ΠΎΠΊ. Π­Ρ‚Ρƒ Π²Π΅Π»ΠΈΡ‡ΠΈΠ½Ρƒ Π½Π°Π·Ρ‹Π²Π°ΡŽΡ‚ элСктричСским сопротивлСниСм ΠΏΡ€ΠΎΠ²ΠΎΠ΄Π½ΠΈΠΊΠ°, ΠΈΠ»ΠΈ просто сопротивлСниСм. ΠžΠ±ΠΎΠ·Π½Π°Ρ‡Π°Π΅Ρ‚ΡΡ сопротивлСниС Π±ΡƒΠΊΠ²ΠΎΠΉ ​\( R \)​.

Π‘ΠΎΠΏΡ€ΠΎΡ‚ΠΈΠ²Π»Π΅Π½ΠΈΠ΅ΠΌ ΠΏΡ€ΠΎΠ²ΠΎΠ΄Π½ΠΈΠΊΠ° ​\( (R) \)​ Π½Π°Π·Ρ‹Π²Π°ΡŽΡ‚ Ρ„ΠΈΠ·ΠΈΡ‡Π΅ΡΠΊΡƒΡŽ Π²Π΅Π»ΠΈΡ‡ΠΈΠ½Ρƒ, Ρ€Π°Π²Π½ΡƒΡŽ ΠΎΡ‚Π½ΠΎΡˆΠ΅Π½ΠΈΡŽ напряТСния ​\( (U) \)​ Π½Π° ΠΊΠΎΠ½Ρ†Π°Ρ… ΠΏΡ€ΠΎΠ²ΠΎΠ΄Π½ΠΈΠΊΠ° ΠΊ силС Ρ‚ΠΎΠΊΠ° ​\( (I) \)​ Π² Π½Ρ‘ΠΌ. ​\( R=U/I \)​.

Π—Π° Π΅Π΄ΠΈΠ½ΠΈΡ†Ρƒ сопротивлСния ΠΏΡ€ΠΈΠ½ΠΈΠΌΠ°ΡŽΡ‚ Ом (1 Ом).

Один Ом β€” сопротивлСниС Ρ‚Π°ΠΊΠΎΠ³ΠΎ ΠΏΡ€ΠΎΠ²ΠΎΠ΄Π½ΠΈΠΊΠ°, Π² ΠΊΠΎΡ‚ΠΎΡ€ΠΎΠΌ сила Ρ‚ΠΎΠΊΠ° Ρ€Π°Π²Π½Π° 1 А ΠΏΡ€ΠΈ напряТСнии Π½Π° Π΅Π³ΠΎ ΠΊΠΎΠ½Ρ†Π°Ρ… 1 Π’: 1 Ом = 1 Π’/1 А.

ΠŸΡ€ΠΈΡ‡ΠΈΠ½Π° Ρ‚ΠΎΠ³ΠΎ, Ρ‡Ρ‚ΠΎ ΠΏΡ€ΠΎΠ²ΠΎΠ΄Π½ΠΈΠΊ ΠΎΠ±Π»Π°Π΄Π°Π΅Ρ‚ сопротивлСниСм, Π·Π°ΠΊΠ»ΡŽΡ‡Π°Π΅Ρ‚ΡΡ Π² Ρ‚ΠΎΠΌ, Ρ‡Ρ‚ΠΎ Π½Π°ΠΏΡ€Π°Π²Π»Π΅Π½Π½ΠΎΠΌΡƒ двиТСнию элСктричСских зарядов Π² Π½Ρ‘ΠΌ ΠΏΡ€Π΅ΠΏΡΡ‚ΡΡ‚Π²ΡƒΡŽΡ‚ ΠΈΠΎΠ½Ρ‹ кристалличСской Ρ€Π΅ΡˆΠ΅Ρ‚ΠΊΠΈ, ΡΠΎΠ²Π΅Ρ€ΡˆΠ°ΡŽΡ‰ΠΈΠ΅ бСспорядочноС Π΄Π²ΠΈΠΆΠ΅Π½ΠΈΠ΅.2}{ΠΌ} \)​.

ИзмСняя Π΄Π»ΠΈΠ½Ρƒ ΠΏΡ€ΠΎΠ²ΠΎΠ΄Π½ΠΈΠΊΠ°, Π° ΡΠ»Π΅Π΄ΠΎΠ²Π°Ρ‚Π΅Π»ΡŒΠ½ΠΎ Π΅Π³ΠΎ сопротивлСниС, ΠΌΠΎΠΆΠ½ΠΎ Ρ€Π΅Π³ΡƒΠ»ΠΈΡ€ΠΎΠ²Π°Ρ‚ΡŒ силу Ρ‚ΠΎΠΊΠ° Π² Ρ†Π΅ΠΏΠΈ. ΠŸΡ€ΠΈΠ±ΠΎΡ€, с ΠΏΠΎΠΌΠΎΡ‰ΡŒΡŽ ΠΊΠΎΡ‚ΠΎΡ€ΠΎΠ³ΠΎ это ΠΌΠΎΠΆΠ½ΠΎ ΡΠ΄Π΅Π»Π°Ρ‚ΡŒ, называСтся рСостатом (рис. 84).

9. Как ΠΏΠΎΠΊΠ°Π·Π°Π½ΠΎ Π²Ρ‹ΡˆΠ΅, сила Ρ‚ΠΎΠΊΠ° Π² ΠΏΡ€ΠΎΠ²ΠΎΠ΄Π½ΠΈΠΊΠ΅ зависит ΠΎΡ‚ напряТСния Π½Π° Π΅Π³ΠΎ ΠΊΠΎΠ½Ρ†Π°Ρ…. Если Π² ΠΎΠΏΡ‹Ρ‚Π΅ ΠΌΠ΅Π½ΡΡ‚ΡŒ ΠΏΡ€ΠΎΠ²ΠΎΠ΄Π½ΠΈΠΊΠΈ, оставляя напряТСниС Π½Π° Π½ΠΈΡ… Π½Π΅ΠΈΠ·ΠΌΠ΅Π½Π½Ρ‹ΠΌ, Ρ‚ΠΎ ΠΌΠΎΠΆΠ½ΠΎ ΠΏΠΎΠΊΠ°Π·Π°Ρ‚ΡŒ, Ρ‡Ρ‚ΠΎ ΠΏΡ€ΠΈ постоянном напряТСнии Π½Π° ΠΊΠΎΠ½Ρ†Π°Ρ… ΠΏΡ€ΠΎΠ²ΠΎΠ΄Π½ΠΈΠΊΠ° сила Ρ‚ΠΎΠΊΠ° ΠΎΠ±Ρ€Π°Ρ‚Π½ΠΎ ΠΏΡ€ΠΎΠΏΠΎΡ€Ρ†ΠΈΠΎΠ½Π°Π»ΡŒΠ½Π° Π΅Π³ΠΎ ΡΠΎΠΏΡ€ΠΎΡ‚ΠΈΠ²Π»Π΅Π½ΠΈΡŽ. ΠžΠ±ΡŠΠ΅Π΄ΠΈΠ½ΠΈΠ²Β Π·Π°Π²ΠΈΡΠΈΠΌΠΎΡΡ‚ΡŒ силы Ρ‚ΠΎΠΊΠ° ΠΎΡ‚ напряТСния ΠΈ Π΅Π³ΠΎ Π·Π°Π²ΠΈΡΠΈΠΌΠΎΡΡ‚ΡŒ ΠΎΡ‚ сопротивлСния ΠΏΡ€ΠΎΠ²ΠΎΠ΄Π½ΠΈΠΊΠ°, ΠΌΠΎΠΆΠ½ΠΎ Π·Π°ΠΏΠΈΡΠ°Ρ‚ΡŒ: ​\( I=\frac{U}{R} \)​. Π­Ρ‚ΠΎΡ‚ Π·Π°ΠΊΠΎΠ½, установлСнный ΡΠΊΡΠΏΠ΅Ρ€ΠΈΠΌΠ΅Π½Ρ‚Π°Π»ΡŒΠ½ΠΎ, называСтся Π·Π°ΠΊΠΎΠ½ΠΎΠΌ Ома (для участка Ρ†Π΅ΠΏΠΈ): сила Ρ‚ΠΎΠΊΠ° Π½Π° участкС Ρ†Π΅ΠΏΠΈ прямо ΠΏΡ€ΠΎΠΏΠΎΡ€Ρ†ΠΈΠΎΠ½Π°Π»ΡŒΠ½Π° Π½Π°ΠΏΡ€ΡΠΆΠ΅Π½ΠΈΡŽ Π½Π° ΠΊΠΎΠ½Ρ†Π°Ρ… этого участка ΠΈ ΠΎΠ±Ρ€Π°Ρ‚Π½ΠΎ ΠΏΡ€ΠΎΠΏΠΎΡ€Ρ†ΠΈΠΎΠ½Π°Π»ΡŒΠ½Π° Π΅Π³ΠΎ ΡΠΎΠΏΡ€ΠΎΡ‚ΠΈΠ²Π»Π΅Π½ΠΈΡŽ.

ΠŸΠ Π˜ΠœΠ•Π Π« Π—ΠΠ”ΠΠΠ˜Π™

Π§Π°ΡΡ‚ΡŒ 1

1. На рисункС ΠΏΡ€ΠΈΠ²Π΅Π΄Π΅Π½Π° схСма элСктричСской Ρ†Π΅ΠΏΠΈ, состоящСй ΠΈΠ· источника Ρ‚ΠΎΠΊΠ°, ΠΊΠ»ΡŽΡ‡Π° ΠΈ Π΄Π²ΡƒΡ… ΠΏΠ°Ρ€Π°Π»Π»Π΅Π»ΡŒΠ½ΠΎ соСдинённых рСзисторов. Для измСрСния напряТСния Π½Π° рСзисторС ​\( R_2 \)​ Π²ΠΎΠ»ΡŒΡ‚ΠΌΠ΅Ρ‚Ρ€ ΠΌΠΎΠΆΠ½ΠΎ Π²ΠΊΠ»ΡŽΡ‡ΠΈΡ‚ΡŒ ΠΌΠ΅ΠΆΠ΄Ρƒ Ρ‚ΠΎΡ‡ΠΊΠ°ΠΌΠΈ

1) Ρ‚ΠΎΠ»ΡŒΠΊΠΎ Π‘ ΠΈ Π’
2) Ρ‚ΠΎΠ»ΡŒΠΊΠΎ А ΠΈ Π’
3) Π‘ ΠΈ Π“ ΠΈΠ»ΠΈ Π‘ ΠΈ Π’
4) А ΠΈ Π“ ΠΈΠ»ΠΈ А ΠΈ Π’

2. На рисункС прСдставлСна элСктричСская Ρ†Π΅ΠΏΡŒ, состоящая ΠΈΠ· источника Ρ‚ΠΎΠΊΠ°, рСзистора ΠΈ Π΄Π²ΡƒΡ… Π°ΠΌΠΏΠ΅Ρ€ΠΌΠ΅Ρ‚Ρ€ΠΎΠ². Π‘ΠΈΠ»Π° Ρ‚ΠΎΠΊΠ°, показываСмая Π°ΠΌΠΏΠ΅Ρ€ΠΌΠ΅Ρ‚Ρ€ΠΎΠΌ А1, Ρ€Π°Π²Π½Π° 0,5 А. АмпСрмСтр А2 ΠΏΠΎΠΊΠ°ΠΆΠ΅Ρ‚ силу Ρ‚ΠΎΠΊΠ°

1) мСньшС 0,5 А
2) большС 0,5 А
3) 0,5 А
4) 0 А

3. Π£Ρ‡Π΅Π½ΠΈΠΊ исслСдовал Π·Π°Π²ΠΈΡΠΈΠΌΠΎΡΡ‚ΡŒ силы Ρ‚ΠΎΠΊΠ° Π² элСктроплиткС ΠΎΡ‚ ΠΏΡ€ΠΈΠ»ΠΎΠΆΠ΅Π½Π½ΠΎΠ³ΠΎ напряТСния ΠΈ ΠΏΠΎΠ»ΡƒΡ‡ΠΈΠ» ΡΠ»Π΅Π΄ΡƒΡŽΡ‰ΠΈΠ΅ Π΄Π°Π½Π½Ρ‹Π΅.

ΠŸΡ€ΠΎΠ°Π½Π°Π»ΠΈΠ·ΠΈΡ€ΠΎΠ²Π°Π² ΠΏΠΎΠ»ΡƒΡ‡Π΅Π½Π½Ρ‹Π΅ значСния, ΠΎΠ½ высказал прСдполоТСния:

А. Π—Π°ΠΊΠΎΠ½ Ома справСдлив для ΠΏΠ΅Ρ€Π²Ρ‹Ρ… Ρ‚Ρ€Ρ‘Ρ… ΠΈΠ·ΠΌΠ΅Ρ€Π΅Π½ΠΈΠΉ.
Π‘. Π—Π°ΠΊΠΎΠ½ Ома справСдлив для послСдних Ρ‚Ρ€Ρ‘Ρ… ΠΈΠ·ΠΌΠ΅Ρ€Π΅Π½ΠΈΠΉ.

Какая(-ΠΈΠ΅) ΠΈΠ· высказанных ΡƒΡ‡Π΅Π½ΠΈΠΊΠΎΠΌ Π³ΠΈΠΏΠΎΡ‚Π΅Π· Π²Π΅Ρ€Π½Π°(-Ρ‹)?

1) Ρ‚ΠΎΠ»ΡŒΠΊΠΎ А
2) Ρ‚ΠΎΠ»ΡŒΠΊΠΎ Π‘
3) ΠΈ А, ΠΈ Π‘
4) Π½ΠΈ А, Π½ΠΈ Π‘

4. На рисункС ΠΈΠ·ΠΎΠ±Ρ€Π°ΠΆΡ‘Π½ Π³Ρ€Π°Ρ„ΠΈΠΊ зависимости силы Ρ‚ΠΎΠΊΠ° Π² ΠΏΡ€ΠΎΠ²ΠΎΠ΄Π½ΠΈΠΊΠ΅ ΠΎΡ‚ напряТСния Π½Π° Π΅Π³ΠΎ ΠΊΠΎΠ½Ρ†Π°Ρ…. Π§Π΅ΠΌΡƒ Ρ€Π°Π²Π½ΠΎ сопротивлСниС ΠΏΡ€ΠΎΠ²ΠΎΠ΄Π½ΠΈΠΊΠ°?

1) 0,25 Ом
2) 2 Ом
3) 4 Ом
4) 8 Ом

5. На Π΄ΠΈΠ°Π³Ρ€Π°ΠΌΠΌΠ°Ρ… ΠΈΠ·ΠΎΠ±Ρ€Π°ΠΆΠ΅Π½Ρ‹ значСния силы Ρ‚ΠΎΠΊΠ° ΠΈ напряТСния Π½Π° ΠΊΠΎΠ½Ρ†Π°Ρ… Π΄Π²ΡƒΡ… ΠΏΡ€ΠΎΠ²ΠΎΠ΄Π½ΠΈΠΊΠΎΠ². Π‘Ρ€Π°Π²Π½ΠΈΡ‚Π΅ сопротивлСния этих ΠΏΡ€ΠΎΠ²ΠΎΠ΄Π½ΠΈΠΊΠΎΠ².

1) ​\( R_1=R_2 \)​
2) \( R_1=2R_2 \)​
3) \( R_1=4R_2 \)​
4) \( 4R_1=R_2 \)​

6. На рисункС ΠΏΡ€ΠΈΠ²Π΅Π΄Π΅Π½Π° столбчатая Π΄ΠΈΠ°Π³Ρ€Π°ΠΌΠΌΠ°. На Π½Π΅ΠΉ прСдставлСны значСния мощности Ρ‚ΠΎΠΊΠ° для Π΄Π²ΡƒΡ… ΠΏΡ€ΠΎΠ²ΠΎΠ΄Π½ΠΈΠΊΠΎΠ² (1) ΠΈ (2) ΠΎΠ΄ΠΈΠ½Π°ΠΊΠΎΠ²ΠΎΠ³ΠΎ сопротивлСния. Π‘Ρ€Π°Π²Π½ΠΈΡ‚Π΅ значСния напряТСния ​\( U_1 \)​ ΠΈ ​\( U_2 \)​ Π½Π° ΠΊΠΎΠ½Ρ†Π°Ρ… этих ΠΏΡ€ΠΎΠ²ΠΎΠ΄Π½ΠΈΠΊΠΎΠ².

1) ​\( U_2=\sqrt{3}U_1 \)​
2) \( U_1=3U_2 \)
3) \( U_2=9U_1 \)
4) \( U_2=3U_1 \)

7. НСобходимо ΡΠΊΡΠΏΠ΅Ρ€ΠΈΠΌΠ΅Π½Ρ‚Π°Π»ΡŒΠ½ΠΎ ΠΎΠ±Π½Π°Ρ€ΡƒΠΆΠΈΡ‚ΡŒ Π·Π°Π²ΠΈΡΠΈΠΌΠΎΡΡ‚ΡŒ элСктричСского сопротивлСния ΠΊΡ€ΡƒΠ³Π»ΠΎΠ³ΠΎ ΡƒΠ³ΠΎΠ»ΡŒΠ½ΠΎΠ³ΠΎ стСрТня ΠΎΡ‚ Π΅Π³ΠΎ Π΄Π»ΠΈΠ½Ρ‹. ΠšΠ°ΠΊΡƒΡŽ ΠΈΠ· ΡƒΠΊΠ°Π·Π°Π½Π½Ρ‹Ρ… ΠΏΠ°Ρ€ стСрТнСй ΠΌΠΎΠΆΠ½ΠΎ ΠΈΡΠΏΠΎΠ»ΡŒΠ·ΠΎΠ²Π°Ρ‚ΡŒ для этой Ρ†Π΅Π»ΠΈ?

1) А ΠΈ Π“
2) Π‘ ΠΈ Π’
3) Π‘ ΠΈ Π“
4) Π’ ΠΈ Π“

8. Π”Π²Π° Π°Π»ΡŽΠΌΠΈΠ½ΠΈΠ΅Π²Ρ‹Ρ… ΠΏΡ€ΠΎΠ²ΠΎΠ΄Π½ΠΈΠΊΠ° ΠΎΠ΄ΠΈΠ½Π°ΠΊΠΎΠ²ΠΎΠΉ Π΄Π»ΠΈΠ½Ρ‹ ΠΈΠΌΠ΅ΡŽΡ‚ Ρ€Π°Π·Π½ΡƒΡŽ ΠΏΠ»ΠΎΡ‰Π°Π΄ΡŒ ΠΏΠΎΠΏΠ΅Ρ€Π΅Ρ‡Π½ΠΎΠ³ΠΎ сСчСния: ΠΏΠ»ΠΎΡ‰Π°Π΄ΡŒ ΠΏΠΎΠΏΠ΅Ρ€Π΅Ρ‡Π½ΠΎΠ³ΠΎ сСчСния ΠΏΠ΅Ρ€Π²ΠΎΠ³ΠΎ ΠΏΡ€ΠΎΠ²ΠΎΠ΄Π½ΠΈΠΊΠ° 0,5 ΠΌΠΌ2, Π° Π²Ρ‚ΠΎΡ€ΠΎΠ³ΠΎ ΠΏΡ€ΠΎΠ²ΠΎΠ΄Π½ΠΈΠΊΠ° 4 ΠΌΠΌ2. Π‘ΠΎΠΏΡ€ΠΎΡ‚ΠΈΠ²Π»Π΅Π½ΠΈΠ΅ ΠΊΠ°ΠΊΠΎΠ³ΠΎ ΠΈΠ· ΠΏΡ€ΠΎΠ²ΠΎΠ΄Π½ΠΈΠΊΠΎΠ² большС ΠΈ Π²ΠΎ сколько Ρ€Π°Π·?

1) Π‘ΠΎΠΏΡ€ΠΎΡ‚ΠΈΠ²Π»Π΅Π½ΠΈΠ΅ ΠΏΠ΅Ρ€Π²ΠΎΠ³ΠΎ ΠΏΡ€ΠΎΠ²ΠΎΠ΄Π½ΠΈΠΊΠ° Π² 64 Ρ€Π°Π·Π° большС, Ρ‡Π΅ΠΌ Π²Ρ‚ΠΎΡ€ΠΎΠ³ΠΎ.
2) Π‘ΠΎΠΏΡ€ΠΎΡ‚ΠΈΠ²Π»Π΅Π½ΠΈΠ΅ ΠΏΠ΅Ρ€Π²ΠΎΠ³ΠΎ ΠΏΡ€ΠΎΠ²ΠΎΠ΄Π½ΠΈΠΊΠ° Π² 8 Ρ€Π°Π· большС, Ρ‡Π΅ΠΌ Π²Ρ‚ΠΎΡ€ΠΎΠ³ΠΎ.
3) Π‘ΠΎΠΏΡ€ΠΎΡ‚ΠΈΠ²Π»Π΅Π½ΠΈΠ΅ Π²Ρ‚ΠΎΡ€ΠΎΠ³ΠΎ ΠΏΡ€ΠΎΠ²ΠΎΠ΄Π½ΠΈΠΊΠ° Π² 64 Ρ€Π°Π·Π° большС, Ρ‡Π΅ΠΌ ΠΏΠ΅Ρ€Π²ΠΎΠ³ΠΎ.
4) Π‘ΠΎΠΏΡ€ΠΎΡ‚ΠΈΠ²Π»Π΅Π½ΠΈΠ΅ Π²Ρ‚ΠΎΡ€ΠΎΠ³ΠΎ ΠΏΡ€ΠΎΠ²ΠΎΠ΄Π½ΠΈΠΊΠ° Π² 8 Ρ€Π°Π· большС, Ρ‡Π΅ΠΌ ΠΏΠ΅Ρ€Π²ΠΎΠ³ΠΎ.

9. Π’ Ρ‚Π΅Ρ‡Π΅Π½ΠΈΠ΅ 600 с Ρ‡Π΅Ρ€Π΅Π· ΠΏΠΎΡ‚Ρ€Π΅Π±ΠΈΡ‚Π΅Π»ΡŒ элСктричСского Ρ‚ΠΎΠΊΠ° ΠΏΡ€ΠΎΡ…ΠΎΠ΄ΠΈΡ‚ заряд 12 Кл. Π§Π΅ΠΌΡƒ Ρ€Π°Π²Π½Π° сила Ρ‚ΠΎΠΊΠ° Π² ΠΏΠΎΡ‚Ρ€Π΅Π±ΠΈΡ‚Π΅Π»Π΅?

1) 0,02 А
2) 0,2 А
3) 5 А
4) 50 А

10. Π’ Ρ‚Π°Π±Π»ΠΈΡ†Π΅ ΠΏΡ€ΠΈΠ²Π΅Π΄Π΅Π½Ρ‹ Ρ€Π΅Π·ΡƒΠ»ΡŒΡ‚Π°Ρ‚Ρ‹ ΡΠΊΡΠΏΠ΅Ρ€ΠΈΠΌΠ΅Π½Ρ‚Π°Π»ΡŒΠ½Ρ‹Ρ… ΠΈΠ·ΠΌΠ΅Ρ€Π΅Π½ΠΈΠΉ ΠΏΠ»ΠΎΡ‰Π°Π΄ΠΈ ΠΏΠΎΠΏΠ΅Ρ€Π΅Ρ‡Π½ΠΎΠ³ΠΎ сСчСния ​\( S \)​, Π΄Π»ΠΈΠ½Ρ‹ ​\( L \)​ ΠΈ элСктричСского сопротивлСния ​\( R \)​ для Ρ‚Ρ€Ρ‘Ρ… ΠΏΡ€ΠΎΠ²ΠΎΠ΄Π½ΠΈΠΊΠΎΠ², ΠΈΠ·Π³ΠΎΡ‚ΠΎΠ²Π»Π΅Π½Π½Ρ‹Ρ… ΠΈΠ· ΠΆΠ΅Π»Π΅Π·Π° ΠΈΠ»ΠΈ Π½ΠΈΠΊΠ΅Π»ΠΈΠ½Π°.

На основании ΠΏΡ€ΠΎΠ²Π΅Π΄Ρ‘Π½Π½Ρ‹Ρ… ΠΈΠ·ΠΌΠ΅Ρ€Π΅Π½ΠΈΠΉ ΠΌΠΎΠΆΠ½ΠΎ ΡƒΡ‚Π²Π΅Ρ€ΠΆΠ΄Π°Ρ‚ΡŒ, Ρ‡Ρ‚ΠΎ элСктричСскоС сопротивлСниС ΠΏΡ€ΠΎΠ²ΠΎΠ΄Π½ΠΈΠΊΠ°

1) зависит ΠΎΡ‚ ΠΌΠ°Ρ‚Π΅Ρ€ΠΈΠ°Π»Π° ΠΏΡ€ΠΎΠ²ΠΎΠ΄Π½ΠΈΠΊΠ°
2) Π½Π΅ зависит ΠΎΡ‚ ΠΌΠ°Ρ‚Π΅Ρ€ΠΈΠ°Π»Π° ΠΏΡ€ΠΎΠ²ΠΎΠ΄Π½ΠΈΠΊΠ°
3) увСличиваСтся ΠΏΡ€ΠΈ ΡƒΠ²Π΅Π»ΠΈΡ‡Π΅Π½ΠΈΠΈ Π΅Π³ΠΎ Π΄Π»ΠΈΠ½Ρ‹
4) ΡƒΠΌΠ΅Π½ΡŒΡˆΠ°Π΅Ρ‚ΡΡ ΠΏΡ€ΠΈ ΡƒΠ²Π΅Π»ΠΈΡ‡Π΅Π½ΠΈΠΈ Π΅Π³ΠΎ ΠΏΠ»ΠΎΡ‰Π°Π΄ΠΈ ΠΏΠΎΠΏΠ΅Ρ€Π΅Ρ‡Π½ΠΎΠ³ΠΎ сСчСния

11. Для изготовлСния рСзисторов использовался Ρ€ΡƒΠ»ΠΎΠ½ Π½ΠΈΡ…Ρ€ΠΎΠΌΠΎΠ²ΠΎΠΉ ΠΏΡ€ΠΎΠ²ΠΎΠ»ΠΎΠΊΠΈ. ΠŸΠΎΠΎΡ‡Π΅Ρ€Π΅Π΄Π½ΠΎ Π² Ρ†Π΅ΠΏΡŒ (см. рисунок) Π²ΠΊΠ»ΡŽΡ‡Π°Π»ΠΈ ΠΎΡ‚Ρ€Π΅Π·ΠΊΠΈ ΠΏΡ€ΠΎΠ²ΠΎΠ»ΠΎΠΊΠΈ Π΄Π»ΠΈΠ½ΠΎΠΉ 4 ΠΌ, 8 ΠΌ ΠΈ 12 ΠΌ. Для ΠΊΠ°ΠΆΠ΄ΠΎΠ³ΠΎ случая ΠΈΠ·ΠΌΠ΅Ρ€ΡΠ»ΠΈΡΡŒ напряТСниС ΠΈ сила Ρ‚ΠΎΠΊΠ° (см. Ρ‚Π°Π±Π»ΠΈΡ†Ρƒ).

Какой Π²Ρ‹Π²ΠΎΠ΄ ΠΌΠΎΠΆΠ½ΠΎ ΡΠ΄Π΅Π»Π°Ρ‚ΡŒ Π½Π° основании ΠΏΡ€ΠΎΠ²Π΅Π΄Ρ‘Π½Π½Ρ‹Ρ… исслСдований?

1) сопротивлСниС ΠΏΡ€ΠΎΠ²ΠΎΠ΄Π½ΠΈΠΊΠ° ΠΎΠ±Ρ€Π°Ρ‚Π½ΠΎ ΠΏΡ€ΠΎΠΏΠΎΡ€Ρ†ΠΈΠΎΠ½Π°Π»ΡŒΠ½ΠΎ ΠΏΠ»ΠΎΡ‰Π°Π΄ΠΈ Π΅Π³ΠΎ ΠΏΠΎΠΏΠ΅Ρ€Π΅Ρ‡Π½ΠΎΠ³ΠΎ сСчСния
2) сопротивлСниС ΠΏΡ€ΠΎΠ²ΠΎΠ΄Π½ΠΈΠΊΠ° прямо ΠΏΡ€ΠΎΠΏΠΎΡ€Ρ†ΠΈΠΎΠ½Π°Π»ΡŒΠ½ΠΎ Π΅Π³ΠΎ Π΄Π»ΠΈΠ½Π΅
3) сопротивлСниС ΠΏΡ€ΠΎΠ²ΠΎΠ΄Π½ΠΈΠΊΠ° зависит ΠΎΡ‚ силы Ρ‚ΠΎΠΊΠ° Π² ΠΏΡ€ΠΎΠ²ΠΎΠ΄Π½ΠΈΠΊΠ΅
4) сопротивлСниС ΠΏΡ€ΠΎΠ²ΠΎΠ΄Π½ΠΈΠΊΠ° зависит ΠΎΡ‚ напряТСния Π½Π° ΠΊΠΎΠ½Ρ†Π°Ρ… ΠΏΡ€ΠΎΠ²ΠΎΠ΄Π½ΠΈΠΊΠ°
5) сила Ρ‚ΠΎΠΊΠ° Π² ΠΏΡ€ΠΎΠ²ΠΎΠ΄Π½ΠΈΠΊΠ΅ ΠΎΠ±Ρ€Π°Ρ‚Π½ΠΎ ΠΏΡ€ΠΎΠΏΠΎΡ€Ρ†ΠΈΠΎΠ½Π°Π»ΡŒΠ½Π° Π΅Π³ΠΎ ΡΠΎΠΏΡ€ΠΎΡ‚ΠΈΠ²Π»Π΅Π½ΠΈΡŽ

12. Π’ справочникС физичСских свойств Ρ€Π°Π·Π»ΠΈΡ‡Π½Ρ‹Ρ… ΠΌΠ°Ρ‚Π΅Ρ€ΠΈΠ°Π»ΠΎΠ² прСдставлСна ΡΠ»Π΅Π΄ΡƒΡŽΡ‰Π°Ρ Ρ‚Π°Π±Π»ΠΈΡ†Π°.

Π˜ΡΠΏΠΎΠ»ΡŒΠ·ΡƒΡ Π΄Π°Π½Π½Ρ‹Π΅ Ρ‚Π°Π±Π»ΠΈΡ†Ρ‹, Π²Ρ‹Π±Π΅Ρ€ΠΈΡ‚Π΅ ΠΈΠ· ΠΏΡ€Π΅Π΄Π»ΠΎΠΆΠ΅Π½Π½ΠΎΠ³ΠΎ пСрСчня Π΄Π²Π° Π²Π΅Ρ€Π½Ρ‹Ρ… утвСрТдСния. Π£ΠΊΠ°ΠΆΠΈΡ‚Π΅ ΠΈΡ… Π½ΠΎΠΌΠ΅Ρ€Π°.

1) ΠŸΡ€ΠΈ Ρ€Π°Π²Π½Ρ‹Ρ… Ρ€Π°Π·ΠΌΠ΅Ρ€Π°Ρ… ΠΏΡ€ΠΎΠ²ΠΎΠ΄Π½ΠΈΠΊ ΠΈΠ· алюминия Π±ΡƒΠ΄Π΅Ρ‚ ΠΈΠΌΠ΅Ρ‚ΡŒ ΠΌΠ΅Π½ΡŒΡˆΡƒΡŽ массу ΠΈ большСС элСктричСскоС сопротивлСниС ΠΏΠΎ ΡΡ€Π°Π²Π½Π΅Π½ΠΈΡŽ с ΠΏΡ€ΠΎΠ²ΠΎΠ΄Π½ΠΈΠΊΠΎΠΌ ΠΈΠ· ΠΌΠ΅Π΄ΠΈ.
2) ΠŸΡ€ΠΎΠ²ΠΎΠ΄Π½ΠΈΠΊΠΈ ΠΈΠ· Π½ΠΈΡ…Ρ€ΠΎΠΌΠ° ΠΈ Π»Π°Ρ‚ΡƒΠ½ΠΈ ΠΏΡ€ΠΈ ΠΎΠ΄ΠΈΠ½Π°ΠΊΠΎΠ²Ρ‹Ρ… Ρ€Π°Π·ΠΌΠ΅Ρ€Π°Ρ… Π±ΡƒΠ΄ΡƒΡ‚ ΠΈΠΌΠ΅Ρ‚ΡŒ ΠΎΠ΄ΠΈΠ½Π°ΠΊΠΎΠ²Ρ‹Π΅ элСктричСскиС сопротивлСния.
3) ΠŸΡ€ΠΎΠ²ΠΎΠ΄Π½ΠΈΠΊΠΈ ΠΈΠ· константана ΠΈ Π½ΠΈΠΊΠ΅Π»ΠΈΠ½Π° ΠΏΡ€ΠΈ ΠΎΠ΄ΠΈΠ½Π°ΠΊΠΎΠ²Ρ‹Ρ… Ρ€Π°Π·ΠΌΠ΅Ρ€Π°Ρ… Π±ΡƒΠ΄ΡƒΡ‚ ΠΈΠΌΠ΅Ρ‚ΡŒ Ρ€Π°Π·Π½Ρ‹Π΅ массы.
4) ΠŸΡ€ΠΈ Π·Π°ΠΌΠ΅Π½Π΅ Π½ΠΈΠΊΠ΅Π»ΠΈΠ½ΠΎΠ²ΠΎΠΉ спирали элСктроплитки Π½Π° Π½ΠΈΡ…Ρ€ΠΎΠΌΠΎΠ²ΡƒΡŽ Ρ‚Π°ΠΊΠΎΠ³ΠΎ ΠΆΠ΅ Ρ€Π°Π·ΠΌΠ΅Ρ€Π° элСктричСскоС сопротивлСниС спирали ΡƒΠΌΠ΅Π½ΡŒΡˆΠΈΡ‚ΡΡ.
5) ΠŸΡ€ΠΈ Ρ€Π°Π²Π½ΠΎΠΉ ΠΏΠ»ΠΎΡ‰Π°Π΄ΠΈ ΠΏΠΎΠΏΠ΅Ρ€Π΅Ρ‡Π½ΠΎΠ³ΠΎ сСчСния ΠΏΡ€ΠΎΠ²ΠΎΠ΄Π½ΠΈΠΊ ΠΈΠ· константана Π΄Π»ΠΈΠ½ΠΎΠΉ 4 ΠΌ Π±ΡƒΠ΄Π΅Ρ‚ ΠΈΠΌΠ΅Ρ‚ΡŒ Ρ‚Π°ΠΊΠΎΠ΅ ΠΆΠ΅ элСктричСскоС сопротивлСниС, Ρ‡Ρ‚ΠΎ ΠΈ ΠΏΡ€ΠΎΠ²ΠΎΠ΄Π½ΠΈΠΊ ΠΈΠ· Π½ΠΈΠΊΠ΅Π»ΠΈΠ½Π° Π΄Π»ΠΈΠ½ΠΎΠΉ 5 ΠΌ.

Π§Π°ΡΡ‚ΡŒ 2

13. МСняя элСктричСскоС напряТСниС Π½Π° участкС Ρ†Π΅ΠΏΠΈ, состоящСм ΠΈΠ· Π½ΠΈΠΊΠ΅Π»ΠΈΠ½ΠΎΠ²ΠΎΠ³ΠΎ ΠΏΡ€ΠΎΠ²ΠΎΠ΄Π½ΠΈΠΊΠ° Π΄Π»ΠΈΠ½ΠΎΠΉ 5 ΠΌ, ΡƒΡ‡Π΅Π½ΠΈΠΊ ΠΏΠΎΠ»ΡƒΡ‡Π΅Π½Π½Ρ‹Π΅ Π΄Π°Π½Π½Ρ‹Π΅ ΠΈΠ·ΠΌΠ΅Ρ€Π΅Π½ΠΈΠΉ силы Ρ‚ΠΎΠΊΠ° ΠΈ напряТСния записал Π² Ρ‚Π°Π±Π»ΠΈΡ†Ρƒ. Π§Π΅ΠΌΡƒ Ρ€Π°Π²Π½Π° ΠΏΠ»ΠΎΡ‰Π°Π΄ΡŒ ΠΏΠΎΠΏΠ΅Ρ€Π΅Ρ‡Π½ΠΎΠ³ΠΎ сСчСния ΠΏΡ€ΠΎΠ²ΠΎΠ΄Π½ΠΈΠΊΠ°?

ΠžΡ‚Π²Π΅Ρ‚Ρ‹

ΠŸΠΎΡΡ‚ΠΎΡΠ½Π½Ρ‹ΠΉ элСктричСский Ρ‚ΠΎΠΊ. Π‘ΠΈΠ»Π° Ρ‚ΠΎΠΊΠ°. НапряТСниС. ЭлСктричСскоС сопротивлСниС. Π—Π°ΠΊΠΎΠ½ Ома для участка элСктричСской Ρ†Π΅ΠΏΠΈ

3.4 (67.14%) 14 votes

Бвязь ΠΌΠ΅ΠΆΠ΄Ρƒ напряТСниСм, Ρ‚ΠΎΠΊΠΎΠΌ ΠΈ сопротивлСниСм

ВсС ΠΌΠ°Ρ‚Π΅Ρ€ΠΈΠ°Π»Ρ‹ состоят ΠΈΠ· Π°Ρ‚ΠΎΠΌΠΎΠ², ΠΈ всС Π°Ρ‚ΠΎΠΌΡ‹ состоят ΠΈΠ· ΠΏΡ€ΠΎΡ‚ΠΎΠ½ΠΎΠ², Π½Π΅ΠΉΡ‚Ρ€ΠΎΠ½ΠΎΠ² ΠΈ элСктронов. ΠŸΡ€ΠΎΡ‚ΠΎΠ½Ρ‹ ΠΈΠΌΠ΅ΡŽΡ‚ ΠΏΠΎΠ»ΠΎΠΆΠΈΡ‚Π΅Π»ΡŒΠ½Ρ‹ΠΉ элСктричСский заряд. НСйтроны Π½Π΅ ΠΈΠΌΠ΅ΡŽΡ‚ элСктричСского заряда (Ρ‚ΠΎ Π΅ΡΡ‚ΡŒ ΠΎΠ½ΠΈ Π½Π΅ΠΉΡ‚Ρ€Π°Π»ΡŒΠ½Ρ‹), Π° элСктроны ΠΈΠΌΠ΅ΡŽΡ‚ ΠΎΡ‚Ρ€ΠΈΡ†Π°Ρ‚Π΅Π»ΡŒΠ½Ρ‹ΠΉ элСктричСский заряд. Атомы связаны вмСстС ΠΌΠΎΡ‰Π½Ρ‹ΠΌΠΈ силами притяТСния, ΡΡƒΡ‰Π΅ΡΡ‚Π²ΡƒΡŽΡ‰ΠΈΠΌΠΈ ΠΌΠ΅ΠΆΠ΄Ρƒ ядром Π°Ρ‚ΠΎΠΌΠ° ΠΈ элСктронами Π² Π΅Π³ΠΎ внСшнСй ΠΎΠ±ΠΎΠ»ΠΎΡ‡ΠΊΠ΅.

Когда эти ΠΏΡ€ΠΎΡ‚ΠΎΠ½Ρ‹, Π½Π΅ΠΉΡ‚Ρ€ΠΎΠ½Ρ‹ ΠΈ элСктроны вмСстС Π²Π½ΡƒΡ‚Ρ€ΠΈ Π°Ρ‚ΠΎΠΌΠ°, ΠΎΠ½ΠΈ счастливы ΠΈ ΡΡ‚Π°Π±ΠΈΠ»ΡŒΠ½Ρ‹.Но Ссли ΠΌΡ‹ ΠΎΡ‚Π΄Π΅Π»ΠΈΠΌ ΠΈΡ… Π΄Ρ€ΡƒΠ³ ΠΎΡ‚ Π΄Ρ€ΡƒΠ³Π°, ΠΎΠ½ΠΈ захотят Ρ€Π΅Ρ„ΠΎΡ€ΠΌΠΈΡ€ΠΎΠ²Π°Ρ‚ΡŒΡΡ ΠΈ Π½Π°Ρ‡Π°Ρ‚ΡŒ ΠΏΡ€ΠΎΡΠ²Π»ΡΡ‚ΡŒ ΠΏΠΎΡ‚Π΅Π½Ρ†ΠΈΠ°Π» притяТСния, Π½Π°Π·Ρ‹Π²Π°Π΅ΠΌΡ‹ΠΉ Ρ€Π°Π·Π½ΠΎΡΡ‚ΡŒΡŽ ΠΏΠΎΡ‚Π΅Π½Ρ†ΠΈΠ°Π»ΠΎΠ² .

Π’Π΅ΠΏΠ΅Ρ€ΡŒ, Ссли ΠΌΡ‹ создадим Π·Π°ΠΌΠΊΠ½ΡƒΡ‚ΡƒΡŽ Ρ†Π΅ΠΏΡŒ, эти свободныС элСктроны Π½Π°Ρ‡Π½ΡƒΡ‚ Π΄Π²ΠΈΠ³Π°Ρ‚ΡŒΡΡ ΠΈ Π²ΠΎΠ·Π²Ρ€Π°Ρ‰Π°Ρ‚ΡŒΡΡ ΠΊ ΠΏΡ€ΠΎΡ‚ΠΎΠ½Π°ΠΌ ΠΈΠ·-Π·Π° ΠΈΡ… притяТСния, создавая ΠΏΠΎΡ‚ΠΎΠΊ элСктронов. Π­Ρ‚ΠΎΡ‚ ΠΏΠΎΡ‚ΠΎΠΊ элСктронов называСтся элСктричСским Ρ‚ΠΎΠΊΠΎΠΌ . Π­Π»Π΅ΠΊΡ‚Ρ€ΠΎΠ½Ρ‹ Π½Π΅ проходят свободно Ρ‡Π΅Ρ€Π΅Π· Ρ†Π΅ΠΏΡŒ, ΠΏΠΎΡΠΊΠΎΠ»ΡŒΠΊΡƒ ΠΌΠ°Ρ‚Π΅Ρ€ΠΈΠ°Π», Ρ‡Π΅Ρ€Π΅Π· ΠΊΠΎΡ‚ΠΎΡ€Ρ‹ΠΉ ΠΎΠ½ΠΈ проходят, создаСт ΠΎΠ³Ρ€Π°Π½ΠΈΡ‡Π΅Π½ΠΈΠ΅ для ΠΏΠΎΡ‚ΠΎΠΊΠ° элСктронов.Π­Ρ‚ΠΎ ΠΎΠ³Ρ€Π°Π½ΠΈΡ‡Π΅Π½ΠΈΠ΅ называСтся сопротивлСниСм .

Π’ΠΎΠ³Π΄Π° всС основныС элСктричСскиС ΠΈΠ»ΠΈ элСктронныС схСмы состоят ΠΈΠ· Ρ‚Ρ€Π΅Ρ… ΠΎΡ‚Π΄Π΅Π»ΡŒΠ½Ρ‹Ρ…, Π½ΠΎ ΠΎΡ‡Π΅Π½ΡŒ взаимосвязанных элСктричСских Π²Π΅Π»ΠΈΡ‡ΠΈΠ½, Π½Π°Π·Ρ‹Π²Π°Π΅ΠΌΡ‹Ρ…: напряТСниС (v), Ρ‚ΠΎΠΊ (i) ΠΈ сопротивлСниС (Ξ©).

ЭлСктричСскоС напряТСниС

НапряТСниС , (Π’) — это ΠΏΠΎΡ‚Π΅Π½Ρ†ΠΈΠ°Π»ΡŒΠ½Π°Ρ энСргия источника питания, сохранСнная Π² Π²ΠΈΠ΄Π΅ элСктричСского заряда. НапряТСниС ΠΌΠΎΠΆΠ½ΠΎ Ρ€Π°ΡΡΠΌΠ°Ρ‚Ρ€ΠΈΠ²Π°Ρ‚ΡŒ ΠΊΠ°ΠΊ силу, которая Ρ‚ΠΎΠ»ΠΊΠ°Π΅Ρ‚ элСктроны Ρ‡Π΅Ρ€Π΅Π· ΠΏΡ€ΠΎΠ²ΠΎΠ΄Π½ΠΈΠΊ, ΠΈ Ρ‡Π΅ΠΌ большС напряТСниС, Ρ‚Π΅ΠΌ большС Π΅Π³ΠΎ ΡΠΏΠΎΡΠΎΠ±Π½ΠΎΡΡ‚ΡŒ Β«ΠΏΡ€ΠΎΡ‚Π°Π»ΠΊΠΈΠ²Π°Ρ‚ΡŒΒ» элСктроны Ρ‡Π΅Ρ€Π΅Π· Π΄Π°Π½Π½ΡƒΡŽ Ρ†Π΅ΠΏΡŒ.ΠŸΠΎΡΠΊΠΎΠ»ΡŒΠΊΡƒ энСргия ΠΈΠΌΠ΅Π΅Ρ‚ ΡΠΏΠΎΡΠΎΠ±Π½ΠΎΡΡ‚ΡŒ ΡΠΎΠ²Π΅Ρ€ΡˆΠ°Ρ‚ΡŒ Ρ€Π°Π±ΠΎΡ‚Ρƒ, эту ΠΏΠΎΡ‚Π΅Π½Ρ†ΠΈΠ°Π»ΡŒΠ½ΡƒΡŽ ΡΠ½Π΅Ρ€Π³ΠΈΡŽ ΠΌΠΎΠΆΠ½ΠΎ ΠΎΠΏΠΈΡΠ°Ρ‚ΡŒ ΠΊΠ°ΠΊ Ρ€Π°Π±ΠΎΡ‚Ρƒ, Π½Π΅ΠΎΠ±Ρ…ΠΎΠ΄ΠΈΠΌΡƒΡŽ Π² дТоулях для пСрСмСщСния элСктронов Π² Ρ„ΠΎΡ€ΠΌΠ΅ элСктричСского Ρ‚ΠΎΠΊΠ° ΠΏΠΎ Ρ†Π΅ΠΏΠΈ ΠΎΡ‚ ΠΎΠ΄Π½ΠΎΠΉ Ρ‚ΠΎΡ‡ΠΊΠΈ ΠΈΠ»ΠΈ ΡƒΠ·Π»Π° ΠΊ Π΄Ρ€ΡƒΠ³ΠΎΠΉ.

Π’ΠΎΠ³Π΄Π° Ρ€Π°Π·Π½ΠΈΡ†Π° Π² напряТСнии ΠΌΠ΅ΠΆΠ΄Ρƒ Π»ΡŽΠ±Ρ‹ΠΌΠΈ двумя Ρ‚ΠΎΡ‡ΠΊΠ°ΠΌΠΈ, соСдинСниями ΠΈΠ»ΠΈ ΠΏΠ΅Ρ€Π΅Ρ…ΠΎΠ΄Π°ΠΌΠΈ (Π½Π°Π·Ρ‹Π²Π°Π΅ΠΌΡ‹ΠΌΠΈ ΡƒΠ·Π»Π°ΠΌΠΈ) Π² Ρ†Π΅ΠΏΠΈ извСстна ΠΊΠ°ΠΊ Ρ€Π°Π·Π½ΠΎΡΡ‚ΡŒ ΠΏΠΎΡ‚Π΅Π½Ρ†ΠΈΠ°Π»ΠΎΠ² (p.d.), ΠΎΠ±Ρ‹Ρ‡Π½ΠΎ называСмая ΠΏΠ°Π΄Π΅Π½ΠΈΠ΅ΠΌ напряТСния .

Π Π°Π·Π½ΠΈΡ†Π° ΠΏΠΎΡ‚Π΅Π½Ρ†ΠΈΠ°Π»ΠΎΠ² ΠΌΠ΅ΠΆΠ΄Ρƒ двумя Ρ‚ΠΎΡ‡ΠΊΠ°ΠΌΠΈ измСряСтся Π² Π’ΠΎΠ»ΡŒΡ‚ с символом Ρ†Π΅ΠΏΠΈ V ΠΈΠ»ΠΈ строчной Π±ΡƒΠΊΠ²ΠΎΠΉ Β«vΒ», хотя Energy , E строчная Π±ΡƒΠΊΠ²Π° Β«Π΅Β» ΠΈΠ½ΠΎΠ³Π΄Π° ΠΈΡΠΏΠΎΠ»ΡŒΠ·ΡƒΠ΅Ρ‚ΡΡ для обозначСния Π³Π΅Π½Π΅Ρ€ΠΈΡ€ΡƒΠ΅ΠΌΠΎΠΉ Π­Π”Π‘ (элСктродвиТущСй силы).Π’ΠΎΠ³Π΄Π° Ρ‡Π΅ΠΌ большС напряТСниС, Ρ‚Π΅ΠΌ большС Π΄Π°Π²Π»Π΅Π½ΠΈΠ΅ (ΠΈΠ»ΠΈ Ρ‚ΠΎΠ»ΠΊΠ°ΡŽΡ‰Π°Ρ сила) ΠΈ Ρ‚Π΅ΠΌ Π²Ρ‹ΡˆΠ΅ ΡΠΏΠΎΡΠΎΠ±Π½ΠΎΡΡ‚ΡŒ Π²Ρ‹ΠΏΠΎΠ»Π½ΡΡ‚ΡŒ Ρ€Π°Π±ΠΎΡ‚Ρƒ.

Π˜ΡΡ‚ΠΎΡ‡Π½ΠΈΠΊ постоянного напряТСния называСтся НапряТСниС постоянного Ρ‚ΠΎΠΊΠ° с напряТСниСм, ΠΊΠΎΡ‚ΠΎΡ€ΠΎΠ΅ пСриодичСски измСняСтся со Π²Ρ€Π΅ΠΌΠ΅Π½Π΅ΠΌ, называСтся напряТСниСм ΠΏΠ΅Ρ€Π΅ΠΌΠ΅Π½Π½ΠΎΠ³ΠΎ Ρ‚ΠΎΠΊΠ° . НапряТСниС измСряСтся Π² Π²ΠΎΠ»ΡŒΡ‚Π°Ρ…, ΠΏΡ€ΠΈΡ‡Π΅ΠΌ ΠΎΠ΄ΠΈΠ½ Π²ΠΎΠ»ΡŒΡ‚ опрСдСляСтся ΠΊΠ°ΠΊ элСктричСскоС Π΄Π°Π²Π»Π΅Π½ΠΈΠ΅, Π½Π΅ΠΎΠ±Ρ…ΠΎΠ΄ΠΈΠΌΠΎΠ΅ для проталкивания элСктричСского Ρ‚ΠΎΠΊΠ° Π² ΠΎΠ΄ΠΈΠ½ Π°ΠΌΠΏΠ΅Ρ€ Ρ‡Π΅Ρ€Π΅Π· сопротивлСниС Π² ΠΎΠ΄ΠΈΠ½ Ом. НапряТСния ΠΎΠ±Ρ‹Ρ‡Π½ΠΎ Π²Ρ‹Ρ€Π°ΠΆΠ°ΡŽΡ‚ΡΡ Π² Π²ΠΎΠ»ΡŒΡ‚Π°Ρ… с прСфиксами, ΠΈΡΠΏΠΎΠ»ΡŒΠ·ΡƒΠ΅ΠΌΡ‹ΠΌΠΈ для обозначСния частСй, ΠΊΡ€Π°Ρ‚Π½Ρ‹Ρ… Π½Π°ΠΏΡ€ΡΠΆΠ΅Π½ΠΈΡŽ, Ρ‚Π°ΠΊΠΈΡ… ΠΊΠ°ΠΊ ΠΌΠΈΠΊΡ€ΠΎΠ²ΠΎΠ»ΡŒΡ‚ (ΠΌΠΊΠ’ = 10 -6 Π’), ΠΌΠΈΠ»Π»ΠΈΠ²ΠΎΠ»ΡŒΡ‚ (ΠΌΠ’ = 10 -3 Π’) ΠΈΠ»ΠΈ ΠΊΠΈΠ»ΠΎΠ²ΠΎΠ»ΡŒΡ‚ (ΠΊΠ’ = 10 3 Π’).НапряТСниС ΠΌΠΎΠΆΠ΅Ρ‚ Π±Ρ‹Ρ‚ΡŒ ΠΊΠ°ΠΊ ΠΏΠΎΠ»ΠΎΠΆΠΈΡ‚Π΅Π»ΡŒΠ½Ρ‹ΠΌ, Ρ‚Π°ΠΊ ΠΈ ΠΎΡ‚Ρ€ΠΈΡ†Π°Ρ‚Π΅Π»ΡŒΠ½Ρ‹ΠΌ.

Π‘Π°Ρ‚Π°Ρ€Π΅ΠΈ ΠΈΠ»ΠΈ источники питания Π² основном ΠΈΡΠΏΠΎΠ»ΡŒΠ·ΡƒΡŽΡ‚ΡΡ для создания постоянного источника постоянного Ρ‚ΠΎΠΊΠ° (постоянного Ρ‚ΠΎΠΊΠ°), Ρ‚Π°ΠΊΠΎΠ³ΠΎ ΠΊΠ°ΠΊ 5 Π’, 12 Π’, 24 Π’ ΠΈ Ρ‚. Π”., Π’ элСктронных схСмах ΠΈ систСмах. Π’ Ρ‚ΠΎ врСмя ΠΊΠ°ΠΊ источники ΠΏΠ΅Ρ€Π΅ΠΌΠ΅Π½Π½ΠΎΠ³ΠΎ Ρ‚ΠΎΠΊΠ° (ΠΏΠ΅Ρ€Π΅ΠΌΠ΅Π½Π½ΠΎΠ³ΠΎ Ρ‚ΠΎΠΊΠ°) доступны для Π±Ρ‹Ρ‚ΠΎΠ²ΠΎΠ³ΠΎ ΠΈ ΠΏΡ€ΠΎΠΌΡ‹ΡˆΠ»Π΅Π½Π½ΠΎΠ³ΠΎ элСктроснабТСния ΠΈ освСщСния, Π° Ρ‚Π°ΠΊΠΆΠ΅ для ΠΏΠ΅Ρ€Π΅Π΄Π°Ρ‡ΠΈ элСктроэнСргии. НапряТСниС элСктросСти Π² Π‘ΠΎΠ΅Π΄ΠΈΠ½Π΅Π½Π½ΠΎΠΌ ΠšΠΎΡ€ΠΎΠ»Π΅Π²ΡΡ‚Π²Π΅ Π² настоящСС врСмя составляСт 230 Π²ΠΎΠ»ΡŒΡ‚ ΠΏΠ΅Ρ€Π΅ΠΌΠ΅Π½Π½ΠΎΠ³ΠΎ Ρ‚ΠΎΠΊΠ°. ΠΈ 110 Π²ΠΎΠ»ΡŒΡ‚ ΠΏΠ΅Ρ€Π΅ΠΌΠ΅Π½Π½ΠΎΠ³ΠΎ Ρ‚ΠΎΠΊΠ°. Π² БША.

Π­Π»Π΅ΠΊΡ‚Ρ€ΠΎΠ½Π½Ρ‹Π΅ схСмы ΠΎΠ±Ρ‰Π΅Π³ΠΎ назначСния Ρ€Π°Π±ΠΎΡ‚Π°ΡŽΡ‚ ΠΎΡ‚ Π½ΠΈΠ·ΠΊΠΎΠ²ΠΎΠ»ΡŒΡ‚Π½Ρ‹Ρ… Π±Π°Ρ‚Π°Ρ€Π΅ΠΉ постоянного Ρ‚ΠΎΠΊΠ° напряТСниСм ΠΎΡ‚ 1 Π΄ΠΎ 1 мА.5 Π’ ΠΈ 24 Π’ постоянного Ρ‚ΠΎΠΊΠ°. ΠžΠ±ΠΎΠ·Π½Π°Ρ‡Π΅Π½ΠΈΠ΅ схСмы для источника постоянного напряТСния, ΠΎΠ±Ρ‹Ρ‡Π½ΠΎ прСдставляСмоС ΠΊΠ°ΠΊ символ Π±Π°Ρ‚Π°Ρ€Π΅ΠΈ с ΠΏΠΎΠ»ΠΎΠΆΠΈΡ‚Π΅Π»ΡŒΠ½Ρ‹ΠΌ, + ΠΈ ΠΎΡ‚Ρ€ΠΈΡ†Π°Ρ‚Π΅Π»ΡŒΠ½Ρ‹ΠΌ Π·Π½Π°ΠΊΠΎΠΌ -, ΡƒΠΊΠ°Π·Ρ‹Π²Π°ΡŽΡ‰ΠΈΠΌ Π½Π°ΠΏΡ€Π°Π²Π»Π΅Π½ΠΈΠ΅ полярности. ΠžΠ±ΠΎΠ·Π½Π°Ρ‡Π΅Π½ΠΈΠ΅ схСмы для источника ΠΏΠ΅Ρ€Π΅ΠΌΠ΅Π½Π½ΠΎΠ³ΠΎ напряТСния — ΠΊΡ€ΡƒΠ³ с ΡΠΈΠ½ΡƒΡΠΎΠΈΠ΄Π°Π»ΡŒΠ½ΠΎΠΉ Π²ΠΎΠ»Π½ΠΎΠΉ Π²Π½ΡƒΡ‚Ρ€ΠΈ.

Π‘ΠΈΠΌΠ²ΠΎΠ»Ρ‹ напряТСния

МоТно ΡƒΡΡ‚Π°Π½ΠΎΠ²ΠΈΡ‚ΡŒ ΠΏΡ€ΠΎΡΡ‚ΡƒΡŽ взаимосвязь ΠΌΠ΅ΠΆΠ΄Ρƒ Π±Π°ΠΊΠΎΠΌ с Π²ΠΎΠ΄ΠΎΠΉ ΠΈ источником напряТСния. Π§Π΅ΠΌ Π²Ρ‹ΡˆΠ΅ Ρ€Π΅Π·Π΅Ρ€Π²ΡƒΠ°Ρ€ для Π²ΠΎΠ΄Ρ‹ Π½Π°Π΄ выпускным отвСрстиСм, Ρ‚Π΅ΠΌ Π²Ρ‹ΡˆΠ΅ Π΄Π°Π²Π»Π΅Π½ΠΈΠ΅ Π²ΠΎΠ΄Ρ‹ ΠΏΠΎ ΠΌΠ΅Ρ€Π΅ высвобоТдСния большСго количСства энСргии, Ρ‡Π΅ΠΌ Π²Ρ‹ΡˆΠ΅ напряТСниС, Ρ‚Π΅ΠΌ большС ΠΏΠΎΡ‚Π΅Π½Ρ†ΠΈΠ°Π»ΡŒΠ½Π°Ρ энСргия ΠΏΡ€ΠΈ высвобоТдСнии большСго количСства элСктронов.

НапряТСниС всСгда измСряСтся ΠΊΠ°ΠΊ Ρ€Π°Π·Π½ΠΈΡ†Π° ΠΌΠ΅ΠΆΠ΄Ρƒ Π»ΡŽΠ±Ρ‹ΠΌΠΈ двумя Ρ‚ΠΎΡ‡ΠΊΠ°ΠΌΠΈ Π² Ρ†Π΅ΠΏΠΈ, ΠΈ напряТСниС ΠΌΠ΅ΠΆΠ΄Ρƒ этими двумя Ρ‚ΠΎΡ‡ΠΊΠ°ΠΌΠΈ ΠΎΠ±Ρ‹Ρ‡Π½ΠΎ называСтся Β«ΠΏΠ°Π΄Π΅Π½ΠΈΠ΅ΠΌ напряТСния Β». ΠžΠ±Ρ€Π°Ρ‚ΠΈΡ‚Π΅ Π²Π½ΠΈΠΌΠ°Π½ΠΈΠ΅, Ρ‡Ρ‚ΠΎ напряТСниС ΠΌΠΎΠΆΠ΅Ρ‚ ΡΡƒΡ‰Π΅ΡΡ‚Π²ΠΎΠ²Π°Ρ‚ΡŒ Π² Ρ†Π΅ΠΏΠΈ Π±Π΅Π· Ρ‚ΠΎΠΊΠ°, Π½ΠΎ Ρ‚ΠΎΠΊ Π½Π΅ ΠΌΠΎΠΆΠ΅Ρ‚ ΡΡƒΡ‰Π΅ΡΡ‚Π²ΠΎΠ²Π°Ρ‚ΡŒ Π±Π΅Π· напряТСния, ΠΈ поэтому любой источник напряТСния, Π±ΡƒΠ΄ΡŒ Ρ‚ΠΎ постоянный ΠΈΠ»ΠΈ ΠΏΠ΅Ρ€Π΅ΠΌΠ΅Π½Π½Ρ‹ΠΉ Ρ‚ΠΎΠΊ, Π»ΡŽΠ±ΠΈΡ‚ состояниС Ρ€Π°Π·ΠΎΠΌΠΊΠ½ΡƒΡ‚ΠΎΠΉ ΠΈΠ»ΠΈ ΠΏΠΎΠ»ΡƒΠΎΡ‚ΠΊΡ€Ρ‹Ρ‚ΠΎΠΉ Ρ†Π΅ΠΏΠΈ, Π½ΠΎ Π½Π΅Π½Π°Π²ΠΈΠ΄ΠΈΡ‚ Π»ΡŽΠ±Ρ‹Π΅ условия ΠΊΠΎΡ€ΠΎΡ‚ΠΊΠΎΠ³ΠΎ замыкания, ΠΏΠΎΡΠΊΠΎΠ»ΡŒΠΊΡƒ это ΠΌΠΎΠΆΠ΅Ρ‚ Π΅Π³ΠΎ Ρ€Π°Π·Ρ€ΡƒΡˆΠΈΡ‚ΡŒ.

ЭлСктричСский Ρ‚ΠΎΠΊ

ЭлСктричСский Ρ‚ΠΎΠΊ , (I) — это Π΄Π²ΠΈΠΆΠ΅Π½ΠΈΠ΅ ΠΈΠ»ΠΈ ΠΏΠΎΡ‚ΠΎΠΊ элСктричСского заряда ΠΈ измСряСтся Π² АмпСр , символ i , для силы ).Π­Ρ‚ΠΎ Π½Π΅ΠΏΡ€Π΅Ρ€Ρ‹Π²Π½Ρ‹ΠΉ ΠΈ Ρ€Π°Π²Π½ΠΎΠΌΠ΅Ρ€Π½Ρ‹ΠΉ ΠΏΠΎΡ‚ΠΎΠΊ (Π½Π°Π·Ρ‹Π²Π°Π΅ΠΌΡ‹ΠΉ Π΄Ρ€Π΅ΠΉΡ„ΠΎΠΌ) элСктронов (ΠΎΡ‚Ρ€ΠΈΡ†Π°Ρ‚Π΅Π»ΡŒΠ½Ρ‹Ρ… частиц Π°Ρ‚ΠΎΠΌΠ°) Π²ΠΎΠΊΡ€ΡƒΠ³ Ρ†Π΅ΠΏΠΈ, ΠΊΠΎΡ‚ΠΎΡ€Ρ‹Π΅ Β«ΠΏΠΎΠ΄Ρ‚Π°Π»ΠΊΠΈΠ²Π°ΡŽΡ‚ΡΡΒ» источником напряТСния. Π’ Π΄Π΅ΠΉΡΡ‚Π²ΠΈΡ‚Π΅Π»ΡŒΠ½ΠΎΡΡ‚ΠΈ, элСктроны Ρ‚Π΅ΠΊΡƒΡ‚ ΠΎΡ‚ ΠΎΡ‚Ρ€ΠΈΡ†Π°Ρ‚Π΅Π»ΡŒΠ½ΠΎΠ³ΠΎ (–ve) Π²Ρ‹Π²ΠΎΠ΄Π° ΠΊ ΠΏΠΎΠ»ΠΎΠΆΠΈΡ‚Π΅Π»ΡŒΠ½ΠΎΠΌΡƒ (+ ve) Π²Ρ‹Π²ΠΎΠ΄Ρƒ источника питания, ΠΈ для простоты понимания схСмы ΠΎΠ±Ρ‹Ρ‡Π½Ρ‹ΠΉ ΠΏΠΎΡ‚ΠΎΠΊ Ρ‚ΠΎΠΊΠ° ΠΏΡ€Π΅Π΄ΠΏΠΎΠ»Π°Π³Π°Π΅Ρ‚, Ρ‡Ρ‚ΠΎ Ρ‚ΠΎΠΊ Ρ‚Π΅Ρ‡Π΅Ρ‚ ΠΎΡ‚ ΠΏΠΎΠ»ΠΎΠΆΠΈΡ‚Π΅Π»ΡŒΠ½ΠΎΠ³ΠΎ Π²Ρ‹Π²ΠΎΠ΄Π° ΠΊ ΠΎΡ‚Ρ€ΠΈΡ†Π°Ρ‚Π΅Π»ΡŒΠ½ΠΎΠΌΡƒ.

ΠžΠ±Ρ‹Ρ‡Π½ΠΎ Π½Π° ΠΏΡ€ΠΈΠ½Ρ†ΠΈΠΏΠΈΠ°Π»ΡŒΠ½Ρ‹Ρ… схСмах Ρƒ ΠΏΠΎΡ‚ΠΎΠΊΠ° Ρ‚ΠΎΠΊΠ° Ρ‡Π΅Ρ€Π΅Π· Ρ†Π΅ΠΏΡŒ ΠΎΠ±Ρ‹Ρ‡Π½ΠΎ Π΅ΡΡ‚ΡŒ стрСлка, связанная с символом I, ΠΈΠ»ΠΈ строчная Π±ΡƒΠΊΠ²Π° i, ΡƒΠΊΠ°Π·Ρ‹Π²Π°ΡŽΡ‰Π°Ρ фактичСскоС Π½Π°ΠΏΡ€Π°Π²Π»Π΅Π½ΠΈΠ΅ Ρ‚ΠΎΠΊΠ°.Однако эта стрСлка ΠΎΠ±Ρ‹Ρ‡Π½ΠΎ ΡƒΠΊΠ°Π·Ρ‹Π²Π°Π΅Ρ‚ Π½Π°ΠΏΡ€Π°Π²Π»Π΅Π½ΠΈΠ΅ ΠΎΠ±Ρ‹Ρ‡Π½ΠΎΠ³ΠΎ ΠΏΠΎΡ‚ΠΎΠΊΠ° Ρ‚ΠΎΠΊΠ°, Π° Π½Π΅ ΠΎΠ±ΡΠ·Π°Ρ‚Π΅Π»ΡŒΠ½ΠΎ Π½Π°ΠΏΡ€Π°Π²Π»Π΅Π½ΠΈΠ΅ фактичСского ΠΏΠΎΡ‚ΠΎΠΊΠ°.

ΠžΠ±Ρ‹Ρ‡Π½Ρ‹ΠΉ Ρ‚ΠΎΠΊ

ΠžΠ±Ρ‹Ρ‡Π½ΠΎ это ΠΏΠΎΡ‚ΠΎΠΊ ΠΏΠΎΠ»ΠΎΠΆΠΈΡ‚Π΅Π»ΡŒΠ½ΠΎΠ³ΠΎ заряда ΠΏΠΎ Ρ†Π΅ΠΏΠΈ, пСрСходящий ΠΎΡ‚ ΠΏΠΎΠ»ΠΎΠΆΠΈΡ‚Π΅Π»ΡŒΠ½ΠΎΠ³ΠΎ ΠΊ ΠΎΡ‚Ρ€ΠΈΡ†Π°Ρ‚Π΅Π»ΡŒΠ½ΠΎΠΌΡƒ. На схСмС слСва ΠΏΠΎΠΊΠ°Π·Π°Π½ΠΎ Π΄Π²ΠΈΠΆΠ΅Π½ΠΈΠ΅ ΠΏΠΎΠ»ΠΎΠΆΠΈΡ‚Π΅Π»ΡŒΠ½ΠΎΠ³ΠΎ заряда (отвСрстий) ΠΏΠΎ Π·Π°ΠΌΠΊΠ½ΡƒΡ‚ΠΎΠΉ Ρ†Π΅ΠΏΠΈ, ΠΏΡ€ΠΎΡ‚Π΅ΠΊΠ°ΡŽΡ‰Π΅Π³ΠΎ ΠΎΡ‚ ΠΏΠΎΠ»ΠΎΠΆΠΈΡ‚Π΅Π»ΡŒΠ½ΠΎΠΉ ΠΊΠ»Π΅ΠΌΠΌΡ‹ Π±Π°Ρ‚Π°Ρ€Π΅ΠΈ Ρ‡Π΅Ρ€Π΅Π· Ρ†Π΅ΠΏΡŒ ΠΈ Π²ΠΎΠ·Π²Ρ€Π°Ρ‰Π°ΡŽΡ‰Π΅Π³ΠΎΡΡ ΠΊ ΠΎΡ‚Ρ€ΠΈΡ†Π°Ρ‚Π΅Π»ΡŒΠ½ΠΎΠΉ ΠΊΠ»Π΅ΠΌΠΌΠ΅ Π±Π°Ρ‚Π°Ρ€Π΅ΠΈ.Π­Ρ‚ΠΎΡ‚ ΠΏΠΎΡ‚ΠΎΠΊ Ρ‚ΠΎΠΊΠ° ΠΎΡ‚ ΠΏΠΎΠ»ΠΎΠΆΠΈΡ‚Π΅Π»ΡŒΠ½ΠΎΠ³ΠΎ ΠΊ ΠΎΡ‚Ρ€ΠΈΡ†Π°Ρ‚Π΅Π»ΡŒΠ½ΠΎΠΌΡƒ ΠΎΠ±Ρ‹Ρ‡Π½ΠΎ извСстСн ΠΊΠ°ΠΊ ΠΎΠ±Ρ‹Ρ‡Π½Ρ‹ΠΉ ΠΏΠΎΡ‚ΠΎΠΊ Ρ‚ΠΎΠΊΠ°.

Π­Ρ‚ΠΎ Π±Ρ‹Π»ΠΎ соглашСниС, Π²Ρ‹Π±Ρ€Π°Π½Π½ΠΎΠ΅ ΠΏΡ€ΠΈ ΠΎΡ‚ΠΊΡ€Ρ‹Ρ‚ΠΈΠΈ элСктричСства, ΠΏΡ€ΠΈ ΠΊΠΎΡ‚ΠΎΡ€ΠΎΠΌ элСктричСский Ρ‚ΠΎΠΊ считался Π½Π°ΠΏΡ€Π°Π²Π»Π΅Π½Π½Ρ‹ΠΌ Π² Ρ†Π΅ΠΏΠΈ. Π§Ρ‚ΠΎΠ±Ρ‹ ΠΏΡ€ΠΎΠ΄ΠΎΠ»ΠΆΠΈΡ‚ΡŒ эту ΠΌΡ‹ΡΠ»ΡŒ, Π½Π° всСх ΠΏΡ€ΠΈΠ½Ρ†ΠΈΠΏΠΈΠ°Π»ΡŒΠ½Ρ‹Ρ… схСмах ΠΈ схСмах стрСлки, ΠΏΠΎΠΊΠ°Π·Π°Π½Π½Ρ‹Π΅ Π½Π° символах для Ρ‚Π°ΠΊΠΈΡ… ΠΊΠΎΠΌΠΏΠΎΠ½Π΅Π½Ρ‚ΠΎΠ², ΠΊΠ°ΠΊ Π΄ΠΈΠΎΠ΄Ρ‹ ΠΈ транзисторы, ΡƒΠΊΠ°Π·Ρ‹Π²Π°ΡŽΡ‚ Π² Π½Π°ΠΏΡ€Π°Π²Π»Π΅Π½ΠΈΠΈ ΠΎΠ±Ρ‹Ρ‡Π½ΠΎΠ³ΠΎ Ρ‚ΠΎΠΊΠ°.

Π—Π°Ρ‚Π΅ΠΌ ΠžΠ±Ρ‹Ρ‡Π½Ρ‹ΠΉ ΠΏΠΎΡ‚ΠΎΠΊ Ρ‚ΠΎΠΊΠ° опрСдСляСт ΠΏΠΎΡ‚ΠΎΠΊ элСктричСского Ρ‚ΠΎΠΊΠ° ΠΎΡ‚ ΠΏΠΎΠ»ΠΎΠΆΠΈΡ‚Π΅Π»ΡŒΠ½ΠΎΠ³ΠΎ ΠΊ ΠΎΡ‚Ρ€ΠΈΡ†Π°Ρ‚Π΅Π»ΡŒΠ½ΠΎΠΌΡƒ, ΠΏΡ€ΠΎΡ‚ΠΈΠ²ΠΎΠΏΠΎΠ»ΠΎΠΆΠ½Ρ‹ΠΉ ΠΏΠΎ Π½Π°ΠΏΡ€Π°Π²Π»Π΅Π½ΠΈΡŽ фактичСскому ΠΏΠΎΡ‚ΠΎΠΊΡƒ элСктронов.

Π­Π»Π΅ΠΊΡ‚Ρ€ΠΎΠ½Π½Ρ‹ΠΉ ΠΏΠΎΡ‚ΠΎΠΊ

ΠŸΠΎΡ‚ΠΎΠΊ элСктронов ΠΏΠΎ Ρ†Π΅ΠΏΠΈ ΠΏΡ€ΠΎΡ‚ΠΈΠ²ΠΎΠΏΠΎΠ»ΠΎΠΆΠ΅Π½ Π½Π°ΠΏΡ€Π°Π²Π»Π΅Π½ΠΈΡŽ ΠΎΠ±Ρ‹Ρ‡Π½ΠΎΠ³ΠΎ ΠΏΠΎΡ‚ΠΎΠΊΠ° Ρ‚ΠΎΠΊΠ° — ΠΎΡ‚ ΠΎΡ‚Ρ€ΠΈΡ†Π°Ρ‚Π΅Π»ΡŒΠ½ΠΎΠ³ΠΎ ΠΊ ΠΏΠΎΠ»ΠΎΠΆΠΈΡ‚Π΅Π»ΡŒΠ½ΠΎΠΌΡƒ. ЀактичСский Ρ‚ΠΎΠΊ, ΠΏΡ€ΠΎΡ‚Π΅ΠΊΠ°ΡŽΡ‰ΠΈΠΉ Π² элСктричСской Ρ†Π΅ΠΏΠΈ, состоит ΠΈΠ· элСктронов, ΠΊΠΎΡ‚ΠΎΡ€Ρ‹Π΅ Ρ‚Π΅ΠΊΡƒΡ‚ ΠΎΡ‚ ΠΎΡ‚Ρ€ΠΈΡ†Π°Ρ‚Π΅Π»ΡŒΠ½ΠΎΠ³ΠΎ полюса Π±Π°Ρ‚Π°Ρ€Π΅ΠΈ (ΠΊΠ°Ρ‚ΠΎΠ΄Π°) ΠΈ Π²ΠΎΠ·Π²Ρ€Π°Ρ‰Π°ΡŽΡ‚ΡΡ ΠΎΠ±Ρ€Π°Ρ‚Π½ΠΎ. ΠΎΠ±Ρ€Π°Ρ‚Π½ΠΎ ΠΊ ΠΏΠΎΠ»ΠΎΠΆΠΈΡ‚Π΅Π»ΡŒΠ½ΠΎΠΌΡƒ ΠΏΠΎΠ»ΡŽΡΡƒ (Π°Π½ΠΎΠ΄Ρƒ) Π±Π°Ρ‚Π°Ρ€Π΅ΠΈ.

Π­Ρ‚ΠΎ происходит ΠΏΠΎΡ‚ΠΎΠΌΡƒ, Ρ‡Ρ‚ΠΎ заряд элСктрона ΠΏΠΎ ΠΎΠΏΡ€Π΅Π΄Π΅Π»Π΅Π½ΠΈΡŽ ΠΎΡ‚Ρ€ΠΈΡ†Π°Ρ‚Π΅Π»ΡŒΠ½Ρ‹ΠΉ, поэтому ΠΎΠ½ притягиваСтся ΠΊ ΠΏΠΎΠ»ΠΎΠΆΠΈΡ‚Π΅Π»ΡŒΠ½ΠΎΠΌΡƒ ΠΏΠΎΠ»ΡŽΡΡƒ.Π­Ρ‚ΠΎΡ‚ ΠΏΠΎΡ‚ΠΎΠΊ элСктронов называСтся Π­Π»Π΅ΠΊΡ‚Ρ€ΠΎΠ½Π½Ρ‹ΠΉ ΠΏΠΎΡ‚ΠΎΠΊ . Π’Π°ΠΊΠΈΠΌ ΠΎΠ±Ρ€Π°Π·ΠΎΠΌ, элСктроны фактичСски Ρ‚Π΅ΠΊΡƒΡ‚ ΠΏΠΎ Ρ†Π΅ΠΏΠΈ ΠΎΡ‚ ΠΎΡ‚Ρ€ΠΈΡ†Π°Ρ‚Π΅Π»ΡŒΠ½ΠΎΠ³ΠΎ Π²Ρ‹Π²ΠΎΠ΄Π° ΠΊ ΠΏΠΎΠ»ΠΎΠΆΠΈΡ‚Π΅Π»ΡŒΠ½ΠΎΠΌΡƒ.

И ΠΎΠ±Ρ‹Ρ‡Π½ΠΎΠ³ΠΎ Ρ‚ΠΎΠΊΠ° , ΠΈ элСктронного ΠΏΠΎΡ‚ΠΎΠΊΠ° ΠΈΡΠΏΠΎΠ»ΡŒΠ·ΡƒΡŽΡ‚ΡΡ Π²ΠΎ ΠΌΠ½ΠΎΠ³ΠΈΡ… ΡƒΡ‡Π΅Π±Π½ΠΈΠΊΠ°Ρ…. ЀактичСски, Π½Π΅ ΠΈΠΌΠ΅Π΅Ρ‚ значСния, Π² ΠΊΠ°ΠΊΠΎΠΌ Π½Π°ΠΏΡ€Π°Π²Π»Π΅Π½ΠΈΠΈ Ρ‚ΠΎΠΊ Ρ‚Π΅Ρ‡Π΅Ρ‚ ΠΏΠΎ Ρ†Π΅ΠΏΠΈ, Ссли Π½Π°ΠΏΡ€Π°Π²Π»Π΅Π½ΠΈΠ΅ ΠΈΡΠΏΠΎΠ»ΡŒΠ·ΡƒΠ΅Ρ‚ΡΡ ΠΏΠΎΡΠ»Π΅Π΄ΠΎΠ²Π°Ρ‚Π΅Π»ΡŒΠ½ΠΎ. НаправлСниС Ρ‚ΠΎΠΊΠ° Π½Π΅ влияСт Π½Π° Ρ‚ΠΎ, Ρ‡Ρ‚ΠΎ Ρ‚ΠΎΠΊ Π΄Π΅Π»Π°Π΅Ρ‚ Π² Ρ†Π΅ΠΏΠΈ.Как ΠΏΡ€Π°Π²ΠΈΠ»ΠΎ, Π³ΠΎΡ€Π°Π·Π΄ΠΎ ΠΏΡ€ΠΎΡ‰Π΅ ΠΏΠΎΠ½ΡΡ‚ΡŒ ΠΎΠ±Ρ‹Ρ‡Π½Ρ‹ΠΉ Ρ‚ΠΎΠΊ — ΠΎΡ‚ ΠΏΠΎΠ»ΠΎΠΆΠΈΡ‚Π΅Π»ΡŒΠ½ΠΎΠ³ΠΎ Π΄ΠΎ ΠΎΡ‚Ρ€ΠΈΡ†Π°Ρ‚Π΅Π»ΡŒΠ½ΠΎΠ³ΠΎ.

Π’ элСктронных схСмах источник Ρ‚ΠΎΠΊΠ° — это элСмСнт схСмы, ΠΊΠΎΡ‚ΠΎΡ€Ρ‹ΠΉ обСспСчиваСт Π·Π°Π΄Π°Π½Π½ΠΎΠ΅ количСство Ρ‚ΠΎΠΊΠ°, Π½Π°ΠΏΡ€ΠΈΠΌΠ΅Ρ€ 1 А, 5 А, 10 А ΠΈ Ρ‚. Π”., ΠŸΡ€ΠΈ этом символ Ρ†Π΅ΠΏΠΈ для источника постоянного Ρ‚ΠΎΠΊΠ° прСдставлСн Π² Π²ΠΈΠ΄Π΅ ΠΊΡ€ΡƒΠ³Π° со стрСлкой Π²Π½ΡƒΡ‚Ρ€ΠΈ, ΡƒΠΊΠ°Π·Ρ‹Π²Π°ΡŽΡ‰Π΅ΠΉ Π΅Π³ΠΎ Π½Π°ΠΏΡ€Π°Π²Π»Π΅Π½ΠΈΠ΅.

Π’ΠΎΠΊ измСряСтся Π² А. , Π° Π°ΠΌΠΏΠ΅Ρ€ ΠΈΠ»ΠΈ Π°ΠΌΠΏΠ΅Ρ€ опрСдСляСтся ΠΊΠ°ΠΊ количСство элСктронов ΠΈΠ»ΠΈ заряда (Q Π² ΠΊΡƒΠ»ΠΎΠ½Π°Ρ…), проходящих Ρ‡Π΅Ρ€Π΅Π· ΠΎΠΏΡ€Π΅Π΄Π΅Π»Π΅Π½Π½ΡƒΡŽ Ρ‚ΠΎΡ‡ΠΊΡƒ Π² Ρ†Π΅ΠΏΠΈ Π·Π° ΠΎΠ΄Π½Ρƒ сСкунду (t Π² сСкундах).

ЭлСктричСский Ρ‚ΠΎΠΊ ΠΎΠ±Ρ‹Ρ‡Π½ΠΎ выраТаСтся Π² Π°ΠΌΠΏΠ΅Ρ€Π°Ρ… с прСфиксом, ΠΈΡΠΏΠΎΠ»ΡŒΠ·ΡƒΠ΅ΠΌΡ‹ΠΌ для обозначСния ΠΌΠΈΠΊΡ€ΠΎΠ°ΠΌΠΏΠ΅Ρ€ (мкА = 10 -6 A) ΠΈΠ»ΠΈ ΠΌΠΈΠ»Π»ΠΈΠ°ΠΌΠΏΠ΅Ρ€ (мА = 10 -3 A). ΠžΠ±Ρ€Π°Ρ‚ΠΈΡ‚Π΅ Π²Π½ΠΈΠΌΠ°Π½ΠΈΠ΅, Ρ‡Ρ‚ΠΎ элСктричСский Ρ‚ΠΎΠΊ ΠΌΠΎΠΆΠ΅Ρ‚ Π±Ρ‹Ρ‚ΡŒ ΠΊΠ°ΠΊ ΠΏΠΎΠ»ΠΎΠΆΠΈΡ‚Π΅Π»ΡŒΠ½Ρ‹ΠΌ, Ρ‚Π°ΠΊ ΠΈ ΠΎΡ‚Ρ€ΠΈΡ†Π°Ρ‚Π΅Π»ΡŒΠ½Ρ‹ΠΌ, Π² зависимости ΠΎΡ‚ направлСния Π΅Π³ΠΎ протСкания ΠΏΠΎ Ρ†Π΅ΠΏΠΈ.

Π’ΠΎΠΊ, ΠΏΡ€ΠΎΡ‚Π΅ΠΊΠ°ΡŽΡ‰ΠΈΠΉ Π² ΠΎΠ΄Π½ΠΎΠΌ Π½Π°ΠΏΡ€Π°Π²Π»Π΅Π½ΠΈΠΈ, называСтся ΠŸΠΎΡΡ‚ΠΎΡΠ½Π½Ρ‹ΠΉ Ρ‚ΠΎΠΊ ΠΈΠ»ΠΈ D.C. , Π° Ρ‚ΠΎΠΊ, ΠΊΠΎΡ‚ΠΎΡ€Ρ‹ΠΉ ΠΏΠΎΠΏΠ΅Ρ€Π΅ΠΌΠ΅Π½Π½ΠΎ пСрСмСщаСтся ΠΏΠΎ Ρ†Π΅ΠΏΠΈ, извСстСн ΠΊΠ°ΠΊ ΠŸΠ΅Ρ€Π΅ΠΌΠ΅Π½Π½Ρ‹ΠΉ Ρ‚ΠΎΠΊ, ΠΈΠ»ΠΈ А.Π‘. . НСзависимо ΠΎΡ‚ Ρ‚ΠΎΠ³ΠΎ, ΠΏΡ€ΠΎΡ‚Π΅ΠΊΠ°Π΅Ρ‚ Π»ΠΈ ΠΏΠ΅Ρ€Π΅ΠΌΠ΅Π½Π½Ρ‹ΠΉ ΠΈΠ»ΠΈ постоянный Ρ‚ΠΎΠΊ Ρ‡Π΅Ρ€Π΅Π· Ρ†Π΅ΠΏΡŒ, Ρ‚ΠΎΠ»ΡŒΠΊΠΎ ΠΊΠΎΠ³Π΄Π° ΠΊ Π½Π΅ΠΉ ΠΏΠΎΠ΄ΠΊΠ»ΡŽΡ‡Π΅Π½ источник напряТСния, ΠΈ Π΅Π³ΠΎ Β«ΠΏΠΎΡ‚ΠΎΠΊΒ» ΠΎΠ³Ρ€Π°Π½ΠΈΡ‡Π΅Π½ ΠΊΠ°ΠΊ сопротивлСниСм Ρ†Π΅ΠΏΠΈ, Ρ‚Π°ΠΊ ΠΈ Ρ‚ΠΎΠ»ΠΊΠ°ΡŽΡ‰ΠΈΠΌ Π΅Π΅ источником напряТСния.

ΠšΡ€ΠΎΠΌΠ΅ Ρ‚ΠΎΠ³ΠΎ, ΠΏΠΎΡΠΊΠΎΠ»ΡŒΠΊΡƒ ΠΏΠ΅Ρ€Π΅ΠΌΠ΅Π½Π½Ρ‹Π΅ Ρ‚ΠΎΠΊΠΈ (ΠΈ напряТСния) ΡΠ²Π»ΡΡŽΡ‚ΡΡ пСриодичСскими ΠΈ ΠΌΠ΅Π½ΡΡŽΡ‚ΡΡ Π²ΠΎ Π²Ρ€Π΅ΠΌΠ΅Π½ΠΈ, «эффСктивноС» ΠΈΠ»ΠΈ «срСднСквадратичноС» Π·Π½Π°Ρ‡Π΅Π½ΠΈΠ΅, Π·Π°Π΄Π°Π½Π½ΠΎΠ΅ ΠΊΠ°ΠΊ I Π΄Π΅ΠΉΡΡ‚Π²ΡƒΡŽΡ‰Π΅Π΅ Π·Π½Π°Ρ‡Π΅Π½ΠΈΠ΅ , Π΄Π°Π΅Ρ‚ Ρ‚Π°ΠΊΡƒΡŽ ​​ТС ΡΡ€Π΅Π΄Π½ΡŽΡŽ ΠΏΠΎΡ‚Π΅Ρ€ΡŽ мощности, ΡΠΊΠ²ΠΈΠ²Π°Π»Π΅Π½Ρ‚Π½ΡƒΡŽ постоянному Ρ‚ΠΎΠΊΡƒ I Π² срСднСм . Π˜ΡΡ‚ΠΎΡ‡Π½ΠΈΠΊΠΈ Ρ‚ΠΎΠΊΠ° ΠΏΡ€ΠΎΡ‚ΠΈΠ²ΠΎΠΏΠΎΠ»ΠΎΠΆΠ½Ρ‹ источникам напряТСния Π² Ρ‚ΠΎΠΌ, Ρ‡Ρ‚ΠΎ ΠΎΠ½ΠΈ Π»ΡŽΠ±ΡΡ‚ условия ΠΊΠΎΡ€ΠΎΡ‚ΠΊΠΎΠ³ΠΎ замыкания ΠΈΠ»ΠΈ Π·Π°ΠΌΠΊΠ½ΡƒΡ‚ΠΎΠΉ Ρ†Π΅ΠΏΠΈ, Π½ΠΎ нСнавидят условия Ρ€Π°Π·ΠΎΠΌΠΊΠ½ΡƒΡ‚ΠΎΠΉ Ρ†Π΅ΠΏΠΈ, ΠΏΠΎΡΠΊΠΎΠ»ΡŒΠΊΡƒ Ρ‚ΠΎΠΊ Π½Π΅ Π±ΡƒΠ΄Π΅Ρ‚ Ρ‚Π΅Ρ‡ΡŒ.

Π˜ΡΠΏΠΎΠ»ΡŒΠ·ΡƒΡ ΡΠΎΠΎΡ‚Π½ΠΎΡˆΠ΅Π½ΠΈΠ΅ Β«Ρ€Π΅Π·Π΅Ρ€Π²ΡƒΠ°Ρ€ Π²ΠΎΠ΄Ρ‹Β», Ρ‚ΠΎΠΊ эквивалСнтСн ΠΏΠΎΡ‚ΠΎΠΊΡƒ Π²ΠΎΠ΄Ρ‹ ΠΏΠΎ Ρ‚Ρ€ΡƒΠ±Π΅, ΠΏΡ€ΠΈ этом ΠΏΠΎΡ‚ΠΎΠΊ ΠΎΠ΄ΠΈΠ½Π°ΠΊΠΎΠ² ΠΏΠΎ всСй Ρ‚Ρ€ΡƒΠ±Π΅. Π§Π΅ΠΌ быстрСС Ρ‚Π΅Ρ‡Π΅Ρ‚ Π²ΠΎΠ΄Π°, Ρ‚Π΅ΠΌ сильнСС Ρ‚Π΅Ρ‡Π΅Π½ΠΈΠ΅. ΠžΠ±Ρ€Π°Ρ‚ΠΈΡ‚Π΅ Π²Π½ΠΈΠΌΠ°Π½ΠΈΠ΅, Ρ‡Ρ‚ΠΎ Ρ‚ΠΎΠΊ Π½Π΅ ΠΌΠΎΠΆΠ΅Ρ‚ ΡΡƒΡ‰Π΅ΡΡ‚Π²ΠΎΠ²Π°Ρ‚ΡŒ Π±Π΅Π· напряТСния, поэтому любой источник Ρ‚ΠΎΠΊΠ°, Π±ΡƒΠ΄ΡŒ Ρ‚ΠΎ постоянный ΠΈΠ»ΠΈ ΠΏΠ΅Ρ€Π΅ΠΌΠ΅Π½Π½Ρ‹ΠΉ Ρ‚ΠΎΠΊ, Π»ΡŽΠ±ΠΈΡ‚ состояниС ΠΊΠΎΡ€ΠΎΡ‚ΠΊΠΎΠ³ΠΎ ΠΈΠ»ΠΈ ΠΏΠΎΠ»ΡƒΠΊΠΎΡ€ΠΎΡ‚ΠΊΠΎΠ³ΠΎ замыкания, Π½ΠΎ Π½Π΅Π½Π°Π²ΠΈΠ΄ΠΈΡ‚ любоС состояниС Ρ€Π°Π·ΠΎΠΌΠΊΠ½ΡƒΡ‚ΠΎΠΉ Ρ†Π΅ΠΏΠΈ, ΠΏΠΎΡΠΊΠΎΠ»ΡŒΠΊΡƒ это прСпятствуСт Π΅Π³ΠΎ Ρ‚Π΅Ρ‡Π΅Π½ΠΈΡŽ.

Π‘ΠΎΠΏΡ€ΠΎΡ‚ΠΈΠ²Π»Π΅Π½ΠΈΠ΅

Π‘ΠΎΠΏΡ€ΠΎΡ‚ΠΈΠ²Π»Π΅Π½ΠΈΠ΅ , (R) — это ΡΠΏΠΎΡΠΎΠ±Π½ΠΎΡΡ‚ΡŒ ΠΌΠ°Ρ‚Π΅Ρ€ΠΈΠ°Π»Π° ΡΠΎΠΏΡ€ΠΎΡ‚ΠΈΠ²Π»ΡΡ‚ΡŒΡΡ ΠΈΠ»ΠΈ ΠΏΡ€Π΅Π΄ΠΎΡ‚Π²Ρ€Π°Ρ‰Π°Ρ‚ΡŒ ΠΏΡ€ΠΎΡ‚Π΅ΠΊΠ°Π½ΠΈΠ΅ Ρ‚ΠΎΠΊΠ° ΠΈΠ»ΠΈ, Π±ΠΎΠ»Π΅Π΅ ΠΊΠΎΠ½ΠΊΡ€Π΅Ρ‚Π½ΠΎ, ΠΏΡ€ΠΎΡ‚Π΅ΠΊΠ°Π½ΠΈΠ΅ элСктричСского заряда Π² Ρ†Π΅ΠΏΠΈ.Π­Π»Π΅ΠΌΠ΅Π½Ρ‚ схСмы, ΠΊΠΎΡ‚ΠΎΡ€Ρ‹ΠΉ Π΄Π΅Π»Π°Π΅Ρ‚ это идСально, называСтся «рСзистор».

Π‘ΠΎΠΏΡ€ΠΎΡ‚ΠΈΠ²Π»Π΅Π½ΠΈΠ΅ — это элСмСнт схСмы, измСряСмый Π² Ом , грСчСский символ ( Ом , ОмСга) с прСфиксами, ΠΈΡΠΏΠΎΠ»ΡŒΠ·ΡƒΠ΅ΠΌΡ‹ΠΌΠΈ для обозначСния ΠΊΠΈΠ»ΠΎΠΎΠΌΠΎΠ² (кОм = 10 3 Ом) ΠΈ МОм (МОм = 10). 6 Ом). ΠžΠ±Ρ€Π°Ρ‚ΠΈΡ‚Π΅ Π²Π½ΠΈΠΌΠ°Π½ΠΈΠ΅, Ρ‡Ρ‚ΠΎ сопротивлСниС Π½Π΅ ΠΌΠΎΠΆΠ΅Ρ‚ Π±Ρ‹Ρ‚ΡŒ ΠΎΡ‚Ρ€ΠΈΡ†Π°Ρ‚Π΅Π»ΡŒΠ½Ρ‹ΠΌ ΠΏΠΎ Π²Π΅Π»ΠΈΡ‡ΠΈΠ½Π΅, Ρ‚ΠΎΠ»ΡŒΠΊΠΎ ΠΏΠΎΠ»ΠΎΠΆΠΈΡ‚Π΅Π»ΡŒΠ½Ρ‹ΠΌ.

Π‘ΠΈΠΌΠ²ΠΎΠ»Ρ‹ рСзисторов

Π’Π΅Π»ΠΈΡ‡ΠΈΠ½Π° сопротивлСния рСзистора опрСдСляСтся ΠΎΡ‚Π½ΠΎΡˆΠ΅Π½ΠΈΠ΅ΠΌ ΠΏΡ€ΠΎΡ‚Π΅ΠΊΠ°ΡŽΡ‰Π΅Π³ΠΎ Ρ‡Π΅Ρ€Π΅Π· Π½Π΅Π³ΠΎ Ρ‚ΠΎΠΊΠ° ΠΊ Π½Π°ΠΏΡ€ΡΠΆΠ΅Π½ΠΈΡŽ Π½Π° Π½Π΅ΠΌ, ΠΊΠΎΡ‚ΠΎΡ€ΠΎΠ΅ опрСдСляСт, являСтся Π»ΠΈ элСмСнт схСмы Β«Ρ…ΠΎΡ€ΠΎΡˆΠΈΠΌ ΠΏΡ€ΠΎΠ²ΠΎΠ΄Π½ΠΈΠΊΠΎΠΌΒ» — с Π½ΠΈΠ·ΠΊΠΈΠΌ сопротивлСниСм, ΠΈΠ»ΠΈ Β«ΠΏΠ»ΠΎΡ…ΠΈΠΌ ΠΏΡ€ΠΎΠ²ΠΎΠ΄Π½ΠΈΠΊΠΎΠΌΒ» — с высоким сопротивлСниСм.НизкоС сопротивлСниС, Π½Π°ΠΏΡ€ΠΈΠΌΠ΅Ρ€ 1 Ом ΠΈΠ»ΠΈ ΠΌΠ΅Π½Π΅Π΅, ΠΎΠ·Π½Π°Ρ‡Π°Π΅Ρ‚, Ρ‡Ρ‚ΠΎ Ρ†Π΅ΠΏΡŒ являСтся Ρ…ΠΎΡ€ΠΎΡˆΠΈΠΌ ΠΏΡ€ΠΎΠ²ΠΎΠ΄Π½ΠΈΠΊΠΎΠΌ, сдСланным ΠΈΠ· Ρ‚Π°ΠΊΠΈΡ… ΠΌΠ°Ρ‚Π΅Ρ€ΠΈΠ°Π»ΠΎΠ², ΠΊΠ°ΠΊ мСдь, алюминий ΠΈΠ»ΠΈ ΡƒΠ³Π»Π΅Ρ€ΠΎΠ΄, Π² Ρ‚ΠΎ врСмя ΠΊΠ°ΠΊ высокоС сопротивлСниС, 1 МОм ΠΈΠ»ΠΈ Π±ΠΎΠ»Π΅Π΅, ΠΎΠ·Π½Π°Ρ‡Π°Π΅Ρ‚, Ρ‡Ρ‚ΠΎ Ρ†Π΅ΠΏΡŒ являСтся ΠΏΠ»ΠΎΡ…ΠΈΠΌ ΠΏΡ€ΠΎΠ²ΠΎΠ΄Π½ΠΈΠΊΠΎΠΌ, сдСланным ΠΈΠ· изоляционных ΠΌΠ°Ρ‚Π΅Ρ€ΠΈΠ°Π»ΠΎΠ², Ρ‚Π°ΠΊΠΈΡ… ΠΊΠ°ΠΊ стСкло, Ρ„Π°Ρ€Ρ„ΠΎΡ€. ΠΈΠ»ΠΈ пластик.

Π‘ Π΄Ρ€ΡƒΠ³ΠΎΠΉ стороны, Β«ΠΏΠΎΠ»ΡƒΠΏΡ€ΠΎΠ²ΠΎΠ΄Π½ΠΈΠΊΒ», Ρ‚Π°ΠΊΠΎΠΉ ΠΊΠ°ΠΊ ΠΊΡ€Π΅ΠΌΠ½ΠΈΠΉ ΠΈΠ»ΠΈ Π³Π΅Ρ€ΠΌΠ°Π½ΠΈΠΉ, — это ΠΌΠ°Ρ‚Π΅Ρ€ΠΈΠ°Π», сопротивлСниС ΠΊΠΎΡ‚ΠΎΡ€ΠΎΠ³ΠΎ находится посСрСдинС ΠΌΠ΅ΠΆΠ΄Ρƒ сопротивлСниСм Ρ…ΠΎΡ€ΠΎΡˆΠ΅Π³ΠΎ ΠΏΡ€ΠΎΠ²ΠΎΠ΄Π½ΠΈΠΊΠ° ΠΈ Ρ…ΠΎΡ€ΠΎΡˆΠ΅Π³ΠΎ изолятора. ΠžΡ‚ΡΡŽΠ΄Π° ΠΈ Π½Π°Π·Π²Π°Π½ΠΈΠ΅ Β«ΠΏΠΎΠ»ΡƒΠΏΡ€ΠΎΠ²ΠΎΠ΄Π½ΠΈΠΊΒ». ΠŸΠΎΠ»ΡƒΠΏΡ€ΠΎΠ²ΠΎΠ΄Π½ΠΈΠΊΠΈ ΠΈΡΠΏΠΎΠ»ΡŒΠ·ΡƒΡŽΡ‚ΡΡ для изготовлСния Π΄ΠΈΠΎΠ΄ΠΎΠ², транзисторов ΠΈ Ρ‚. Π”.

Π‘ΠΎΠΏΡ€ΠΎΡ‚ΠΈΠ²Π»Π΅Π½ΠΈΠ΅ ΠΌΠΎΠΆΠ΅Ρ‚ Π±Ρ‹Ρ‚ΡŒ Π»ΠΈΠ½Π΅ΠΉΠ½Ρ‹ΠΌ ΠΈΠ»ΠΈ Π½Π΅Π»ΠΈΠ½Π΅ΠΉΠ½Ρ‹ΠΌ ΠΏΠΎ своСй ΠΏΡ€ΠΈΡ€ΠΎΠ΄Π΅, Π½ΠΎ Π½Π΅ ΠΎΡ‚Ρ€ΠΈΡ†Π°Ρ‚Π΅Π»ΡŒΠ½Ρ‹ΠΌ. Π›ΠΈΠ½Π΅ΠΉΠ½ΠΎΠ΅ сопротивлСниС подчиняСтся Π·Π°ΠΊΠΎΠ½Ρƒ Ома, ΠΏΠΎΡΠΊΠΎΠ»ΡŒΠΊΡƒ напряТСниС Π½Π° рСзисторС Π»ΠΈΠ½Π΅ΠΉΠ½ΠΎ ΠΏΡ€ΠΎΠΏΠΎΡ€Ρ†ΠΈΠΎΠ½Π°Π»ΡŒΠ½ΠΎ ΠΏΡ€ΠΎΡ‚Π΅ΠΊΠ°ΡŽΡ‰Π΅ΠΌΡƒ Ρ‡Π΅Ρ€Π΅Π· Π½Π΅Π³ΠΎ Ρ‚ΠΎΠΊΡƒ. НСлинСйноС сопротивлСниС Π½Π΅ подчиняСтся Π·Π°ΠΊΠΎΠ½Ρƒ Ома, Π½ΠΎ ΠΈΠΌΠ΅Π΅Ρ‚ ΠΏΠ°Π΄Π΅Π½ΠΈΠ΅ напряТСния Π½Π° Π½Π΅ΠΌ, ΠΏΡ€ΠΎΠΏΠΎΡ€Ρ†ΠΈΠΎΠ½Π°Π»ΡŒΠ½ΠΎΠ΅ Π½Π΅ΠΊΠΎΡ‚ΠΎΡ€ΠΎΠΉ мощности Ρ‚ΠΎΠΊΠ°.

Π‘ΠΎΠΏΡ€ΠΎΡ‚ΠΈΠ²Π»Π΅Π½ΠΈΠ΅ являСтся чистым ΠΈ Π½Π΅ зависит ΠΎΡ‚ частоты, ΠΏΡ€ΠΈ этом ΠΏΠΎΠ»Π½ΠΎΠ΅ сопротивлСниС ΠΏΠ΅Ρ€Π΅ΠΌΠ΅Π½Π½ΠΎΠ³ΠΎ Ρ‚ΠΎΠΊΠ° сопротивлСния Ρ€Π°Π²Π½ΠΎ Π΅Π³ΠΎ ΡΠΎΠΏΡ€ΠΎΡ‚ΠΈΠ²Π»Π΅Π½ΠΈΡŽ постоянному Ρ‚ΠΎΠΊΡƒ ΠΈ, ΡΠ»Π΅Π΄ΠΎΠ²Π°Ρ‚Π΅Π»ΡŒΠ½ΠΎ, Π½Π΅ ΠΌΠΎΠΆΠ΅Ρ‚ Π±Ρ‹Ρ‚ΡŒ ΠΎΡ‚Ρ€ΠΈΡ†Π°Ρ‚Π΅Π»ΡŒΠ½Ρ‹ΠΌ.ΠŸΠΎΠΌΠ½ΠΈΡ‚Π΅, Ρ‡Ρ‚ΠΎ сопротивлСниС всСгда ΠΏΠΎΠ»ΠΎΠΆΠΈΡ‚Π΅Π»ΡŒΠ½ΠΎΠ΅ ΠΈ Π½ΠΈΠΊΠΎΠ³Π΄Π° Π½Π΅ ΠΎΡ‚Ρ€ΠΈΡ†Π°Ρ‚Π΅Π»ΡŒΠ½ΠΎΠ΅.

РСзистор классифицируСтся ΠΊΠ°ΠΊ пассивный элСмСнт схСмы ΠΈ, ΠΊΠ°ΠΊ Ρ‚Π°ΠΊΠΎΠ²ΠΎΠΉ, Π½Π΅ ΠΌΠΎΠΆΠ΅Ρ‚ ΠΏΠ΅Ρ€Π΅Π΄Π°Π²Π°Ρ‚ΡŒ ΠΈΠ»ΠΈ Π½Π°ΠΊΠ°ΠΏΠ»ΠΈΠ²Π°Ρ‚ΡŒ ΡΠ½Π΅Ρ€Π³ΠΈΡŽ. ВмСсто этого рСзисторы ΠΏΠΎΠ³Π»ΠΎΡ‰Π°Π»ΠΈ ΠΌΠΎΡ‰Π½ΠΎΡΡ‚ΡŒ, которая ΠΏΡ€ΠΎΡΠ²Π»ΡΠ»Π°ΡΡŒ Π² Π²ΠΈΠ΄Π΅ Ρ‚Π΅ΠΏΠ»Π° ΠΈ свСта. ΠœΠΎΡ‰Π½ΠΎΡΡ‚ΡŒ Π² сопротивлСнии всСгда ΠΏΠΎΠ»ΠΎΠΆΠΈΡ‚Π΅Π»ΡŒΠ½Π°, нСзависимо ΠΎΡ‚ полярности напряТСния ΠΈ направлСния Ρ‚ΠΎΠΊΠ°.

Для ΠΎΡ‡Π΅Π½ΡŒ Π½ΠΈΠ·ΠΊΠΈΡ… Π·Π½Π°Ρ‡Π΅Π½ΠΈΠΉ сопротивлСния, Π½Π°ΠΏΡ€ΠΈΠΌΠ΅Ρ€ миллиОм (мОм), ΠΈΠ½ΠΎΠ³Π΄Π° Π³ΠΎΡ€Π°Π·Π΄ΠΎ ΠΏΡ€ΠΎΡ‰Π΅ ΠΈΡΠΏΠΎΠ»ΡŒΠ·ΠΎΠ²Π°Ρ‚ΡŒ Π²Π΅Π»ΠΈΡ‡ΠΈΠ½Ρƒ, ΠΎΠ±Ρ€Π°Ρ‚Π½ΡƒΡŽ ΡΠΎΠΏΡ€ΠΎΡ‚ΠΈΠ²Π»Π΅Π½ΠΈΡŽ (1 / R), Ρ‡Π΅ΠΌ само сопротивлСниС (R).Π’Π΅Π»ΠΈΡ‡ΠΈΠ½Π° сопротивлСния называСтся ΠŸΡ€ΠΎΠ²ΠΎΠ΄ΠΈΠΌΠΎΡΡ‚ΡŒ , символ ( G ) ΠΈ прСдставляСт ΡΠΏΠΎΡΠΎΠ±Π½ΠΎΡΡ‚ΡŒ ΠΏΡ€ΠΎΠ²ΠΎΠ΄Π½ΠΈΠΊΠ° ΠΈΠ»ΠΈ устройства ΠΏΡ€ΠΎΠ²ΠΎΠ΄ΠΈΡ‚ΡŒ элСктричСство.

Π”Ρ€ΡƒΠ³ΠΈΠΌΠΈ словами, Π»Π΅Π³ΠΊΠΎΡΡ‚ΡŒ, с ΠΊΠΎΡ‚ΠΎΡ€ΠΎΠΉ Ρ‚Π΅Ρ‡Π΅Ρ‚ Ρ‚ΠΎΠΊ. ВысокиС значСния проводимости ΠΏΠΎΠ΄Ρ€Π°Π·ΡƒΠΌΠ΅Π²Π°ΡŽΡ‚ Ρ…ΠΎΡ€ΠΎΡˆΠΈΠΉ ΠΏΡ€ΠΎΠ²ΠΎΠ΄Π½ΠΈΠΊ, Π½Π°ΠΏΡ€ΠΈΠΌΠ΅Ρ€ мСдь, Π² Ρ‚ΠΎ врСмя ΠΊΠ°ΠΊ Π½ΠΈΠ·ΠΊΠΈΠ΅ значСния проводимости ΠΏΠΎΠ΄Ρ€Π°Π·ΡƒΠΌΠ΅Π²Π°ΡŽΡ‚ ΠΏΠ»ΠΎΡ…ΠΎΠΉ ΠΏΡ€ΠΎΠ²ΠΎΠ΄Π½ΠΈΠΊ, Π½Π°ΠΏΡ€ΠΈΠΌΠ΅Ρ€, Π΄Π΅Ρ€Π΅Π²ΠΎ. Бтандартная Π΅Π΄ΠΈΠ½ΠΈΡ†Π° измСрСния проводимости — Siemen , символ ( S ).

Π•Π΄ΠΈΠ½ΠΈΡ†Π΅ΠΉ измСрСния проводимости являСтся mho (ΠΎΠΌ, записанный Π² ΠΎΠ±Ρ€Π°Ρ‚Π½ΠΎΠΌ Π½Π°ΠΏΡ€Π°Π²Π»Π΅Π½ΠΈΠΈ), ΠΊΠΎΡ‚ΠΎΡ€Ρ‹ΠΉ обозначаСтся ΠΏΠ΅Ρ€Π΅Π²Π΅Ρ€Π½ΡƒΡ‚Ρ‹ΠΌ Π·Π½Π°ΠΊΠΎΠΌ ΠΎΠΌΠ°.ΠœΠΎΡ‰Π½ΠΎΡΡ‚ΡŒ Ρ‚Π°ΠΊΠΆΠ΅ ΠΌΠΎΠΆΠ΅Ρ‚ Π±Ρ‹Ρ‚ΡŒ Π²Ρ‹Ρ€Π°ΠΆΠ΅Π½Π° с использованиСм проводимости ΠΊΠ°ΠΊ: p = i 2 / G = v 2 G.

Π‘ΠΎΠΎΡ‚Π½ΠΎΡˆΠ΅Π½ΠΈΠ΅ ΠΌΠ΅ΠΆΠ΄Ρƒ напряТСниСм (v) ΠΈ Ρ‚ΠΎΠΊΠΎΠΌ (i) Π² Ρ†Π΅ΠΏΠΈ постоянного сопротивлСния (R) Π±ΡƒΠ΄Π΅Ρ‚ Π΄Π°Π²Π°Ρ‚ΡŒ ΠΏΡ€ΡΠΌΡƒΡŽ Π·Π°Π²ΠΈΡΠΈΠΌΠΎΡΡ‚ΡŒ i-v с Π½Π°ΠΊΠ»ΠΎΠ½ΠΎΠΌ, Ρ€Π°Π²Π½Ρ‹ΠΌ Π·Π½Π°Ρ‡Π΅Π½ΠΈΡŽ сопротивлСния, ΠΊΠ°ΠΊ ΠΏΠΎΠΊΠ°Π·Π°Π½ΠΎ.

Π‘Π²ΠΎΠ΄ΠΊΠ° ΠΏΠΎ Π½Π°ΠΏΡ€ΡΠΆΠ΅Π½ΠΈΡŽ, Ρ‚ΠΎΠΊΡƒ ΠΈ ΡΠΎΠΏΡ€ΠΎΡ‚ΠΈΠ²Π»Π΅Π½ΠΈΡŽ

НадСюсь, ΠΊ настоящСму Π²Ρ€Π΅ΠΌΠ΅Π½ΠΈ Π²Ρ‹ Π΄ΠΎΠ»ΠΆΠ½Ρ‹ ΠΈΠΌΠ΅Ρ‚ΡŒ Π½Π΅ΠΊΠΎΡ‚ΠΎΡ€ΠΎΠ΅ прСдставлСниС ΠΎ Ρ‚ΠΎΠΌ, ΠΊΠ°ΠΊ элСктричСскоС напряТСниС, Ρ‚ΠΎΠΊ ΠΈ сопротивлСниС тСсно связаны Π΄Ρ€ΡƒΠ³ с Π΄Ρ€ΡƒΠ³ΠΎΠΌ.Π‘ΠΎΠΎΡ‚Π½ΠΎΡˆΠ΅Π½ΠΈΠ΅ ΠΌΠ΅ΠΆΠ΄Ρƒ НапряТСниС , Π’ΠΎΠΊ ΠΈ Π‘ΠΎΠΏΡ€ΠΎΡ‚ΠΈΠ²Π»Π΅Π½ΠΈΠ΅ составляСт основу Π·Π°ΠΊΠΎΠ½Π° Ома. Π’ Π»ΠΈΠ½Π΅ΠΉΠ½ΠΎΠΉ Ρ†Π΅ΠΏΠΈ с фиксированным сопротивлСниСм, Ссли ΠΌΡ‹ ΡƒΠ²Π΅Π»ΠΈΡ‡ΠΈΠ²Π°Π΅ΠΌ напряТСниС, Ρ‚ΠΎΠΊ возрастаСт, ΠΈ Π°Π½Π°Π»ΠΎΠ³ΠΈΡ‡Π½ΠΎ, Ссли ΠΌΡ‹ ΡƒΠΌΠ΅Π½ΡŒΡˆΠ°Π΅ΠΌ напряТСниС, Ρ‚ΠΎΠΊ ΠΏΠ°Π΄Π°Π΅Ρ‚. Π­Ρ‚ΠΎ ΠΎΠ·Π½Π°Ρ‡Π°Π΅Ρ‚, Ρ‡Ρ‚ΠΎ Ссли напряТСниС высокоС, Ρ‚ΠΎΠΊ большой, Π° Ссли напряТСниС Π½ΠΈΠ·ΠΊΠΎΠ΅, Ρ‚ΠΎ Ρ‚ΠΎΠΊ Π½ΠΈΠ·ΠΊΠΈΠΉ.

Аналогично, Ссли ΠΌΡ‹ ΡƒΠ²Π΅Π»ΠΈΡ‡ΠΈΠ²Π°Π΅ΠΌ сопротивлСниС, Ρ‚ΠΎΠΊ сниТаСтся для Π΄Π°Π½Π½ΠΎΠ³ΠΎ напряТСния, Π° Ссли ΠΌΡ‹ ΡƒΠΌΠ΅Π½ΡŒΡˆΠ°Π΅ΠΌ сопротивлСниС, Ρ‚ΠΎΠΊ возрастаСт.Π­Ρ‚ΠΎ ΠΎΠ·Π½Π°Ρ‡Π°Π΅Ρ‚, Ρ‡Ρ‚ΠΎ Ссли сопротивлСниС Π²Π΅Π»ΠΈΠΊΠΎ, Ρ‚ΠΎΠΊ Π½ΠΈΠ·ΠΊΠΈΠΉ, Π° Ссли сопротивлСниС Π½ΠΈΠ·ΠΊΠΎΠ΅, Ρ‚ΠΎΠΊ высокий.

Π’ΠΎΠ³Π΄Π° ΠΌΡ‹ ΠΌΠΎΠΆΠ΅ΠΌ Π²ΠΈΠ΄Π΅Ρ‚ΡŒ, Ρ‡Ρ‚ΠΎ Ρ‚ΠΎΠΊ, ΠΏΡ€ΠΎΡ‚Π΅ΠΊΠ°ΡŽΡ‰ΠΈΠΉ ΠΏΠΎ Ρ†Π΅ΠΏΠΈ, прямо ΠΏΡ€ΠΎΠΏΠΎΡ€Ρ†ΠΈΠΎΠ½Π°Π»Π΅Π½ (∝) Π½Π°ΠΏΡ€ΡΠΆΠ΅Π½ΠΈΡŽ (V ↑ Π²Ρ‹Π·Ρ‹Π²Π°Π΅Ρ‚ I ↑), Π½ΠΎ ΠΎΠ±Ρ€Π°Ρ‚Π½ΠΎ ΠΏΡ€ΠΎΠΏΠΎΡ€Ρ†ΠΈΠΎΠ½Π°Π»Π΅Π½ (1 / ∝) ΡΠΎΠΏΡ€ΠΎΡ‚ΠΈΠ²Π»Π΅Π½ΠΈΡŽ, ΠΊΠ°ΠΊ, (R ↑ Π²Ρ‹Π·Ρ‹Π²Π°Π΅Ρ‚ I ↓).

НиТС приводится ΠΊΡ€Π°Ρ‚ΠΊΠΎΠ΅ описаниС Ρ‚Ρ€Π΅Ρ… ΠΌΠΎΠ΄ΡƒΠ»Π΅ΠΉ.

  • НапряТСниС ΠΈΠ»ΠΈ Ρ€Π°Π·Π½ΠΎΡΡ‚ΡŒ ΠΏΠΎΡ‚Π΅Π½Ρ†ΠΈΠ°Π»ΠΎΠ² — это ΠΌΠ΅Ρ€Π° ΠΏΠΎΡ‚Π΅Π½Ρ†ΠΈΠ°Π»ΡŒΠ½ΠΎΠΉ энСргии ΠΌΠ΅ΠΆΠ΄Ρƒ двумя Ρ‚ΠΎΡ‡ΠΊΠ°ΠΌΠΈ Π² Ρ†Π΅ΠΏΠΈ ΠΈ ΠΎΠ±Ρ‹Ρ‡Π½ΠΎ обозначаСтся ΠΊΠ°ΠΊ Β«ΠΏΠ°Π΄Π΅Π½ΠΈΠ΅ Π²ΠΎΠ»ΡŒΡ‚, Β».
  • Когда источник напряТСния ΠΏΠΎΠ΄ΠΊΠ»ΡŽΡ‡Π΅Π½ ΠΊ Π·Π°ΠΌΠΊΠ½ΡƒΡ‚ΠΎΠΉ Ρ†Π΅ΠΏΠΈ, напряТСниС Π±ΡƒΠ΄Π΅Ρ‚ ΡΠΎΠ·Π΄Π°Π²Π°Ρ‚ΡŒ Ρ‚ΠΎΠΊ, ΠΏΡ€ΠΎΡ‚Π΅ΠΊΠ°ΡŽΡ‰ΠΈΠΉ ΠΏΠΎ Ρ†Π΅ΠΏΠΈ.
  • Π’ источниках постоянного напряТСния символы + ve (ΠΏΠΎΠ»ΠΎΠΆΠΈΡ‚Π΅Π»ΡŒΠ½Ρ‹ΠΉ) ΠΈ -ve (ΠΎΡ‚Ρ€ΠΈΡ†Π°Ρ‚Π΅Π»ΡŒΠ½Ρ‹ΠΉ) ΠΈΡΠΏΠΎΠ»ΡŒΠ·ΡƒΡŽΡ‚ΡΡ для обозначСния полярности источника напряТСния.
  • НапряТСниС измСряСтся Π² Π²ΠΎΠ»ΡŒΡ‚Π°Ρ… ΠΈ ΠΎΠ±ΠΎΠ·Π½Π°Ρ‡Π΅Π½ΠΎ символом V для напряТСния ΠΈΠ»ΠΈ E для элСктричСской энСргии.
  • ΠŸΠΎΡ‚ΠΎΠΊ Ρ‚ΠΎΠΊΠ° — это комбинация ΠΏΠΎΡ‚ΠΎΠΊΠ° элСктронов ΠΈ ΠΏΠΎΡ‚ΠΎΠΊΠ° Π΄Ρ‹Ρ€ΠΎΠΊ Π² Ρ†Π΅ΠΏΠΈ.
  • Π’ΠΎΠΊ прСдставляСт собой Π½Π΅ΠΏΡ€Π΅Ρ€Ρ‹Π²Π½Ρ‹ΠΉ ΠΈ Ρ€Π°Π²Π½ΠΎΠΌΠ΅Ρ€Π½Ρ‹ΠΉ ΠΏΠΎΡ‚ΠΎΠΊ заряда ΠΏΠΎ Ρ†Π΅ΠΏΠΈ ΠΈ измСряСтся Π² АмпСрах ΠΈΠ»ΠΈ АмпСрах ΠΈ обозначаСтся символом I.
  • Π’ΠΎΠΊ прямо ΠΏΡ€ΠΎΠΏΠΎΡ€Ρ†ΠΈΠΎΠ½Π°Π»Π΅Π½ Π½Π°ΠΏΡ€ΡΠΆΠ΅Π½ΠΈΡŽ (I ∝ Π’)
  • Π­Ρ„Ρ„Π΅ΠΊΡ‚ΠΈΠ²Π½ΠΎΠ΅ (срСднСквадратичноС) Π·Π½Π°Ρ‡Π΅Π½ΠΈΠ΅ ΠΏΠ΅Ρ€Π΅ΠΌΠ΅Π½Π½ΠΎΠ³ΠΎ Ρ‚ΠΎΠΊΠ° ΠΈΠΌΠ΅Π΅Ρ‚ Ρ‚Π°ΠΊΡƒΡŽ ​​ТС ΡΡ€Π΅Π΄Π½ΡŽΡŽ ΠΏΠΎΡ‚Π΅Ρ€ΡŽ мощности, Ρ‡Ρ‚ΠΎ ΠΈ постоянный Ρ‚ΠΎΠΊ, ΠΏΡ€ΠΎΡ‚Π΅ΠΊΠ°ΡŽΡ‰ΠΈΠΉ Ρ‡Π΅Ρ€Π΅Π· рСзистивный элСмСнт.
  • Π‘ΠΎΠΏΡ€ΠΎΡ‚ΠΈΠ²Π»Π΅Π½ΠΈΠ΅ — это противодСйствиС Ρ‚ΠΎΠΊΡƒ, ΠΏΡ€ΠΎΡ‚Π΅ΠΊΠ°ΡŽΡ‰Π΅ΠΌΡƒ ΠΏΠΎ Ρ†Π΅ΠΏΠΈ.
  • НизкиС значСния сопротивлСния ΠΏΠΎΠ΄Ρ€Π°Π·ΡƒΠΌΠ΅Π²Π°ΡŽΡ‚ ΠΏΡ€ΠΎΠ²ΠΎΠ΄Π½ΠΈΠΊ, Π° высокиС значСния сопротивлСния — изолятор.
  • Π’ΠΎΠΊ ΠΎΠ±Ρ€Π°Ρ‚Π½ΠΎ ΠΏΡ€ΠΎΠΏΠΎΡ€Ρ†ΠΈΠΎΠ½Π°Π»Π΅Π½ ΡΠΎΠΏΡ€ΠΎΡ‚ΠΈΠ²Π»Π΅Π½ΠΈΡŽ (I 1 / ∝ R)
  • Π‘ΠΎΠΏΡ€ΠΎΡ‚ΠΈΠ²Π»Π΅Π½ΠΈΠ΅ измСряСтся Π² Ом ΠΈ ΠΈΠΌΠ΅Π΅Ρ‚ грСчСский символ Ξ© ΠΈΠ»ΠΈ Π±ΡƒΠΊΠ²Ρƒ R.
Кол-Π²ΠΎ Π‘ΠΈΠΌΠ²ΠΎΠ» Π•Π΄ΠΈΠ½ΠΈΡ†Π° измСрСния Π‘ΠΎΠΊΡ€Π°Ρ‰Π΅Π½ΠΈΠ΅
НапряТСниС V ΠΈΠ»ΠΈ E Π’ΠΎΠ»ΡŒΡ‚ Π’
Π’Π΅ΠΊΡƒΡ‰ΠΈΠΉ I АмпСр А
Π‘ΠΎΠΏΡ€ΠΎΡ‚ΠΈΠ²Π»Π΅Π½ΠΈΠ΅ R Ом Ом

Π’ ΡΠ»Π΅Π΄ΡƒΡŽΡ‰Π΅ΠΌ ΡƒΡ€ΠΎΠΊΠ΅ ΠΎ цСпях постоянного Ρ‚ΠΎΠΊΠ° ΠΌΡ‹ рассмотрим Π·Π°ΠΊΠΎΠ½ Ома, ΠΊΠΎΡ‚ΠΎΡ€Ρ‹ΠΉ прСдставляСт собой матСматичСскоС ΡƒΡ€Π°Π²Π½Π΅Π½ΠΈΠ΅, ΠΎΠ±ΡŠΡΡΠ½ΡΡŽΡ‰Π΅Π΅ взаимосвязь ΠΌΠ΅ΠΆΠ΄Ρƒ напряТСниСм, Ρ‚ΠΎΠΊΠΎΠΌ ΠΈ сопротивлСниСм Π² элСктричСских цСпях ΠΈ ΡΠ²Π»ΡΡŽΡ‰Π΅Π΅ΡΡ основой элСктроники ΠΈ элСктротСхники. Π—Π°ΠΊΠΎΠ½ Ома опрСдСляСтся ΠΊΠ°ΠΊ: V = I * R.

Π—Π°ΠΊΠΎΠ½

Ома: ΠΎΠΏΡ€Π΅Π΄Π΅Π»Π΅Π½ΠΈΠ΅ ΠΈ взаимосвязь ΠΌΠ΅ΠΆΠ΄Ρƒ напряТСниСм, Ρ‚ΠΎΠΊΠΎΠΌ ΠΈ сопротивлСниСм — Π²ΠΈΠ΄Π΅ΠΎ ΠΈ стСнограмма ΡƒΡ€ΠΎΠΊΠ°

Π—Π°ΠΊΠΎΠ½ Ома

Π’Π·Π°ΠΈΠΌΠΎΡΠ²ΡΠ·ΡŒ ΠΌΠ΅ΠΆΠ΄Ρƒ напряТСниСм, Ρ‚ΠΎΠΊΠΎΠΌ ΠΈ сопротивлСниСм описываСтся Π·Π°ΠΊΠΎΠ½ΠΎΠΌ Ома . Π­Ρ‚ΠΎ ΡƒΡ€Π°Π²Π½Π΅Π½ΠΈΠ΅, i = v / r , Π³ΠΎΠ²ΠΎΡ€ΠΈΡ‚ Π½Π°ΠΌ, Ρ‡Ρ‚ΠΎ Ρ‚ΠΎΠΊ, i , ΠΏΡ€ΠΎΡ‚Π΅ΠΊΠ°ΡŽΡ‰ΠΈΠΉ ΠΏΠΎ Ρ†Π΅ΠΏΠΈ, прямо ΠΏΡ€ΠΎΠΏΠΎΡ€Ρ†ΠΈΠΎΠ½Π°Π»Π΅Π½ Π½Π°ΠΏΡ€ΡΠΆΠ΅Π½ΠΈΡŽ, v , ΠΈ ΠΎΠ±Ρ€Π°Ρ‚Π½ΠΎ ΠΏΡ€ΠΎΠΏΠΎΡ€Ρ†ΠΈΠΎΠ½Π°Π»Π΅Π½ ΡΠΎΠΏΡ€ΠΎΡ‚ΠΈΠ²Π»Π΅Π½ΠΈΡŽ, r .Π”Ρ€ΡƒΠ³ΠΈΠΌΠΈ словами, Ссли ΠΌΡ‹ ΡƒΠ²Π΅Π»ΠΈΡ‡ΠΈΠΌ напряТСниС, Ρ‚ΠΎ увСличится ΠΈ Ρ‚ΠΎΠΊ. Но, Ссли ΡƒΠ²Π΅Π»ΠΈΡ‡ΠΈΡ‚ΡŒ сопротивлСниС, Ρ‚ΠΎ Ρ‚ΠΎΠΊ ΡƒΠΌΠ΅Π½ΡŒΡˆΠΈΡ‚ΡΡ. ΠœΡ‹ ΡƒΠ²ΠΈΠ΄Π΅Π»ΠΈ эти ΠΊΠΎΠ½Ρ†Π΅ΠΏΡ†ΠΈΠΈ Π² дСйствии с садовым шлангом. Π£Π²Π΅Π»ΠΈΡ‡Π΅Π½ΠΈΠ΅ давлСния ΠΏΡ€ΠΈΠ²Π΅Π»ΠΎ ΠΊ ΡƒΠ²Π΅Π»ΠΈΡ‡Π΅Π½ΠΈΡŽ ΠΏΠΎΡ‚ΠΎΠΊΠ°, Π½ΠΎ ΠΈΠ·Π³ΠΈΠ± шланга ΡƒΠ²Π΅Π»ΠΈΡ‡ΠΈΠ» сопротивлСниС, Ρ‡Ρ‚ΠΎ ΠΏΡ€ΠΈΠ²Π΅Π»ΠΎ ΠΊ ΡƒΠΌΠ΅Π½ΡŒΡˆΠ΅Π½ΠΈΡŽ ΠΏΠΎΡ‚ΠΎΠΊΠ°.

Π­Ρ‚Π° Π΄ΠΈΠ°Π³Ρ€Π°ΠΌΠΌΠ° — простой способ Ρ€Π΅ΡˆΠ°Ρ‚ΡŒ уравнСния.

Как написано здСсь ΡƒΡ€Π°Π²Π½Π΅Π½ΠΈΠ΅, Π±Ρ‹Π»ΠΎ Π±Ρ‹ Π»Π΅Π³ΠΊΠΎ ΠΈΡΠΏΠΎΠ»ΡŒΠ·ΠΎΠ²Π°Ρ‚ΡŒ Π·Π°ΠΊΠΎΠ½ Ома, Ρ‡Ρ‚ΠΎΠ±Ρ‹ Π²Ρ‹Ρ‡ΠΈΡΠ»ΠΈΡ‚ΡŒ Ρ‚ΠΎΠΊ, Ссли Π±Ρ‹ ΠΌΡ‹ Π·Π½Π°Π»ΠΈ напряТСниС ΠΈ сопротивлСниС.Но Ρ‡Ρ‚ΠΎ, Ссли Π±Ρ‹ ΠΌΡ‹ вмСсто этого Π·Π°Ρ…ΠΎΡ‚Π΅Π»ΠΈ Π²Ρ‹Ρ‡ΠΈΡΠ»ΠΈΡ‚ΡŒ напряТСниС ΠΈΠ»ΠΈ сопротивлСниС? Один ΠΈΠ· способов ΡΠ΄Π΅Π»Π°Ρ‚ΡŒ это — ΠΏΠ΅Ρ€Π΅ΡΡ‚Π°Π²ΠΈΡ‚ΡŒ Ρ‡Π»Π΅Π½Ρ‹ уравнСния для Ρ€Π΅ΡˆΠ΅Π½ΠΈΡ Π΄Ρ€ΡƒΠ³ΠΈΡ… ΠΏΠ°Ρ€Π°ΠΌΠ΅Ρ‚Ρ€ΠΎΠ², Π½ΠΎ Π΅ΡΡ‚ΡŒ Π±ΠΎΠ»Π΅Π΅ простой способ. ΠŸΡ€ΠΈΠ²Π΅Π΄Π΅Π½Π½Π°Ρ Π²Ρ‹ΡˆΠ΅ Π΄ΠΈΠ°Π³Ρ€Π°ΠΌΠΌΠ° даст Π½Π°ΠΌ ΡΠΎΠΎΡ‚Π²Π΅Ρ‚ΡΡ‚Π²ΡƒΡŽΡ‰Π΅Π΅ ΡƒΡ€Π°Π²Π½Π΅Π½ΠΈΠ΅ для Ρ€Π΅ΡˆΠ΅Π½ΠΈΡ любого нСизвСстного ΠΏΠ°Ρ€Π°ΠΌΠ΅Ρ‚Ρ€Π° Π±Π΅Π· использования Π°Π»Π³Π΅Π±Ρ€Ρ‹. Π§Ρ‚ΠΎΠ±Ρ‹ ΠΈΡΠΏΠΎΠ»ΡŒΠ·ΠΎΠ²Π°Ρ‚ΡŒ эту Π΄ΠΈΠ°Π³Ρ€Π°ΠΌΠΌΡƒ, ΠΌΡ‹ просто Π·Π°ΠΊΡ€Ρ‹Π²Π°Π΅ΠΌ ΠΏΠ°Ρ€Π°ΠΌΠ΅Ρ‚Ρ€, ΠΊΠΎΡ‚ΠΎΡ€Ρ‹ΠΉ пытаСмся Π½Π°ΠΉΡ‚ΠΈ, Ρ‡Ρ‚ΠΎΠ±Ρ‹ ΠΏΠΎΠ»ΡƒΡ‡ΠΈΡ‚ΡŒ ΠΏΡ€Π°Π²ΠΈΠ»ΡŒΠ½ΠΎΠ΅ ΡƒΡ€Π°Π²Π½Π΅Π½ΠΈΠ΅. Π­Ρ‚ΠΎ станСт Π±ΠΎΠ»Π΅Π΅ понятным, ΠΊΠΎΠ³Π΄Π° ΠΌΡ‹ Π½Π°Ρ‡Π½Π΅ΠΌ Π΅Π³ΠΎ ΠΈΡΠΏΠΎΠ»ΡŒΠ·ΠΎΠ²Π°Ρ‚ΡŒ, поэтому Π΄Π°Π²Π°ΠΉΡ‚Π΅ рассмотрим нСсколько ΠΏΡ€ΠΈΠΌΠ΅Ρ€ΠΎΠ².

Π—Π°ΠΊΠΎΠ½ Ома Π² дСйствии

НиТС прСдставлСна ​​простая элСктричСская схСма, ΠΊΠΎΡ‚ΠΎΡ€ΡƒΡŽ ΠΌΡ‹ Π±ΡƒΠ΄Π΅ΠΌ ΠΈΡΠΏΠΎΠ»ΡŒΠ·ΠΎΠ²Π°Ρ‚ΡŒ для выполнСния Π½Π°ΡˆΠΈΡ… ΠΏΡ€ΠΈΠΌΠ΅Ρ€ΠΎΠ². Наш источник напряТСния — это аккумулятор, ΠΏΠΎΠ΄ΠΊΠ»ΡŽΡ‡Π΅Π½Π½Ρ‹ΠΉ ΠΊ Π»Π°ΠΌΠΏΠΎΡ‡ΠΊΠ΅, которая обСспСчиваСт сопротивлСниС элСктричСскому Ρ‚ΠΎΠΊΡƒ. Для Π½Π°Ρ‡Π°Π»Π° ΠΏΡ€Π΅Π΄ΠΏΠΎΠ»ΠΎΠΆΠΈΠΌ, Ρ‡Ρ‚ΠΎ наша батарСя ΠΈΠΌΠ΅Π΅Ρ‚ напряТСниС 10 Π²ΠΎΠ»ΡŒΡ‚, элСктричСская Π»Π°ΠΌΠΏΠΎΡ‡ΠΊΠ° ΠΈΠΌΠ΅Π΅Ρ‚ сопротивлСниС 20 Ом, ΠΈ Π½Π°ΠΌ Π½ΡƒΠΆΠ½ΠΎ Π²Ρ‹Ρ‡ΠΈΡΠ»ΠΈΡ‚ΡŒ Ρ‚ΠΎΠΊ, ΠΏΡ€ΠΎΡ‚Π΅ΠΊΠ°ΡŽΡ‰ΠΈΠΉ ΠΏΠΎ Ρ†Π΅ΠΏΠΈ. Π˜ΡΠΏΠΎΠ»ΡŒΠ·ΡƒΡ Π½Π°ΡˆΡƒ Π΄ΠΈΠ°Π³Ρ€Π°ΠΌΠΌΡƒ, ΠΌΡ‹ Π·Π°ΠΊΡ€Ρ‹Π²Π°Π΅ΠΌ ΠΏΠ°Ρ€Π°ΠΌΠ΅Ρ‚Ρ€, ΠΊΠΎΡ‚ΠΎΡ€Ρ‹ΠΉ ΠΌΡ‹ пытаСмся Π½Π°ΠΉΡ‚ΠΈ, Ρ‚ΠΎ Π΅ΡΡ‚ΡŒ Ρ‚ΠΎΠΊ, ΠΈΠ»ΠΈ i , ΠΈ это оставляСт Π½Π°ΠΌ напряТСниС v Π½Π°Π΄ сопротивлСниСм r .Π”Ρ€ΡƒΠ³ΠΈΠΌΠΈ словами, Ρ‡Ρ‚ΠΎΠ±Ρ‹ Π½Π°ΠΉΡ‚ΠΈ Ρ‚ΠΎΠΊ, Π½Π°ΠΌ Π½ΡƒΠΆΠ½ΠΎ Ρ€Π°Π·Π΄Π΅Π»ΠΈΡ‚ΡŒ напряТСниС Π½Π° сопротивлСниС. ДСлая ΠΌΠ°Ρ‚Π΅ΠΌΠ°Ρ‚ΠΈΠΊΡƒ, 10 Π²ΠΎΠ»ΡŒΡ‚, Ρ€Π°Π·Π΄Π΅Π»Π΅Π½Π½Ρ‹Π΅ Π½Π° 20 Ом, Π΄Π°ΡŽΡ‚ ΠΏΠΎΠ»ΠΎΠ²ΠΈΠ½Ρƒ Π°ΠΌΠΏΠ΅Ρ€Π° Ρ‚ΠΎΠΊΠ°, ΠΏΡ€ΠΎΡ‚Π΅ΠΊΠ°ΡŽΡ‰Π΅Π³ΠΎ Π² Ρ†Π΅ΠΏΠΈ.

Π§Ρ‚ΠΎΠ±Ρ‹ Π½Π°ΠΉΡ‚ΠΈ Ρ‚ΠΎΠΊ, Ρ€Π°Π·Π΄Π΅Π»ΠΈΡ‚Π΅ напряТСниС (20 Π²ΠΎΠ»ΡŒΡ‚) Π½Π° сопротивлСниС (20 Ом).

Π’Π΅ΠΏΠ΅Ρ€ΡŒ Π΄Π°Π²Π°ΠΉΡ‚Π΅ ΡƒΠ²Π΅Π»ΠΈΡ‡ΠΈΠΌ напряТСниС, Ρ‡Ρ‚ΠΎΠ±Ρ‹ ΠΏΠΎΡΠΌΠΎΡ‚Ρ€Π΅Ρ‚ΡŒ, Ρ‡Ρ‚ΠΎ происходит с Ρ‚ΠΎΠΊΠΎΠΌ. ΠœΡ‹ Π±ΡƒΠ΄Π΅ΠΌ ΠΈΡΠΏΠΎΠ»ΡŒΠ·ΠΎΠ²Π°Ρ‚ΡŒ Ρ‚Ρƒ ΠΆΠ΅ Π»Π°ΠΌΠΏΠΎΡ‡ΠΊΡƒ, Π½ΠΎ ΠΏΠ΅Ρ€Π΅ΠΉΠ΄Π΅ΠΌ Π½Π° 20-Π²ΠΎΠ»ΡŒΡ‚ΠΎΠ²ΡƒΡŽ Π±Π°Ρ‚Π°Ρ€Π΅ΡŽ.Π˜ΡΠΏΠΎΠ»ΡŒΠ·ΡƒΡ Ρ‚ΠΎ ΠΆΠ΅ ΡƒΡ€Π°Π²Π½Π΅Π½ΠΈΠ΅, Ρ‡Ρ‚ΠΎ ΠΈ Ρ€Π°Π½ΡŒΡˆΠ΅, ΠΌΡ‹ Ρ€Π°Π·Π΄Π΅Π»ΠΈΠΌ 20 Π²ΠΎΠ»ΡŒΡ‚ Π½Π° 20 Ом, ΠΈ ΠΌΡ‹ ΠΏΠΎΠ»ΡƒΡ‡ΠΈΠΌ 1 Π°ΠΌΠΏΠ΅Ρ€ Ρ‚ΠΎΠΊΠ°. Как ΠΌΡ‹ Π²ΠΈΠ΄ΠΈΠΌ, ΡƒΠ΄Π²ΠΎΠ΅Π½ΠΈΠ΅ напряТСния ΠΏΡ€ΠΈΠ²Π΅Π»ΠΎ ΠΊ ΡƒΠ΄Π²ΠΎΠ΅Π½ΠΈΡŽ Ρ‚ΠΎΠΊΠ°. Π­Ρ‚ΠΎ ΠΈΠΌΠ΅Π΅Ρ‚ смысл, ΠΊΠΎΠ³Π΄Π° ΠΌΡ‹ Π΄ΡƒΠΌΠ°Π΅ΠΌ ΠΎ садовом шлангС. Если Π±Ρ‹ ΠΌΡ‹ ΡƒΠ²Π΅Π»ΠΈΡ‡ΠΈΠ»ΠΈ Π΄Π°Π²Π»Π΅Π½ΠΈΠ΅ Π² шлангС, ΠΌΠΎΠΆΠ½ΠΎ Π±Ρ‹Π»ΠΎ Π±Ρ‹ ΠΎΠΆΠΈΠ΄Π°Ρ‚ΡŒ, Ρ‡Ρ‚ΠΎ ΠΏΠΎΡ‚ΠΎΠΊ Π²ΠΎΠ΄Ρ‹ Ρ‚Π°ΠΊΠΆΠ΅ увСличится. ВсСгда ΠΏΠΎΠ»Π΅Π·Π½ΠΎ ΠΏΠ΅Ρ€Π΅ΠΏΡ€ΠΎΠ²Π΅Ρ€ΠΈΡ‚ΡŒ свою Ρ€Π°Π±ΠΎΡ‚Ρƒ, спросив, ΡΠΎΠΎΡ‚Π²Π΅Ρ‚ΡΡ‚Π²ΡƒΡŽΡ‚ Π»ΠΈ Ρ€Π΅Π·ΡƒΠ»ΡŒΡ‚Π°Ρ‚Ρ‹ Ρ‚ΠΎΠΌΡƒ, Ρ‡Ρ‚ΠΎ Π²Ρ‹ ΠΎΠΆΠΈΠ΄Π°Π»ΠΈ.

Если Π±Ρ‹ ΠΌΡ‹ ΡƒΠ²Π΅Π»ΠΈΡ‡ΠΈΠ»ΠΈ сопротивлСниС Π»Π°ΠΌΠΏΠΎΡ‡ΠΊΠΈ, Ρ‡Ρ‚ΠΎ Π±Ρ‹ Π²Ρ‹ ΠΎΠΆΠΈΠ΄Π°Π»ΠΈ, Ρ‡Ρ‚ΠΎ ΠΏΡ€ΠΎΠΈΠ·ΠΎΠΉΠ΄Π΅Ρ‚ с Ρ‚ΠΎΠΊΠΎΠΌ? Π§Ρ‚ΠΎΠ±Ρ‹ Π²Ρ‹ΡΡΠ½ΠΈΡ‚ΡŒ это, Π΄Π°Π²Π°ΠΉΡ‚Π΅ помСняСм ΡΡƒΡ‰Π΅ΡΡ‚Π²ΡƒΡŽΡ‰ΡƒΡŽ Π»Π°ΠΌΠΏΠΎΡ‡ΠΊΡƒ Π½Π° Π΄Ρ€ΡƒΠ³ΡƒΡŽ с сопротивлСниСм 40 Ом.ΠŸΠΎΡΠΊΠΎΠ»ΡŒΠΊΡƒ ΠΌΡ‹ всС Π΅Ρ‰Π΅ ΠΈΡ‰Π΅ΠΌ Ρ‚ΠΎΠΊ, ΠΌΡ‹ ΠΈΡΠΏΠΎΠ»ΡŒΠ·ΡƒΠ΅ΠΌ Ρ‚ΠΎ ΠΆΠ΅ ΡƒΡ€Π°Π²Π½Π΅Π½ΠΈΠ΅, Ρ‡Ρ‚ΠΎ ΠΈ Ρ€Π°Π½ΡŒΡˆΠ΅. Π Π°Π·Π΄Π΅Π»ΠΈΠ² 20 Π²ΠΎΠ»ΡŒΡ‚ Π½Π° 40 Ом, ΠΌΡ‹ ΠΏΠΎΠ»ΡƒΡ‡ΠΈΠΌ ΠΏΠΎΠ»ΠΎΠ²ΠΈΠ½Ρƒ Π°ΠΌΠΏΠ΅Ρ€Π° Ρ‚ΠΎΠΊΠ°. Π­Ρ‚ΠΎΡ‚ Ρ€Π΅Π·ΡƒΠ»ΡŒΡ‚Π°Ρ‚ Π³ΠΎΠ²ΠΎΡ€ΠΈΡ‚ Π½Π°ΠΌ, Ρ‡Ρ‚ΠΎ ΡƒΠ΄Π²ΠΎΠ΅Π½ΠΈΠ΅ сопротивлСния ΡƒΠΌΠ΅Π½ΡŒΡˆΠΈΠ»ΠΎ Ρ‚ΠΎΠΊ Π²Π΄Π²ΠΎΠ΅. Π’Ρ‹ этого ΠΎΠΆΠΈΠ΄Π°Π»ΠΈ? Если Π²Π΅Ρ€Π½ΡƒΡ‚ΡŒΡΡ ΠΊ Π½Π°ΡˆΠ΅ΠΌΡƒ ΡˆΠ»Π°Π½Π³Ρƒ, Π»ΠΎΠ³ΠΈΡ‡Π½ΠΎ ΠΏΡ€Π΅Π΄ΠΏΠΎΠ»ΠΎΠΆΠΈΡ‚ΡŒ, Ρ‡Ρ‚ΠΎ ΠΏΠ΅Ρ€Π΅Π³ΠΈΠ± Π² шлангС ΡƒΠΌΠ΅Π½ΡŒΡˆΠΈΡ‚ ΠΏΠΎΡ‚ΠΎΠΊ Π²ΠΎΠ΄Ρ‹, Ρ‚ΠΎΡ‡Π½ΠΎ Ρ‚Π°ΠΊ ΠΆΠ΅, ΠΊΠ°ΠΊ ΡƒΠ²Π΅Π»ΠΈΡ‡Π΅Π½ΠΈΠ΅ сопротивлСния Π² Ρ†Π΅ΠΏΠΈ ΡƒΠΌΠ΅Π½ΡŒΡˆΠΈΡ‚ Ρ‚ΠΎΠΊ.

Π”ΠΎ сих ΠΏΠΎΡ€ ΠΌΡ‹ Ρ‚ΠΎΠ»ΡŒΠΊΠΎ рассчитали Ρ‚ΠΎΠΊ Π² Ρ†Π΅ΠΏΠΈ, Π½ΠΎ Ρ‡Ρ‚ΠΎ, Ссли Π±Ρ‹ ΠΊΡ‚ΠΎ-Ρ‚ΠΎ помСнял Π½Π°ΡˆΡƒ Π»Π°ΠΌΠΏΠΎΡ‡ΠΊΡƒ, ΠΊΠΎΠ³Π΄Π° ΠΌΡ‹ Π½Π΅ смотрСли, ΠΈ Π½Π°ΠΌ Π½ΡƒΠΆΠ½ΠΎ Π±Ρ‹Π»ΠΎ Π²Ρ‹Ρ‡ΠΈΡΠ»ΠΈΡ‚ΡŒ сопротивлСниС Π½ΠΎΠ²ΠΎΠΉ? Π§Ρ‚ΠΎ ΠΆ, ΠΌΡ‹ Π·Π½Π°Π΅ΠΌ, Ρ‡Ρ‚ΠΎ напряТСниС нашСй Π±Π°Ρ‚Π°Ρ€Π΅ΠΈ составляСт 20 Π²ΠΎΠ»ΡŒΡ‚, ΠΈ ΠΌΡ‹ ΠΌΠΎΠΆΠ΅ΠΌ ΠΈΠ·ΠΌΠ΅Ρ€ΠΈΡ‚ΡŒ Ρ‚ΠΎΠΊ Π² Ρ†Π΅ΠΏΠΈ с ΠΏΠΎΠΌΠΎΡ‰ΡŒΡŽ инструмСнта, Π½Π°Π·Ρ‹Π²Π°Π΅ΠΌΠΎΠ³ΠΎ Π°ΠΌΠΏΠ΅Ρ€ΠΌΠ΅Ρ‚Ρ€ΠΎΠΌ, поэтому всС, Ρ‡Ρ‚ΠΎ Π½Π°ΠΌ ΠΎΡΡ‚Π°Π»ΠΎΡΡŒ, — это Π²Ρ‹ΠΏΠΎΠ»Π½ΠΈΡ‚ΡŒ Π½Π΅ΠΊΠΎΡ‚ΠΎΡ€Ρ‹Π΅ вычислСния.Π˜ΡΠΏΠΎΠ»ΡŒΠ·ΡƒΡ Π½Π°ΡˆΡƒ Π΄ΠΈΠ°Π³Ρ€Π°ΠΌΠΌΡƒ, ΠΌΡ‹ скрываСм ΠΏΠ°Ρ€Π°ΠΌΠ΅Ρ‚Ρ€, ΠΊΠΎΡ‚ΠΎΡ€Ρ‹ΠΉ ΠΌΡ‹ пытаСмся Π½Π°ΠΉΡ‚ΠΈ, Π° ΠΈΠΌΠ΅Π½Π½ΠΎ сопротивлСниС, r . Π‘Ρ…Π΅ΠΌΠ° Ρ‚Π΅ΠΏΠ΅Ρ€ΡŒ ΠΏΠΎΠΊΠ°Π·Ρ‹Π²Π°Π΅Ρ‚ Π½Π°ΠΌ, Ρ‡Ρ‚ΠΎ Π½Π°ΠΌ Π½ΡƒΠΆΠ½ΠΎ Ρ€Π°Π·Π΄Π΅Π»ΠΈΡ‚ΡŒ напряТСниС Π½Π° Ρ‚ΠΎΠΊ. Если наш Π°ΠΌΠΏΠ΅Ρ€ΠΌΠ΅Ρ‚Ρ€ ΠΈΠ·ΠΌΠ΅Ρ€ΠΈΠ» Ρ‚ΠΎΠΊ Π² 5 Π°ΠΌΠΏΠ΅Ρ€, ΠΏΡ€ΠΎΡ‚Π΅ΠΊΠ°ΡŽΡ‰ΠΈΠΉ ΠΏΠΎ Ρ†Π΅ΠΏΠΈ, Ρ‚ΠΎ сопротивлСниС Π±ΡƒΠ΄Π΅Ρ‚ Ρ€Π°Π²Π½ΠΎ 20 Π²ΠΎΠ»ΡŒΡ‚, Ρ€Π°Π·Π΄Π΅Π»Π΅Π½Π½Ρ‹ΠΌ Π½Π° 5 Π°ΠΌΠΏΠ΅Ρ€, Ρ‡Ρ‚ΠΎ составляСт 4 Ом

Π§Ρ‚ΠΎΠ±Ρ‹ ΠΎΠΏΡ€Π΅Π΄Π΅Π»ΠΈΡ‚ΡŒ напряТСниС, ΡƒΠΌΠ½ΠΎΠΆΡŒΡ‚Π΅ силу Ρ‚ΠΎΠΊΠ° (3 Π°ΠΌΠΏΠ΅Ρ€Π°) Π½Π° сопротивлСниС (4 Ом).

НаконСц, ΠΏΡ€Π΅Π΄ΡΡ‚Π°Π²ΡŒΡ‚Π΅, Ρ‡Ρ‚ΠΎ ΠΊΡ‚ΠΎ-Ρ‚ΠΎ Π·Π°ΠΌΠ΅Π½ΠΈΠ» Π½Π°ΡˆΡƒ Π±Π°Ρ‚Π°Ρ€Π΅ΡŽ, ΠΈ Π½Π°ΠΌ Π½ΡƒΠΆΠ½ΠΎ Π²Ρ‹ΡΡΠ½ΠΈΡ‚ΡŒ Π΅Π΅ напряТСниС.ΠŸΡ€ΠΎΡ†Π΅ΡΡ ΠΏΠΎΡ‡Ρ‚ΠΈ Ρ‚Π°ΠΊΠΎΠΉ ΠΆΠ΅. ΠœΡ‹ Π·Π½Π°Π΅ΠΌ, Ρ‡Ρ‚ΠΎ наша новая Π»Π°ΠΌΠΏΠΎΡ‡ΠΊΠ° ΠΈΠΌΠ΅Π΅Ρ‚ сопротивлСниС 4 Ом, ΠΈ ΠΌΡ‹ ΠΌΠΎΠΆΠ΅ΠΌ ΠΈΠ·ΠΌΠ΅Ρ€ΠΈΡ‚ΡŒ Ρ‚ΠΎΠΊ Π² Ρ†Π΅ΠΏΠΈ с ΠΏΠΎΠΌΠΎΡ‰ΡŒΡŽ Π°ΠΌΠΏΠ΅Ρ€ΠΌΠ΅Ρ‚Ρ€Π°. Π˜ΡΠΏΠΎΠ»ΡŒΠ·ΡƒΡ Π΄ΠΈΠ°Π³Ρ€Π°ΠΌΠΌΡƒ, ΠΌΡ‹ ΠΏΠΎΠΊΡ€Ρ‹Π²Π°Π΅ΠΌ напряТСниС v , ΠΊΠΎΡ‚ΠΎΡ€ΠΎΠ΅ Π³ΠΎΠ²ΠΎΡ€ΠΈΡ‚ Π½Π°ΠΌ, Ρ‡Ρ‚ΠΎ Π½Π°ΠΌ Π½ΡƒΠΆΠ½ΠΎ ΡƒΠΌΠ½ΠΎΠΆΠΈΡ‚ΡŒ Ρ‚ΠΎΠΊ Π½Π° сопротивлСниС. Если Π±Ρ‹ Π°ΠΌΠΏΠ΅Ρ€ΠΌΠ΅Ρ‚Ρ€ ΠΈΠ·ΠΌΠ΅Ρ€ΠΈΠ» Ρ‚ΠΎΠΊ Π² 3 Π°ΠΌΠΏΠ΅Ρ€Π°, Ρ‚ΠΎΠ³Π΄Π° напряТСниС Π±Ρ‹Π»ΠΎ Π±Ρ‹ 3 Π°ΠΌΠΏΠ΅Ρ€Π°, ΡƒΠΌΠ½ΠΎΠΆΠ΅Π½Π½Ρ‹ΠΌ Π½Π° 4 Ом, Ρ‡Ρ‚ΠΎ составляСт 12 Π²ΠΎΠ»ΡŒΡ‚. Π’ΠΎΡ‚ ΠΈ всС. Зная Π»ΡŽΠ±Ρ‹Π΅ Π΄Π²Π° ΠΈΠ· Ρ‚Ρ€Π΅Ρ… ΠΏΠ°Ρ€Π°ΠΌΠ΅Ρ‚Ρ€ΠΎΠ², ΠΌΡ‹ всСгда ΠΌΠΎΠΆΠ΅ΠΌ Π²Ρ‹Ρ‡ΠΈΡΠ»ΠΈΡ‚ΡŒ Ρ‚Ρ€Π΅Ρ‚ΠΈΠΉ, ΠΈΡΠΏΠΎΠ»ΡŒΠ·ΡƒΡ Π·Π°ΠΊΠΎΠ½ Ома.

РСзюмС ΡƒΡ€ΠΎΠΊΠ°

Π—Π°ΠΊΠΎΠ½ Ома опрСдСляСт ΡΠΎΠΎΡ‚Π½ΠΎΡˆΠ΅Π½ΠΈΠ΅ ΠΌΠ΅ΠΆΠ΄Ρƒ напряТСниСм, Ρ‚ΠΎΠΊΠΎΠΌ ΠΈ сопротивлСниСм Π² элСктричСской Ρ†Π΅ΠΏΠΈ: i = v / r . Π’ΠΎΠΊ прямо ΠΏΡ€ΠΎΠΏΠΎΡ€Ρ†ΠΈΠΎΠ½Π°Π»Π΅Π½ Π½Π°ΠΏΡ€ΡΠΆΠ΅Π½ΠΈΡŽ ΠΈ ΠΎΠ±Ρ€Π°Ρ‚Π½ΠΎ ΠΏΡ€ΠΎΠΏΠΎΡ€Ρ†ΠΈΠΎΠ½Π°Π»Π΅Π½ ΡΠΎΠΏΡ€ΠΎΡ‚ΠΈΠ²Π»Π΅Π½ΠΈΡŽ. Π­Ρ‚ΠΎ ΠΎΠ·Π½Π°Ρ‡Π°Π΅Ρ‚, Ρ‡Ρ‚ΠΎ ΡƒΠ²Π΅Π»ΠΈΡ‡Π΅Π½ΠΈΠ΅ напряТСния ΠΏΡ€ΠΈΠ²Π΅Π΄Π΅Ρ‚ ΠΊ ΡƒΠ²Π΅Π»ΠΈΡ‡Π΅Π½ΠΈΡŽ Ρ‚ΠΎΠΊΠ°, Π° ΡƒΠ²Π΅Π»ΠΈΡ‡Π΅Π½ΠΈΠ΅ сопротивлСния ΠΏΡ€ΠΈΠ²Π΅Π΄Π΅Ρ‚ ΠΊ ΡƒΠΌΠ΅Π½ΡŒΡˆΠ΅Π½ΠΈΡŽ Ρ‚ΠΎΠΊΠ°. Зная Π»ΡŽΠ±Ρ‹Π΅ Π΄Π²Π° ΠΈΠ· Ρ‚Ρ€Π΅Ρ… ΠΏΠ°Ρ€Π°ΠΌΠ΅Ρ‚Ρ€ΠΎΠ², ΠΌΡ‹ ΠΌΠΎΠΆΠ΅ΠΌ Π²Ρ‹Ρ‡ΠΈΡΠ»ΠΈΡ‚ΡŒ Ρ‚Ρ€Π΅Ρ‚ΠΈΠΉ, нСизвСстный ΠΏΠ°Ρ€Π°ΠΌΠ΅Ρ‚Ρ€.ΠœΡ‹ ΠΌΠΎΠΆΠ΅ΠΌ ΡΠ΄Π΅Π»Π°Ρ‚ΡŒ это, пСрСставив Ρ‡Π»Π΅Π½Ρ‹ Π² ΡƒΡ€Π°Π²Π½Π΅Π½ΠΈΠΈ Π·Π°ΠΊΠΎΠ½Π° Ома ΠΈΠ»ΠΈ ΠΈΡΠΏΠΎΠ»ΡŒΠ·ΡƒΡ Π΄ΠΈΠ°Π³Ρ€Π°ΠΌΠΌΡƒ, ΠΏΡ€ΠΈΠ²Π΅Π΄Π΅Π½Π½ΡƒΡŽ Π²Ρ‹ΡˆΠ΅ Π² ΡƒΡ€ΠΎΠΊΠ΅. Π‘ΠΊΡ€Ρ‹Ρ‚ΠΈΠ΅ ΠΏΠ°Ρ€Π°ΠΌΠ΅Ρ‚Ρ€Π°, ΠΊΠΎΡ‚ΠΎΡ€Ρ‹ΠΉ ΠΌΡ‹ пытаСмся Π½Π°ΠΉΡ‚ΠΈ, ΠΏΠΎΠΊΠ°Π·Ρ‹Π²Π°Π΅Ρ‚ Π½Π°ΠΌ ΡΠΎΠΎΡ‚Π²Π΅Ρ‚ΡΡ‚Π²ΡƒΡŽΡ‰Π΅Π΅ ΡƒΡ€Π°Π²Π½Π΅Π½ΠΈΠ΅ с использованиСм Π΄Π²ΡƒΡ… извСстных ΠΏΠ°Ρ€Π°ΠΌΠ΅Ρ‚Ρ€ΠΎΠ².

Π Π΅Π·ΡƒΠ»ΡŒΡ‚Π°Ρ‚Ρ‹ обучСния

По Π·Π°Π²Π΅Ρ€ΡˆΠ΅Π½ΠΈΠΈ этого ΡƒΡ€ΠΎΠΊΠ° Π²Ρ‹ смоТСтС:

  • ΠžΠΏΠΈΡΡ‹Π²Π°Ρ‚ΡŒ взаимосвязь ΠΌΠ΅ΠΆΠ΄Ρƒ напряТСниСм, Ρ‚ΠΎΠΊΠΎΠΌ ΠΈ сопротивлСниСм, ΠΈΡΠΏΠΎΠ»ΡŒΠ·ΡƒΡ Π·Π°ΠΊΠΎΠ½ Ома
  • ΠΠ°ΠΏΠΈΡˆΠΈΡ‚Π΅ ΡƒΡ€Π°Π²Π½Π΅Π½ΠΈΠ΅ Π·Π°ΠΊΠΎΠ½Π° Ома
  • ΠžΠ±ΡŠΡΡΠ½ΠΈΡ‚Π΅, ΠΊΠ°ΠΊ ΠΌΠΎΠΆΠ½ΠΎ Π½Π°ΠΉΡ‚ΠΈ Π»ΡŽΠ±ΡƒΡŽ ΠΈΠ· Ρ‚Ρ€Π΅Ρ… ΠΏΠ΅Ρ€Π΅ΠΌΠ΅Π½Π½Ρ‹Ρ… Π² ΡƒΡ€Π°Π²Π½Π΅Π½ΠΈΠΈ Π·Π°ΠΊΠΎΠ½Π° Ома, Ссли Π²Ρ‹ Π·Π½Π°Π΅Ρ‚Π΅ Π΄Π²Π΅ Π΄Ρ€ΡƒΠ³ΠΈΠ΅
  • РассчитайтС Π»ΡŽΠ±ΡƒΡŽ ΠΈΠ· Ρ‚Ρ€Π΅Ρ… ΠΏΠ΅Ρ€Π΅ΠΌΠ΅Π½Π½Ρ‹Ρ…, ΠΈΡΠΏΠΎΠ»ΡŒΠ·ΡƒΡ ΡƒΡ€Π°Π²Π½Π΅Π½ΠΈΠ΅ Π·Π°ΠΊΠΎΠ½Π° Ома

Π’Π·Π°ΠΈΠΌΠΎΡΠ²ΡΠ·ΡŒ ΠΈ Ρ€Π°Π·Π½ΠΈΡ†Π° ΠΌΠ΅ΠΆΠ΄Ρƒ напряТСниСм, Ρ‚ΠΎΠΊΠΎΠΌ ΠΈ сопротивлСниСм —

ΠŸΡ€ΠΈΡΡ‚ΡƒΠΏΠ°Ρ ΠΊ исслСдованию ΠΌΠΈΡ€Π° элСктроники ΠΈ элСктричСства, ΠΎΡ‡Π΅Π½ΡŒ Π²Π°ΠΆΠ½ΠΎ Π½Π°Ρ‡Π°Ρ‚ΡŒ с понимания основ напряТСния (v), Ρ‚ΠΎΠΊΠ° (c) ΠΈ сопротивлСния (R) .Π­Ρ‚ΠΈ Ρ‚Ρ€ΠΈ Ρ„ΡƒΠ½Π΄Π°ΠΌΠ΅Π½Ρ‚Π°Π»ΡŒΠ½Ρ‹Ρ… ΠΏΡ€ΠΈΠ½Ρ†ΠΈΠΏΠ° ΡΠ²Π»ΡΡŽΡ‚ΡΡ основными ΡΡ‚Ρ€ΠΎΠΈΡ‚Π΅Π»ΡŒΠ½Ρ‹ΠΌΠΈ Π±Π»ΠΎΠΊΠ°ΠΌΠΈ, Π½Π΅ΠΎΠ±Ρ…ΠΎΠ΄ΠΈΠΌΡ‹ΠΌΠΈ для использования элСктричСства. ΠŸΠΎΠ½Π°Ρ‡Π°Π»Ρƒ эти ΠΊΠΎΠ½Ρ†Π΅ΠΏΡ†ΠΈΠΈ ΠΌΠΎΠ³ΡƒΡ‚ Π±Ρ‹Ρ‚ΡŒ Ρ‚Ρ€ΡƒΠ΄Π½Ρ‹ΠΌΠΈ для понимания, ΠΏΠΎΡ‚ΠΎΠΌΡƒ Ρ‡Ρ‚ΠΎ ΠΌΡ‹ Π½Π΅ ΠΌΠΎΠΆΠ΅ΠΌ ΠΈΡ… Π²ΠΈΠ΄Π΅Ρ‚ΡŒ. НСвозмоТно ΡƒΠ²ΠΈΠ΄Π΅Ρ‚ΡŒ ΠΏΠΎΡ‚ΠΎΠΊ энСргии ΠΏΠΎ ΠΏΡ€ΠΎΠ²ΠΎΠ΄Ρƒ Π³Π»Π°Π·Π°ΠΌΠΈ.

Π”Π°ΠΆΠ΅ молния Π² Π½Π΅Π±Π΅, хотя ΠΈ видимая, Π½Π° самом Π΄Π΅Π»Π΅ Π½Π΅ являСтся ΠΎΠ±ΠΌΠ΅Π½ΠΎΠΌ энСргиСй, происходящим ΠΎΡ‚ ΠΎΠ±Π»Π°ΠΊΠ° ΠΊ ΠΏΠ»Π°Π½Π΅Ρ‚Π΅. Однако это рСакция Π²ΠΎΠ·Π΄ΡƒΡ…Π° Π½Π° ΡΠ½Π΅Ρ€Π³ΠΈΡŽ, ΠΏΡ€ΠΎΡ…ΠΎΠ΄ΡΡ‰ΡƒΡŽ Ρ‡Π΅Ρ€Π΅Π· Π½Π΅Π³ΠΎ. Π˜Ρ‚Π°ΠΊ, Ρ‡Ρ‚ΠΎΠ±Ρ‹ Π·Π°ΠΌΠ΅Ρ‚ΠΈΡ‚ΡŒ эту ΠΏΠ΅Ρ€Π΅Π΄Π°Ρ‡Ρƒ энСргии, ΠΌΡ‹ Π΄ΠΎΠ»ΠΆΠ½Ρ‹ ΠΈΡΠΏΠΎΠ»ΡŒΠ·ΠΎΠ²Π°Ρ‚ΡŒ инструмСнты измСрСния, Ρ‚Π°ΠΊΠΈΠ΅ ΠΊΠ°ΠΊ Π°Π½Π°Π»ΠΈΠ·Π°Ρ‚ΠΎΡ€ спСктра, ΠΌΡƒΠ»ΡŒΡ‚ΠΈΠΌΠ΅Ρ‚Ρ€ ΠΈ осциллограф.Π­Ρ‚ΠΎΡ‚ осциллограф ΠΈΡΠΏΠΎΠ»ΡŒΠ·ΡƒΠ΅Ρ‚ΡΡ для Π²ΠΈΠ·ΡƒΠ°Π»ΠΈΠ·Π°Ρ†ΠΈΠΈ Ρ‚ΠΎΠ³ΠΎ, Ρ‡Ρ‚ΠΎ происходит с зарядом Π² систСмС. Π­Ρ‚Π° ΡΡ‚Π°Ρ‚ΡŒΡ расскаТСт Π²Π°ΠΌ ΠΎΠ± основных ΠΎΡ‚Π½ΠΎΡˆΠ΅Π½ΠΈΡΡ… ΠΈ различиях ΠΌΠ΅ΠΆΠ΄Ρƒ напряТСниСм ΠΈ Ρ‚ΠΎΠΊΠΎΠΌ, Π° Ρ‚Π°ΠΊΠΆΠ΅ ΠΎ ΡΠΎΠΎΡ‚Π½ΠΎΡˆΠ΅Π½ΠΈΠΈ ΠΌΠ΅ΠΆΠ΄Ρƒ Ρ‚ΠΎΠΊΠΎΠΌ ΠΈ напряТСниСм

Π’Π·Π°ΠΈΠΌΠΎΡΠ²ΡΠ·ΡŒ ΠΌΠ΅ΠΆΠ΄Ρƒ напряТСниСм, Ρ‚ΠΎΠΊΠΎΠΌ ΠΈ сопротивлСниСм

Основная взаимосвязь ΠΌΠ΅ΠΆΠ΄Ρƒ V, I ΠΈ R Π² элСктричСской Ρ†Π΅ΠΏΠΈ называСтся Π·Π°ΠΊΠΎΠ½ΠΎΠΌ Ома. ВсС ΠΌΠ°Ρ‚Π΅Ρ€ΠΈΠ°Π»Ρ‹ состоят ΠΈΠ· Π°Ρ‚ΠΎΠΌΠΎΠ², ΠΊΠ°ΠΆΠ΄Ρ‹ΠΉ Π°Ρ‚ΠΎΠΌ состоит ΠΈΠ· ΠΏΡ€ΠΎΡ‚ΠΎΠ½ΠΎΠ², Π½Π΅ΠΉΡ‚Ρ€ΠΎΠ½ΠΎΠ² ΠΈ элСктронов. Если ΠΏΡ€ΠΎΡ‚ΠΎΠ½Ρ‹ ΠΈΠΌΠ΅ΡŽΡ‚ ΠΏΠΎΠ»ΠΎΠΆΠΈΡ‚Π΅Π»ΡŒΠ½Ρ‹ΠΉ элСктричСский заряд, Π½Π΅ΠΉΡ‚Ρ€ΠΎΠ½Ρ‹ Π½Π΅ ΠΈΠΌΠ΅ΡŽΡ‚ элСктричСского заряда, Π° элСктроны ΠΈΠΌΠ΅ΡŽΡ‚ ΠΎΡ‚Ρ€ΠΈΡ†Π°Ρ‚Π΅Π»ΡŒΠ½Ρ‹ΠΉ элСктричСский заряд.Π­Ρ‚ΠΈ Ρ‚Ρ€ΠΎΠ΅ вмСстС Π² Π°Ρ‚ΠΎΠΌΠ΅. Но Ссли ΠΌΡ‹ ΠΎΡ‚Π΄Π΅Π»ΠΈΠΌ ΠΈΡ… Π΄Ρ€ΡƒΠ³ ΠΎΡ‚ Π΄Ρ€ΡƒΠ³Π°, ΠΎΠ½ΠΈ захотят Ρ€Π΅Ρ„ΠΎΡ€ΠΌΠΈΡ€ΠΎΠ²Π°Ρ‚ΡŒΡΡ, Ρ‡Ρ‚ΠΎΠ±Ρ‹ ΠΏΡ€ΠΎΡΠ²ΠΈΡ‚ΡŒ ΠΏΠΎΡ‚Π΅Π½Ρ†ΠΈΠ°Π» притяТСния, Π½Π°Π·Ρ‹Π²Π°Π΅ΠΌΡ‹ΠΉ Ρ€Π°Π·Π½ΠΎΡΡ‚ΡŒΡŽ ΠΏΠΎΡ‚Π΅Π½Ρ†ΠΈΠ°Π»ΠΎΠ².

Когда ΠΌΡ‹ строим Π·Π°ΠΌΠΊΠ½ΡƒΡ‚ΡƒΡŽ Ρ†Π΅ΠΏΡŒ, эти элСктроны двиТутся ΠΈ Π²ΠΎΠ·Π²Ρ€Π°Ρ‰Π°ΡŽΡ‚ΡΡ ΠΊ ΠΏΡ€ΠΎΡ‚ΠΎΠ½Π°ΠΌ ΠΈΠ·-Π·Π° ΠΈΡ… притяТСния, Ρ‡Ρ‚ΠΎΠ±Ρ‹ ΡΠΎΠ·Π΄Π°Ρ‚ΡŒ ΠΏΠΎΡ‚ΠΎΠΊ элСктронов, это называСтся элСктричСским Ρ‚ΠΎΠΊΠΎΠΌ. Π­Π»Π΅ΠΊΡ‚Ρ€ΠΎΠ½Ρ‹ Π½Π΅ Ρ‚Π΅ΠΊΡƒΡ‚ свободно ΠΈΠ·-Π·Π° ограничСния ΠΏΠΎΡ‚ΠΎΠΊΠ° элСктронов, это называСтся сопротивлСниСм.

Π’ΠΎΠ³Π΄Π° всС основныС схСмы состоят ΠΈΠ· Ρ‚Ρ€Π΅Ρ… ΠΎΡ‚Π΄Π΅Π»ΡŒΠ½Ρ‹Ρ… Π²Π΅Π»ΠΈΡ‡ΠΈΠ½, Π° ΠΈΠΌΠ΅Π½Π½ΠΎ напряТСния, Ρ‚ΠΎΠΊΠ° ΠΈ сопротивлСния.

ЭлСктричСский заряд

ЭлСктричСство — это Π΄Π²ΠΈΠΆΠ΅Π½ΠΈΠ΅ элСктронов, ΠΎΠ½ΠΎ создаСт заряд, ΠΊΠΎΡ‚ΠΎΡ€Ρ‹ΠΉ ΠΌΡ‹ ΠΌΠΎΠΆΠ΅ΠΌ ΠΏΠΎΠ΄ΠΊΠ»ΡŽΡ‡ΠΈΡ‚ΡŒ для выполнСния Ρ€Π°Π±ΠΎΡ‚Ρ‹, вашСго свСта, Ρ‚Π΅Π»Π΅Ρ„ΠΎΠ½Π°, стСрСосистСмы ΠΈ Ρ‚. Π”. ВсС ΠΎΠ½ΠΈ Ρ€Π°Π±ΠΎΡ‚Π°ΡŽΡ‚ с использованиСм основного источника энСргии, Ρ‚ΠΎ Π΅ΡΡ‚ΡŒ двиТСния элСктронов. Π’Ρ€ΠΈ основных ΠΏΡ€ΠΈΠ½Ρ†ΠΈΠΏΠ°, Ρ‚Π°ΠΊΠΈΠ΅ ΠΊΠ°ΠΊ напряТСниС, Ρ‚ΠΎΠΊ ΠΈ сопротивлСниС, ΠΌΠΎΠΆΠ½ΠΎ ΠΎΠ±ΡΡƒΠ΄ΠΈΡ‚ΡŒ, ΠΈΡΠΏΠΎΠ»ΡŒΠ·ΡƒΡ элСктроны ΠΈΠ»ΠΈ, Ρ‚ΠΎΡ‡Π½Π΅Π΅, заряд, ΠΊΠΎΡ‚ΠΎΡ€Ρ‹ΠΉ ΠΎΠ½ΠΈ ΡΠΎΠ·Π΄Π°ΡŽΡ‚. ОсновноС Ρ€Π°Π·Π»ΠΈΡ‡ΠΈΠ΅ ΠΌΠ΅ΠΆΠ΄Ρƒ Ρ‚ΠΎΠΊΠΎΠΌ ΠΈ напряТСниСм Π·Π°ΠΊΠ»ΡŽΡ‡Π°Π΅Ρ‚ΡΡ Π² Ρ‚ΠΎΠΌ, Ρ‡Ρ‚ΠΎ Ссли ΠΏΡ€ΠΈΠ»ΠΎΠΆΠΈΡ‚ΡŒ Ρ€Π°Π·Π½ΠΎΡΡ‚ΡŒ ΠΏΠΎΡ‚Π΅Π½Ρ†ΠΈΠ°Π»ΠΎΠ² ΠΌΠ΅ΠΆΠ΄Ρƒ двумя Ρ‚ΠΎΡ‡ΠΊΠ°ΠΌΠΈ Π² любом ΠΌΠ°Ρ‚Π΅Ρ€ΠΈΠ°Π»Π΅, Π² ΠΏΡ€ΠΈΠ½Ρ†ΠΈΠΏΠ΅ ΠΌΠΎΠΆΠ΅Ρ‚ ΡΡƒΡ‰Π΅ΡΡ‚Π²ΠΎΠ²Π°Ρ‚ΡŒ Ρ‚ΠΎΠΊ.

  • НапряТСниС опрСдСляСтся ΠΊΠ°ΠΊ Ρ€Π°Π·Π½ΠΎΡΡ‚ΡŒ ΠΏΠΎΡ‚Π΅Π½Ρ†ΠΈΠ°Π»ΠΎΠ² ΠΌΠ΅ΠΆΠ΄Ρƒ двумя Ρ‚ΠΎΡ‡ΠΊΠ°ΠΌΠΈ элСктричСского заряда.
  • Π’ΠΎΠΊ — это ΠΏΠΎΡ‚ΠΎΠΊ элСктронов
  • Π‘ΠΎΠΏΡ€ΠΎΡ‚ΠΈΠ²Π»Π΅Π½ΠΈΠ΅ опрСдСляСтся ΠΊΠ°ΠΊ ΡΠΏΠΎΡΠΎΠ±Π½ΠΎΡΡ‚ΡŒ ΠΌΠ°Ρ‚Π΅Ρ€ΠΈΠ°Π»Π° ΠΎΠ³Ρ€Π°Π½ΠΈΡ‡ΠΈΠ²Π°Ρ‚ΡŒ ΠΏΡ€ΠΎΡ…ΠΎΠΆΠ΄Π΅Π½ΠΈΠ΅ Ρ‚ΠΎΠΊΠ°.

Π˜Ρ‚Π°ΠΊ, ΠΊΠΎΠ³Π΄Π° ΠΌΡ‹ обсуТдаСм эти значСния, ΠΏΠΎΠ²Π΅Π΄Π΅Π½ΠΈΠ΅ элСктронов Π² Π·Π°ΠΌΠΊΠ½ΡƒΡ‚ΠΎΠΉ Ρ†Π΅ΠΏΠΈ позволяСт заряду ΠΏΠ΅Ρ€Π΅ΠΌΠ΅Ρ‰Π°Ρ‚ΡŒΡΡ ΠΈΠ· ΠΎΠ΄Π½ΠΎΠ³ΠΎ мСста Π² Π΄Ρ€ΡƒΠ³ΠΎΠ΅. ΠžΡΠ½ΠΎΠ²Π½Ρ‹Π΅ ΠΊΠΎΠΌΠΏΠΎΠ½Π΅Π½Ρ‚Ρ‹, ΠΈΡΠΏΠΎΠ»ΡŒΠ·ΡƒΠ΅ΠΌΡ‹Π΅ Π² схСмС, ΠΏΠΎΠ·Π²ΠΎΠ»ΡΡŽΡ‚ Π½Π°ΠΌ ΠΊΠΎΠ½Ρ‚Ρ€ΠΎΠ»ΠΈΡ€ΠΎΠ²Π°Ρ‚ΡŒ заряд ΠΈ ΠΈΡΠΏΠΎΠ»ΡŒΠ·ΠΎΠ²Π°Ρ‚ΡŒ Π΅Π³ΠΎ для Ρ€Π°Π±ΠΎΡ‚Ρ‹.Π˜Ρ‚Π°ΠΊ, баварский ΡƒΡ‡Π΅Π½Ρ‹ΠΉ Π“Π΅ΠΎΡ€Π³ Ом ΠΈΠ·ΡƒΡ‡Π°Π» элСктричСство. Он описал Π΅Π΄ΠΈΠ½ΠΈΡ†Ρƒ сопротивлСния, которая опрСдСляСтся напряТСниСм ΠΈ Ρ‚ΠΎΠΊΠΎΠΌ. Π Π°Π·Π½ΠΈΡ†Π° ΠΌΠ΅ΠΆΠ΄Ρƒ напряТСниСм ΠΈ Ρ‚ΠΎΠΊΠΎΠΌ ΠΈ сопротивлСниСм обсуТдаСтся Π½ΠΈΠΆΠ΅.

Π—Π°ΠΊΠΎΠ½ Ома

Π—Π°ΠΊΠΎΠ½

Ома опрСдСляСтся ΠΊΠ°ΠΊ ΡΠΎΠΎΡ‚Π½ΠΎΡˆΠ΅Π½ΠΈΠ΅ ΠΌΠ΅ΠΆΠ΄Ρƒ трСмя Π²Π΅Π»ΠΈΡ‡ΠΈΠ½Π°ΠΌΠΈ, Ρ‚Π°ΠΊΠΈΠΌΠΈ ΠΊΠ°ΠΊ сопротивлСниС, напряТСниС ΠΈ Ρ‚ΠΎΠΊ. Он выводится ΠΏΠΎ Ρ„ΠΎΡ€ΠΌΡƒΠ»Π΅ V = IR, Π³Π΄Π΅

Π—Π°ΠΊΠΎΠ½ Ома

  • НапряТСниС V измСряСтся Π² Π²ΠΎΠ»ΡŒΡ‚Π°Ρ…
  • Π’ΠΎΠΊ I измСряСтся Π² Π°ΠΌΠΏΠ΅Ρ€Π°Ρ…
  • Π‘ΠΎΠΏΡ€ΠΎΡ‚ΠΈΠ²Π»Π΅Π½ΠΈΠ΅ R измСряСтся Π² Ом.

Π’ этом ΡƒΡ€Π°Π²Π½Π΅Π½ΠΈΠΈ напряТСниС Ρ€Π°Π²Π½ΠΎ Ρ‚ΠΎΠΊΡƒ, ΡƒΠΌΠ½ΠΎΠΆΠ΅Π½Π½ΠΎΠΌΡƒ Π½Π° сопротивлСниС. Π˜ΡΠΏΠΎΠ»ΡŒΠ·ΡƒΡ ΠΌΠ΅Ρ‚ΠΎΠ΄Ρ‹ Π°Π»Π³Π΅Π±Ρ€Ρ‹, ΠΌΡ‹ ΠΌΠΎΠΆΠ΅ΠΌ ΠΈΡΠΏΠΎΠ»ΡŒΠ·ΠΎΠ²Π°Ρ‚ΡŒ это ΡƒΡ€Π°Π²Π½Π΅Π½ΠΈΠ΅ Π² Π΄Π²ΡƒΡ… Π²Π°Ρ€ΠΈΠ°Π½Ρ‚Π°Ρ…, ΠΎΡ‚Π΄Π΅Π»ΡŒΠ½ΠΎ для Ρ€Π΅ΡˆΠ΅Π½ΠΈΡ Ρ‚ΠΎΠΊΠ° ΠΈ напряТСния.

Π’ = ИК

Из ΠΏΡ€ΠΈΠ²Π΅Π΄Π΅Π½Π½ΠΎΠ³ΠΎ Π²Ρ‹ΡˆΠ΅ уравнСния Π·Π°ΠΊΠΎΠ½Π° Ома ΠΌΡ‹ ΠΌΠΎΠΆΠ΅ΠΌ Ρ€Π°ΡΡΡ‡ΠΈΡ‚Π°Ρ‚ΡŒ значСния Ρ‚ΠΎΠΊΠ° ΠΈ напряТСния, ΠΈΡΠΏΠΎΠ»ΡŒΠ·ΡƒΡ ΡΠ»Π΅Π΄ΡƒΡŽΡ‰ΠΈΠ΅ уравнСния.
I = V / R
R = V / I
Π’ ΠΏΡ€ΠΈΠ²Π΅Π΄Π΅Π½Π½ΠΎΠΉ Π½ΠΈΠΆΠ΅ схСмС Π΅ΡΡ‚ΡŒ источники напряТСния, сопротивлСния ΠΈ Ρ‚ΠΎΠΊΠ°, Π° ΠΈΠΌΠ΅Π½Π½ΠΎ аккумулятор для напряТСния, Π»Π°ΠΌΠΏΠ° являСтся Ρ‚ΠΎΠ»ΡŒΠΊΠΎ ΠΎΠ΄Π½ΠΈΠΌ источником сопротивлСния ΠΈ Ρ‚ΠΎΠΊΠ°.Π­Ρ‚ΠΎ ΡƒΠΏΡ€ΠΎΡ‰Π°Π΅Ρ‚ ΠΏΡ€ΠΈΠΌΠ΅Π½Π΅Π½ΠΈΠ΅ Π·Π°ΠΊΠΎΠ½Π° Ома. Если ΠΌΡ‹ Π·Π½Π°Π΅ΠΌ Π»ΡŽΠ±Ρ‹Π΅ Π΄Π²Π° значСния напряТСния, Ρ‚ΠΎΠΊΠ° ΠΈ сопротивлСния, ΠΌΡ‹ ΠΌΠΎΠΆΠ΅ΠΌ Π²Ρ‹Ρ‡ΠΈΡΠ»ΠΈΡ‚ΡŒ Ρ‚Ρ€Π΅Ρ‚ΡŒΠ΅, ΠΈΡΠΏΠΎΠ»ΡŒΠ·ΡƒΡ Π·Π°ΠΊΠΎΠ½ Ома.

ΠŸΡ€ΠΈΠ½Ρ†ΠΈΠΏΠΈΠ°Π»ΡŒΠ½Π°Ρ элСктричСская схСма V, I ΠΈ R

Π’ ΠΏΡ€ΠΈΠ²Π΅Π΄Π΅Π½Π½ΠΎΠΉ Π²Ρ‹ΡˆΠ΅ схСмС, ΠΊΠΎΠ³Π΄Π° ΡƒΠΊΠ°Π·Π°Π½Ρ‹ значСния напряТСния ΠΈ сопротивлСния, ΠΌΡ‹ ΠΌΠΎΠΆΠ΅ΠΌ Π²Ρ‹Ρ‡ΠΈΡΠ»ΠΈΡ‚ΡŒ Π²Π΅Π»ΠΈΡ‡ΠΈΠ½Ρƒ Ρ‚ΠΎΠΊΠ°. Π’Π΅Π»ΠΈΡ‡ΠΈΠ½Π° Ρ‚ΠΎΠΊΠ° Π² ΡƒΠΊΠ°Π·Π°Π½Π½ΠΎΠΉ Π²Ρ‹ΡˆΠ΅ Ρ†Π΅ΠΏΠΈ составляСт
Π’ = 12 Π’, R = 3 Ом
I = V / R => 12 Π’ / 3 Ом = 4 А

Π Π°Π·Π½ΠΈΡ†Π° ΠΌΠ΅ΠΆΠ΄Ρƒ напряТСниСм, Ρ‚ΠΎΠΊΠΎΠΌ ΠΈ сопротивлСниСм

ОсновноС Ρ€Π°Π·Π»ΠΈΡ‡ΠΈΠ΅ ΠΌΠ΅ΠΆΠ΄Ρƒ Π½ΠΈΠΌΠΈ Π² основном связано с ΠΎΠΏΡ€Π΅Π΄Π΅Π»Π΅Π½ΠΈΠ΅ΠΌ напряТСния, сопротивлСния ΠΈ Ρ‚ΠΎΠΊΠ°.Различия ΠΌΠ΅ΠΆΠ΄Ρƒ V, I ΠΈ R ΠΎΠ±ΡΡƒΠΆΠ΄Π°ΡŽΡ‚ΡΡ Π½ΠΈΠΆΠ΅.

НапряТСниС опрСдСляСтся ΠΊΠ°ΠΊ Ρ€Π°Π·Π½ΠΎΡΡ‚ΡŒ зарядов ΠΌΠ΅ΠΆΠ΄Ρƒ двумя Ρ‚ΠΎΡ‡ΠΊΠ°ΠΌΠΈ Ρ†Π΅ΠΏΠΈ, Ρ‚Π°ΠΊΠΆΠ΅ называСмая элСктродвиТущСй силой. Он измСряСтся Π² Π²ΠΎΠ»ΡŒΡ‚Π°Ρ… (1 Π’ = 1 Π΄ΠΆΠΎΡƒΠ»ΡŒ / ΠΊΡƒΠ»ΠΎΠ½ (Π’ = Π’Ρ‚ / Кл)). Одна Ρ‚ΠΎΡ‡ΠΊΠ° ΠΈΠΌΠ΅Π΅Ρ‚ большС заряда, Ρ‡Π΅ΠΌ другая. Π•Π΄ΠΈΠ½ΠΈΡ†Π° Π²ΠΎΠ»ΡŒΡ‚ Π½Π°Π·Π²Π°Π½Π° Π² Ρ‡Π΅ΡΡ‚ΡŒ изобрСтСния ΠΈΡ‚Π°Π»ΡŒΡΠ½ΡΠΊΠΎΠ³ΠΎ Ρ„ΠΈΠ·ΠΈΠΊΠ° АлСссандро Π’ΠΎΠ»ΡŒΡ‚Π°. Π’Π΅Ρ€ΠΌΠΈΠ½ «Π²ΠΎΠ»ΡŒΡ‚» Π½Π° схСмах ΠΎΠ±ΠΎΠ·Π½Π°Ρ‡Π΅Π½ Π±ΡƒΠΊΠ²ΠΎΠΉ V. Π˜Π·ΠΌΠ΅Ρ€ΠΈΡ‚Π΅Π»Π΅ΠΌ напряТСния являСтся Π²ΠΎΠ»ΡŒΡ‚ΠΌΠ΅Ρ‚Ρ€.НапряТСниС — это источник, Π° Ρ‚ΠΎΠΊ — Π΅Π³ΠΎ Ρ€Π΅Π·ΡƒΠ»ΡŒΡ‚Π°Ρ‚, это ΠΌΠΎΠΆΠ΅Ρ‚ ΠΏΡ€ΠΎΠΈΡΡ…ΠΎΠ΄ΠΈΡ‚ΡŒ Π±Π΅Π· Ρ‚ΠΎΠΊΠ°. НапряТСниС распрСдСляСтся ΠΏΠΎ Ρ€Π°Π·Π»ΠΈΡ‡Π½Ρ‹ΠΌ элСктронным ΠΊΠΎΠΌΠΏΠΎΠ½Π΅Π½Ρ‚Π°ΠΌ, ΠΊΠΎΡ‚ΠΎΡ€Ρ‹Π΅ соСдинСны ΠΏΠΎΡΠ»Π΅Π΄ΠΎΠ²Π°Ρ‚Π΅Π»ΡŒΠ½ΠΎ Π² Ρ†Π΅ΠΏΠΈ, ΠΈ напряТСниС Π² ΠΏΠ°Ρ€Π°Π»Π»Π΅Π»ΡŒΠ½ΠΎΠΉ Ρ†Π΅ΠΏΠΈ ΠΎΠ΄ΠΈΠ½Π°ΠΊΠΎΠ²ΠΎ для всСх ΠΊΠΎΠΌΠΏΠΎΠ½Π΅Π½Ρ‚ΠΎΠ², ΠΊΠΎΡ‚ΠΎΡ€Ρ‹Π΅ соСдинСны ΠΏΠ°Ρ€Π°Π»Π»Π΅Π»ΡŒΠ½ΠΎ.

Π’ΠΎΠΊ опрСдСляСтся ΠΊΠ°ΠΊ ΡΠΊΠΎΡ€ΠΎΡΡ‚ΡŒ протСкания элСктричСского заряда Π² Ρ†Π΅ΠΏΠΈ. ΠžΠ±ΠΎΠ·Π½Π°Ρ‡Π°Π΅Ρ‚ΡΡ символом Β«IΒ». Π•Π΄ΠΈΠ½ΠΈΡ†Π° измСрСния Ρ‚ΠΎΠΊΠ° — Π°ΠΌΠΏΠ΅Ρ€ ΠΈΠ»ΠΈ сила Ρ‚ΠΎΠΊΠ°, Π° 1 Π°ΠΌΠΏΠ΅Ρ€ = 1 ΠΊΡƒΠ»ΠΎΠ½ Π² сСкунду. Π˜Π·ΠΌΠ΅Ρ€ΠΈΡ‚Π΅Π»ΡŒ силы Ρ‚ΠΎΠΊΠ° — Π°ΠΌΠΏΠ΅Ρ€ΠΌΠ΅Ρ‚Ρ€.Π’ΠΎΠΊ ΠΎΠ΄ΠΈΠ½Π°ΠΊΠΎΠ² Π²ΠΎ всСх ΠΏΠΎΡΠ»Π΅Π΄ΠΎΠ²Π°Ρ‚Π΅Π»ΡŒΠ½ΠΎ соСдинСнных ΠΊΠΎΠΌΠΏΠΎΠ½Π΅Π½Ρ‚Π°Ρ…. И Ρ‚ΠΎΠΊ распрСдСляСтся ΠΏΡ€ΠΈ ΠΏΠ°Ρ€Π°Π»Π»Π΅Π»ΡŒΠ½ΠΎΠΌ ΠΏΠΎΠ΄ΠΊΠ»ΡŽΡ‡Π΅Π½ΠΈΠΈ ΠΊΠΎΠΌΠΏΠΎΠ½Π΅Π½Ρ‚ΠΎΠ².

Π‘ΠΎΠΏΡ€ΠΎΡ‚ΠΈΠ²Π»Π΅Π½ΠΈΠ΅ опрСдСляСтся ΠΊΠ°ΠΊ сопротивлСниС, ΠΊΠΎΡ‚ΠΎΡ€ΠΎΠ΅ вСщСство ΠΎΠΊΠ°Π·Ρ‹Π²Π°Π΅Ρ‚ ΠΏΠΎΡ‚ΠΎΠΊΡƒ элСктричСского Ρ‚ΠΎΠΊΠ°. ΠžΠ±ΠΎΠ·Π½Π°Ρ‡Π°Π΅Ρ‚ΡΡ Π±ΡƒΠΊΠ²ΠΎΠΉ R. Π•Π΄ΠΈΠ½ΠΈΡ†Π΅ΠΉ измСрСния сопротивлСния являСтся ΠΎΠΌ, Π° ΠΈΠ·ΠΌΠ΅Ρ€ΠΈΡ‚Π΅Π»Π΅ΠΌ сопротивлСния — ΠΌΡƒΠ»ΡŒΡ‚ΠΈΠΌΠ΅Ρ‚Ρ€.

Π Π°Π·Π½ΠΈΡ†Π° ΠΌΠ΅ΠΆΠ΄Ρƒ напряТСниСм, Ρ‚ΠΎΠΊΠΎΠΌ ΠΈ сопротивлСниСм

ΠŸΡ€ΠΈ описании Ρ€Π°Π·Π½ΠΈΡ†Ρ‹ ΠΌΠ΅ΠΆΠ΄Ρƒ напряТСниСм, сопротивлСниСм ΠΈ Ρ‚ΠΎΠΊΠΎΠΌ ΠΏΠΎ ΠΎΠ±Ρ‰Π΅ΠΉ Π°Π½Π°Π»ΠΎΠ³ΠΈΠΈ ΠΈΡΠΏΠΎΠ»ΡŒΠ·ΡƒΠ΅Ρ‚ΡΡ Ρ€Π΅Π·Π΅Ρ€Π²ΡƒΠ°Ρ€ для Π²ΠΎΠ΄Ρ‹.Рассмотрим Ρ€Π΅Π·Π΅Ρ€Π²ΡƒΠ°Ρ€ для Π²ΠΎΠ΄Ρ‹ Π½Π° ΠΎΠΏΡ€Π΅Π΄Π΅Π»Π΅Π½Π½ΠΎΠΉ высотС ΠΎΡ‚ Π·Π΅ΠΌΠ»ΠΈ. Π’Π½ΠΈΠ·Ρƒ этого Ρ€Π΅Π·Π΅Ρ€Π²ΡƒΠ°Ρ€Π° для Π²ΠΎΠ΄Ρ‹ Π΅ΡΡ‚ΡŒ Ρ‚Ρ€ΡƒΠ±ΠΊΠ°. Π’ этом Ρ€Π΅Π·Π΅Ρ€Π²ΡƒΠ°Ρ€Π΅ заряд обозначаСтся количСством Π²ΠΎΠ΄Ρ‹, напряТСниС обозначаСтся Π΄Π°Π²Π»Π΅Π½ΠΈΠ΅ΠΌ Π²ΠΎΠ΄Ρ‹, Π° Ρ‚ΠΎΠΊ обозначаСтся ΠΏΠΎΡ‚ΠΎΠΊΠΎΠΌ Π²ΠΎΠ΄Ρ‹. Для этого Π·Π°ΠΏΠΎΠΌΠ½ΠΈΡ‚Π΅ ΡΠ»Π΅Π΄ΡƒΡŽΡ‰ΠΈΠ΅ Ρ‚Π΅Ρ€ΠΌΠΈΠ½Ρ‹: заряд — это Π²ΠΎΠ΄Π°, напряТСниС — это Π΄Π°Π²Π»Π΅Π½ΠΈΠ΅, Π° Ρ‚ΠΎΠΊ — это ΠΏΠΎΡ‚ΠΎΠΊ Π²ΠΎΠ΄Ρ‹.

ΠžΠ±Ρ‰ΠΈΠΉ Ρ€Π΅Π·Π΅Ρ€Π²ΡƒΠ°Ρ€ для Π²ΠΎΠ΄Ρ‹

Π­Ρ‚ΠΎ всС ΠΎ взаимосвязи ΠΈ Ρ€Π°Π·Π½ΠΈΡ†Π΅ ΠΌΠ΅ΠΆΠ΄Ρƒ напряТСниСм, Ρ‚ΠΎΠΊΠΎΠΌ ΠΈ сопротивлСниСм.Π’Π΅ΠΏΠ΅Ρ€ΡŒ Π²Ρ‹ Π΄ΠΎΠ»ΠΆΠ½Ρ‹ ΠΏΠΎΠ½ΡΡ‚ΡŒ основныС ΠΊΠΎΠ½Ρ†Π΅ΠΏΡ†ΠΈΠΈ этих Ρ‚Ρ€Π΅Ρ… Ρ‚Π΅Ρ€ΠΌΠΈΠ½ΠΎΠ² ΠΈ Ρ‚ΠΎ, ΠΊΠ°ΠΊ ΠΎΠ½ΠΈ связаны. Π—Π°ΠΊΠΎΠ½ Ома — это основной ΠΏΡ€ΠΈΠ½Ρ†ΠΈΠΏ Π°Π½Π°Π»ΠΈΠ·Π° элСктричСских Ρ†Π΅ΠΏΠ΅ΠΉ. ΠšΡ€ΠΎΠΌΠ΅ Ρ‚ΠΎΠ³ΠΎ, Ссли Π²Ρ‹ Ρ…ΠΎΡ‚ΠΈΡ‚Π΅ ΠΈΠ·ΡƒΡ‡ΠΈΡ‚ΡŒ Π±ΠΎΠ»Π΅Π΅ слоТныС прилоТСния Π·Π°ΠΊΠΎΠ½Π° Ома, ΠΎΡΡ‚Π°Π²ΡŒΡ‚Π΅ свой ΠΎΡ‚Π·Ρ‹Π² Π² Ρ€Π°Π·Π΄Π΅Π»Π΅ ΠΊΠΎΠΌΠΌΠ΅Π½Ρ‚Π°Ρ€ΠΈΠ΅Π² Π½ΠΈΠΆΠ΅. Π’ΠΎΡ‚ Π²Π°ΠΌ вопрос, ΠΊΠ°ΠΊΠΈΠ΅ ΠΏΡ€ΠΈΠ±ΠΎΡ€Ρ‹ ΠΈΡΠΏΠΎΠ»ΡŒΠ·ΡƒΡŽΡ‚ΡΡ для измСрСния напряТСния, Ρ‚ΠΎΠΊΠ° ΠΈ сопротивлСния.

Π€ΠΎΡ‚ΠΎ:

3. Π’ΠΎΠΊ, напряТСниС ΠΈ сопротивлСниС

ЭлСктричСство ΠΌΠΎΠΆΠ½ΠΎ ΠΎΠΏΠΈΡΠ°Ρ‚ΡŒ Π² Ρ‚Π΅Ρ€ΠΌΠΈΠ½Π°Ρ… Π’ΠΎΠΊ, НапряТСниС ΠΈ Π‘ΠΎΠΏΡ€ΠΎΡ‚ΠΈΠ²Π»Π΅Π½ΠΈΠ΅

Вок (символ = I)

  • ЭлСктричСский Ρ‚ΠΎΠΊ Ρ‚Π΅Ρ‡Π΅Ρ‚, ΠΊΠΎΠ³Π΄Π° заряд двиТСтся
  • Π’ΠΎΠΊ — это ΠΌΠ΅Ρ€Π° скорости, с ΠΊΠΎΡ‚ΠΎΡ€ΠΎΠΉ заряд ΠΏΡ€ΠΎΡ…ΠΎΠ΄ΠΈΡ‚ Ρ‡Π΅Ρ€Π΅Π· Ρ‚ΠΎΡ‡ΠΊΡƒ Π² Ρ†Π΅ΠΏΠΈ
  • Π’ΠΎΠΊ измСряСтся Π² АмпСрах (А)
  • Π§Π΅ΠΌ Π²Ρ‹ΡˆΠ΅ Ρ‚ΠΎΠΊ, Ρ‚Π΅ΠΌ быстрСС Π² этой Ρ‚ΠΎΡ‡ΠΊΠ΅ Ρ†Π΅ΠΏΠΈ Ρ‚Π΅Ρ‡Π΅Ρ‚ заряд.
  • Π’ Ρ†Π΅ΠΏΠΈ Ρ‚ΠΎΠΊ Ρ‚Π΅Ρ‡Π΅Ρ‚ ΠΎΡ‚ ΠΏΠΎΠ»ΠΎΠΆΠΈΡ‚Π΅Π»ΡŒΠ½ΠΎΠ³ΠΎ ΠΊΠΎΠ½Ρ†Π° источника питания ΠΊ ΠΎΡ‚Ρ€ΠΈΡ†Π°Ρ‚Π΅Π»ΡŒΠ½ΠΎΠΌΡƒ ΠΊΠΎΠ½Ρ†Ρƒ источника
  • НапряТСниС — это ΠΌΠ΅Ρ€Π° энСргии, пСрСносимой ΠΈΠ»ΠΈ ΠΈΡΠΏΠΎΠ»ΡŒΠ·ΡƒΠ΅ΠΌΠΎΠΉ зарядом.
  • НапряТСниС ΠΌΠΎΠΆΠ½ΠΎ ΠΏΡ€Π΅Π΄ΡΡ‚Π°Π²ΠΈΡ‚ΡŒ ΠΊΠ°ΠΊ Ρ€Π°Π·Π½ΠΈΡ†Ρƒ Π² энСргии (Π΄Π°Π²Π»Π΅Π½ΠΈΠΈ) Π² Π½Π°Ρ‡Π°Π»Π΅ ΠΈ Π² ΠΊΠΎΠ½Ρ†Π΅ Ρ†Π΅ΠΏΠΈ.
    • Начало Ρ†Π΅ΠΏΠΈ Π±ΡƒΠ΄Π΅Ρ‚ ΠΏΠΎΠ΄ высоким Π΄Π°Π²Π»Π΅Π½ΠΈΠ΅ΠΌ, ΠΈ ΠΏΠΎ ΠΌΠ΅Ρ€Π΅ прохоТдСния Ρ‚ΠΎΠΊΠ° ΠΏΠΎ Ρ†Π΅ΠΏΠΈ Π΄Π°Π²Π»Π΅Π½ΠΈΠ΅ Π±ΡƒΠ΄Π΅Ρ‚ ΡΠ½ΠΈΠΆΠ°Ρ‚ΡŒΡΡ. ΠΊ рСзисторам, Ρ‚Π°ΠΊΠΈΠΌ ΠΊΠ°ΠΊ ΡˆΠ°Ρ€ΠΈΠΊΠΈ
    • ЭлСктрохимичСский элСмСнт ΠΈΠ»ΠΈ батарСя Π²ΠΎΡΡΡ‚Π°Π½Π°Π²Π»ΠΈΠ²Π°ΡŽΡ‚ Π΄Π°Π²Π»Π΅Π½ΠΈΠ΅ заряда, Ρ‡Ρ‚ΠΎΠ±Ρ‹ ΠΎΠ±Π΅ΡΠΏΠ΅Ρ‡ΠΈΡ‚ΡŒ Ρ†Π΅ΠΏΡŒ энСргиСй для поддСрТания ΠΏΠΎΡ‚ΠΎΠΊΠ°
  • НапряТСниС измСряСтся Π² Π’ΠΎΠ»ΡŒΡ‚ (Π’)
  • Для Ρ‚ΠΎΠ³ΠΎ, Ρ‡Ρ‚ΠΎΠ±Ρ‹ амСриканскиС Π³ΠΎΡ€ΠΊΠΈ поднялись Π½Π° подъСм, трСбуСтся энСргия, Π½ΠΎ, оказавшись Π½Π° Π²Π΅Ρ€ΡˆΠΈΠ½Π΅, ΠΎΠ½ΠΈ ΡΠΊΠ°Ρ‚Ρ‹Π²Π°ΡŽΡ‚ΡΡ Π²Π½ΠΈΠ· ΠΏΠΎ склону Π±Π΅Π· ΠΊΠ°ΠΊΠΈΡ…-Π»ΠΈΠ±ΠΎ Π΄ΠΎΠΏΠΎΠ»Π½ΠΈΡ‚Π΅Π»ΡŒΠ½Ρ‹Ρ… дСйствий.
    • Π’ Ρ†Π΅ΠΏΠΈ батарСя Π±ΡƒΠ΄Π΅Ρ‚ ΠΏΠΎΡΡ‚Π°Π²Π»ΡΡ‚ΡŒ достаточно энСргии для пСрСмСщСния заряда ΠΈΠ· мСста с Π½ΠΈΠ·ΠΊΠΈΠΌ ΡƒΡ€ΠΎΠ²Π½Π΅ΠΌ энСргии Π² мСсто с высоким ΡƒΡ€ΠΎΠ²Π½Π΅ΠΌ энСргии, Ρ‚ΠΎΡ‡Π½ΠΎ Ρ‚Π°ΠΊ ΠΆΠ΅, ΠΊΠ°ΠΊ Π΄Π²ΠΈΠ³Π°Ρ‚Π΅Π»ΡŒ, Ρ‚ΠΎΠ»ΠΊΠ°ΡŽΡ‰ΠΈΠΉ амСриканскиС Π³ΠΎΡ€ΠΊΠΈ ΠΊ Π²Π΅Ρ€ΡˆΠΈΠ½Π΅ подъСма.
    • Π’ ΠΊΠ»ΠΈΠΏΠ΅ Π½ΠΈΠΆΠ΅ взрослый Π΄ΠΎΠ»ΠΆΠ΅Π½ ΠΏΠΎΠ΄Ρ‚ΠΎΠ»ΠΊΠ½ΡƒΡ‚ΡŒ Ρ‚Π΅Π»Π΅ΠΆΠΊΡƒ ΠΊ Π²Π΅Ρ€Ρ…Ρƒ, ΠΏΡ€Π΅ΠΆΠ΄Π΅ Ρ‡Π΅ΠΌ ΠΎΠ½Π° скатится, ΠΈ ΠΏΠΎ Ρ€Π΅Π»ΡŒΡΠ°ΠΌ

Π‘ΠΎΠΏΡ€ΠΎΡ‚ΠΈΠ²Π»Π΅Π½ΠΈΠ΅ (символ = R)

  • Π‘ΠΎΠΏΡ€ΠΎΡ‚ΠΈΠ²Π»Π΅Π½ΠΈΠ΅ являСтся ΠΌΠ΅Ρ€ΠΎΠΉ большой Π½Π°Π³Ρ€ΡƒΠ·ΠΊΠΈ, Π½Π°ΠΏΡ€ΠΈΠΌΠ΅Ρ€, свСт ΠΎΠ³Ρ€Π°Π½ΠΈΡ‡ΠΈΠ²Π°Π΅Ρ‚ ΠΏΡ€ΠΎΡ…ΠΎΠΆΠ΄Π΅Π½ΠΈΠ΅ Ρ‚ΠΎΠΊΠ°
    • Π‘ΠΎΠΏΡ€ΠΎΡ‚ΠΈΠ²Π»Π΅Π½ΠΈΠ΅ опрСдСляСт, насколько Ρ…ΠΎΡ€ΠΎΡˆΠΎ ΠΏΡ€ΠΎΠ²ΠΎΠ΄Π½ΠΈΠΊ ΠΏΡ€ΠΎΠ²ΠΎΠ΄ΠΈΡ‚ элСктричСство.
  • Π‘ΠΎΠΏΡ€ΠΎΡ‚ΠΈΠ²Π»Π΅Π½ΠΈΠ΅ измСряСтся Π² Ом (Ом)

Π‘ΠœΠžΠ’Π Π•Π’Π¬ Π’Π˜Π”Π•Πž, ΠžΠŸΠ˜Π‘ΠΠΠΠžΠ• Π’ΠΠ˜ΠœΠΠΠ˜Π•: ΠžΠ’ΠΠžΠ¨Π•ΠΠ˜Π• ΠœΠ•Π–Π”Π£ ВОКОМ, ΠΠΠŸΠ Π―Π–Π•ΠΠ˜Π•Πœ, ΠΠΠŸΠ Π―Π–Π•ΠΠ˜Π•Πœ

0

03

02

03

02

Π—Π°ΠΊΠΎΠ½ Ома…Π—Π°Π²ΠΈΡΠΈΠΌΠΎΡΡ‚ΡŒ ΠΌΠ΅ΠΆΠ΄Ρƒ напряТСниСм, Ρ‚ΠΎΠΊΠΎΠΌ ΠΈ сопротивлСниСм

ВрСмя чтСния: 5 ΠΌΠΈΠ½ΡƒΡ‚

ВСорСтичСскиС Ρ‚Π΅Ρ€ΠΌΠΈΠ½Ρ‹ ΠΈ опрСдСлСния

Π‘Π»Π΅Π΄ΡƒΡŽΡ‰ΠΈΠ΅ опрСдСлСния относятся ΠΊ основной Ρ‚Π΅ΠΎΡ€ΠΈΠΈ элСктричСства. Π’Π°ΠΆΠ½ΠΎ, Ρ‡Ρ‚ΠΎΠ±Ρ‹ установщики ΠΈ инспСкторы ΠΎΠ±Π»Π°Π΄Π°Π»ΠΈ практичСскими знаниями Ρ‚Π΅ΠΎΡ€ΠΈΠΈ элСктричСства. Π’Π°ΠΊΠΈΠ΅ знания часто ΠΈΠΌΠ΅ΡŽΡ‚ ΠΆΠΈΠ·Π½Π΅Π½Π½ΠΎ Π²Π°ΠΆΠ½ΠΎΠ΅ Π·Π½Π°Ρ‡Π΅Π½ΠΈΠ΅ для опрСдСлСния ΠΏΡ€Π°Π²ΠΈΠ»ΡŒΠ½ΠΎΠ³ΠΎ сСчСния ΠΏΡ€ΠΎΠ²ΠΎΠ΄ΠΎΠ² для Ρ†Π΅ΠΏΠ΅ΠΉ с Ρ€Π°Π·Π»ΠΈΡ‡Π½ΠΎΠΉ Π½Π°Π³Ρ€ΡƒΠ·ΠΊΠΎΠΉ.

Π’ΠΎΠ»ΡŒΡ‚ — Π΅Π΄ΠΈΠ½ΠΈΡ†Π° элСктричСского давлСния — это Π΄Π°Π²Π»Π΅Π½ΠΈΠ΅, Π½Π΅ΠΎΠ±Ρ…ΠΎΠ΄ΠΈΠΌΠΎΠ΅ для Ρ‚ΠΎΠ³ΠΎ, Ρ‡Ρ‚ΠΎΠ±Ρ‹ Π·Π°ΡΡ‚Π°Π²ΠΈΡ‚ΡŒ ΠΎΠ΄ΠΈΠ½ Π°ΠΌΠΏΠ΅Ρ€ ΠΏΡ€ΠΎΠΉΡ‚ΠΈ Ρ‡Π΅Ρ€Π΅Π· сопротивлСниС Π² ΠΎΠ΄ΠΈΠ½ ΠΎΠΌ; сокращСнно Β«EΒ», пСрвая Π±ΡƒΠΊΠ²Π° Ρ‚Π΅Ρ€ΠΌΠΈΠ½Π° ΡΠ»Π΅ΠΊΡ‚Ρ€ΠΎΠ΄Π²ΠΈΠ³Π°Ρ‚Π΅Π»ΡŒ сила .

АмпСр — Π΅Π΄ΠΈΠ½ΠΈΡ†Π° измСрСния элСктричСского Ρ‚ΠΎΠΊΠ°, ΠΊΠΎΡ‚ΠΎΡ€Ρ‹ΠΉ ΠΏΡ€ΠΎΡ‚Π΅ΠΊΠ°Π΅Ρ‚ Ρ‡Π΅Ρ€Π΅Π· ΠΎΠ΄ΠΈΠ½ Ом ΠΏΠΎΠ΄ Π΄Π°Π²Π»Π΅Π½ΠΈΠ΅ΠΌ Π² ΠΎΠ΄ΠΈΠ½ Π²ΠΎΠ»ΡŒΡ‚ Π·Π° ΠΎΠ΄Π½Ρƒ сСкунду; сокращСнно Β«IΒ», пСрвая Π±ΡƒΠΊΠ²Π° Ρ‚Π΅Ρ€ΠΌΠΈΠ½Π° сила Ρ‚ΠΎΠΊΠ° .

Ом — Π΅Π΄ΠΈΠ½ΠΈΡ†Π° элСктричСского сопротивлСния — это сопротивлСниС, Ρ‡Π΅Ρ€Π΅Π· ΠΊΠΎΡ‚ΠΎΡ€ΠΎΠ΅ ΠΎΠ΄ΠΈΠ½ Π²ΠΎΠ»ΡŒΡ‚ заставит ΠΎΠ΄ΠΈΠ½ Π°ΠΌΠΏΠ΅Ρ€; сокращСнно Β«RΒ», пСрвая Π±ΡƒΠΊΠ²Π° Ρ‚Π΅Ρ€ΠΌΠΈΠ½Π° сопротивлСниС .

Π’Ρ‚ — это Π΅Π΄ΠΈΠ½ΠΈΡ†Π° измСрСния энСргии, ΠΏΡ€ΠΎΡ‚Π΅ΠΊΠ°ΡŽΡ‰Π΅ΠΉ Π² элСктричСской Ρ†Π΅ΠΏΠΈ Π² любой Π΄Π°Π½Π½Ρ‹ΠΉ ΠΌΠΎΠΌΠ΅Π½Ρ‚.Π­Ρ‚ΠΎ Ρ‚Π°ΠΊΠΆΠ΅ объСм Ρ€Π°Π±ΠΎΡ‚Ρ‹, выполняСмой Π² элСктричСской Ρ†Π΅ΠΏΠΈ. Π’Π΅Ρ€ΠΌΠΈΠ½Ρ‹ Π²Π°Ρ‚Ρ‚ ΠΈΠ»ΠΈ ΠΊΠΈΠ»ΠΎΠ²Π°Ρ‚Ρ‚ ΠΈΡΠΏΠΎΠ»ΡŒΠ·ΡƒΡŽΡ‚ΡΡ Ρ‡Π°Ρ‰Π΅ для обозначСния объСма Ρ€Π°Π±ΠΎΡ‚Ρ‹, выполняСмой Π² элСктричСской Ρ†Π΅ΠΏΠΈ, Π° Π½Π΅ Π΄ΠΆΠΎΡƒΠ»Π΅ΠΉ . Π’Π°Ρ‚Ρ‚Ρ‹ — это ΠΏΡ€ΠΎΠΈΠ·Π²Π΅Π΄Π΅Π½ΠΈΠ΅ Π²ΠΎΠ»ΡŒΡ‚ ΠΈ Π°ΠΌΠΏΠ΅Ρ€, ΠΊΠΎΡ‚ΠΎΡ€ΠΎΠ΅ ΠΈΠ½ΠΎΠ³Π΄Π° Π½Π°Π·Ρ‹Π²Π°ΡŽΡ‚ Π²ΠΎΠ»ΡŒΡ‚-Π°ΠΌΠΏΠ΅Ρ€. Одна тысяча Π²ΠΎΠ»ΡŒΡ‚-Π°ΠΌΠΏΠ΅Ρ€ упоминаСтся ΠΊΠ°ΠΊ ΠΎΠ΄ΠΈΠ½ ΠΊΠΈΠ»ΠΎΠ²ΠΎΠ»ΡŒΡ‚-Π°ΠΌΠΏΠ΅Ρ€ ΠΈΠ»ΠΈ ΠΎΠ΄Π½Π° кВА.

Π—Π°ΠΊΠΎΠ½ Ома

Π”ΠΆΠΎΡ€Π΄ΠΆ Π‘Π°ΠΉΠΌΠΎΠ½ Ом ΠΎΠ±Π½Π°Ρ€ΡƒΠΆΠΈΠ» взаимосвязь ΠΌΠ΅ΠΆΠ΄Ρƒ Ρ‚ΠΎΠΊΠΎΠΌ, напряТСниСм ΠΈ сопротивлСниСм Π² элСктричСской Ρ†Π΅ΠΏΠΈ Π² 1826 Π³ΠΎΠ΄Ρƒ.Он ΠΎΠ±Π½Π°Ρ€ΡƒΠΆΠΈΠ» ΡΠΊΡΠΏΠ΅Ρ€ΠΈΠΌΠ΅Π½Ρ‚Π°Π»ΡŒΠ½Ρ‹ΠΌ ΠΏΡƒΡ‚Π΅ΠΌ, Ρ‡Ρ‚ΠΎ Π΄Π°Π²Π»Π΅Π½ΠΈΠ΅ Ρ€Π°Π²Π½ΠΎ ΠΏΡ€ΠΎΠΈΠ·Π²Π΅Π΄Π΅Π½ΠΈΡŽ силы Ρ‚ΠΎΠΊΠ° ΠΈ сопротивлСния; это ΡΠΎΠΎΡ‚Π½ΠΎΡˆΠ΅Π½ΠΈΠ΅ называСтся Π·Π°ΠΊΠΎΠ½ΠΎΠΌ Ома. Π­Ρ‚ΠΎΡ‚ Π·Π°ΠΊΠΎΠ½ являСтся практичСской основой Π±ΠΎΠ»ΡŒΡˆΠΈΠ½ΡΡ‚Π²Π° элСктричСских расчСтов. Π€ΠΎΡ€ΠΌΡƒΠ»Π° ΠΌΠΎΠΆΠ΅Ρ‚ Π±Ρ‹Ρ‚ΡŒ Π²Ρ‹Ρ€Π°ΠΆΠ΅Π½Π° Π² Ρ€Π°Π·Π»ΠΈΡ‡Π½Ρ‹Ρ… Ρ„ΠΎΡ€ΠΌΠ°Ρ… ΠΈ Π΅Π΅ использовании, ΠΊΠ°ΠΊ Π² Ρ‚Ρ€Π΅Ρ… ΠΏΡ€ΠΈΠΌΠ΅Ρ€Π°Ρ…, ΠΏΠΎΠΊΠ°Π·Π°Π½Π½Ρ‹Ρ… Π½Π° рисункС 1.

Рисунок 1. ΠžΡΠ½ΠΎΠ²Π½Ρ‹Π΅ ΠΏΡ€ΠΈΠΌΠ΅Ρ€Ρ‹ ΠΈ ΠΏΡ€ΠΈΠΌΠ΅Π½Π΅Π½ΠΈΠ΅ Π·Π°ΠΊΠΎΠ½Π° Ома

Если извСстны Π»ΡŽΠ±Ρ‹Π΅ Π΄Π²Π° значСния, Ρ‚Ρ€Π΅Ρ‚ΡŒΠ΅ ΠΌΠΎΠΆΠ½ΠΎ Π½Π°ΠΉΡ‚ΠΈ с ΠΏΠΎΠΌΠΎΡ‰ΡŒΡŽ Ρ„ΠΎΡ€ΠΌΡƒΠ»Ρ‹. НапримСр, Ссли извСстны сопротивлСниС ΠΈ напряТСниС, Ρ‚ΠΎΠΊ ΠΌΠΎΠΆΠ½ΠΎ ΠΎΠΏΡ€Π΅Π΄Π΅Π»ΠΈΡ‚ΡŒ, Ρ€Π°Π·Π΄Π΅Π»ΠΈΠ² напряТСниС Π½Π° сопротивлСниС.Π­Ρ‚ΠΎ ΠΌΠΎΠΆΠ΅Ρ‚ Π±Ρ‹Ρ‚ΡŒ ΠΏΠΎΠ»Π΅Π·Π½ΠΎ ΠΏΡ€ΠΈ ΠΎΠΏΡ€Π΅Π΄Π΅Π»Π΅Π½ΠΈΠΈ Π²Π΅Π»ΠΈΡ‡ΠΈΠ½Ρ‹ Ρ‚ΠΎΠΊΠ°, ΠΊΠΎΡ‚ΠΎΡ€Ρ‹ΠΉ Π±ΡƒΠ΄Π΅Ρ‚ ΠΏΡ€ΠΎΡ‚Π΅ΠΊΠ°Ρ‚ΡŒ Π² Ρ†Π΅ΠΏΠΈ, для ΠΏΡ€Π°Π²ΠΈΠ»ΡŒΠ½ΠΎΠ³ΠΎ опрСдСлСния Ρ€Π°Π·ΠΌΠ΅Ρ€ΠΎΠ² ΠΏΡ€ΠΎΠ²ΠΎΠ΄Π½ΠΈΠΊΠΎΠ², Π° Ρ‚Π°ΠΊΠΆΠ΅ устройств ΠΏΠ΅Ρ€Π΅Π³Ρ€ΡƒΠ·ΠΊΠΈ ΠΏΠΎ Ρ‚ΠΎΠΊΡƒ.

Π».с. ΠœΠ΅Ρ…Π°Π½ΠΈΡ‡Π΅ΡΠΊΠ°Ρ ΠΌΠΎΡ‰Π½ΠΎΡΡ‚ΡŒ ΠΎΠ±Ρ‹Ρ‡Π½ΠΎ выраТаСтся Π² Π»ΠΎΡˆΠ°Π΄ΠΈΠ½Ρ‹Ρ… силах, Π° элСктричСская ΠΌΠΎΡ‰Π½ΠΎΡΡ‚ΡŒ — Π² Π²Π°Ρ‚Ρ‚Π°Ρ…. Π’Π΅Ρ€ΠΌΠΈΠ½ Π»ΠΎΡˆΠ°Π΄ΠΈΠ½Ρ‹Ρ… сил Π²ΠΎΠ·Π½ΠΈΠΊ ΠΊΠ°ΠΊ объСм Ρ€Π°Π±ΠΎΡ‚Ρ‹, ΠΊΠΎΡ‚ΠΎΡ€ΡƒΡŽ сильная лондонская тягловая лошадь ΠΌΠΎΠ³Π»Π° Π²Ρ‹ΠΏΠΎΠ»Π½ΡΡ‚ΡŒ Π·Π° ΠΊΠΎΡ€ΠΎΡ‚ΠΊΠΈΠΉ ΠΏΡ€ΠΎΠΌΠ΅ΠΆΡƒΡ‚ΠΎΠΊ Π²Ρ€Π΅ΠΌΠ΅Π½ΠΈ. Он Ρ‚Π°ΠΊΠΆΠ΅ использовался для измСрСния мощности ΠΏΠ°Ρ€ΠΎΠ²Ρ‹Ρ… Π΄Π²ΠΈΠ³Π°Ρ‚Π΅Π»Π΅ΠΉ. Одна лошадиная сила, сокращСнно Β«HPΒ», Ρ€Π°Π²Π½Π° Ρ€Π°Π±ΠΎΡ‚Π΅, Π½Π΅ΠΎΠ±Ρ…ΠΎΠ΄ΠΈΠΌΠΎΠΉ для поднятия 33 000 Ρ„ΡƒΠ½Ρ‚ΠΎΠ² Π½Π° ΠΎΠ΄ΠΈΠ½ Ρ„ΡƒΡ‚ (33 000 Ρ„ΡƒΡ‚-Ρ„ΡƒΠ½Ρ‚ΠΎΠ²) Π·Π° ΠΎΠ΄Π½Ρƒ ΠΌΠΈΠ½ΡƒΡ‚Ρƒ.Π­Ρ‚ΠΎ Ρ‚ΠΎ ΠΆΠ΅ самоС, Ρ‡Ρ‚ΠΎ ΠΏΠΎΠ΄Π½ΠΈΠΌΠ°Ρ‚ΡŒ ΠΎΠ΄ΠΈΠ½ Ρ„ΡƒΡ‚ Π½Π° 550 Ρ„ΡƒΠ½Ρ‚ΠΎΠ² Π·Π° сСкунду.

Часто Π±Ρ‹Π²Π°Π΅Ρ‚ Π½Π΅ΠΎΠ±Ρ…ΠΎΠ΄ΠΈΠΌΠΎ ΠΏΡ€Π΅ΠΎΠ±Ρ€Π°Π·ΠΎΠ²Π°Ρ‚ΡŒ ΠΌΠΎΡ‰Π½ΠΎΡΡ‚ΡŒ ΠΎΡ‚ ΠΎΠ΄Π½ΠΎΠ³ΠΎ устройства ΠΊ Π΄Ρ€ΡƒΠ³ΠΎΠΌΡƒ, ΠΈ ΡƒΡ€Π°Π²Π½Π΅Π½ΠΈΠ΅ Π½Π° рисункС 2 ΠΈΡΠΏΠΎΠ»ΡŒΠ·ΡƒΠ΅Ρ‚ΡΡ для прСобразования мощности Π² Π²Π°Ρ‚Ρ‚Ρ‹ ΠΈΠ»ΠΈ Π²Π°Ρ‚Ρ‚ΠΎΠ² Π² Π»ΠΎΡˆΠ°Π΄ΠΈΠ½Ρ‹Π΅ силы.


Рис. 2. Базовая Ρ„ΠΎΡ€ΠΌΡƒΠ»Π° HP

Π€ΠΎΡ€ΠΌΡƒΠ»Π° Π».с. ΠΏΡ€ΠΈΠΌΠ΅Π½ΠΈΠΌΠ° ΠΊ Π»Π°Π±ΠΎΡ€Π°Ρ‚ΠΎΡ€Π½Ρ‹ΠΌ условиям, ΠΏΠΎΡΠΊΠΎΠ»ΡŒΠΊΡƒ Π΄Π²ΠΈΠ³Π°Ρ‚Π΅Π»ΠΈ ΠΏΠΎΡ‚Ρ€Π΅Π±Π»ΡΡŽΡ‚ большС мощности, Ρ‡Π΅ΠΌ Π΄ΠΎΡΡ‚Π°Π²Π»ΡΡŽΡ‚. Π­Ρ‚ΠΎ связано с Ρ‚Π΅ΠΌ, Ρ‡Ρ‚ΠΎ ΠΌΠΎΡ‰Π½ΠΎΡΡ‚ΡŒ, потрСбляСмая Π΄Π²ΠΈΠ³Π°Ρ‚Π΅Π»Π΅ΠΌ Π² Π²ΠΈΠ΄Π΅ Ρ‚Π΅ΠΏΠ»Π°, ΠΏΡ€Π΅ΠΎΠ΄ΠΎΠ»Π΅Π²Π°Π΅Ρ‚ Ρ‚Ρ€Π΅Π½ΠΈΠ΅ Π² ΠΏΠΎΠ΄ΡˆΠΈΠΏΠ½ΠΈΠΊΠ°Ρ…, сопротивлСниС Π²Π΅Ρ‚Ρ€Ρƒ ΠΈ Π΄Ρ€ΡƒΠ³ΠΈΠ΅ Ρ„Π°ΠΊΡ‚ΠΎΡ€Ρ‹.НапримСр, Π΄Π²ΠΈΠ³Π°Ρ‚Π΅Π»ΡŒ ΠΌΠΎΡ‰Π½ΠΎΡΡ‚ΡŒΡŽ 1 Π».с. (746 Π’Ρ‚) ΠΌΠΎΠΆΠ΅Ρ‚ ΠΏΠΎΡ‚Ρ€Π΅Π±Π»ΡΡ‚ΡŒ ΠΏΠΎΡ‡Ρ‚ΠΈ 1000 Π’Ρ‚, Ρ€Π°Π·Π½ΠΈΡ†Π° расходуСтся Π½Π° ΠΏΡ€Π΅ΠΎΠ΄ΠΎΠ»Π΅Π½ΠΈΠ΅ ΡƒΠΆΠ΅ ΡƒΠΊΠ°Π·Π°Π½Π½Ρ‹Ρ… Ρ„Π°ΠΊΡ‚ΠΎΡ€ΠΎΠ². Для опрСдСлСния истинной мощности ΠΎΠ΄Π½ΠΎΡ„Π°Π·Π½Ρ‹Ρ… Π΄Π²ΠΈΠ³Π°Ρ‚Π΅Π»Π΅ΠΉ Π½Π΅ΠΎΠ±Ρ…ΠΎΠ΄ΠΈΠΌΠΎ ΡƒΡ‡ΠΈΡ‚Ρ‹Π²Π°Ρ‚ΡŒ коэффициСнт ΠΏΠΎΠ»Π΅Π·Π½ΠΎΠ³ΠΎ дСйствия двигатСля (см. Рисунок 3).


Рисунок 3. ΠžΡΠ½ΠΎΠ²Π½Ρ‹Π΅ Ρ„ΠΎΡ€ΠΌΡƒΠ»Ρ‹ коэффициСнта мощности

КолСсо Π’Π°Ρ‚Ρ‚

КолСсо Π’Π°Ρ‚Ρ‚Π° Π±Ρ‹Π»ΠΎ Ρ€Π°Π·Ρ€Π°Π±ΠΎΡ‚Π°Π½ΠΎ ΠΈ ΠΎΠΏΡƒΠ±Π»ΠΈΠΊΠΎΠ²Π°Π½ΠΎ Π²ΠΎ ΠΌΠ½ΠΎΠ³ΠΈΡ… руководствах ΠΈ Π² Π½Π΅ΡΠΊΠΎΠ»ΡŒΠΊΠΈΡ… Π²Π°Ρ€ΠΈΠ°Π½Ρ‚Π°Ρ… для ΠΈΠ»Π»ΡŽΡΡ‚Ρ€Π°Ρ†ΠΈΠΈ Π²Π°Ρ‚Ρ‚ ΠΈΠ»ΠΈ мощности ΠΈ ΠΈΡ… связи с элСмСнтами Π·Π°ΠΊΠΎΠ½Π° Ома.Как ΠΏΠΎΠΊΠ°Π·Π°Π½ΠΎ Π² этом тСкстС, это Π²Π΅Ρ€Π½ΠΎ для Ρ†Π΅ΠΏΠ΅ΠΉ постоянного Ρ‚ΠΎΠΊΠ° ΠΈ для рСзистивных Π½Π°Π³Ρ€ΡƒΠ·ΠΎΠΊ Ρ†Π΅ΠΏΠ΅ΠΉ ΠΏΠ΅Ρ€Π΅ΠΌΠ΅Π½Π½ΠΎΠ³ΠΎ Ρ‚ΠΎΠΊΠ°, Π³Π΄Π΅ коэффициСнт мощности Π±Π»ΠΈΠ·ΠΎΠΊ ΠΊ 100 ΠΏΡ€ΠΎΡ†Π΅Π½Ρ‚Π°ΠΌ ΠΈΠ»ΠΈ Π΅Π΄ΠΈΠ½ΠΈΡ†Π΅ (см. Рисунок 4). НС ΠΏΡ‹Ρ‚Π°ΠΉΡ‚Π΅ΡΡŒ ΠΈΡΠΏΠΎΠ»ΡŒΠ·ΠΎΠ²Π°Ρ‚ΡŒ Π΅Π³ΠΎ для Π½Π°Π³Ρ€ΡƒΠ·ΠΎΠΊ двигатСля, ΠΏΠΎΡΠΊΠΎΠ»ΡŒΠΊΡƒ Π² Ρ„ΠΎΡ€ΠΌΡƒΠ»Π΅ Π½Π΅ΠΎΠ±Ρ…ΠΎΠ΄ΠΈΠΌΠΎ ΡƒΡ‡ΠΈΡ‚Ρ‹Π²Π°Ρ‚ΡŒ ΠΊΠ°ΠΊ коэффициСнт мощности, Ρ‚Π°ΠΊ ΠΈ ΠšΠŸΠ” двигатСля (см. Рисунок 3).


Рисунок 4. КолСсо Π’Π°Ρ‚Ρ‚Π° ΠΈ Π·Π°ΠΊΠΎΠ½ Ома

Π’ цСпях ΠΏΠ΅Ρ€Π΅ΠΌΠ΅Π½Π½ΠΎΠ³ΠΎ Ρ‚ΠΎΠΊΠ° ΠΌΡ‹ ΠΈΡΠΏΠΎΠ»ΡŒΠ·ΡƒΠ΅ΠΌ Ρ‚Π΅Ρ€ΠΌΠΈΠ½ импСданс , Π° Π½Π΅ Ом для обозначСния сопротивлСния Ρ†Π΅ΠΏΠΈ. ИмпСданс — это ΠΏΠΎΠ»Π½ΠΎΠ΅ сопротивлСниС Ρ‚ΠΎΠΊΡƒ Π² Ρ†Π΅ΠΏΠΈ ΠΏΠ΅Ρ€Π΅ΠΌΠ΅Π½Π½ΠΎΠ³ΠΎ Ρ‚ΠΎΠΊΠ°; ΠΎΠ½ измСряСтся Π² ΠΎΠΌΠ°Ρ….ИмпСданс Π²ΠΊΠ»ΡŽΡ‡Π°Π΅Ρ‚ сопротивлСниС, СмкостноС Ρ€Π΅Π°ΠΊΡ‚ΠΈΠ²Π½ΠΎΠ΅ сопротивлСниС ΠΈ ΠΈΠ½Π΄ΡƒΠΊΡ‚ΠΈΠ²Π½ΠΎΠ΅ Ρ€Π΅Π°ΠΊΡ‚ΠΈΠ²Π½ΠΎΠ΅ сопротивлСниС. ПослСдниС Π΄Π²Π° Ρ„Π°ΠΊΡ‚ΠΎΡ€Π° ΡƒΠ½ΠΈΠΊΠ°Π»ΡŒΠ½Ρ‹ для Ρ†Π΅ΠΏΠ΅ΠΉ ΠΏΠ΅Ρ€Π΅ΠΌΠ΅Π½Π½ΠΎΠ³ΠΎ Ρ‚ΠΎΠΊΠ° ΠΈ ΠΎΠ±Ρ‹Ρ‡Π½ΠΎ ΠΌΠΎΠ³ΡƒΡ‚ ΠΈΠ³Π½ΠΎΡ€ΠΈΡ€ΠΎΠ²Π°Ρ‚ΡŒΡΡ Π² цСпях, Ρ‚Π°ΠΊΠΈΡ… ΠΊΠ°ΠΊ Π»Π°ΠΌΠΏΡ‹ накаливания ΠΈ Ρ†Π΅ΠΏΠΈ нагрСватСля, состоящиС ΠΈΠ· рСзистивных Π½Π°Π³Ρ€ΡƒΠ·ΠΎΠΊ. ΠŸΠΎΠ΄Ρ€ΠΎΠ±Π½ΠΎΠ΅ объяснСниС Смкостного Ρ€Π΅Π°ΠΊΡ‚ΠΈΠ²Π½ΠΎΠ³ΠΎ сопротивлСния ΠΈ ΠΈΠ½Π΄ΡƒΠΊΡ‚ΠΈΠ²Π½ΠΎΠ³ΠΎ Ρ€Π΅Π°ΠΊΡ‚ΠΈΠ²Π½ΠΎΠ³ΠΎ сопротивлСния Π²Ρ‹Ρ…ΠΎΠ΄ΠΈΡ‚ Π·Π° Ρ€Π°ΠΌΠΊΠΈ этого тСкста, Π½ΠΎ Π΅Π³ΠΎ ΠΌΠΎΠΆΠ½ΠΎ Π½Π°ΠΉΡ‚ΠΈ Π²ΠΎ ΠΌΠ½ΠΎΠ³ΠΈΡ… прСкрасных тСкстах ΠΏΠΎ Ρ‚Π΅ΠΎΡ€ΠΈΠΈ элСктричСства.

Π—Π°ΠΊΠΎΠ½ Ома ΠΈ основная элСктричСская тСория

ЭлСктричСский Ρ‚ΠΎΠΊ, ΠΏΡ€ΠΎΡ‚Π΅ΠΊΠ°ΡŽΡ‰ΠΈΠΉ Ρ‡Π΅Ρ€Π΅Π· Π»ΡŽΠ±ΡƒΡŽ ΡΠ»Π΅ΠΊΡ‚Ρ€ΠΈΡ‡Π΅ΡΠΊΡƒΡŽ Ρ†Π΅ΠΏΡŒ, ΠΌΠΎΠΆΠ½ΠΎ ΡΡ€Π°Π²Π½ΠΈΡ‚ΡŒ с Π²ΠΎΠ΄ΠΎΠΉ ΠΏΠΎΠ΄ Π΄Π°Π²Π»Π΅Π½ΠΈΠ΅ΠΌ, ΠΏΡ€ΠΎΡ‚Π΅ΠΊΠ°ΡŽΡ‰Π΅ΠΉ Ρ‡Π΅Ρ€Π΅Π· ΠΏΠΎΠΆΠ°Ρ€Π½Ρ‹ΠΉ шланг.Π’ΠΎΠ΄Π°, ΠΏΡ€ΠΎΡ‚Π΅ΠΊΠ°ΡŽΡ‰Π°Ρ Ρ‡Π΅Ρ€Π΅Π· ΠΏΠΎΠΆΠ°Ρ€Π½Ρ‹ΠΉ шланг, измСряСтся Π² Π³Π°Π»Π»ΠΎΠ½Π°Ρ… Π² ΠΌΠΈΠ½ΡƒΡ‚Ρƒ (GPM), Π° элСктричСство, ΠΏΡ€ΠΎΡ‚Π΅ΠΊΠ°ΡŽΡ‰Π΅Π΅ Ρ‡Π΅Ρ€Π΅Π· ΠΊΠΎΠ½Ρ‚ΡƒΡ€, измСряСтся Π² Π°ΠΌΠΏΠ΅Ρ€Π°Ρ… (A).

Π’ΠΎΠ΄Π° Ρ‚Π΅Ρ‡Π΅Ρ‚ ΠΏΠΎ ΡˆΠ»Π°Π½Π³Ρƒ, ΠΊΠΎΠ³Π΄Π° Π½Π° Π½Π΅Π³ΠΎ оказываСтся Π΄Π°Π²Π»Π΅Π½ΠΈΠ΅ ΠΈ открываСтся ΠΊΠ»Π°ΠΏΠ°Π½. Π”Π°Π²Π»Π΅Π½ΠΈΠ΅ Π²ΠΎΠ΄Ρ‹ измСряСтся Π² Ρ„ΡƒΠ½Ρ‚Π°Ρ… Π½Π° ΠΊΠ²Π°Π΄Ρ€Π°Ρ‚Π½Ρ‹ΠΉ дюйм (psi). ЭлСктричСский Ρ‚ΠΎΠΊ Ρ‚Π΅Ρ‡Π΅Ρ‚ ΠΏΠΎ элСктричСскому ΠΏΡ€ΠΎΠ²ΠΎΠ΄Π½ΠΈΠΊΡƒ, ΠΊΠΎΠ³Π΄Π° ΠΊ Π½Π΅ΠΌΡƒ прикладываСтся элСктричСскоС Π΄Π°Π²Π»Π΅Π½ΠΈΠ΅, ΠΈ создаСтся ΠΏΡƒΡ‚ΡŒ для прохоТдСния Ρ‚ΠΎΠΊΠ°. Подобно Ρ‚ΠΎΠΌΡƒ, ΠΊΠ°ΠΊ Β«Ρ„ΡƒΠ½Ρ‚Ρ‹ Π½Π° ΠΊΠ²Π°Π΄Ρ€Π°Ρ‚Π½Ρ‹ΠΉ дюйм» (Π΄Π°Π²Π»Π΅Π½ΠΈΠ΅) Π²Ρ‹Π·Ρ‹Π²Π°ΡŽΡ‚ ΠΏΠΎΡ‚ΠΎΠΊ Π³Π°Π»Π»ΠΎΠ½ΠΎΠ² Π² ΠΌΠΈΠ½ΡƒΡ‚Ρƒ, Ρ‚Π°ΠΊ Β«Π²ΠΎΠ»ΡŒΡ‚Β» (Π΄Π°Π²Π»Π΅Π½ΠΈΠ΅) заставляСт Ρ‚Π΅Ρ‡ΡŒ Β«Π°ΠΌΠΏΠ΅Ρ€Ρ‹Β» (Ρ‚ΠΎΠΊ).

Π§Ρ‚ΠΎΠ±Ρ‹ ΠΏΡ€ΠΎΠΏΡƒΡΡ‚ΠΈΡ‚ΡŒ Ρ‚Π°ΠΊΠΎΠ΅ ΠΆΠ΅ количСство Π²ΠΎΠ΄Ρ‹ Ρ‡Π΅Ρ€Π΅Π· малСнький шланг, трСбуСтся большСС Π΄Π°Π²Π»Π΅Π½ΠΈΠ΅, Ρ‡Π΅ΠΌ Ρ‡Π΅Ρ€Π΅Π· шланг большСго Ρ€Π°Π·ΠΌΠ΅Ρ€Π°. МалСнький шланг, ΠΊ ΠΊΠΎΡ‚ΠΎΡ€ΠΎΠΌΡƒ ΠΏΡ€ΠΈΠ»ΠΎΠΆΠ΅Π½ΠΎ Ρ‚Π°ΠΊΠΎΠ΅ ΠΆΠ΅ Π΄Π°Π²Π»Π΅Π½ΠΈΠ΅, ΠΏΠΎ ΡΡ€Π°Π²Π½Π΅Π½ΠΈΡŽ с большим шлангом, Π±ΡƒΠ΄Π΅Ρ‚ ΠΏΡ€ΠΎΠΏΡƒΡΠΊΠ°Ρ‚ΡŒ Π³ΠΎΡ€Π°Π·Π΄ΠΎ мСньшС Π²ΠΎΠ΄Ρ‹ Π·Π° ΠΎΠΏΡ€Π΅Π΄Π΅Π»Π΅Π½Π½Ρ‹ΠΉ ΠΏΠ΅Ρ€ΠΈΠΎΠ΄. ΠžΡ‚ΡΡŽΠ΄Π° слСдуСт, Ρ‡Ρ‚ΠΎ малСнький шланг ΠΎΠΊΠ°Π·Ρ‹Π²Π°Π΅Ρ‚ большСС сопротивлСниС ΠΏΠΎΡ‚ΠΎΠΊΡƒ Π²ΠΎΠ΄Ρ‹.

Π’ элСктричСской Ρ†Π΅ΠΏΠΈ большСС элСктричСскоС Π΄Π°Π²Π»Π΅Π½ΠΈΠ΅ (Π²ΠΎΠ»ΡŒΡ‚) заставит ΠΎΠΏΡ€Π΅Π΄Π΅Π»Π΅Π½Π½ΠΎΠ΅ количСство Ρ‚ΠΎΠΊΠ° (Π² Π°ΠΌΠΏΠ΅Ρ€Π°Ρ…) ΠΏΡ€ΠΎΡ…ΠΎΠ΄ΠΈΡ‚ΡŒ Ρ‡Π΅Ρ€Π΅Π· нСбольшой ΠΏΡ€ΠΎΠ²ΠΎΠ΄Π½ΠΈΠΊ (сопротивлСниС), Ρ‡Π΅ΠΌ Π½Π΅ΠΎΠ±Ρ…ΠΎΠ΄ΠΈΠΌΠΎΠ΅ для протСкания Ρ‚ΠΎΠ³ΠΎ ΠΆΠ΅ количСства Ρ‚ΠΎΠΊΠ° (Π² Π°ΠΌΠΏΠ΅Ρ€Π°Ρ…) Ρ‡Π΅Ρ€Π΅Π· ΠΏΡ€ΠΎΠ²ΠΎΠ΄Π½ΠΈΠΊ большСго Ρ€Π°Π·ΠΌΠ΅Ρ€Π° (сопротивлСниС). .ΠŸΡ€ΠΎΠ²ΠΎΠ΄Π½ΠΈΠΊ мСньшСго Ρ€Π°Π·ΠΌΠ΅Ρ€Π° ΠΏΠΎΠ·Π²ΠΎΠ»ΠΈΡ‚ ΠΏΡ€ΠΎΡ…ΠΎΠ΄ΠΈΡ‚ΡŒ ΠΌΠ΅Π½ΡŒΡˆΠ΅ΠΌΡƒ Ρ‚ΠΎΠΊΡƒ (Π² Π°ΠΌΠΏΠ΅Ρ€Π°Ρ…), Ρ‡Π΅ΠΌ ΠΏΡ€ΠΎΠ²ΠΎΠ΄Π½ΠΈΠΊ большСго Ρ€Π°Π·ΠΌΠ΅Ρ€Π°, Ссли ΠΊ ΠΊΠ°ΠΆΠ΄ΠΎΠΌΡƒ ΠΏΡ€ΠΎΠ²ΠΎΠ΄Π½ΠΈΠΊΡƒ Π² Ρ‚Π΅Ρ‡Π΅Π½ΠΈΠ΅ Ρ‚ΠΎΠ³ΠΎ ΠΆΠ΅ ΠΏΠ΅Ρ€ΠΈΠΎΠ΄Π° ΠΏΡ€ΠΈΠ»ΠΎΠΆΠΈΡ‚ΡŒ ΠΎΠ΄ΠΈΠ½Π°ΠΊΠΎΠ²ΠΎΠ΅ элСктричСскоС Π΄Π°Π²Π»Π΅Π½ΠΈΠ΅ (Π²ΠΎΠ»ΡŒΡ‚). МоТно ΠΏΡ€Π΅Π΄ΠΏΠΎΠ»ΠΎΠΆΠΈΡ‚ΡŒ, Ρ‡Ρ‚ΠΎ мСньший ΠΏΡ€ΠΎΠ²ΠΎΠ΄Π½ΠΈΠΊ ΠΈΠΌΠ΅Π΅Ρ‚ большСС сопротивлСниС (Ом), Ρ‡Π΅ΠΌ ΠΏΡ€ΠΎΠ²ΠΎΠ΄ большСго Ρ€Π°Π·ΠΌΠ΅Ρ€Π°. Π’Π°ΠΊΠΈΠΌ ΠΎΠ±Ρ€Π°Π·ΠΎΠΌ, ΠΌΡ‹ ΠΌΠΎΠΆΠ΅ΠΌ ΠΎΠΏΡ€Π΅Π΄Π΅Π»ΠΈΡ‚ΡŒ сопротивлСниС ΠΊΠ°ΠΊ «свойство Ρ‚Π΅Π»Π°, ΠΊΠΎΡ‚ΠΎΡ€ΠΎΠ΅ сопротивляСтся ΠΈΠ»ΠΈ ΠΎΠ³Ρ€Π°Π½ΠΈΡ‡ΠΈΠ²Π°Π΅Ρ‚ ΠΏΠΎΡ‚ΠΎΠΊ элСктричСства Ρ‡Π΅Ρ€Π΅Π· Π½Π΅Π³ΠΎΒ». Π‘ΠΎΠΏΡ€ΠΎΡ‚ΠΈΠ²Π»Π΅Π½ΠΈΠ΅ измСряСтся Π² Ом — Ρ‚Π΅Ρ€ΠΌΠΈΠ½, Π°Π½Π°Π»ΠΎΠ³ΠΈΡ‡Π½Ρ‹ΠΉ Ρ‚Ρ€Π΅Π½ΠΈΡŽ Π² шлангС ΠΈΠ»ΠΈ Ρ‚Ρ€ΡƒΠ±Π΅.

Π’Ρ‹Π΄Π΅Ρ€ΠΆΠΊΠ° ΠΈΠ· ЭлСктричСскиС систСмы для ΠΎΠ΄Π½ΠΎ- ΠΈ Π΄Π²ΡƒΡ…ΠΊΠ²Π°Ρ€Ρ‚ΠΈΡ€Π½Ρ‹Ρ… Π΄ΠΎΠΌΠΎΠ² , 8 -Π΅ ΠΈΠ·Π΄Π°Π½ΠΈΠ΅ . Π­Ρ‚Π° ΠΊΠ½ΠΈΠ³Π° доступна ΠΏΠΎ адрСсу www.iaei.org/web/shop ΠΈΠ»ΠΈ Amazon.com .

Π˜Π·ΡƒΡ‡Π΅Π½ΠΈΠ΅ Π·Π°ΠΊΠΎΠ½Π° Ома | BCHydro Power Smart для школ

ΠžΠ±Π·ΠΎΡ€

ΠŸΠΎΡΠΌΠΎΡ‚Ρ€ΠΈΡ‚Π΅ Π²ΠΈΠ΄Π΅ΠΎ, ΠΎΠ±ΡŠΡΡΠ½ΡΡŽΡ‰Π΅Π΅ Π·Π°ΠΊΠΎΠ½ Ома, Π·Π°Ρ‚Π΅ΠΌ постройтС схСму ΠΈ ΠΏΡ€ΠΎΠ²Π΅Π΄ΠΈΡ‚Π΅ Π΄Π΅ΠΌΠΎΠ½ΡΡ‚Ρ€Π°Ρ†ΠΈΡŽ, Ρ‡Ρ‚ΠΎΠ±Ρ‹ учащиСся ΠΌΠΎΠ³Π»ΠΈ Π½Π°Π±Π»ΡŽΠ΄Π°Ρ‚ΡŒ взаимосвязь ΠΌΠ΅ΠΆΠ΄Ρƒ напряТСниСм, Ρ‚ΠΎΠΊΠΎΠΌ ΠΈ сопротивлСниСм.

Π˜Π½ΡΡ‚Ρ€ΡƒΠΊΡ†ΠΈΠΈ
РазъяснСниС закона Ома

ΠŸΠΎΡΠΌΠΎΡ‚Ρ€ΠΈΡ‚Π΅ Π²ΠΈΠ΄Π΅ΠΎ «РазъяснСниС Π·Π°ΠΊΠΎΠ½Π° Ома» , Ρ‡Ρ‚ΠΎΠ±Ρ‹ ΠΏΠΎΠ·Π½Π°ΠΊΠΎΠΌΠΈΡ‚ΡŒ студСнтов с Π·Π°ΠΊΠΎΠ½ΠΎΠΌ Ома.

Π’Π²Π΅Π΄Π΅Π½ΠΈΠ΅ Π² Ρ‚Π΅ΠΌΡƒ

НастройтС схСму, ΠΊΠ°ΠΊ ΠΏΠΎΠΊΠ°Π·Π°Π½ΠΎ здСсь:

ΠŸΡ€ΠΎΡΠΌΠΎΡ‚Ρ€ΠΈΡ‚Π΅ Ρ€Π°Π±ΠΎΡ‡ΠΈΠΉ лист Β«Π˜Π·ΡƒΡ‡Π΅Π½ΠΈΠ΅ Π·Π°ΠΊΠΎΠ½Π° Ома» со студСнтами.

ΠŸΡ€ΠΎΠ²Π΅Π΄ΠΈΡ‚Π΅ Π΄Π΅ΠΌΠΎΠ½ΡΡ‚Ρ€Π°Ρ†ΠΈΡŽ
  • Π˜ΡΠΏΠΎΠ»ΡŒΠ·ΡƒΡ Π°ΠΌΠΏΠ΅Ρ€ΠΌΠ΅Ρ‚Ρ€ ΠΈ Π²ΠΎΠ»ΡŒΡ‚ΠΌΠ΅Ρ‚Ρ€, ΠΏΠΎΠΊΠ°ΠΆΠΈΡ‚Π΅ учащимся, ΠΊΠ°ΠΊ ΡΡ‡ΠΈΡ‚Ρ‹Π²Π°Ρ‚ΡŒ значСния Ρ‚ΠΎΠΊΠ° ΠΈ напряТСния Π² Ρ†Π΅ΠΏΠΈ.Пока Π²Ρ‹ ΠΏΡ€ΠΎΠ²ΠΎΠ΄ΠΈΡ‚Π΅ измСрСния, Π·Π°ΠΏΠΈΡˆΠΈΡ‚Π΅ Π΄Π°Π½Π½Ρ‹Π΅ Π½Π° доскС ΠΈ попроситС учащихся Π·Π°ΠΏΠΈΡΠ°Ρ‚ΡŒ Π΄Π°Π½Π½Ρ‹Π΅ Π½Π° своих Ρ€Π°Π±ΠΎΡ‡ΠΈΡ… листах. НапомнитС ΠΈΠΌ ΠΏΡ€Π΅ΠΎΠ±Ρ€Π°Π·ΠΎΠ²Π°Ρ‚ΡŒ мА Π² А; 1 Π°ΠΌΠΏΠ΅Ρ€ = 1000 ΠΌΠΈΠ»Π»ΠΈΠ°ΠΌΠΏΠ΅Ρ€.
  • ΠŸΠΎΡΠ»Π΅Π΄ΠΎΠ²Π°Ρ‚Π΅Π»ΡŒΠ½ΠΎ Π΄ΠΎΠ±Π°Π²ΡŒΡ‚Π΅ ΡΡƒΡ…ΡƒΡŽ ячСйку ΠΈ ΠΏΠΎΠ²Ρ‚ΠΎΡ€ΠΈΡ‚Π΅ измСрСния.
  • Если Ρƒ вас большС сухих ячССк, добавляйтС ΠΈΡ… ΠΏΠΎΡΠ»Π΅Π΄ΠΎΠ²Π°Ρ‚Π΅Π»ΡŒΠ½ΠΎ ΠΏΠΎ ΠΎΠ΄Π½ΠΎΠΉ ΠΈ повторяйтС измСрСния ΠΊΠ°ΠΆΠ΄Ρ‹ΠΉ Ρ€Π°Π·.
ΠŸΠΎΡΡ‚Ρ€ΠΎΠΉΡ‚Π΅ Π³Ρ€Π°Ρ„ΠΈΠΊ сопротивлСния

Π˜ΡΠΏΠΎΠ»ΡŒΠ·ΡƒΡ Π΄Π°Π½Π½Ρ‹Π΅ ΠΈΠ· Ρ‚Π°Π±Π»ΠΈΡ†Ρ‹, попроситС учащихся ΠΏΠΎΡΡ‚Ρ€ΠΎΠΈΡ‚ΡŒ Π³Ρ€Π°Ρ„ΠΈΠΊ зависимости напряТСния ΠΎΡ‚ Ρ‚ΠΎΠΊΠ° (V vs.Π―). Π£Π±Π΅Π΄ΠΈΡ‚Π΅ΡΡŒ, Ρ‡Ρ‚ΠΎ ΠΎΠ½ΠΈ ΠΏΠΎΠΌΠ΅Ρ‡Π°ΡŽΡ‚ всС части своСго Π³Ρ€Π°Ρ„ΠΈΠΊΠ°. ΠžΠ±ΡŠΡΡΠ½ΠΈΡ‚Π΅, какая линия Π»ΡƒΡ‡ΡˆΠ΅ всСго ΠΏΠΎΠ΄Ρ…ΠΎΠ΄ΠΈΡ‚, ΠΈ попроситС учащихся Π½Π°Ρ€ΠΈΡΠΎΠ²Π°Ρ‚ΡŒ Π΅Π΅ Π½Π° своСм Π³Ρ€Π°Ρ„ΠΈΠΊΠ΅.

ΠŸΠΎΠΏΡ€ΠΎΡΠΈΡ‚Π΅ учащихся Π²Ρ‹Ρ‡ΠΈΡΠ»ΠΈΡ‚ΡŒ Π½Π°ΠΊΠ»ΠΎΠ½ Π»ΠΈΠ½ΠΈΠΈ ΠΏΠΎ Π½Π°ΠΈΠ±ΠΎΠ»Π΅Π΅ подходящСй Π»ΠΈΠ½ΠΈΠΈ:

  • Π’Ρ‹Π±Π΅Ρ€ΠΈΡ‚Π΅ Π΄Π²Π΅ Ρ‚ΠΎΡ‡ΠΊΠΈ Π½Π° прямой (Ρ‚ΠΎΡ‡ΠΊΠ° A ΠΈ Ρ‚ΠΎΡ‡ΠΊΠ° B).
  • РассчитайтС Ρ€Π°Π·Π½ΠΈΡ†Ρƒ ΠΌΠ΅ΠΆΠ΄Ρƒ напряТСниями Π² Π΄Π²ΡƒΡ… Ρ‚ΠΎΡ‡ΠΊΠ°Ρ… (ΠΠΠ ΠΠ©Π˜Π’ΠΠΠ˜Π• Π½Π°ΠΊΠ»ΠΎΠ½Π°).
  • ВычислитС Ρ€Π°Π·Π½ΠΈΡ†Ρƒ ΠΌΠ΅ΠΆΠ΄Ρƒ Ρ‚ΠΎΠΊΠΎΠΌ Π² Π΄Π²ΡƒΡ… Ρ‚ΠΎΡ‡ΠΊΠ°Ρ… (ΠŸΠ•Π Π•Π”ΠΠ§Π ΠΠΠšΠ›ΠžΠΠ).
  • Π Π°Π·Π΄Π΅Π»ΠΈΡ‚Π΅ ΠŸΠžΠ”ΠͺΠ•Πœ Π½Π° Π‘Π•Π“. Π­Ρ‚ΠΎ Π½Π°ΠΊΠ»ΠΎΠ½ Π»ΠΈΠ½ΠΈΠΈ.

Π‘ΠΎΠΏΡ€ΠΎΡ‚ΠΈΠ²Π»Π΅Π½ΠΈΠ΅ Ρ†Π΅ΠΏΠΈ матСматичСски отобраТаСтся Π² Π²ΠΈΠ΄Π΅ алгСбраичСского уравнСния:

  • Π‘ΠΎΠΏΡ€ΠΎΡ‚ΠΈΠ²Π»Π΅Π½ΠΈΠ΅ = напряТСниС / Ρ‚ΠΎΠΊ.
Π˜Π½Ρ‚Π΅Ρ€ΠΏΡ€Π΅Ρ‚Π°Ρ†ΠΈΡ Π΄Π°Π½Π½Ρ‹Ρ…

Π‘Ρ€Π°Π²Π½ΠΈΡ‚Π΅ Π½Π°ΠΊΠ»ΠΎΠ½ Π³Ρ€Π°Ρ„ΠΈΠΊΠΎΠ², созданных вашими ΡƒΡ‡Π΅Π½ΠΈΠΊΠ°ΠΌΠΈ, с заявлСнным сопротивлСниСм рСзистора, ΠΊΠΎΡ‚ΠΎΡ€Ρ‹ΠΉ Π²Ρ‹ использовали. Π¦ΠΈΡ„Ρ€Ρ‹ Π΄ΠΎΠ»ΠΆΠ½Ρ‹ Π±Ρ‹Ρ‚ΡŒ ΠΏΠΎΡ…ΠΎΠΆΠΈΠΌΠΈ (Ρ€Π°Π·Π½Ρ‹Π΅ числа ΡΠ²Π»ΡΡŽΡ‚ΡΡ Ρ€Π΅Π·ΡƒΠ»ΡŒΡ‚Π°Ρ‚ΠΎΠΌ ΠΈΠ½Π΄ΠΈΠ²ΠΈΠ΄ΡƒΠ°Π»ΡŒΠ½Ρ‹Ρ… Ρ€Π°Π·Π»ΠΈΡ‡ΠΈΠΉ Π² Π²Ρ‹Π±ΠΎΡ€Π΅ Π½Π°ΠΈΠ±ΠΎΠ»Π΅Π΅ подходящСй Π»ΠΈΠ½ΠΈΠΈ).

Бвязь ΠΌΠ΅ΠΆΠ΄Ρƒ напряТСниСм ΠΈ Ρ‚ΠΎΠΊΠΎΠΌ — это Π·Π°ΠΊΠΎΠ½ Ома, Π° Π½Π°ΠΊΠ»ΠΎΠ½ Π»ΠΈΠ½ΠΈΠΈ Π½Π° Π³Ρ€Π°Ρ„ΠΈΠΊΠ΅ этих Π΄Π²ΡƒΡ… Π²Π΅Π»ΠΈΡ‡ΠΈΠ½ являСтся Π·Π½Π°Ρ‡Π΅Π½ΠΈΠ΅ΠΌ сопротивлСния Π² Ρ†Π΅ΠΏΠΈ. Π£Ρ€Π°Π²Π½Π΅Π½ΠΈΠ΅ Π·Π°ΠΊΠΎΠ½Π° Ома ΠΌΠΎΠΆΠ½ΠΎ ΠΏΡ€Π΅Π΄ΡΡ‚Π°Π²ΠΈΡ‚ΡŒ трСмя способами:

  • R = V / I (сопротивлСниС = напряТСниС, Π΄Π΅Π»Π΅Π½Π½ΠΎΠ΅ Π½Π° Ρ‚ΠΎΠΊ)
  • V = I x R (напряТСниС = Ρ‚ΠΎΠΊ x сопротивлСниС)
  • I = V / R (Ρ‚ΠΎΠΊ = напряТСниС, Π΄Π΅Π»Π΅Π½Π½ΠΎΠ΅ Π½Π° сопротивлСниС)

Как соотносятся напряТСниС, Ρ‚ΠΎΠΊ ΠΈ сопротивлСниС: Π—Π°ΠΊΠΎΠ½ Ома

Π’ΠΎΠΌ I — ΠžΠΊΡ€ΡƒΠ³ ΠšΠΎΠ»ΡƒΠΌΠ±ΠΈΡ Β»Π—ΠΠšΠžΠ ОМА»

ЭлСктричСская Ρ†Π΅ΠΏΡŒ образуСтся, ΠΊΠΎΠ³Π΄Π° создаСтся токопроводящий ΠΏΡƒΡ‚ΡŒ для ΠΏΠΎΠ·Π²ΠΎΠ»ΡΡŽΡ‚ свободным элСктронам Π½Π΅ΠΏΡ€Π΅Ρ€Ρ‹Π²Π½ΠΎ Π΄Π²ΠΈΠ³Π°Ρ‚ΡŒΡΡ.Π­Ρ‚ΠΎ Π½Π΅ΠΏΡ€Π΅Ρ€Ρ‹Π²Π½ΠΎΠ΅ Π΄Π²ΠΈΠΆΠ΅Π½ΠΈΠ΅ Π‘Π²ΠΎΠ±ΠΎΠ΄Π½Ρ‹Π΅ элСктроны, проходящиС Ρ‡Π΅Ρ€Π΅Π· ΠΏΡ€ΠΎΠ²ΠΎΠ΄Π½ΠΈΠΊΠΈ Ρ†Π΅ΠΏΠΈ, Π½Π°Π·Ρ‹Π²Π°ΡŽΡ‚ Ρ‚ΠΎΠΊΠΎΠΌ , ΠΈ Π΅Π³ΠΎ часто Π½Π°Π·Ρ‹Π²Π°ΡŽΡ‚ Β«ΠΏΠΎΡ‚ΠΎΠΊΠΎΠΌΒ», ΠΊΠ°ΠΊ ΠΏΠΎΡ‚ΠΎΠΊ Тидкости Ρ‡Π΅Ρ€Π΅Π· ΠΏΠΎΠ»ΡƒΡŽ Ρ‚Ρ€ΡƒΠ±Ρƒ.

Π‘ΠΈΠ»Π°, ΠΏΠΎΠ±ΡƒΠΆΠ΄Π°ΡŽΡ‰Π°Ρ элСктроны Β«Ρ‚Π΅Ρ‡ΡŒΒ» Π² Ρ†Π΅ΠΏΠΈ, называСтся напряТСниСм , напряТСниСм . НапряТСниС — это особая ΠΌΠ΅Ρ€Π° ΠΏΠΎΡ‚Π΅Π½Ρ†ΠΈΠ°Π»ΡŒΠ½ΠΎΠΉ энСргии, которая всСгда ΠΎΡ‚Π½ΠΎΡΠΈΡ‚Π΅Π»ΡŒΠ½Ρ‹ΠΉ ΠΌΠ΅ΠΆΠ΄Ρƒ двумя Ρ‚ΠΎΡ‡ΠΊΠ°ΠΌΠΈ. Когда ΠΌΡ‹ Π³ΠΎΠ²ΠΎΡ€ΠΈΠΌ ΠΎΠ± ΠΎΠΏΡ€Π΅Π΄Π΅Π»Π΅Π½Π½ΠΎΠΌ количСствС напряТСниС, ΠΏΡ€ΠΈΡΡƒΡ‚ΡΡ‚Π²ΡƒΡŽΡ‰Π΅Π΅ Π² Ρ†Π΅ΠΏΠΈ, ΠΌΡ‹ ΠΈΠΌΠ΅Π΅ΠΌ Π² Π²ΠΈΠ΄Ρƒ ΠΈΠ·ΠΌΠ΅Ρ€Π΅Π½ΠΈΠ΅ ΠΎ Ρ‚ΠΎΠΌ, сколько ΠΏΠΎΡ‚Π΅Π½Ρ†ΠΈΠ°Π»ΡŒΠ½ΠΎΠΉ энСргии сущСствуСт для пСрСмСщСния элСктронов ΠΈΠ· ΠΎΠ΄Π½ΠΎΠΉ ΠΊΠΎΠ½ΠΊΡ€Π΅Ρ‚Π½ΠΎΠΉ Ρ‚ΠΎΡ‡ΠΊΠΈ Π² этой Ρ†Π΅ΠΏΠΈ Π² Π΄Ρ€ΡƒΠ³ΡƒΡŽ ΠΊΠΎΠ½ΠΊΡ€Π΅Ρ‚Π½ΡƒΡŽ Ρ‚ΠΎΡ‡ΠΊΡƒ.Π‘Π΅Π· ссылки Π½Π° , Π΄Π²Π° ΠΊΠΎΠ½ΠΊΡ€Π΅Ρ‚Π½Ρ‹Ρ… ΠΏΡƒΠ½ΠΊΡ‚Π° , Ρ‚Π΅Ρ€ΠΌΠΈΠ½ «напряТСниС» Π½Π΅ ΠΈΠΌΠ΅Π΅Ρ‚ значСния.

Π‘Π²ΠΎΠ±ΠΎΠ΄Π½Ρ‹Π΅ элСктроны ΠΈΠΌΠ΅ΡŽΡ‚ Ρ‚Π΅Π½Π΄Π΅Π½Ρ†ΠΈΡŽ ΠΏΠ΅Ρ€Π΅ΠΌΠ΅Ρ‰Π°Ρ‚ΡŒΡΡ ΠΏΠΎ ΠΏΡ€ΠΎΠ²ΠΎΠ΄Π½ΠΈΠΊΠ°ΠΌ с Π½Π΅ΠΊΠΎΡ‚ΠΎΡ€ΠΎΠΉ ΡΡ‚Π΅ΠΏΠ΅Π½ΡŒΡŽ Ρ‚Ρ€Π΅Π½ΠΈΠ΅ ΠΈΠ»ΠΈ противодСйствиС двиТСнию. Π­Ρ‚ΠΎ противодСйствиС двиТСнию большС ΠΏΡ€Π°Π²ΠΈΠ»ΡŒΠ½ΠΎ называСтся сопротивлСниС . ΠšΠΎΠ»ΠΈΡ‡Π΅ΡΡ‚Π²ΠΎ Ρ‚ΠΎΠΊΠ° Π² Ρ†Π΅ΠΏΠΈ зависит ΠΎΡ‚ количСства доступного напряТСния, Ρ‡Ρ‚ΠΎΠ±Ρ‹ ΠΌΠΎΡ‚ΠΈΠ²ΠΈΡ€ΠΎΠ²Π°Ρ‚ΡŒ элСктронов, Π° Ρ‚Π°ΠΊΠΆΠ΅ количСство сопротивлСния Π² Ρ†Π΅ΠΏΠΈ, Ρ‡Ρ‚ΠΎΠ±Ρ‹ ΠΏΡ€ΠΎΡ‚ΠΈΠ²ΠΎΡΡ‚ΠΎΡΡ‚ΡŒ элСктронный ΠΏΠΎΡ‚ΠΎΠΊ.Как ΠΈ напряТСниС, сопротивлСниС — Π²Π΅Π»ΠΈΡ‡ΠΈΠ½Π° ΠΎΡ‚Π½ΠΎΡΠΈΡ‚Π΅Π»ΡŒΠ½Π°Ρ. ΠΌΠ΅ΠΆΠ΄Ρƒ двумя Ρ‚ΠΎΡ‡ΠΊΠ°ΠΌΠΈ. По этой ΠΏΡ€ΠΈΡ‡ΠΈΠ½Π΅ Π²Π΅Π»ΠΈΡ‡ΠΈΠ½Ρ‹ напряТСния ΠΈ сопротивлСниС часто указываСтся ΠΊΠ°ΠΊ Β«ΠΌΠ΅ΠΆΠ΄ΡƒΒ» ΠΈΠ»ΠΈ Β«ΠΏΠΎΠΏΠ΅Ρ€Π΅ΠΊΒ» Π΄Π²ΡƒΡ… Ρ‚ΠΎΡ‡Π΅ΠΊ Π² Ρ†Π΅ΠΏΠΈ.

Π§Ρ‚ΠΎΠ±Ρ‹ ΠΈΠΌΠ΅Ρ‚ΡŒ Π²ΠΎΠ·ΠΌΠΎΠΆΠ½ΠΎΡΡ‚ΡŒ Π΄Π΅Π»Π°Ρ‚ΡŒ Π·Π½Π°Ρ‡ΠΈΠΌΡ‹Π΅ заявлСния ΠΎΠ± этих количСствах Π² Ρ†Π΅ΠΏΠ΅ΠΉ, ΠΌΡ‹ Π΄ΠΎΠ»ΠΆΠ½Ρ‹ ΠΈΠΌΠ΅Ρ‚ΡŒ Π²ΠΎΠ·ΠΌΠΎΠΆΠ½ΠΎΡΡ‚ΡŒ ΠΎΠΏΠΈΡΡ‹Π²Π°Ρ‚ΡŒ ΠΈΡ… количСство Π² ΠΎΠ΄Π½ΠΎΠΌ ΠΈ Ρ‚ΠΎΠΌ ΠΆΠ΅ способ, ΠΊΠΎΡ‚ΠΎΡ€Ρ‹ΠΌ ΠΌΡ‹ ΠΌΠΎΠ³Π»ΠΈ Π±Ρ‹ количСствСнно ΠΎΠΏΡ€Π΅Π΄Π΅Π»ΠΈΡ‚ΡŒ массу, Ρ‚Π΅ΠΌΠΏΠ΅Ρ€Π°Ρ‚ΡƒΡ€Ρƒ, объСм, Π΄Π»ΠΈΠ½Ρƒ ΠΈΠ»ΠΈ любой Π΄Ρ€ΡƒΠ³ΠΎΠΉ Π΄Ρ€ΡƒΠ³ΠΎΠΉ Π²ΠΈΠ΄ физичСской Π²Π΅Π»ΠΈΡ‡ΠΈΠ½Ρ‹. Для массы ΠΌΡ‹ ΠΌΠΎΠΆΠ΅ΠΌ ΠΈΡΠΏΠΎΠ»ΡŒΠ·ΠΎΠ²Π°Ρ‚ΡŒ Π΅Π΄ΠΈΠ½ΠΈΡ†Ρ‹ Β«Ρ„ΡƒΠ½Ρ‚Β» ΠΈΠ»ΠΈ Β«Π³Ρ€Π°ΠΌΠΌΒ».»Π”ля Ρ‚Π΅ΠΌΠΏΠ΅Ρ€Π°Ρ‚ΡƒΡ€Ρ‹ ΠΌΡ‹ ΠΌΠΎΠΆΠ΅ΠΌ ΠΈΡΠΏΠΎΠ»ΡŒΠ·ΠΎΠ²Π°Ρ‚ΡŒ градусы Π€Π°Ρ€Π΅Π½Π³Π΅ΠΉΡ‚Π° ΠΈΠ»ΠΈ градусов ЦСльсия. Π’ΠΎΡ‚ стандартныС Π΅Π΄ΠΈΠ½ΠΈΡ†Ρ‹ измСрСния для элСктричСский Ρ‚ΠΎΠΊ, напряТСниС ΠΈ сопротивлСниС:

Β«Π‘ΠΈΠΌΠ²ΠΎΠ»Β», ΡƒΠΊΠ°Π·Π°Π½Π½Ρ‹ΠΉ для ΠΊΠ°ΠΆΠ΄ΠΎΠ³ΠΎ количСства, являСтся стандартным Π±ΡƒΠΊΠ²Π΅Π½Π½Ρ‹ΠΌ ΠΎΠ±ΠΎΠ·Π½Π°Ρ‡Π΅Π½ΠΈΠ΅ΠΌ. Π±ΡƒΠΊΠ²Π°, ΠΈΡΠΏΠΎΠ»ΡŒΠ·ΡƒΠ΅ΠΌΠ°Ρ для обозначСния этой Π²Π΅Π»ΠΈΡ‡ΠΈΠ½Ρ‹ Π² алгСбраичСском ΡƒΡ€Π°Π²Π½Π΅Π½ΠΈΠΈ. ΠŸΠΎΠ΄ΠΎΠ±Π½Ρ‹Π΅ стандартизированныС Π±ΡƒΠΊΠ²Ρ‹ распространСны Π² дисциплинах Ρ„ΠΈΠ·ΠΈΠΊΠ° ΠΈ Ρ‚Π΅Ρ…Π½ΠΈΠΊΠ°, ΠΈ ΠΏΡ€ΠΈΠ·Π½Π°Π½Ρ‹ Π²ΠΎ всСм ΠΌΠΈΡ€Π΅. Π•Π΄ΠΈΠ½ΠΈΡ†Π° Π°Π±Π±Ρ€Π΅Π²ΠΈΠ°Ρ‚ΡƒΡ€Π° «Π΄Π»Ρ ΠΊΠ°ΠΆΠ΄ΠΎΠ³ΠΎ количСства прСдставляСт собой ΠΈΡΠΏΠΎΠ»ΡŒΠ·ΡƒΠ΅ΠΌΡ‹ΠΉ Π°Π»Ρ„Π°Π²ΠΈΡ‚Π½Ρ‹ΠΉ символ. ΠΊΠ°ΠΊ сокращСнноС ΠΎΠ±ΠΎΠ·Π½Π°Ρ‡Π΅Π½ΠΈΠ΅ ΠΊΠΎΠ½ΠΊΡ€Π΅Ρ‚Π½ΠΎΠΉ Π΅Π΄ΠΈΠ½ΠΈΡ†Ρ‹ измСрСния.И, Π΄Π°, этот странный Π½Π° Π²ΠΈΠ΄ символ «ΠΏΠΎΠ΄ΠΊΠΎΠ²Π°» — заглавная грСчСская Π±ΡƒΠΊΠ²Π° Ξ©, просто символ иностранного Π°Π»Ρ„Π°Π²ΠΈΡ‚Π° (извинСния ΠΏΠ΅Ρ€Π΅Π΄ читатСлями-Π³Ρ€Π΅ΠΊΠ°ΠΌΠΈ).

КаТдая Π΅Π΄ΠΈΠ½ΠΈΡ†Π° измСрСния Π½Π°Π·Π²Π°Π½Π° Π² Ρ‡Π΅ΡΡ‚ΡŒ извСстного экспСримСнтатора Π² области элСктричСства: amp Π² Ρ‡Π΅ΡΡ‚ΡŒ Ρ„Ρ€Π°Π½Ρ†ΡƒΠ·Π° АндрС М. АмпСра, Π²ΠΎΠ»ΡŒΡ‚ Π² Ρ‡Π΅ΡΡ‚ΡŒ ΠΈΡ‚Π°Π»ΡŒΡΠ½Ρ†Π° АлСссандро Π’ΠΎΠ»ΡŒΡ‚Π° ΠΈ Ом Π² Ρ‡Π΅ΡΡ‚ΡŒ Π½Π΅ΠΌΡ†Π° Π“Π΅ΠΎΡ€Π³Π° Π‘ΠΈΠΌΠΎΠ½Π° Ома.

ΠœΠ°Ρ‚Π΅ΠΌΠ°Ρ‚ΠΈΡ‡Π΅ΡΠΊΠΈΠΉ символ для ΠΊΠ°ΠΆΠ΄ΠΎΠΉ Π²Π΅Π»ΠΈΡ‡ΠΈΠ½Ρ‹ Ρ‚Π°ΠΊΠΆΠ΅ ΠΈΠΌΠ΅Π΅Ρ‚ Π·Π½Π°Ρ‡Π΅Π½ΠΈΠ΅.Π’ Β«RΒ» для сопротивлСния ΠΈ Β«VΒ» для напряТСния говорят сами Π·Π° сСбя, Ρ‚ΠΎΠ³Π΄Π° ΠΊΠ°ΠΊ «I» для Ρ‚ΠΎΠΊΠ° каТСтся Π½Π΅ΠΌΠ½ΠΎΠ³ΠΎ странным. БчитаСтся, Ρ‡Ρ‚ΠΎ «Ρ» Π΄ΠΎΠ»ΠΆΠ½ΠΎ Π±Ρ‹Π»ΠΎ ΠΏΡ€Π΅Π΄ΡΡ‚Π°Π²Π»ΡΡ‚ΡŒ Β«Π˜Π½Ρ‚Π΅Π½ΡΠΈΠ²Π½ΠΎΡΡ‚ΡŒΒ» (ΠΏΠΎΡ‚ΠΎΠΊΠ° элСктронов) ΠΈ Π΄Ρ€ΡƒΠ³ΠΎΠΉ символ напряТСния, Β«EΒ». Ρ€Π°ΡΡˆΠΈΡ„Ρ€ΠΎΠ²Ρ‹Π²Π°Π΅Ρ‚ΡΡ ΠΊΠ°ΠΊ «ЭлСктродвиТущая сила». Из ΠΊΠ°ΠΊΠΈΡ… исслСдований я смог Π”Π°, ΠΏΠΎΡ…ΠΎΠΆΠ΅, Π΅ΡΡ‚ΡŒ Π½Π΅ΠΊΠΎΡ‚ΠΎΡ€Ρ‹Π΅ споры ΠΎ Π·Π½Π°Ρ‡Π΅Π½ΠΈΠΈ «я». Π‘ΠΈΠΌΠ²ΠΎΠ»Ρ‹ Β«EΒ» ΠΈ Β«VΒ» ΠΏΠΎ большСй части взаимозамСняСмы, хотя Π½Π΅ΠΊΠΎΡ‚ΠΎΡ€Ρ‹Π΅ тСксты Π·Π°Ρ€Π΅Π·Π΅Ρ€Π²ΠΈΡ€ΡƒΠΉΡ‚Π΅ «E» для обозначСния напряТСния Π½Π° источникС (Ρ‚Π°ΠΊΠΎΠΌ ΠΊΠ°ΠΊ батарСя ΠΈΠ»ΠΈ Π³Π΅Π½Π΅Ρ€Π°Ρ‚ΠΎΡ€) ΠΈ «V» для обозначСния напряТСния Π½Π° любом Π΄Ρ€ΡƒΠ³ΠΎΠΌ элСмСнтС.

ВсС эти символы Π²Ρ‹Ρ€Π°ΠΆΠ°ΡŽΡ‚ΡΡ Π·Π°Π³Π»Π°Π²Π½Ρ‹ΠΌΠΈ Π±ΡƒΠΊΠ²Π°ΠΌΠΈ, Π·Π° ΠΈΡΠΊΠ»ΡŽΡ‡Π΅Π½ΠΈΠ΅ΠΌ случаСв, ΠΊΠΎΠ³Π΄Π° Π²Π΅Π»ΠΈΡ‡ΠΈΠ½Π° (особСнно напряТСниС ΠΈΠ»ΠΈ Ρ‚ΠΎΠΊ) описываСтся Π² Ρ‚Π΅Ρ€ΠΌΠΈΠ½Π°Ρ… ΠΊΠΎΡ€ΠΎΡ‚ΠΊΠΎΠ³ΠΎ ΠΏΠ΅Ρ€ΠΈΠΎΠ΄Π° Π²Ρ€Π΅ΠΌΠ΅Π½ΠΈ (Π½Π°Π·Ρ‹Π²Π°Π΅ΠΌΠΎΠ³ΠΎ Β«ΠΌΠ³Π½ΠΎΠ²Π΅Π½Π½ΠΎΠ΅Β» Π·Π½Π°Ρ‡Π΅Π½ΠΈΠ΅). НапримСр, напряТСниС Π±Π°Ρ‚Π°Ρ€Π΅ΠΈ, ΠΊΠΎΡ‚ΠΎΡ€ΠΎΠ΅ ΡΡ‚Π°Π±ΠΈΠ»ΡŒΠ½Ρ‹ΠΉ Π² Ρ‚Π΅Ρ‡Π΅Π½ΠΈΠ΅ Π΄Π»ΠΈΡ‚Π΅Π»ΡŒΠ½ΠΎΠ³ΠΎ ΠΏΠ΅Ρ€ΠΈΠΎΠ΄Π° Π²Ρ€Π΅ΠΌΠ΅Π½ΠΈ, Π±ΡƒΠ΄Π΅Ρ‚ ΠΎΠ±ΠΎΠ·Π½Π°Ρ‡Π°Ρ‚ΡŒΡΡ Π·Π°Π³Π»Π°Π²Π½ΠΎΠΉ Π±ΡƒΠΊΠ²ΠΎΠΉ Π±ΡƒΠΊΠ²Π° Β«Π•Β», Π° ΠΏΠΈΠΊ напряТСния ΡƒΠ΄Π°Ρ€Π° ΠΌΠΎΠ»Π½ΠΈΠΈ Π² самом ΠΌΠΎΠΌΠ΅Π½Ρ‚, ΠΊΠΎΠ³Π΄Π° ΠΎΠ½ ΠΏΠΎΠΏΠ°Π΄Π΅Ρ‚ Π² линию элСктропСрСдачи, скорСС всСго, Π±ΡƒΠ΄Π΅Ρ‚ ΠΎΠ±ΠΎΠ·Π½Π°Ρ‡Π΅Π½ строчная Π±ΡƒΠΊΠ²Π° Β«Π΅Β» (ΠΈΠ»ΠΈ строчная Π±ΡƒΠΊΠ²Π° Β«vΒ») для обозначСния этого значСния ΠΊΠ°ΠΊ Π½Π°Ρ…ΠΎΠ΄ΡΡΡŒ Π² ΠΎΠ΄ΠΈΠ½ ΠΌΠΎΠΌΠ΅Π½Ρ‚ Π²Ρ€Π΅ΠΌΠ΅Π½ΠΈ.Π­Ρ‚ΠΎ ΠΆΠ΅ соглашСниС ΠΎ Π½ΠΈΠΆΠ½Π΅ΠΌ рСгистрС выполняСтся Π²Π΅Ρ€Π½ΠΎ ΠΈ для Ρ‚ΠΎΠΊΠ°, строчная Π±ΡƒΠΊΠ²Π° Β«iΒ» ΠΎΠ±ΠΎΠ·Π½Π°Ρ‡Π°Π΅Ρ‚ Ρ‚ΠΎΠΊ Π² Π½Π΅ΠΊΠΎΡ‚ΠΎΡ€Ρ‹ΠΉ ΠΌΠΎΠΌΠ΅Π½Ρ‚ Π²Ρ€Π΅ΠΌΠ΅Π½ΠΈ. Однако Π±ΠΎΠ»ΡŒΡˆΠΈΠ½ΡΡ‚Π²ΠΎ ΠΈΠ·ΠΌΠ΅Ρ€Π΅Π½ΠΈΠΉ постоянного Ρ‚ΠΎΠΊΠ° (DC), ΠΊΠΎΡ‚ΠΎΡ€Ρ‹Π΅ ΡΡ‚Π°Π±ΠΈΠ»ΡŒΠ½Ρ‹ Π²ΠΎ Π²Ρ€Π΅ΠΌΠ΅Π½ΠΈ, Π±ΡƒΠ΄ΡƒΡ‚ ΠΎΠ±ΠΎΠ·Π½Π°Ρ‡Π΅Π½Ρ‹ Π·Π°Π³Π»Π°Π²Π½Ρ‹ΠΌΠΈ Π±ΡƒΠΊΠ²Π°ΠΌΠΈ.

Одна ΠΎΡΠ½ΠΎΠ²ΠΎΠΏΠΎΠ»Π°Π³Π°ΡŽΡ‰Π°Ρ Π΅Π΄ΠΈΠ½ΠΈΡ†Π° элСктричСского измСрСния, ΠΊΠΎΡ‚ΠΎΡ€ΠΎΠΉ часто ΡƒΡ‡Π°Ρ‚ Π² Π½Π°Ρ‡Π°Π»ΠΎ курсов элСктроники, Π½ΠΎ впослСдствии Ρ€Π΅Π΄ΠΊΠΎ ΠΈΡΠΏΠΎΠ»ΡŒΠ·ΡƒΠ΅ΠΌΠΎΠ΅, Π±Π»ΠΎΠΊ ΠΊΡƒΠ»ΠΎΠ½ , ΠΊΠΎΡ‚ΠΎΡ€Ρ‹ΠΉ являСтся ΠΌΠ΅Ρ€ΠΎΠΉ элСктричСского заряда, ΠΏΡ€ΠΎΠΏΠΎΡ€Ρ†ΠΈΠΎΠ½Π°Π»ΡŒΠ½ΠΎΠΉ количСству элСктроны Π² нСсбалансированном состоянии.Один ΠΊΡƒΠ»ΠΎΠ½ заряда Ρ€Π°Π²Π΅Π½ 6 250 000 000 000 000 000 элСктронов. Π‘ΠΈΠΌΠ²ΠΎΠ» элСктричСского заряда количСство — заглавная Π±ΡƒΠΊΠ²Π° «Q» с Π΅Π΄ΠΈΠ½ΠΈΡ†Π΅ΠΉ измСрСния ΠΊΡƒΠ»ΠΎΠ½Ρ‹. сокращСнно Π·Π°Π³Π»Π°Π²Π½ΠΎΠΉ Π±ΡƒΠΊΠ²ΠΎΠΉ «C». Π’Π°ΠΊ ΠΏΠΎΠ»ΡƒΡ‡ΠΈΠ»ΠΎΡΡŒ, Ρ‡Ρ‚ΠΎ Π°Π³Ρ€Π΅Π³Π°Ρ‚ для ΠΏΠΎΡ‚ΠΎΠΊ элСктронов, amp, Ρ€Π°Π²Π΅Π½ 1 ΠΊΡƒΠ»ΠΎΠ½Ρƒ элСктронов, проходящих Ρ‡Π΅Ρ€Π΅Π· Π΄Π°Π½Π½Ρ‹ΠΉ ΠΌΠΎΠΌΠ΅Π½Ρ‚ Π² Ρ†Π΅ΠΏΠΈ Π·Π° 1 сСкунду Π²Ρ€Π΅ΠΌΠ΅Π½ΠΈ. Π’ этих Ρ‚Π΅Ρ€ΠΌΠΈΠ½Π°Ρ… Ρ‚ΠΎΠΊ — это ΡΠΊΠΎΡ€ΠΎΡΡ‚ΡŒ двиТСния элСктричСского заряда ΠΏΠΎ ΠΏΡ€ΠΎΠ²ΠΎΠ΄Π½ΠΈΠΊΡƒ.

Как ΡƒΠΊΠ°Π·Ρ‹Π²Π°Π»ΠΎΡΡŒ Ρ€Π°Π½Π΅Π΅, напряТСниС являСтся ΠΌΠ΅Ρ€ΠΎΠΉ ΠΏΠΎΡ‚Π΅Π½Ρ†ΠΈΠ°Π»ΡŒΠ½ΠΎΠΉ энСргии Π½Π° Π΅Π΄ΠΈΠ½ΠΈΡ†Ρƒ заряда , доступной для пСрСмСщСния элСктронов ΠΈΠ· ΠΎΠ΄Π½ΠΎΠΉ Ρ‚ΠΎΡ‡ΠΊΠΈ Π² Π΄Ρ€ΡƒΠ³ΡƒΡŽ.ΠŸΡ€Π΅ΠΆΠ΄Π΅ Ρ‡Π΅ΠΌ ΠΌΡ‹ смоТСм Ρ‚ΠΎΡ‡Π½ΠΎ ΠΎΠΏΡ€Π΅Π΄Π΅Π»ΠΈΡ‚ΡŒ, Ρ‡Ρ‚ΠΎ Ρ‚Π°ΠΊΠΎΠ΅ Β«Π²ΠΎΠ»ΡŒΡ‚Β» Ρ‚ΠΎ Π΅ΡΡ‚ΡŒ, ΠΌΡ‹ Π΄ΠΎΠ»ΠΆΠ½Ρ‹ ΠΏΠΎΠ½ΡΡ‚ΡŒ, ΠΊΠ°ΠΊ ΠΈΠ·ΠΌΠ΅Ρ€ΠΈΡ‚ΡŒ эту Π²Π΅Π»ΠΈΡ‡ΠΈΠ½Ρƒ, ΠΊΠΎΡ‚ΠΎΡ€ΡƒΡŽ ΠΌΡ‹ Π½Π°Π·Ρ‹Π²Π°Π΅ΠΌ «ΠΏΠΎΡ‚Π΅Π½Ρ†ΠΈΠ°Π» энСргия Β». ΠžΠ±Ρ‰Π°Ρ Π΅Π΄ΠΈΠ½ΠΈΡ†Π° измСрСния энСргии любого Π²ΠΈΠ΄Π° — Π΄ΠΆΠΎΡƒΠ»ΡŒ , Ρ€Π°Π²Π½ΠΎ количСству Ρ€Π°Π±ΠΎΡ‚Ρ‹, Π²Ρ‹ΠΏΠΎΠ»Π½Π΅Π½Π½ΠΎΠΉ ΠΏΡ€ΠΈΠ»ΠΎΠΆΠ΅Π½Π½ΠΎΠΉ силой Π² 1 Π½ΡŒΡŽΡ‚ΠΎΠ½ Ρ‡Π΅Ρ€Π΅Π· Π΄Π²ΠΈΠΆΠ΅Π½ΠΈΠ΅ Π½Π° 1 ΠΌΠ΅Ρ‚Ρ€ (Π² Ρ‚ΠΎΠΌ ΠΆΠ΅ Π½Π°ΠΏΡ€Π°Π²Π»Π΅Π½ΠΈΠΈ). Π’ британских частях это Ρ‡ΡƒΡ‚ΡŒ мСньшС 3/4 Ρ„ΡƒΠ½Ρ‚Π° силы, ΠΏΡ€ΠΈΠ»ΠΎΠΆΠ΅Π½Π½ΠΎΠΉ Π½Π° расстоянии 1 Ρ„ΡƒΡ‚. ΠŸΡ€ΠΎΡ‰Π΅ говоря, трСбуСтся ΠΎΠΊΠΎΠ»ΠΎ 1 дТоуля энСргии для ΠΏΠΎΠ΄Π½ΠΈΠΌΠΈΡ‚Π΅ Π³ΠΈΡ€ΡŽ 3/4 Ρ„ΡƒΠ½Ρ‚Π° Π½Π° 1 Ρ„ΡƒΡ‚ ΠΎΡ‚ Π·Π΅ΠΌΠ»ΠΈ ΠΈΠ»ΠΈ ΠΏΠ΅Ρ€Π΅Ρ‚Π°Ρ‰ΠΈΡ‚Π΅ Ρ‡Ρ‚ΠΎ-Π½ΠΈΠ±ΡƒΠ΄ΡŒ расстояниС Π² 1 Ρ„ΡƒΡ‚ с использованиСм ΠΏΠ°Ρ€Π°Π»Π»Π΅Π»ΡŒΠ½ΠΎΠ³ΠΎ тягового усилия 3/4 Ρ„ΡƒΠ½Ρ‚Π°.ΠžΠΏΡ€Π΅Π΄Π΅Π»Π΅Π½Π½Ρ‹ΠΉ Π² этих Π½Π°ΡƒΡ‡Π½Ρ‹Ρ… Ρ‚Π΅Ρ€ΠΌΠΈΠ½Π°Ρ… 1 Π²ΠΎΠ»ΡŒΡ‚ Ρ€Π°Π²Π΅Π½ 1 дТоуля элСктричСской ΠΏΠΎΡ‚Π΅Π½Ρ†ΠΈΠ°Π»ΡŒΠ½ΠΎΠΉ энСргии Π½Π° (Π΄Π΅Π»Π΅Π½Π½Ρ‹ΠΉ Π½Π°) 1 ΠΊΡƒΠ»ΠΎΠ½ заряда. Π’Π°ΠΊΠΈΠΌ ΠΎΠ±Ρ€Π°Π·ΠΎΠΌ, батарСя Π½Π° 9 Π²ΠΎΠ»ΡŒΡ‚ выдСляСт 9 Π΄ΠΆΠΎΡƒΠ»Π΅ΠΉ энСргии Π½Π° ΠΊΠ°ΠΆΠ΄Ρ‹ΠΉ ΠΊΡƒΠ»ΠΎΠ½ элСктронов, ΠΏΠ΅Ρ€Π΅ΠΌΠ΅Ρ‰Π°Π΅ΠΌΡ‹Ρ… ΠΏΠΎ Ρ†Π΅ΠΏΠΈ.

Π­Ρ‚ΠΈ Π΅Π΄ΠΈΠ½ΠΈΡ†Ρ‹ ΠΈ символы элСктричСских Π²Π΅Π»ΠΈΡ‡ΠΈΠ½ станут ΠΎΡ‡Π΅Π½ΡŒ Π²Π°ΠΆΠ½ΠΎ Π·Π½Π°Ρ‚ΡŒ, ΠΊΠΎΠ³Π΄Π° ΠΌΡ‹ Π½Π°Ρ‡ΠΈΠ½Π°Π΅ΠΌ ΠΈΡΡΠ»Π΅Π΄ΠΎΠ²Π°Ρ‚ΡŒ ΠΎΡ‚Π½ΠΎΡˆΠ΅Π½ΠΈΡ ΠΌΠ΅ΠΆΠ΄Ρƒ Π½ΠΈΠΌΠΈ Π² схСмах. ΠŸΠ΅Ρ€Π²Ρ‹Π΅ ΠΈ, ΠΏΠΎΠΆΠ°Π»ΡƒΠΉ, самыС Π²Π°ΠΆΠ½Ρ‹Π΅ ΠΎΡ‚Π½ΠΎΡˆΠ΅Π½ΠΈΡ ΠœΠ΅ΠΆΠ΄Ρƒ Ρ‚ΠΎΠΊΠΎΠΌ, напряТСниСм ΠΈ сопротивлСниСм называСтся Π·Π°ΠΊΠΎΠ½ Ома, ΠΎΡ‚ΠΊΡ€Ρ‹Ρ‚Ρ‹ΠΉ Π“Π΅ΠΎΡ€Π³ΠΎΠΌ Π‘Π°ΠΉΠΌΠΎΠ½ΠΎΠΌ Омом ΠΈ ΠΎΠΏΡƒΠ±Π»ΠΈΠΊΠΎΠ²Π°Π½Π½Ρ‹ΠΉ Π² Π΅Π³ΠΎ ΡΡ‚Π°Ρ‚ΡŒΠ΅ 1827 Π³ΠΎΠ΄Π°, . Π“Π°Π»ΡŒΠ²Π°Π½ΠΈΡ‡Π΅ΡΠΊΠ°Ρ Ρ†Π΅ΠΏΡŒ, исслСдованная матСматичСски, .Π“Π»Π°Π²Π½ΠΎΠ΅ ΠΎΡ‚ΠΊΡ€Ρ‹Ρ‚ΠΈΠ΅ Ома Π·Π°ΠΊΠ»ΡŽΡ‡Π°Π»ΠΎΡΡŒ Π² Ρ‚ΠΎΠΌ, Ρ‡Ρ‚ΠΎ Π²Π΅Π»ΠΈΡ‡ΠΈΠ½Π° элСктричСского Ρ‚ΠΎΠΊΠ° Ρ‡Π΅Ρ€Π΅Π· мСталличСский ΠΏΡ€ΠΎΠ²ΠΎΠ΄Π½ΠΈΠΊ Π² Ρ†Π΅ΠΏΠΈ прямо ΠΏΡ€ΠΎΠΏΠΎΡ€Ρ†ΠΈΠΎΠ½Π°Π»ΡŒΠ½ΠΎ напряТСниС, ΠΏΡ€ΠΈΠ»ΠΎΠΆΠ΅Π½Π½ΠΎΠ΅ ΠΊ Π½Π΅ΠΌΡƒ, для любой Π·Π°Π΄Π°Π½Π½ΠΎΠΉ Ρ‚Π΅ΠΌΠΏΠ΅Ρ€Π°Ρ‚ΡƒΡ€Ρ‹. Ом Π²Ρ‹Ρ€Π°ΠΆΠ΅Π½ Π΅Π³ΠΎ ΠΎΡ‚ΠΊΡ€Ρ‹Ρ‚ΠΈΠ΅ Π² Π²ΠΈΠ΄Π΅ простого уравнСния, ΠΎΠΏΠΈΡΡ‹Π²Π°ΡŽΡ‰Π΅Π³ΠΎ, ΠΊΠ°ΠΊ напряТСниС, Ρ‚ΠΎΠΊ ΠΈ сопротивлСниС взаимосвязаны:

Π’ этом алгСбраичСском Π²Ρ‹Ρ€Π°ΠΆΠ΅Π½ΠΈΠΈ напряТСниС (E) Ρ€Π°Π²Π½ΠΎ Ρ‚ΠΎΠΊΡƒ (I) ΡƒΠΌΠ½ΠΎΠΆΠ΅Π½Π½ΠΎΠ΅ Π½Π° сопротивлСниС (R). Π˜ΡΠΏΠΎΠ»ΡŒΠ·ΡƒΡ ΠΌΠ΅Ρ‚ΠΎΠ΄Ρ‹ Π°Π»Π³Π΅Π±Ρ€Ρ‹, ΠΌΡ‹ ΠΌΠΎΠΆΠ΅ΠΌ ΠΏΡ€Π΅ΠΎΠ±Ρ€Π°Π·ΠΎΠ²Π°Ρ‚ΡŒ это ΡƒΡ€Π°Π²Π½Π΅Π½ΠΈΠ΅ Π² Π΄Π²Π° Π²Π°Ρ€ΠΈΠ°Π½Ρ‚Π°, Ρ€Π΅ΡˆΠ°Ρ для I ΠΈ R, соотвСтствСнно:

Π”Π°Π²Π°ΠΉΡ‚Π΅ посмотрим, ΠΊΠ°ΠΊ эти уравнСния ΠΌΠΎΠ³ΡƒΡ‚ Ρ€Π°Π±ΠΎΡ‚Π°Ρ‚ΡŒ, Ρ‡Ρ‚ΠΎΠ±Ρ‹ ΠΏΠΎΠΌΠΎΡ‡ΡŒ Π½Π°ΠΌ Π°Π½Π°Π»ΠΈΠ·ΠΈΡ€ΠΎΠ²Π°Ρ‚ΡŒ простыС схСмы:

Π’ ΠΏΡ€ΠΈΠ²Π΅Π΄Π΅Π½Π½ΠΎΠΉ Π²Ρ‹ΡˆΠ΅ схСмС Π΅ΡΡ‚ΡŒ Ρ‚ΠΎΠ»ΡŒΠΊΠΎ ΠΎΠ΄ΠΈΠ½ источник напряТСния (аккумулятор слСва) ΠΈ Ρ‚ΠΎΠ»ΡŒΠΊΠΎ ΠΎΠ΄ΠΈΠ½ источник сопротивлСния Ρ‚ΠΎΠΊΡƒ. (Π»Π°ΠΌΠΏΠ° справа).Π­Ρ‚ΠΎ позволяСт ΠΎΡ‡Π΅Π½ΡŒ Π»Π΅Π³ΠΊΠΎ ΠΏΡ€ΠΈΠΌΠ΅Π½ΡΡ‚ΡŒ Π·Π°ΠΊΠΎΠ½ Ома. Если ΠΌΡ‹ Π·Π½Π°Π΅ΠΌ значСния Π»ΡŽΠ±Ρ‹Ρ… Π΄Π²ΡƒΡ… ΠΈΠ· Ρ‚Ρ€Π΅Ρ… Π²Π΅Π»ΠΈΡ‡ΠΈΠ½ (напряТСния, Ρ‚ΠΎΠΊΠ° ΠΈ сопротивлСния) Π² этой Ρ†Π΅ΠΏΠΈ, ΠΌΡ‹ ΠΌΠΎΠΆΠ΅ΠΌ ΠΈΡΠΏΠΎΠ»ΡŒΠ·ΠΎΠ²Π°Ρ‚ΡŒ Π·Π°ΠΊΠΎΠ½ Ома для опрСдСлСния Ρ‚Ρ€Π΅Ρ‚ΡŒΠ΅ΠΉ.

Π’ этом ΠΏΠ΅Ρ€Π²ΠΎΠΌ ΠΏΡ€ΠΈΠΌΠ΅Ρ€Π΅ ΠΌΡ‹ рассчитаСм Π²Π΅Π»ΠΈΡ‡ΠΈΠ½Ρƒ Ρ‚ΠΎΠΊΠ° (I) Π² Ρ†Π΅ΠΏΠΈ, учитывая значСния напряТСния (E) ΠΈ сопротивлСния (R):

Какая Π²Π΅Π»ΠΈΡ‡ΠΈΠ½Π° Ρ‚ΠΎΠΊΠ° (I) Π² этой Ρ†Π΅ΠΏΠΈ?

Π’ этом Π²Ρ‚ΠΎΡ€ΠΎΠΌ ΠΏΡ€ΠΈΠΌΠ΅Ρ€Π΅ ΠΌΡ‹ рассчитаСм Π²Π΅Π»ΠΈΡ‡ΠΈΠ½Ρƒ сопротивлСния (R) Π² Ρ†Π΅ΠΏΠΈ, учитывая значСния напряТСния (E) ΠΈ Ρ‚ΠΎΠΊΠ° (I):

КакоС сопротивлСниС (R) ΠΏΡ€Π΅Π΄Π»Π°Π³Π°Π΅Ρ‚ Π»Π°ΠΌΠΏΠ°?

Π’ послСднСм ΠΏΡ€ΠΈΠΌΠ΅Ρ€Π΅ ΠΌΡ‹ рассчитаСм Π²Π΅Π»ΠΈΡ‡ΠΈΠ½Ρƒ напряТСния, ΠΏΠΎΠ΄Π°Π²Π°Π΅ΠΌΠΎΠ³ΠΎ Π±Π°Ρ‚Π°Ρ€Π΅Π΅ΠΉ, с ΡƒΡ‡Π΅Ρ‚ΠΎΠΌ Π·Π½Π°Ρ‡Π΅Π½ΠΈΠΉ Ρ‚ΠΎΠΊΠ° (I) ΠΈ сопротивлСния (R):

КакоС напряТСниС обСспСчиваСт аккумулятор?

Π—Π°ΠΊΠΎΠ½ Ома — ΠΎΡ‡Π΅Π½ΡŒ простой ΠΈ ΠΏΠΎΠ»Π΅Π·Π½Ρ‹ΠΉ инструмСнт для Π°Π½Π°Π»ΠΈΠ·Π° элСктричСских схСмы.Он Ρ‚Π°ΠΊ часто ΠΈΡΠΏΠΎΠ»ΡŒΠ·ΡƒΠ΅Ρ‚ΡΡ ΠΏΡ€ΠΈ ΠΈΠ·ΡƒΡ‡Π΅Π½ΠΈΠΈ элСктричСства ΠΈ элСктроники, ΠΊΠΎΡ‚ΠΎΡ€ΡƒΡŽ Π½ΡƒΠΆΠ½ΠΎ ΡΠΎΡ…Ρ€Π°Π½ΠΈΡ‚ΡŒ Π² памяти ΡΠ΅Ρ€ΡŒΠ΅Π·Π½Ρ‹ΠΌΠΈ студСнт. Для Ρ‚Π΅Ρ…, ΠΊΡ‚ΠΎ Π΅Ρ‰Π΅ Π½Π΅ Π·Π½Π°ΠΊΠΎΠΌ с Π°Π»Π³Π΅Π±Ρ€ΠΎΠΉ, Π΅ΡΡ‚ΡŒ Ρ‚Ρ€ΡŽΠΊ с Π·Π°ΠΏΠΎΠΌΠΈΠ½Π°Π½ΠΈΠ΅ΠΌ Ρ‚ΠΎΠ³ΠΎ, ΠΊΠ°ΠΊ Ρ€Π΅ΡˆΠΈΡ‚ΡŒ для любого ΠΎΠ΄Π½ΠΎΠ³ΠΎ количСства, учитывая Π΄Ρ€ΡƒΠ³ΠΎΠ΅ Π΄Π²Π°. Π‘Π½Π°Ρ‡Π°Π»Π° располоТитС Π±ΡƒΠΊΠ²Ρ‹ E, I ΠΈ R Π² Π²ΠΈΠ΄Π΅ Ρ‚Ρ€Π΅ΡƒΠ³ΠΎΠ»ΡŒΠ½ΠΈΠΊΠ° ΡΠ»Π΅Π΄ΡƒΡŽΡ‰ΠΈΠΌ ΠΎΠ±Ρ€Π°Π·ΠΎΠΌ:

Если Π²Ρ‹ Π·Π½Π°Π΅Ρ‚Π΅ E ΠΈ I ΠΈ Ρ…ΠΎΡ‚ΠΈΡ‚Π΅ ΠΎΠΏΡ€Π΅Π΄Π΅Π»ΠΈΡ‚ΡŒ R, просто ΡƒΠ΄Π°Π»ΠΈΡ‚Π΅ R с ΠΊΠ°Ρ€Ρ‚ΠΈΠ½ΠΊΠΈ ΠΈ посмотритС, Ρ‡Ρ‚ΠΎ ΠΎΡΡ‚Π°Π»ΠΎΡΡŒ:

Если Π²Ρ‹ Π·Π½Π°Π΅Ρ‚Π΅ E ΠΈ R ΠΈ Ρ…ΠΎΡ‚ΠΈΡ‚Π΅ ΠΎΠΏΡ€Π΅Π΄Π΅Π»ΠΈΡ‚ΡŒ I, ΡƒΠ΄Π°Π»ΠΈΡ‚Π΅ I ΠΈ посмотритС, Ρ‡Ρ‚ΠΎ ΠΎΡΡ‚Π°Π»ΠΎΡΡŒ:

НаконСц, Ссли Π²Ρ‹ Π·Π½Π°Π΅Ρ‚Π΅ I ΠΈ R ΠΈ Ρ…ΠΎΡ‚ΠΈΡ‚Π΅ ΠΎΠΏΡ€Π΅Π΄Π΅Π»ΠΈΡ‚ΡŒ E, ΡƒΠ΄Π°Π»ΠΈΡ‚Π΅ E ΠΈ посмотритС, Ρ‡Ρ‚ΠΎ ΠΎΡΡ‚Π°Π»ΠΎΡΡŒ:

Π’ ΠΊΠΎΠ½Ρ†Π΅ ΠΊΠΎΠ½Ρ†ΠΎΠ², Π²Π°ΠΌ придСтся ΠΏΠΎΠ·Π½Π°ΠΊΠΎΠΌΠΈΡ‚ΡŒΡΡ с Π°Π»Π³Π΅Π±Ρ€ΠΎΠΉ, Ρ‡Ρ‚ΠΎΠ±Ρ‹ ΡΠ΅Ρ€ΡŒΠ΅Π·Π½ΠΎ ΠΈΠ·ΡƒΡ‡Π°Ρ‚ΡŒ элСктричСство ΠΈ элСктронику, Π½ΠΎ этот совСт ΠΌΠΎΠΆΠ΅Ρ‚ ΡΠ΄Π΅Π»Π°Ρ‚ΡŒ ваш ΠΏΠ΅Ρ€Π²Ρ‹ΠΉ расчСты Π·Π°ΠΏΠΎΠΌΠ½ΠΈΡ‚ΡŒ Π½Π΅ΠΌΠ½ΠΎΠ³ΠΎ Π»Π΅Π³Ρ‡Π΅.Если Ρ‚Π΅Π±Π΅ ΠΊΠΎΠΌΡ„ΠΎΡ€Ρ‚Π½ΠΎ с Π°Π»Π³Π΅Π±Ρ€Ρ‹, всС, Ρ‡Ρ‚ΠΎ Π²Π°ΠΌ Π½ΡƒΠΆΠ½ΠΎ ΡΠ΄Π΅Π»Π°Ρ‚ΡŒ, это Π·Π°Ρ„ΠΈΠΊΡΠΈΡ€ΠΎΠ²Π°Ρ‚ΡŒ E = IR Π² памяти ΠΈ ΠΏΠΎΠ»ΡƒΡ‡ΠΈΡ‚ΡŒ Π΄Ρ€ΡƒΠ³ΠΈΠ΅ Π΄Π²Π΅ Ρ„ΠΎΡ€ΠΌΡƒΠ»Ρ‹ ΠΈΠ· Ρ‚ΠΎΠ³ΠΎ, ΠΊΠΎΠ³Π΄Π° ΠΎΠ½ΠΈ Π²Π°ΠΌ понадобятся!

  • ΠžΠ‘Π—ΠžΠ :
  • НапряТСниС измСряСтся Π² Π²ΠΎΠ»ΡŒΡ‚Π°Ρ… , обозначаСтся Π±ΡƒΠΊΠ²Π°ΠΌΠΈ Β«EΒ» ΠΈΠ»ΠΈ Β«VΒ».

Π”ΠΎΠ±Π°Π²ΠΈΡ‚ΡŒ ΠΊΠΎΠΌΠΌΠ΅Π½Ρ‚Π°Ρ€ΠΈΠΉ

Π’Π°Ρˆ адрСс email Π½Π΅ Π±ΡƒΠ΄Π΅Ρ‚ ΠΎΠΏΡƒΠ±Π»ΠΈΠΊΠΎΠ²Π°Π½. ΠžΠ±ΡΠ·Π°Ρ‚Π΅Π»ΡŒΠ½Ρ‹Π΅ поля ΠΏΠΎΠΌΠ΅Ρ‡Π΅Π½Ρ‹ *